Download as pdf or txt
Download as pdf or txt
You are on page 1of 348

CUMMINGS

OTOLARYNGOLOGY
HEAD & NECK SURGERY
FOURTH EDITION REVIEW
CUMMINGS O T O L A R Y N G O L O G Y — H E A D & N E C K SURGERY
FOURTH E D I T I O N REVIEW

VOLUME O N E
Part One: General Considerations i n Head and Neck
Charles W. Cummings, Editor
K. Thomas Robbins, Associate Editor
Part Two: Face
David E. Schuller, Editor
J. Regan Thomas, Associate Editor

VOLUME TWO
Part Three: Nose
David E. Schuller, Editor
J. Regan Thomas, Associate Editor
Part Four: Paranasal Sinuses
David E. Schuller, Editor
J. Regan Thomas, Associate Editor
Part Five: Salivary Glands
Bruce H. Haughey, Editor
Part Six: Oral Cavity/Pharynx/Esophagus
Bruce H. Haughey, Editor

VOLUME T H R E E
Part Seven: Larynx/Trachea/Bronchus
Paul W. Flint, Editor
Part Eight: Neck
K. Thomas Robbins, Editor
Part Nine: Thyroid/Parathyroid
K. Thomas Robbins, Editor

VOLUME F O U R
Part Ten: General
Lee A. Harker, Editor
Part Eleven: Infectious Processes
Lee A. Harker, Editor
Part Twelve: Vestibular System
Lee A. Harker, Editor
Part Thirteen: Facial Nerve
Lee A. Harker, Editor
Part Fourteen: Auditory System
Lee A. Harker, Editor
Part Fifteen: Cochlear Implants
Lee A. Harker, Editor
Part Sixteen: Skull Base
Lee A. Harker, Editor
Part Seventeen: Pediatric Otolaryngology
Mark A. Richardson, Editor
CUMMINGS
OTOLARYNGOLOGY
H E A D & N E C K SURGERY
FOURTH EDITION REVIEW
Charles W. Cummings, M.D.
Distingushed Service Professor
Department of Otolaryngology—Head and Neck Surgery
Johns Hopkins University School of Medicine
Baltimore, Maryland
Paul W. Flint, M.D. Lee A. Harker, M.D.
Professor Deputy Director
Department of Otolaryngology—Head and Neck Surgery Boys Town National Research Hospital
Director, Center for Airway, Laryngeal, and Voice Disorders Vice Chairman
Co-Director Minimally Invasive Surgical Training Center Department of Otolaryngology and Human
Johns Hopkins University School of Medicine Communication
Baltimore, Maryland Creighton University School of Medicine
Omaha, Nebraska
Bruce H. Haughey, MBChB, FACS, F R A C S
Professor and Director Mark A. Richardson, M.D.
Head and Neck Surgical Oncology Professor and Chairman
Department of Otolaryngology—Head and Neck Surgery Department of Otolaryngology—Head and Neck Surgery
Washington University School of Medicine Oregon Health and Science University
St. Louis, Missouri Portland, Oregon
K. Thomas Robbins, M.D. David E . Schuller, M.D.
Professor and Chair Professor and Chairman
Division of Otolaryngology, Department of Surgery Department of Otolaryngology—Head
Southern Illinois University School of Medicine and Neck Surgery
Springfield, Illinois Executive Director, Arthur G. James Cancer Hospital
J . Regan Thomas, M.D. and Richard J. Solove Research Institute
Francis L. Lederer Professor and Chairman Deputy Director, Comprehensive Cancer Center
Department of Otolaryngology—Head and Neck Surgery The Ohio State University
University of Illinois Columbus, Ohio
Chicago, Illinois

ELSEVIER
M O S B Y
ELSEVIER
M O S B Y
The Curtis Center
170 S Independence Mall W 300E
Philadelphia, Pennsylvania 19106

Cummings Otolaryngology—Head & Neck Surgery Fourth Edition Review


Copyright © 2005, Elsevier Inc. All rights reserved.

No part of this publication may be reproduced or transmitted in any form or by any means, electronic or
mechanical, including photocopying, recording, or any information storage and retrieval system, without
permission in writing from the publisher. Permissions may be sought directly from Elsevier's Health Sciences
Rights Department in Philadelphia, PA, USA: phone: (+1) 215 238 7869, fax: (+1) 215 238 2239, e-mail:
healthpermissions@elsevier.com. You may also complete your request on-line via the Elsevier homepage
(http://www.elsevier.com), by selecting 'Customer Support' and then 'Obtaining Permissions.'

NOTICE

Otolaryngology is an ever-changing field. Standard safety precautions must be followed, but as new research
and clinical experience broaden our knowledge, changes in treatment and drug therapy may become neces-
sary or appropriate. Readers are advised to check the most current product information provided by the man-
ufacturer of each drug to be administered to verify the recommended dose, the method and duration of
administration, and contraindications. It is the responsibility of the licensed prescriber, relying on experience
and knowledge of the patient, to determine dosages and the best treatment for each individual patient.
Neither the publisher nor the author assumes any liability for any injury and/or damage to persons or prop-
erty arising from this publication.

Previous edition copyrighted 1998.

International Standard Book Number 0-323-03006-8

Acquisitions Editor: Rebecca Schmidt Gaertner


Developmental Editor: Mary Beth Murphy
Editorial Assistant: Suzanne Flint

Printed in the United States of America

Last digit is the print number: 9 8 7 6 5 4 3 2 1


Contributors

George L . Adams, M.D. Thomas J . Balkany, M.D.


Minneapolis, Minnesota Miami, Florida

Peter A. Adamson, M.D., F.R.C.S.C., F.A.C.S. Fuad M. Baroody, M.D.


Toronto, Ontario, Canada Chicago, Illinois

Antoine Adenis, M.D., Ph.D. Roberto L . Barretto, M.D.


Lille, France Orange, California

Seth Akst, M.D. Jonathan Z. Baskin, M.D.


Baltimore, Maryland Syracuse, New York

David M. Albert, F.R.C.S. Robert W. Bastian, M.D.


London, United Kingdom Downers Grove, Illinois

lee Ching Anderson, M.D.* Carol A. Bauer, M.D.


Baltimore, Maryland Springfield, Illinois

William B. Armstrong, M.D. Aaron Benson, M.D.


Orange, California Chicago, Illinois

Moises A. Arriaga, M.D., F.A.C.S. Nasir I. Bhatti, M.D.


Pittsburgh, Pennsylvania Baltimore, Maryland

Agustin J . Arrieta, M.D. Carol M. Bier-Laning, M.D.


Tampa, Florida Barrington, Illinois

H. Alexander Arts, M.D., F.A.C.S. James E . Blaugrund, M.D.


A n n Arbor, Michigan Pittsburgh, Pennsylvania

Yasmine A. Ashram, M.D., D.A.B.N.M. Nikolas H. Blevins, M.D.


Alexandria, Egypt Boston, Massachusetts

Nafi Aygun, M.D. Andrew Blitzer, M.D., D.D.S., F.A.C.S.


Baltimore, Maryland New York, New York

Douglas D. Backous, M.D. Derald E . Brackmann, M.D.


Seattle, Washington Los Angeles, California

Shan R. Baker, M.D. Carol R. Bradford, M.D., F.A.C.S.


Livonia, Michigan A n n Arbor, Michigan

Barton F. Branstetter IV, M.D.


* Review-book author only. Pittsburgh, Pennsylvania

v
vi Contributors

Hilary A. Brodie, M.D., Ph.D. Richard A. Chole, M.D., Ph.D.


Davis, California St. Louis, Missouri

Carolyn J . Brown, Ph.D. Martin J . Citardi, M.D.


Iowa City, Iowa Cleveland, Ohio

Karla Brown, M.D. Seth Cohen, M.D.*


New Orleans, Louisiana Nashville, Tennessee

Orval E . Brown, M.D. Savita Collins, M.D.


Dallas, Texas Gainesville, Florida

J . Dale Browne, M.D., F.A.C.S. Philippe Contencin, M.D.


Winston-Salem, North Carolina Paris, France

John Buatti, M.D. Raymond D. Cook, M.D.


Iowa City, Iowa
Chapel Hill, North Carolina

Daniel Buchbinder, D.M.D., M.D.


Ted A. Cook, M.D., F.A.C.S.
New York, New York
Portland, Oregon

Patrick J . Byrne, M.D.


Baltimore, Maryland
Robin T. Cotton, M.D.
Cincinnati, Ohio
Joseph A. Califano III, M.D.
Baltimore, Maryland Marion Everett Couch, M.D., Ph.D.
Chapel Hill, North Carolina
John P. Carey, M.D.
Baltimore, Maryland Mark S. Courey, M.D.
Nashville, Tennessee
Eric R. Carlson, D.M.D., M.D.
Knoxville, Tennessee Roger L . Crumley, M.D.
Orange, California
Ricardo L . Carrau, M.D.
Pittsburgh, Pennsylvania Oswaldo Laercio M. Cruz, M.D.
Sao Paulo, Brazil
Roy R. Casiano, M.D.
Miami, Florida Bernard J . Cummings, M.B., Ch.B., F.R.A.N.Z.C.R.,
F.R.C.R., F.R.C.P.C.
Jon B. Chadwell, M.D. Toronto, Ontario, Canada
Cincinnati, Ohio
Charles W. Cummings, M.D.
Christopher Y. Chang, M.D.
Baltimore, Maryland
Durham, North Carolina

Calhoun D. Cunningham III, M.D.


Kristi E . Chang, M.D.
Charleston, South Carolina
Iowa City, Iowa

Burke E . Chegar, M.D. Jeffrey Cutler, M.D.*


Syracuse, New York Franklin, Tennessee

Anton Chen, M.D.* Larry E . Davis, M.D.


Nashville, Tennessee Albuquerque, New Mexico

Sukgi S. Choi, M.D. Terry A. Day, M.D.


Washington DC Charleston, South Carolina

Antonio De la Cruz, M.D.


* Review-book author only. Los Angeles, California
Contributors vii

Charles C. Delia Santina, M.D., Ph.D. Arlene A. Forastiere, M.D.


Baltimore, Maryland Baltimore, Maryland

Craig S. Derkay, M.D. L . Arick Forrest, M.D.


Norfolk, Virginia Columbus, Ohio

Robert A. Dobie, M.D., F.A.C.S. Oren Friedman, M.D.


Sacramento, California Rochester, Minnesota

Newton O. Duncan III, M.D. John L . Frodel, Jr., M.D., F.A.C.S.


Houston, Texas Danville, Pennsylvania

Lisa Earnest, M.D.* Gerry F. Funk, M.D.


Baltimore, Maryland Iowa City, Iowa

Scott D. Z. Eggers, M.D. Thomas J . Gal, M.D., M.P.H., Major, USAF-MC


Rochester, Minnesota Lackland, Texas

David W. Eisele, M.D. Suzanne K. Doud Galli, M.D., Ph.D.


San Francisco, California Toronto, Ontario, Canada

Hussam K. El-Kashlan, M.D. Bruce J . Gantz, M.D.


A n n Arbor, Michigan Iowa City, Iowa

Ravindhra G. Elluru, M.D., Ph.D. C. Gaelyn Garrett, M.D.


Cincinnati, Ohio Nashville, Tennessee

Ramon M. Esclamado, M.D. Holger G. Gassner, M.D.


Cleveland, Ohio Rochester, Minnesota

Chun Y. Fan, M.D., Ph.D. George A. Gates, M.D.


Little Rock, Arkansas Seattle, Washington

Edward H. Farrior, M.D., F.A.C.S. William Donald Gay, D.D.S., F.A.C.D.


Tampa, Florida St. Louis, Missouri

Richard T. Farrior, M.D. Eric M. Genden, M.D.


Boca Grande, Florida New York, New York

Russell A. Faust, Ph.D., M.D. Elisa M. Ghezzi, D.D.S.


Detroit, Michigan A n n Arbor, Michigan

Willard E . Fee, Jr., M.D. Timothy G. Gillum, M.D.


Stanford, California Marion, Indiana

Berrylin J . Ferguson, M.D. Marian Girardi, Ph.D.


Pittsburgh, Pennsylvania Alexandria, Virginia

Jill B. Firszt, Ph.D. Douglas A. Girod, M.D.


Milwaukee, Wisconsin Kansas City, Kansas

Paul W. Flint, M.D. George S. Goding, Jr., M.D.


Baltimore, Maryland Minneapolis, Minnesota

Robert L . Folmer, Ph.D. Andrew N. Goldberg, M.D., F.A.C.S.


Portland, Oregon San Francisco, California

David Goldenberg, M.D.


* Review-book author only. Baltimore, Maryland
viii Contributors

W. Jarrard Goodwin, Jr., M.D. Kevin J . Hulett, M.D.


Miami, Florida Chicago, Illinois

Daniel O. Graney, Ph.D. Murad Husein, M.D., M . S c , F.R.C.S.(C)


Seattle, Washington London, Ontario, Canada

Patrick K. Ha, M.D. Steven W. Ing, M.D.


Baltimore, Maryland Wilkes-Barre, Pennsylvania

Jeffrey R. Haller, M.D. Andrew F. Inglis, Jr., M.D.


Missoula, Montana Seattle, Washington

Jongwook Ham, M.D. Robert K. Jackler, M.D.


Elgin, Illinois Stanford, California

Ehab Y. Hanna, M.D., F.A.C.S. Herman A. Jenkins, M.D.


Houston, Texas Denver, Colorado

Marian R. Hansen, M.D. John K. Joe, M.D.


Iowa City, Iowa New Haven, Connecticut

Lee A. Harker, M.D. Stephanie Joe, M.D.


Omaha, Nebraska Chicago, Illinois

Robert V. Harrison, Ph.D., D.Sc. Jonas T. Johnson, M.D.


Toronto, Ontario, Canada Pittsburgh, Pennsylvania

Bruce H. Haughey, MBChB, FACS, F R A C S Timothy M. Johnson, M.D.


St. Louis, Missouri Ann Arbor, Michigan

Gerald B. Healy, M.D., F.A.C.S. Kim Richard Jones, M.D., Ph.D.


Boston, Massachusetts Chapel Hill, North Carolina

Michael L . Hinni, M.D. Sheldon S. Kabaker, M.D.


Scottsdale, Arizona Oakland, California

Henry T. Hoffman, M.D. Lucy H. Karnell, M.D.


Iowa City, Iowa Iowa City, Iowa

Eric H. Holbrook, M.D. Matthew L . Kashima, M.D.


Boston, Massachusetts Baltimore, Maryland

Lauren D. Holinger, M.D. Robert M. Kellman, M.D.


Chicago, Illinois Syracuse, New York

David B. Horn, M.D. Paul E . Kelly, M.D., F.A.C.S.


Minneapolis, Minnesota Riverhead, New York

John W. House, M.D. David W. Kennedy, M.D.


Los Angeles, California Philadelphia, Pennsylvania

J . W. Hudson, D.D.S. Merrill S. Kies, M.D.


Knoxville, Tennessee Houston, Texas

Matthew C. Hull, M.D. Paul R. Kileny, Ph.D., F.A.S.H.A.


Asheville, North Carolina Ann Arbor, Michigan

Timothy E . Hullar, M.D. David W. Kim, M.D.


St. Louis, Missouri San Francisco, California
Contributors ix

John Kim, M.D., F.R.C.P.C. Ken K. Lee, M.D.


Toronto, Ontario, Canada Portland, Oregon

William J . Kimberling, Ph.D. Nancy Y. Lee, M.D.


Omaha, Nebraska New York, New York

Jeffrey L . Koh, M.D. Stephen Lee, M.D.


Portland, Oregon Fayetteville, Arkansas

Peter J . Koltai, M.D., F.A.C.S., F.A.A.P. Jean-Louis Lefebvre, M.D.


Stanford, California Lille, France

Horst R. Konrad, M.D. Susanna Leighton, F.R.C.S.


Springfield, Illinois London, United Kingdom

Frederick K. Kozak, M.D. Donald A. Leopold, M.D.


Vancouver, British Columbia, Canada Omaha, Nebraska

Paul R. Krakovitz, M.D. Daqing L i , M.D.


Cleveland, Ohio Baltimore, Maryland

Russell W. H. Kridel, M.D., F.A.C.S. Timothy S. Lian, M.D.


Houston, Texas Shreveport, Louisiana

Manoj Kumar, M.S., F.R.C.S. Greg R. Licameli, M.D.


Swansea, Wales, United Kingdom Boston, Massachusetts

Parvesh Kumar, M.D. Charles J . Limb, M.D.


Los Angeles, California Baltimore, Maryland

Dario Kunar, M.D. Jerilyn A. Logemann, Ph.D.


Towson, Maryland Evanston, Illinois

Ollivier Laccourreye, M.D. Brenda L . Lonsbury-Martin, Ph.D.


Paris, France Denver, Colorado

Stephen Y. L a i , M.D., Ph.D. Benjamin M. Loos, M.D.


Pittsburgh, Pennsylvania San Francisco, California

Anil K. Lalwani, M.D. Manuel A. Lopez, M.D.


New York, New York Chicago, Illinois

Paul R. Lambert, M.D. Rodney P. Lusk, M.D., F.A.C.S., F.A.A.P.


Charleston, South Carolina Fort Collins, Colorado

George E . Laramore, M.D., Ph.D. Lawrence R. Lustig, M.D.


Seattle, Washington Baltimore, Maryland

Peter E . Larsen, D.D.S. Anna Lysakowski, Ph.D.


Powell, Ohio Chicago, Illinois

Daniel M. Laskin, D.D.S., M.S., D.Sc. Richard L . Mabry, M.D.


Richmond, Virginia San Antonio, Texas

Richard E . Latchaw, M.D. Carol J . MacArthur, M.D.


Sacramento, California Portland, Oregon

Christine L . Lau, M.D. Allison R. MacGregor, M.D.


St. Louis, Missouri Edge wood, Kentucky
x Contributors

Robert H. Maisel, M.D., F.A.C.S. Khosrow (Mark) Mehrany, M.D.


Minneapolis, Minnesota Portland, Oregon

Patrizia Mancini, M.D. Nancy Price Mendenhail, M.D.


Rome, Italy Gainesville, Florida

Susan J . Mandel, M.D., M.P.H. Saumil N. Merchant, M.D.


Philadelphia, Pennsylvania Boston, Massachusetts

Scott C. Manning, M.D. Jennifer L . Mertes, Au.D.


Seattle, Washington Baltimore, Maryland

Lynette J . Mark, M.D. Anna H. Messner, M.D.


Baltimore, Maryland Palo Alto, California

Jeffery C. Markt, D.D.S. Ted A. Meyer, M.D., Ph.D.


Iowa City, Iowa Iowa City, Iowa

Bradley F. Marple, M.D. James Michelson, M.D.


Dallas, Texas Washington DC

Michael A. Marsh, M.D. Henry A. Milczuk, M.D.


Fort Smith, Arkansas Portland, Oregon

Glen K. Martin, Ph.D. Lloyd B. Minor, M.D.


Denver, Colorado Baltimore, Maryland

Douglas D. Massick, M.D. Steven Ross Mobley, M.D.


Columbus, Ohio Salt Lake City, Utah

Douglas E . Mattox, M.D. Jeffrey Morray, M.D.


Atlanta, Georgia Phoenix, Arizona

Thomas V. McCaffrey, M.D., Ph.D. John B. Mulliken, M.D.


Tampa, Florida Boston, Massachusetts

Timothy M. McCulloch, M.D. Harlan R. Muntz, M.D., F.A.A.P., F.A.C.S.


Seattle, Washington Salt Lake City, Utah

Thomas J . McDonald, M.D. Craig S. Murakami, M.D., F.A.C.S.


Rochester, Minnesota Seattle, Washington

JoAnn McGee, Ph.D. Charles M. Myer III, M.D.


Omaha, Nebraska Cincinnati, Ohio

Trevor J . I. McGill, M.D., F.A.C.S. Robert M. Naclerio, M.D.


Boston, Massachusetts Chicago, Illinois

John F. McGuire, M.D., M.B.A. Joseph B. Nadol, Jr., M.D.


Irvine, California Boston, Massachusetts

W. Frederick McGuirt, Sr., M.D., F.A.C.S. Philippe Narcy, M.D.


Winston-Salem, North Carolina Paris, France

Sean O. McMenomey, M.D., F.A.C.S. Paul S. Nassif, M.D., F.A.C.S.


Portland, Oregon Beverly Hills, California

J . Scott McMurray, M.D., F.A.A.P., F.A.C.S. Julian M. Nedzelski, M.D., F.R.C.S.C.


Madison, Wisconsin Toronto, Ontario, Canada
Contributors xi

John K. Niparko, M.D. Stephen W. Perkins, M.D., F.A.C.S.


Baltimore, Maryland Indianapolis, Indiana

Susan J . Norton, Ph.D., C C C - A Shirley S. N. Pignatari, M.D., Ph.D.


Seattle, Washington Sao Paulo, Brazil

Daniel W. Nuss, M.D., F.A.C.S. Randall L . Plant, M.D., M.S., F.A.C.S.


New Orleans, Louisiana Anchorage, Alaska

Brian Nussenbaum, M.D. Steven D. Pletcher, M.D.


St. Louis, Missouri San Francisco, California

Bert W. O'Malley, Jr., M.D. Gregory N. Postma, M.D.


Philadelphia, Pennsylvania Winston-Salem, North Carolina

Matthew O'Malley, M.D.* William P. Potsic, M.D., MM


Nashville, Tennessee Philadelphia, Pennsylvania

Patrick J . Oliverio, M.D. Vito C. Quatela, M.D.


Fairfax, Virginia Rochester, New York

Kerry D. Olsen, M.D. C. Rose Rabinov, M.D.


Bakersfield, California
Rochester, Minnesota

Reza Rahbar, M.D., D.M.D.


Juan Camilo Ospina, M.D.
Boston, Massachusetts
Vancouver, British Columbia, Canada

Gregory W. Randolph, M.D., F.A.C.S.


Robert H. Ossoff, D.M.D., M.D.
Boston, Massachusetts
Nashville, Tennessee
Christopher H. Rassekh, M.D.
Brian O'Sullivan, M.B., F.R.C.P.C.
Morgan town, West Virginia
Toronto, Ontario, Canada
Steven D. Rauch, M.D.
Sara Pai, M.D.*
Boston, Massachusetts
Baltimore, Maryland
Lou Reinisch, Ph.D.
John F. Pallanch, M.D. Christchurch, New Zealand
Sioux City, Iowa
Dale H. Rice, M.D.
James N. Palmer, M.D. Los Angeles, California
Philadelphia, Pennsylvania
Mark A. Richardson, M.D.
Stephen S. Park, M.D., F.A.C.S. Portland, Oregon
Charlottesville, Virginia
K. Thomas Robbins, M.D.
G. Alexander Patterson, M.D. Springfield, Illinois
St. Louis, Missouri
Jason K. Rockhill, M.D., Ph.D.
Bruce W. Pearson, M.D., F.R.C.S., F.A.C.S. Seattle, Washington
Jacksonville, Florida
Richard M. Rosenfeld, M.D., M.P.H.
Phillip K. Pellitteri, D.O., F.A.C.S. Brooldyn, New York
Danville, Pennsylvania
Jay T. Rubinstein, M.D., Ph.D.
Jonathan A. Perkins, D.O. Iowa City, Iowa
Seattle, Washington
Michael J . Ruckenstein, M.D., M . S c , F.A.C.S.,
F.R.C.S.C.
* Review-book author only. Philadelphia, Pennsylvania
xii Contributors

Christina L . Runge-Samuelson, Ph.D. James J . Sciubba, D.M.D., Ph.D.


Milwaukee, Wisconsin Baltimore, Maryland

Cynda Hylton Rushton, D . N . S c , R.N. Jon K. Shallop, Ph.D.


Baltimore, Maryland Rochester, Minnesota

Leonard P. Rybak, M.D., Ph.D. Clough Shelton, M.D., F.A.C.S.


Springfield, Illinois Salt Lake City, Utah

Alain N. Sabri, M.D. Neil T. Shepard, Ph.D.


Rochester, Minnesota Philadelphia, Pennsylvania

John R. Salassa, M.D. Samuel G. Shiley, M.D.


Jacksonville, Florida Portland, Oregon

Thomas J . Salinas, D.D.S., M.S. Edward J . Sliin, M.D.


Omaha, Nebraska Elmhurst, New York

Sandeep Samant, M.D., F.R.C.S. Jonathan A. Ship, D.M.D.


Memphis, Tennessee New York, New York

Robin A. Samlan, M.S., C C C - S L P Kevin A. Shumrick, M.D.


Baltimore, Maryland Cincinnati, Ohio

Ravi N. Samy, M.D. Kathleen C. Y. Sie, M.D.


Dallas, Texas Seattle, Washington

Gabriella Sanchez, M.D.* Patricia Silva, M.D.


Nashville, Tennessee Sacramento, California

Peter A. Santi, Ph.D. Alfred Simental, M.D.


Minneapolis, Minnesota Loma Linda, California

Steven D. Schaefer, M.D. Ranjiv Sivanandan, M.D.


New York, New York Stanford, California

Richard L . Scher, M.D. Marshall E . Smith, M.D., F.A.A.P, F.A.C.S.


Durham, North Carolina Salt Lake City, Utah

David A. Schessel, Ph.D., M.D. Richard J . H. Smith, M.D.


Washington DC Iowa City, Iowa

Joshua S. Schindler, M.D. Russell Smith, M.D.


Nashville, Tennessee Iowa City, Iowa

Cecelia E . Schmalbach, M.D. Robert A. Sofferman, M.D.


A n n Arbor, Michigan Burlington, Vermont

Ilona M. Schmalfuss, M.D. Peter S. Staats, M.D.


Gainesville, Florida Baltimore, Maryland

David E . Schuller, M.D. Hinrich Staecker, M.D.


Columbus, Ohio Baltimore, Maryland

Aldo Cassol Stamm, M.D., Ph.D.


* Review-book author only. Sao Paulo, Brazil
Contributors xiii

James A. Stankiewicz, M.D. Dean M. Toriumi, M.D.


Maywood, Illinois Chicago, Illinois

Laura M. Sterni, M.D. Joseph B. Travers, Ph.D.


Baltimore, Maryland Columbus, Ohio

Holger H. Sudhoff, M.D. Susan P. Travers, Ph.D.


Bochum, Germany Columbus, Ohio

James Y. Suen, M.D., F.A.C.S. Robert J . Troell, M.D., F.A.C.S.


Little Rock, Arkansas Las Vegas, Nevada

John B. Sunwoo, M.D. Terrance T. Tsue, M.D., F.A.C.S.


St. Louis, Missouri Kansas City, Kansas

Neil A. Swanson, M.D. Ralph P. Tufano, M.D.


Portland, Oregon Baltimore, Maryland

Veronica C. Swanson, M.D. David E . Tunkel, M.D., F.A.A.P, F.A.C.S.


Portland, Oregon Baltimore, Maryland

Jonathan M. Sykes, M.D., F.A.C.S. Ravindra Uppaluri, M.D., Ph.D.


Sacramento, California St. Louis, Missouri

M. Eugene Tardy, Jr., M.D., F.A.C.S. Mark L . Urken, M.D.


Chicago, Illinois New York, New York

Sherard A. Tatum III, M.D. Michael F. Vaezi, M.D., Ph.D.


Syracuse, New York Cleveland, Ohio

Helene M. Taylor, M.S., C C C - S L P Thierry Van Den Abbeele, M.D.


Salt Lake City, Utah Paris, France

S. Mark Taylor, M.D., F . R . C . S . C . Jason F. Vollweiler, M.D., Ph.D.


Halifax, Nova Scotia, Canada Cleveland, Ohio

Steven A. Telian, M.D. P. Ashley Wackym, M.D., F.A.C.S.


Ann Arbor, Michigan Milwaukee, Wisconsin

David J . Terris, M.D., F.A.C.S. David L . Walner, M.D., M.S.


Augusta, Georgia Chicago, Illinois

J . Regan Thomas, M.D., F.A.C.S. Edward J . Walsh, Ph.D.


Chicago, Illinois Omaha, Nebraska

James N. Thompson, M.D., F.A.C.S. Tom D. Wang, M.D., F.A.C.S.


Dallas, Texas Portland, Oregon

Robert J . Hbesar, M.D. Randal S. Weber, M.D.


Rochester, Minnesota Houston, Texas

Evan J . Tobin, M.D. Harrison G. Weed, M.S., M.D., F.A.C.P.


Columbus, Ohio Columbus, Ohio

Travis T. Tollefson, M.D. Richard O. Wein, M.D.


Sacramento, California Jackson, Mississippi
xiv Contributors

Gregory S. Weinstein, M.D., F.A.C.S. Gayle Ellen Woodson, M.D.


Philadelphia, Pennsylvania Springfield, Illinois

Ralph F. Wetmore, M.D. Audie L . Woolley, M.D., F.A.C.S.


Philadelphia, Pennsylvania Birmingham, Alabama

Ernest A. Weymuller, Jr., M.D. Charles D. Yingling, Ph.D., D.A.B.N.M.


Seattle, Washington Sausalito, California

Matthew Whitley, M.D.* Bevan Yueh, M.D., M.P.H.


East Haven, Connecticut Seattle, Washington

Brian J . Wiatrak, M.D. Rex C. Yung, M.D., F.C.C.P.


Birmingham, Alabama Baltimore, Maryland

J . Paul Willging, M.D. George H. Zalzal, M.D.


Cincinnati, Ohio Washington DC

Michael A. Williams, M.D. David S. Zee, M.D.


Baltimore, Maryland Baltimore, Maryland

Franz J . Wippold II, M.D., F.A.C.S. Jacob W. Zeiders, M.D.


St. Louis, Missouri Tampa, Florida

Justin Wittkopf, M.D.* Marc S. Zimbler, M.D.


Nashville, Tennessee New York, New York

Matthew Wolpoe, M.D. S. James Zinreich, M.D.


Baltimore, Maryland Baltimore, Maryland

Teresa A. Zwolan, Ph.D.


* Review-book author only. Ann Arbor, Michigan
Preface

This review volume is published to accompany the • Improve delivery of excellent care to patients with
fourth edition of Otolaryngology—Head and Neck otolaryngological disorders.
Surgery. I t contains hundreds of multiple choice • Facilitate preparation by candidates for formal
questions that have been compiled both by authors of examinations i n otolaryngology—head and neck
their respective chapters and, i n some cases, by des- surgery.
ignated associates i n the field of otolaryngology—head
and neck surgery. This has brought additional objec- Please note that these questions have not been pro-
tivity to the process. The editors have reviewed all vided to any examining body for use i n a specific test.
material for consistency and to ensure that i t targets The editors also welcome informed comment from
the core information i n each chapter. (The editors members of the specialty on this review material for
also stress that a correct answer, as designated by the the improvement of future editions.
author of that question, may be open to debate from
other authorities i n the field.) I t is anticipated that by Charles W. Cummings, M.D.
studying and answering these questions, readers will Paul W. Flint, M.D.
be able to: Lee A. Harker, M.D.
Bruce H. Haughey, MBChB, FACS, FRACS
• Measure their fund of knowledge both before and Mark A. Richardson, M.D.
after reading a chapter. K. Thomas Robbins, M.D.
• Identify "key points" i n the contemporary state of David E. Schuller, M.D.
knowledge, or lack of it, for a specific area. J. Regan Thomas, M.D.

xv
Table of Contents

Volume One 10 Head and Neck Manifestations of


GENERAL CONSIDERATIONS Human Immunodeficiency Virus
IN H E A D AND N E C K Infection 13
Steven D. Fletcher, Andrew N. Goldberg
Cummings/Ro b bins
11 Special Considerations in Managing Geriatric
1 History, Physical Examination, and the
Patients 14
Preoperative Evaluation 3
Matthew L . Kashima, W. Jarrard Goodwill,
Marion Everett Couch, James E. Blaugrund,
Jr., Thomas J. Balkany, Roy R. Casiano
Dario Kunar
12 Genetics and Otolaryngology 15
2 Overview of Diagnostic Imaging of the Head
William J. Kimberling
and Neck 4
Questions prepared by: lee Ching Anderson
Nafi Ay gun, Patrick J. Oliverio, S. James
Zinreich
13 Fundamentals of Molecular Biology and Gene
Questions prepared by: Matthew Whitley
Therapy 16
Bert W. O'Malley, Jr., Daqing L i , Hinrich
3 Biophysiology and Clinical Considerations
Staecker
in Radiotherapy 5
Jason K. Rockhill, George E. Laramore
14 Molecular Biology of Head and Neck
Cancer 17
4 Chemotherapy for Head and Neck Cancer 6
Patrick K. Ha, David Goldenberg, Matthew
Arlene A. Forastiere, Merrill S. Kies
Wolpoe, Joseph A. Califano III
Questions prepared by: Matthew Whitley
15 Outcomes Research 18
5 Skin Flap Physiology and Wound Healing 7
Bevan Yueh
George S. Goding, Jr., Patrick J. Byrne
16 Interpreting Medical Data 19
6 Free Tissue Transfer 8
Richard M. Rosen/eld
Douglas A. Girod, Terrance T. Tsue
17 Pain Management in the Head and Neck
7 Laser Surgery: Basic Principles and Safety
Patient 20
Considerations 9
Peter S. Staats
Robert H. Ossqff, C. Gaelyn Garrett,
Questions prepared by: Sara Pai
Lou Reinisch
18 Integrating Palliative and Curative Care
8 General Considerations of Anesthesia and
Strategies in the Practice of
Management of the Difficult Airway 10
Otolaryngology 21
Lynette J. Mark, Seth Akst, James Michelson
Michael A. Williams, Cynda Hylton Rushton
9 Allergy and Immunology of the Upper
Airway 12
Fuad M. Baroody, Robert M. Naclerio

xvii
xviii Contents

19 Graphics and Digital Imaging 32 Management of the Aging Periorbital Area 39


for Otolaryngologists 22 Oren Friedman, Tom D. Wang, Ted A. Cook
Lawrence R. Lustig, Nikolas H. Blevins Questions prepared by: Lisa Earnest
REVIEW QUESTIONS NOT REQUIRED FOR
THIS TOPIC 33 Suction-Assisted Lipocontouring 40
Edward H. Farrior, Stephen S. Park
20 Medical Informatics and Telemedicine 23
David Goldenberg, Marion Everett Couch 34 Mentoplasty and Facial Implants 41
Jonathan M. Sykes, Travis T. Tollefson, John
L . Frodel, Jr.
FACE Questions prepared by: Seth Cohen
Schuller/Thomas
35 Rehabilitation of Facial Paralysis 42
21 Aesthetic Facial Analysis 27 Roger L . Crumley, William B. Armstrong,
Marc S. Zimbler, Jongwook Ham Patrick J. Byrne

22 Recognition and Treatment of Skin 36 Otoplasty 43


Lesions 28 Peter A. Adamson, Suzanne K. Doud Galli
Ken K. Lee, Khosrow (Mark) Mehrany, Neil A.
Swanson
Questions prepared by: Seth Cohen
Volume Two
23 Management of Head and Neck NOSE
Melanoma 29 Schuller/Thomas
Cecelia E. Schmalbach, Timothy M. Johnson,
Carol R. Bradford 37 Physiology of Olfaction 47
Donald A. Leopold, Eric H. Holbrook
24 Scar Revision and Camouflage 30
J. Regan Thomas, Steven Ross Mobley 38 Evaluation of Nasal Breathing Function
with Objective Airway Testing 48
25 Facial Trauma: Soft-Tissue Lacerations Jacob W. Zeiders, John F. Pallanch, Thomas
and Burns 31 V. McCaffrey
Kevin A. Shumrick, Jon B. Chadwell
39 Manifestations of Systemic Diseases of the
26 Maxillofacial Trauma 32 Nose 49
Robert M. Kellman Thomas J. McDonald
Questions prepared by: Seth Cohen
40 Epistaxis 50
27 Reconstruction of Facial Defects 33 Evan J. Tobin, Douglas D. Massick
Shan R. Baker Questions prepared by: Seth Cohen
Questions prepared by: Matthew Whitley
41 Nasal Fractures 51
28 Hair Restoration: Medical and Surgical Burke E. Chegar, Sherard A. Tatum HI
Techniques 34
Benjamin M. Loos, Sheldon S. Kabaker 42 Allergic Rhinitis 52
Richard L . Mabry, Bradley F. Marple
29 Management of Aging Skin 36
Stephen W. Perkins, Timothy G. Gillum 43 Nonailergic Rhinitis 53
Stephanie Joe, Aaron Benson
30 Rhytidectomy 37 Questions prepared by: Seth Cohen
Shan R. Baker
44 The Nasal Septum 54
31 Management of the Aging Brow Russell W. H. Kridel, Paul E. Kelly, Allison
and Forehead 38 R. MacGregor
Paul S. Nassif, J. Regan Thomas
Contents xix

45 Rhinoplasty 55 58 Inflammatory Disorders of the Salivary


M. Eugene Tardy, Jr., J. Regan Thomas Glands 73
Agustin J. Arrieta, Thomas V. McCaffrey
46 Special Rhinoplasty Techniques 56
Richard T. Farrior, Edward H. Farrior, 59 Trauma of the Salivary Glands 74
Raymond D. Cook Jeffrey R. Holler
Questions prepared by: Seth Cohen
47 Revision Rliinoplasty 57
David W. Kim, Manuel A. Lopez, Dean 60 Benign Neoplasms of the Salivary Glands 75
M. Toriumi Ehab Y. Hanna, Stephen Lee, Chun Y. Fan,
James Y. Suen
48 Reconstructive Rhinoplasty 58 Questions prepared by: Joshua Schindler
C. Rose Rabinov, Roger L . Crumley
61 Malignant Neoplasms of the Salivary
Glands 76
PARANASAL SINUSES Alfred Simental, Ricardo L . Carrau
Schuller/Thomas

49 Radiology of the Nasal Cavity and Paranasal ORAL CAVITY/PHARYNX/ESOPHAGUS


Sinuses 61 Haughey
Patrick J. Oliverio, S. James Zinreich
Questions prepared by: Joshua S. Schindler 62 Physiology of the Oral Cavity 79
Joseph B. Travers, Susan P. Travers
50 Infectious Causes of Rhinosinusitis 62
Jonas T. Johnson, Berrylin J. Ferguson 63 Mechanisms of Normal and Abnormal
Swallowing 80
51 Neoplasms 63 JerilynA. Logemann
Ernest A. Weymuller, Jr., Thomas J. Gal
64 Oral Mucosal Lesions 81
52 Medical Management of Nasosinus Infectious James J. Sciubba
and Inflammatory Disease 64
Scott C. Manning 65 Oral Manifestations of Systemic Disease 82
Questions prepared by: Seth Cohen Jonathan A. Ship, Elisa M. Ghezzi

53 Primary Sinus Surgery 65 66 Odontogenesis and Odontogenic Cysts and


Kevin J. Hulett, James A. Stankiewicz Tumors 83
Peter E. Larsen
54 Revision Endoscopic Sinus Surgery 66
David W. Kennedy, James N. Palmer 67 Odontogenic Infections 84
Eric R. Carlson, J. W. Hudson
55 Cerebrospinal Fluid ( C S F ) Rhinorrhea 67
Martin J. Citardi 68 Temporomandibular Joint Disorders 85
Daniel M. Laskin

SALIVARY GLANDS 69 Benign Tumors and Tumor-Like Lesions


Haughey of the Oral Cavity 86
Timothy S. Lian
56 Physiology of the Salivary Glands 71 Questions prepared by: Joshua S. Schindler
Ravindhra G. Elluru, Manoj Kumar
70 Malignant Neoplasms of the Oral
57 Diagnostic Imaging and Fine-Needle Cavity 87
Aspiration of the Salivary Glands 72 Richard O. Wein, Randal S. Weber
Dale H. Rice
xx Contents

71 Reconstruction of the Mandible 84 Reconstruction of Hypopharynx and


and Maxilla 88 Esophagus 102
Mark L . Urken, Daniel Buchbinder, Eric M. Kristi E. Chang, Eric M. Genden, Gerry F. Funk
Genden Questions prepared by: Joshua S. Schindler
Questions prepared by: Joshua S. Schindler

72 Maxillofacial Prosthetics for Head and Neck


Defects 89 Volume Three
Jeffery C. Markt, Thomas J. Salinas, William LARYNX/TRAGHEA/BRONGHUS
Donald Gay
Flint
73 Benign and Malignant Tumors 85A Laryngeal and Pharyngeal Function 105
of the Nasopharynx 90 Gayle Ellen Woodson
Ranjiv Sivanandan, Willard E. Fee, Jr.
85B Evaluation and Management
74 Pharyngitis in Adults 91 of Hyperfunctional Disorders 106
Brian Nussenbaum, Carol R. Bradford Andrew Blitzer
Questions prepared by: Joshua S. Schindler
75 Sleep Apnea and Sleep-Disordered
Breathing 92 86 Visual Documentation of the Larynx 107
Robert J. Troell, David J. Terris Randall L . Plant, Robin A. Samlan

76 Oropharyngeal Malignancy 93 87 Voice Analysis 108


Holger G. Gassner, Alain N. Sabri, Robin A. Samlan
Kerry D. Olsen
Questions prepared by: lee Ching Anderson 88 Diagnostic Imaging of the Larynx 109
Franz J. Wippold II
77 Reconstruction of the Oropharynx 94
Bruce H. Haughey, S. Mark Taylor 89 Neurologic Evaluation of the Larynx
and the Pharynx 110
78 Diagnostic Imaging of the Pharynx Gayle Ellen Woodson, Andrew Blitzer
and Esophagus 95 Questions prepared by: Joshua S. Schindler
Barton F. Branstetter IV
90 Laryngeal and Tracheal Manifestations
79 Endoscopy of the Pharynx and Esophagus 96 of Systemic Disease 111
Ravindhra G. Elluru, J. Paul Willging Kim Richard Jones

80 The Esophagus: Anatomy, Physiology, and 91 Chronic Aspiration 112


Diseases 97 Steven D. Fletcher, David W. Eisele
Jason F. Vollweiler, Michael F. Vaezi
Questions prepared by: Lisa Earnest 92 Laryngeal and Esophageal Trauma 113
Steven D. Schaefer
81 Zenker's Diverticulum 98
Christopher Y. Chang, Richard L . Scher 93 Surgical Management of Upper Airway
Stenosis 114
82 Neoplasms of the Hypopharynx and Cervical David Goldenberg, Ramon M. Esclamado,
Esophagus 99 Paul W. Flint, Charles W. Cummings
Ravindra Uppaluri, John B. Sunwoo Questions prepared by: Joshua S. Schindler
Questions prepared by: Joshua S. Schindler
94 The Professional Voice 115
83 Radiotherapy and Chemotherapy of Gregory N. Postma, Mark S. Courey, Robert
Squamous Cell Carcinomas of the H. Ossoff
Hypopharynx and Esophagus 100
Jean-Louis Lefebvre, Antoine Adenis
Contents xxi

95 Benign Vocal Fold Mucosal Disorders 116 108 Diagnosis and Management of Tracheal
Robert W. Bastian Neoplasms 130
Questions prepared by: Joshua S. Schindler Christine L . Lau, G. Alexander Patterson

96 Medialization Thyroplasty 117 109 Upper Airway Manifestations of


Paul W. Flint, Charles W. Cummings Gastroesophageal Reflux Disease 131
Questions prepared by: Joshua S. Schindler Savita Collins
Questions prepared by: Anton Chen
97 Arytenoid Adduction 118
Gayle Ellen Woodson
Questions prepared by: Joshua S. Schindler NECK
Robbins
98 Laryngeal Reinnervation 119
George S. Goding, Jr. 110 Deep Neck Infection 135
Harrison G. Weed, L . Arick Forrest
99 Malignant Tumors of the Larynx Questions prepared by: Matthew Whitley
and Hypopharynx 120
George L . Adams, Robert H. Maisel 111 Blunt and Penetrating Trauma to the
Neck 136
100 Management of Early Glottic Cancer 121 Robert H. Maisel, David B. Horn
Henry T. Hoffman, Lucy H. Karnell, Timothy
M. McCulloch, John Buatti, Gerry F. Funk 112 Differential Diagnosis of Neck Masses 137
Questions prepared by: Anton Chen W. Frederick McGuirt, Sr.
Questions prepared by: Matthew Whitley
101 Transoral Laser Micro Resection of Advanced
Laryngeal Tumors 122 113 Primary Neoplasms of the Neck 138
Bruce W. Pearson, John R. Salassa, Michael L . Terry A. Day, John K. Joe
Hinni
114 Lymphomas Presenting in the Head
102 Conservation Laryngeal Surgery 123 and Neck 139
Ralph P. Tufano, Gregory S. Weinstein, Nancy Price Mendenhall, Ilona
Ollivier Laccourreye, Christopher H. Rassekh M. Schmalfuss, Matthew C. Hull
Questions prepared by: Matthew Whitley
103 Total Laryngectomy and
Laryngopharyngectomy 124 115 Radiation Therapy and Management
Christopher H. Rassekh, Bruce H. Haughey of the Cervical Lymph Nodes 140
Bernard J. Cummings, John Kim, Brian
104 Radiation Therapy for the Larynx O Sullivan
f

and Hypopharynx 125 Questions prepared by: Matthew Whitley


Parvesh Kumar
Questions prepared by: Anton Chen 116 Neck Dissection 141
K. Thomas Robbins, Sandeep Samant
105 Vocal Rehabilitation Following Total Questions prepared by: Matthew Whitley
Laryngectomy 126
Joshua S. Schindler 117 Surgical Complications of the Neck 142
Questions prepared by: Joshua S. Schindler Carol M. Bier-Laning

106 Management of the Impaired Airway


in the Adult 127
THYROID/PARATHYROID
David Goldenberg, Nasir I. Bhatti
Questions prepared by: Anton Chen Robbins

107 Endoscopy of the Tracheobronchial 118 Disorders of the Thyroid Gland 145
Tree 128 Phillip K. Pellitteri, Steven W. Ing
Rex C. Yung Questions prepared by: Justin Wittkopf
xxii Contents

119 Management of Thyroid Neoplasms 146 131 Pharmacologic Treatment of the Cochlea
Stephen Y Lai, Susan J. Mandel, Randal and Labyrinth 161
S. Weber Anil K. Lalwani, John F. McGuire

120 Surgical Management of Parathyroid


Disorders 147 INFECTIOUS PROCESSES
Phillip K. Pellitteri, Robert A. Sofferman, Harker
Gregory W. Randolph
132 Infections of the External E a r 165
121 Paranasal Sinuses: Management of Thyroid Michael J. Ruckenstein
Eye Disease (Graves' Ophthalmology) 148 Questions prepared by: Justin Wittkopf
Douglas A. Girod
Questions prepared by: Justin Wittkopf 133 Chronic Otitis Media, Mastoiditis,
and Petrositis 166
Richard A. Chole, Holger H. Sudhojf

Volume Four 134 Complications of Temporal Bone


GENERAL Infections 167
Harker Lee A. Harker, Clough Shelton
Questions prepared by: Justin Wittkopf
122 Anatomy of the Skull Base, Temporal
Bone, External Ear, and Middle 135 Infections of the Labyrinth 168
Ear 151 Larry E. Davis
Oswaldo Laercio M. Cruz
Questions prepared by: Justin Wittkopf 136 Tympanoplasty and Ossiculoplasty 169
Hussam K. El-Kashlan, Lee A. Harker
123 Neural Plasticity in Otology 152 Questions prepared by: Justin Wittkopf
Robert V. Harrison
137 Mastoidectomy 170
124 Tinnitus and Hyperacusis 154 Paul R. Lambert
Samuel G. Shiley, Robert L . Folmer, Questions prepared by: Matthew OMalley
Sean O. McMenomey

125 Management of Temporal Bone VESTIBULAR SYSTEM


Trauma 155 Harker
Hilary A. Brodie
Questions prepared by: Justin Wittkopf 138 Anatomy of Vestibular End-Organs
and Neural Pathways 173
126 Otologic Symptoms and Syndromes 156 Anna Lysakowski
Carol A. Bauer, Herman A. Jenkins
139 Principles of Applied Vestibular
127 Otologic Manifestations of Systemic Physiology 174
Disease 157 John P. Carey, Charles C. Delia Santina
Joseph B. Nadol, Jr., Saumil N. Merchant Questions prepared by: Matthew O'Malley

128 Noise-Induced Hearing Loss 158 140 Evaluation of the Patient with Dizziness 175
Brenda L . Lonsbury-Martin, Timothy E. Hullar, Lloyd B. Minor, David
Glen K. Martin S. Zee

129 Autoimmune Inner E a r Disease 159 141 Imbalance and Falls in the Elderly 176
Steven D. Rauch, Michael J. Ruckenstein Marian Girardi, Horst R. Konrad

130 Vestibular and Auditory Toxicity 160


Leonard P. Rybak
Contents xxiii

142 Meniere's Disease and Other Peripheral 155 Sensorineural Hearing Loss: Evaluation
Vestibular Disorders 177 and Management in Adults 195
David A. Schessel, Lloyd B. Minor, Julian M. H. Alexander Arts
Nedzelski
Questions prepared by: Matthew O'Malley 156 Otosclerosis 196
John W. House, Calhoun D. Cunningham III
143 Central Vestibular Disorders 178
Scott D. Z. Eggers, David S. Zee 157 Surgically-Implantable Hearing Aids 197
Lawrence R. Lustig, Charles C. Delia Santina
144 Surgery for Vestibular Disorders 179
Steven A. Telian
C O C H L E A R IMPLANTS
145 Vestibular and Balance Rehabilitation Harker
Therapy: Program Essentials 180
Neil T Shepard, Steven A. Telian 158 Patient Evaluation and Device Selection
for Cochlear Implantation 201
P. Ashley Wackym, Jill B. Firszt, Christina
FACIAL NERVE L. Runge-Samuelson
Harker Questions prepared by: Sara Pai

146 Tests of the Facial Nerve Function 183 159 Medical and Surgical Considerations
Robert A. Dobie in Cochlear Implantation 202
Thomas J. Balkany, Bruce J. Gantz
147 Clinical Disorders of the Facial Nerve 184
Douglas E. Mattox 160 Cochlear Implants: Results, Outcomes,
and Rehabilitation 203
148 Intratemporal Facial Nerve Surgery 185 John K. Niparko, Jennifer L . Mertes, Charles
Bruce J. Gantz, Jay T. Rubinstein, Ravi J. Limb
N. Samy

SKULL BASE
AUDITORY SYSTEM Harker
Harker
161 Diagnostic and Interventional
149 Cochlear Anatomy and Central Auditory Neuroradiology 207
Pathways 189 Richard E. Latchaw, Patricia Silva
Peter A. Santi, Patrizia Mancini
162 Temporal Bone Neoplasms and Lateral
150 Molecular Basis of Auditory Pathology 190 Cranial Base Surgery 208
JoAnn McGee, Edward J. Walsh Michael A. Marsh, Herman A. Jenkins
Questions prepared by: Matthew OMalley Questions prepared by: Joshua S. Schindler

151 Electrophysiologic Assessment of Hearing 191 163 Extra-Axial Neoplasms Involving the Anterior
Carolyn J. Brown and Middle Cranial Fossa 209
Timothy M. McCulloch, Russell Smith
152 Diagnostic and Rehabilitative Audiology 192 Questions prepared by: Jeffrey Cutler
Paul R. Kileny, Teresa A. Zwolan
Questions prepared by: Matthew O'Malley 164 Surgery of the Anterior and Middle Cranial
Base 210
153 Auditory Neuropathy 193 Daniel W Nuss, Bert W. O'Malley, Jr.
Robert J. Tibesar, Jon K. Shallop Questions prepared by: Jeffrey Cutler

154 Evaluation and Surgical Management 165 Extra-Axial Neoplasm of the Posterior
of Conductive Hearing Loss 194 Fossa 211
Douglas D. Backous, John K. Niparko Derald E. Brackmann, Moises A. Arriaga
xxiv Contents

166 Auditory Brainstem Implants 212 111 Velopharyngeal Dysfunction 227


Bruce J. Gantz, Ted A. Meyer Harlan R. Muntz, Helene M. Taylor, Marshall
E. Smith
167 Transnasal Endoscopic-Assisted Surgery Questions prepared by: Joshua S. Schindler
of the Skull Base 213
Aldo Cassol Stamm, Shirley S. N Pignatari 178 Congenital Malformations of the Nose 228
Questions prepared by: Lisa Earnest Karla Brown, Orval E. Brown
Questions prepared by: Joshua S. Schindler
168 Intraoperative Monitoring of Cranial Nerves
in Neurotologic Surgery 214 179 Pediatric Chronic Sinusitis 229
Charles D. Yingling, Yasmine A. Ashram Rodney P Lusk
Questions prepared by: Gabriela Sanchez
180 Salivary Gland Diseases 230
169 Radiation Therapy of the Cranial (Skull) David L . Walner, Charles M. Myer HI
Base 215
Nancy Y Lee, Edward J. Shin 181 Pharyngitis and Adenotonsillar
Questions prepared by: Gabriela Sanchez Disease 231
Brian J. Wiatrak, Audie L . Woolley
Questions prepared by: Joshua S. Schindler
PEDIATRIC OTOLARYNGOLOGY
Richardson 182 Obstructive Sleep Apnea in Children 232
Laura M. Sterni, David E. Tunkel
170 General Considerations 219
J. Scott McMurray, Mark A. Richardson 183 Pediatric Head and Neck Malignancies 233
Carol J. MacArthur, Richard J. H. Smith
111 Developmental Anatomy 220
Daniel O. Graney, Kathleen C. Y Sie 184 Differential Diagnosis of Neck Masses 234
Ralph F. Wetmore, William P. Potsic
172 Anesthesia 222
Jeffrey L . Koh, Veronica C. Swanson, Jeffrey 185 Congenital Disorders of the Larynx 235
Morray Anna H. Messner
Questions prepared by: Gabriela Sanchez
186 Managing the Stridulous Child 236
173 Characteristics of Normal and Abnormal David M. Albert, Susanna Leighton
Postnatal Craniofacial Growth and
Development 223 187 Glottic and Subglottic Stenosis 237
Frederick K Kozak, Juan Camilo Ospina George H. Zalzal, Robin T Cotton
Questions prepared by: Gabriela Sanchez
188 Gastroesophageal Reflux and Laryngeal
174 Hemangiomas and Vascular Malformations Disease 238
of the Head and Neck 224 Philippe Narcy, Philippe Contencin, Thierry
Reza Rahbar, Trevor J. I. McGill, John Van Den Abbeele
B. Mulliken
189 Aspiration and Swallowing Disorders 239
175 Craniofacial Surgery for Congenital Philippe Narcy, Philippe Contencin, Thierry
and Acquired Deformities 225 Van Den Abbeele
Jonathan Z. Baskin, Sherard A. Tatum III,
Questions prepared by: Gabriela Sanchez 190 Voice Disorders 240
Sukgi S. Choi, George H. Zalzal
176 Cleft Lip and Palate 226
Oren Friedman, Tom D. Wang, Henry 191 Congenital Disorders of the Trachea 241
A. Milczuk Reza Rahbar, Gerald B. Healy
Questions prepared by: Gabriela Sanchez
Contributors xxv

192 Tracheal Stenosis 242 199A Reconstruction Surgery of the Ear:


Mark A. Richardson, Greg R. Licameli Microtia Reconstruction 250
Questions prepared by: Gabriela Sanchez Craig S. Murakami, Vito C Quatela
Questions prepared by: Craig S. Murakami
193 Caustic Ingestion 243 and Gabriela Sanchez
J. Dale Browne, James N. Thompson
199B Reconstruction Surgery of the Ear: Auditory
194 Foreign Bodies of the Airway Canal and Tympanum 251
and Esophagus 245 Antonio De la Cruz, Marian R. Hansen
Roberto L . Barretto, Lauren D. Holinger
200 Acute Otitis Media and Otitis Media
195 Infections of the Airway 246 with Effusion 252
Newton O. Duncan III Andrew F. Inglis, Jr., George A. Gates
Questions prepared by: Gabriela Sanchez
201 Genetic Sensorineural Hearing Loss 253
196 Recurrent Respiratory Papillomatosis 247 Murad Husein, Richard J. H. Smith
Craig S. Derkay, Russell A. Faust
202 Pediatric Facial Fractures 254
197 Early Detection and Diagnosis of Infant Peter J. Koltai, Paul R. Krakovitz
Hearing Impairment 248
Susan J. Norton, Jonathan A. Perkins Answers 255
Questions prepared by: Gabriela Sanchez

198 Congenital Malformations of the Inner


Ear 249
Robert K. Jackler
Questions prepared by: Gabriela Sanchez
PART O N E

GENERAL CONSIDERATIONS
IN HEAD A N D NECK
CHAPTER ONE

HISTORY, PHYSICAL E X A M I N A T I O N ,
A N D T H E PREOPERATIVE E V A L U A T I O N

1. When a patient is found to have a Virchow node, 4. Which of the following statements regarding
it will be located hyperthyroidism is false?
a. I n the supraclavicular nodes of level 5a a. Hyperthyroidism may result in hypercalcemia,
b. I n the supraclavicular nodes of level 4 thrombocytopenia, and anemia.
c. I n the level 6 nodes b. Hyperthyroidism may be treated with
d. I n the level 5b supraclavicular nodes mithramycin.
e. I n the submental sublevel l a nodes c. Hyperthyroidism may be treated with
propylthiouracil.
2. Which of the following statements regarding d. Hyperthyroidism may be treated with Lugol's
Addison's disease is false? solution to inhibit iodide organification.
a. I t results i n both glucocorticoid and mineralo- e. Propanolol treats sympathetic hyperactivity
corticoid deficiencies. and decreases T4- to T3-conversion.
b. Hyperpigmentation is caused by overproduc-
tion of adrenocorticotropic hormone and 5. Which of the following is not a risk factor for
P-lipotropin. perioperative cardiovascular complications?
c. Glucocorticoid replacement is required twice a. Recent myocardial infarction
a day. b. Older than 50 years of age
d. Mineralocorticoid therapy is given once a day. c. Third heart sound
e. Stress-dose steroids are not necessary periop- d. Nonsinus rhythm
eratively. e. Valvular aortic stenosis

3. Latex allergies
a. Are seen i n only a small percentage of health-
care workers
b. May lead to anaphylaxis i n the operating
department
c. Require only premedication to adequately
prepare for an operation
d. Do not result i n a positive skin test to latex
proteins
e. Are not true allergies, but rather a side effect
of wearing gloves

3
CHAPTER TWO

O V E R V I E W OF D I A G N O S T I C I M A G I N G
OF T H E H E A D A N D N E C K

1. Which of the following is true of the Caldwell 4. Which of the following are true regarding imag-
view of conventional radiography? ing of the parotid gland?
a. I t provides an excellent view of anterior eth- a. Lesions i n the parotid are better defined on
moid cells. computed tomography (CT) than on MRI.
b. I t is obtained i n the posteroanterior projection b. Ductal anatomy is best delineated by sialo-
with 35 degrees of caudal angulation of the graphy.
x-ray beam. c. Pleomorphic adenoma is hyperintense on
c. I t is not suited for evaluation of the frontal Tl-weighted images and hypointense on
sinuses. T2-weighted images.
d. I t provides an excellent view of maxillary d. Chronic sialadenitis is brighter on
sinuses. Tl-weighted images than on T2-weighted
e. I t provides an excellent view of the posterior images.
ethmoid cells. e. Facial nerve anatomy is best assessed with
ultrasonography.
2. Which of the following is not true of chemical
shift artifact of magnetic resonance imaging 5. Which of the following radiographic properties of
(MRI)? a lymph node on contrast-enhanced CT does not
a. I t occurs i n areas where fat abuts structures support identification of cervical adenopathy as
containing predominantly water. inflammatory (reactive)?
b. I t may produce the appearance of a pseudo- a. Less than 10 m m
capsule around a lesion. b. Well-defined margins
c. I t is seen as a bright band on one side of a c. Central hilar or mild homogeneous enhance-
structure and a black band on the opposite ment
side. d. Central low intensity
d. I t is most apparent on T2-weighted images. e. Calcification
e. I t may cause obscuration of a small-diameter
structure.

3. Which of the following is true of spin (proton)


density images?
a. They use a short repetition time and a long
echo time.
b. They show air and bone as high signal
intensity.
c. They show fluid-containing structures and
muscles as high signal intensity.
d. A solid mass or fluid-filled lesion with a high
protein content demonstrates low signal
intensity.
e. Paranasal sinus inflammation appears very
bright.

4
CHAPTER THREE

BIOPHYSIOLOGY A N D C L I N I C A L
CONSIDERATIONS I N RADIOTHERAPY

1. Which of the following statements is true regard- c. Rapidly proliferating tumors can potentially
ing the high-energy x-rays used i n radiation replace a significant portion of cells killed with
therapy? each dose of radiation.
a. The x-rays, being highly energetic, have a long d. The acute and late effects seen for both accel-
wavelength compared with cellular dimen- erated and continuous hyperaccelerated radio-
sions. therapy are more intense than those seen for
b. The initial interaction of the x-rays with mat- standard fractionation.
ter typically produces a high-energy electron,
which i n turn causes multiple ionizations. 4. Intensity-modulated radiation therapy (IMRT) is
c. The biologic effect of the x-rays is due to heat- rapidly becoming more common i n treating head
ing caused by inducing molecular rotations. and neck cancers. Which of the following state-
d. X-rays have a shorter penetration distance ments is false?
than high-energy electrons, which are also a. Trials to date with IMRT have a significantly
used i n therapy. lower number of treatments than with stan-
e. Bony structures show up better on verification dard radiotherapy.
therapy films than on standard diagnostic b. IMRT treatment plans are designed to have
x-ray films. very steep gradients between isodoses.
c. IMRT can be more conformal than traditional
2. Compared with standard photon and electron three-dimensional radiotherapy.
radiation, high linear energy transfer (LET) radi- d. Initial results suggest that IMRT for nasopha-
ation has the following property: ryngeal carcinoma actually may improve over-
a. The oxygen enhancement ratio is higher, mak- all survival and not just local control.
ing i t more useful i n treating hypoxic tumors.
b. The relative biologic effectiveness factor is 5. Combined-modality therapy with both
lower. chemotherapy and radiotherapy is becoming
c. LET causes double-stranded breaks i n the DNA common for head and neck cancers. Which
of the cells, which are less readily repaired. statement best summarizes the rationale for the
d. LET is more effective i n treating squamous use of combined modality therapy?
cell tumors of the head and neck. a. Chemotherapy, when given concurrently
e. Cell survival curves typically have a larger w i t h radiotherapy, can sensitize both normal
shoulder for high LET irradiation than for low and tumor cells to the effects of radiother-
LET irradiation. apy.
b. Certain agents, such as amifostine, may be
3. There are several different ways of fractionating able to increase the therapeutic ratio by pro-
radiotherapy for head and neck cancers. Which tecting normal tissues.
of the following statements is true? c. Chemotherapy may be effective at addressing
a. Radiotherapy is fractionated to allow tumor micrometastases outside the radiation field.
cells time to repair their DNA and thus move d. I n general, acute affects are more intense with
into a radiosensitive portion of the cell cycle. combined modality therapy than i n single
b. Patients treated with hyperfractionated radio- modality therapy.
therapy finish their treatment quicker than e. a, b, and c
those treated with standard fractionation. f. A l l of the above

5
CHAPTER FOUR

C H E M O T H E R A P Y FOR H E A D
A N D NECK CANCER

1. The dose-limiting toxicity of carboplatin is which 4. Which of the following is a desirable property of
of the following? an intraarterial chemotherapy agent?
a. Hepatotoxicity a. The drug should be activated i n the systemic
b. Myelosuppression circulation.
c. Ototoxicity b. The drug should have a high tissue extraction.
d. Peripheral neuropathy c. The drug should require activation i n the liver.
e. Gastritis d. The drug should not be cleared by the kidney.
e. The drug should not include platinum com-
2. Which of the following is true of induction pounds.
chemotherapy?
a. Patients who are resistant to cisplatin-based 5. Which of the following is not a common toxicity
induction chemotherapy have a high likeli- associated with cisplatin therapy?
hood of not responding to radiotherapy. a. Nausea
b. Treatment increases morbidity of surgery. b. Vomiting
c. Survival is not prolonged i n responders vs c. Renal dysfunction
nonresponders. d. Ototoxicity
d. No affect on organ preservation or quality of e. Severe neutropenia
life has been proven.
e. Distant metastases are slightly more likely.

3. Which of the following is true of concurrent


chemotherapy and radiation therapy?
a. I t is used primarily before surgery i n patients
with larger tumor burden.
b. I t increases incidence i n acute radiation-
induced toxicity (primarily mucosal toxicity).
c. Concurrent bleomycin and radiotherapy is
used for advanced nasopharyngeal carcinoma.
d. Concurrent chemotherapy and radiation ther-
apy is useful primarily for pain relief with pal-
liation as a goal.

6
C H A P T E R FIVE

S K I N FLAP P H Y S I O L O G Y
A N D W O U N D HEALING

1. The failure of a microvascular free flap to survive 4. True/False. After skin flap elevation, lymphatic
after a prolonged ischemia time despite patent obstruction occurs and affects the skin perfusion
anastomoses may be considered a failure of and survival.
a. Zone I
b. Zone I I 5. Neovascularization of transferred flaps occurs i n
c. Zone I I I a. 7 days
d. Zone IV b. 2 days
e. Zone I I or I I I c. 3 weeks
d. 3 months
2. True/False. The paramedian forehead flap is an
example of a random pattern flap.

3. The survival of random pattern flaps is most


directly affected by
a. The period of ischemia after the flap is raised
b. The length/width ratio of the skin flap
c. The perfusion pressure gradient
d. The thickness of the raised flap
e. The smoking status of the patient.

7
J*

1. The first free tissue transfer for oral cavity 4. Donor site selection can be influenced by
reconstruction was reported i n what decade? a. Prior surgery or trauma
a. The early 1960s b. Handedness or footedness
b. The early 1970s c. Occupation
c. The early 1980s d. Hobbies
d. The early 1990s e. A l l of the above

2. The advantage of free tissue transfer over other 5. What is the most reliable method for postopera-
techniques for head and neck reconstruction tive free tissue transfer monitoring?
include a. Visible flap inspection and pinprick
a. Versatility of available tissues b. Doppler monitoring of the flap pedicle
b. Multiple donor site options c. Color flow Doppler monitoring
c. Donor site outside the field of radiation d. Laser Doppler veloeimetry
therapy e. Oxygen tension measurements
d. Single-stage reconstruction even for very large
defects
e. A l l of the above

3. Which soft-tissue donor site has proven to be the


most versatile, reliable, and commonly used?
a. The ulnar forearm flap
b. The radial forearm flap
c. The lateral arm flap
d. The rectus flap

8
CHAPTER EIGHT

DIFFICULT A I R W A Y / I N T U B A T I O N :
I M P L I C A T I O N S FOR ANESTHESIA

1. The postanesthesia care unit nurse calls you to 4. Your patient is an elderly man with hypertension
evaluate your patient for disorientation after a who has a 60 pack/year history of cigarette
uvulopalatopharyngoplasty. On your arrival, the smoking. He is being seen for evaluation of a
patient is snoring heavily, with 9 1 % oxygen satu- vocal cord polyp. Which medication has the
ration (Sa0 ), on 40% oxygen by face mask (FM).
2 potential to cause tachycardia and hypertension
Appropriate immediate management is in this patient?
a. Administer naloxone a. Cocaine
b. Administer 100% S a 0 via FM Ambu Bag; per-
2 b. Etomidate
form chin lift and jaw thrust to alleviate the c. Lidocaine
airway obstruction d. Metoprolol
c. Perform flexible nasopharyngoscopy to evalu-
ate for edema i n the posterior pharynx 5. I n which of the following patients is a rapid-
d. Perform immediate cricothyrotomy to secure sequence induction/intubation not advisable?
an airway i n the presence of mental status a. Diabetic patient with gastroparesis
changes b. Patient with obstructive sleep apnea and
Mallampati class IV airway
2. You are asked to assist with the awake fiberoptic c. Trauma patient i n whom the time of last oral
intubation of a patient with angioedema and res- intake is unknown
piratory distress. The patient is agitated during d. Vomiting patient with a small bowel obstruc-
the process. To note, the patient has a heart rate tion
of 85 beats/min, blood pressure of 150/90 m m
Hg, and Sa0 of 98%. Which of the following
2 6. The American Society of Anesthesiologist's
medications would you consider giving as part of Difficult Airway Algorithm reminds the practi-
this awake intubation? tioner to consider all of the following choices
a. Labetalol except
b. Midazolam a. Awake vs asleep intubation
c. Propofol b. Macintosh vs Miller blade for laryngoscopy
d. Succinylcholine c. Paralyzed vs spontaneously breathing tech-
niques
3. When a local anesthetic is used for a glossopha- d. Surgical vs nonsurgical airway management
ryngeal nerve block, the potential complication
of primary concern is 7. The laryngeal mask airway (LMA) differs from an
a. Edema from subcutaneous infiltration endotracheal tube i n which important respect?
b. Injection into a vein leading to cardiac a. The LMA cannot be used for positive-pressure
arrhythmias ventilation.
c. Toxic absorption of local anesthetic b. The LMA does not protect the patient's airway
d. Seizure caused by arterial injection from aspiration of gastric contents.
c. The LMA is difficult to place i n patients who
are difficult to intubate.
d. The LMA requires more time to place than an
endotracheal tube.

10
CHAPTER SEVEN

LASER SURGERY: BASIC PRINCIPLES


A N D SAFETY C O N S I D E R A T I O N S

1. Which of the following lasers has the highest risk 4. The most effective method of reducing collateral
of injury to the pulmonary vessels during laser thermal damage to tissue surrounding the abla-
bronchoscopy? tion crater of a C 0 laser on laryngeal mucosa
2

a. Carbon dioxide ( C 0 ) laser


2 is to
b. Potassium-titanyl-phosphate (KTP) laser a. Cool the tissue before laser application
c. Neodymium:yttrium-aluminum-garnet b. Reduce the spot size of the laser beam
(Nd:YAG) laser c. Use a short-pulse laser setting
d. Holmium:yttrium-aluminum-garnet (Ho:YAG) d. Reduce the power setting
laser e. Use continuous mode
e. Diode laser
5. Which of the following vocal fold lesions is most
2. The most commonly used lasers i n otolaryngol- appropriate for C 0 laser excision?
2

ogy ( C 0 , KTP, Nd:YAG) exert their biologic


2 a. Polyp
effect through which of the following laser-tissue b. Nodule
interactions? c. Intracordal cyst
a. Transmission d. Sulcus vocalis
b. Scatter e. Papilloma
c. Reflection
d. Absorption
e. Photochemical effect

3. Eye protection for both the patient and operating


department personnel is essential for proper
laser safety. The most important laser parameter
in determining ocular structures at risk during
surgery is
a. Laser wavelength
b. Pulse structure
c. Pulse duration
d. Power setting
e. Delivery system

9
Chapter 8 Difficult Airway/Intubation: Implications for Anesthesia 11

All of the following reduce the risk of airway fire 9. Which of the following is not part of the treat-
except ment of malignant hyperthermia?
a. Avoiding the use of nitrous oxide a. Dantrolene
b. Minimizing the inspired oxygen concentration b. Discontinuation of the volatile anesthetic
c. Using bipolar rather than unipolar cautery c. Succinylcholine
within the airway d. Symptomatic cooling
d. Using special endotracheal tubes during laser
surgery
CHAPTER NINE

ALLERGY A N D I M M U N O L O G Y OF T H E
UPPER A I R W A Y

1. The major histocompatibility complex, which 4. Which of the following immunoglobulins (Ig) is
codes for molecules that allow the immune sys- most important i n memory immune responses?
tem to distinguish between self and nonself, is a. IgG
located on b. IgM
a. Chromosome 5 c. IgA
b. Chromosome 13 d. IgE
c. Chromosome 6 e. IgD
d. Chromosome 10
e. Chromosome 21 5. Eosinophils produce all of the following except
a. Peroxidases
2. Which of the following cells are important i n b. Neurotoxins
antigen presentation (antigen-presenting cells)? c. Proteins
a. Monocytes d. Cytokines
b. Macrophages e. Histamine
c. Dendritic cells
d. B cells
e. Langerhans' cells
f. A l l of the above

3. Which of the following cytokine(s) is secreted by


TH2 CD4+ cells?
a. Interleukin (IL)-2
b. IL-4
c. IL-5
d. Interferon (IFN)-a
e. IL-13

12
CHAPTER TEN

HEAD A N D NECK MANIFESTATIONS


OF H U M A N I M M U N O D E F I C I E N C Y
VIRUS I N F E C T I O N

1. Which of the following statements regarding HIV 4. Which of the following statements regarding HIV
replication is false? infection and sinusitis is false?
a. The reverse transcriptase enzyme is a critical a. HIV-positive patients and the general popula-
enzyme for viral replication that is targeted by tion report similar rates of sinonasal symp-
an tire tro viral medications. toms.
b. HIV typically infects and replicates i n every b. Pseudomonas and fungal infections may rap-
cell i n the body. idly progress to life-threatening infections and
c. Viral proteases are critical for viral replication should be treated aggressively.
and are targeted by an tire tro viral medications. c. I n HIV-positive patients with sinusitis, sphe-
d. A combination of error-prone transcription noid involvement is seen at nearly double the
and prolific replication results in vast genetic rate of that i n the general population.
diversity. d. Surgical intervention is reserved for complica-
tions of sinusitis or life-threatening infection.
2. Which of the following statements regarding cer-
vical adenopathy i n HIV-positive patients is 5. Which of the following statements regarding
true? occupational exposure and HIV infection is true?
a. Open biopsy should be performed i n all HIV- a. Postexposure prophylaxis is universally effec-
positive patients w i t h cervical adenopathy. tive.
b. Fine-needle aspiration is unreliable i n the set- b. Most surgeons follow universal precautions.
ting of H I V c. Inexperienced surgeons are most likely to
c. Idiopathic follicular hyperplasia is the most have injuries from sharp instruments.
common cause of cervical adenopathy i n HIV- d. The rate of seroconversion after a needles tick
positive patients. from an HIV-positive patient is greater
d. I n HIV-positive patients, cervical adenopathy than 1%.
most commonly occurs i n the anterior
triangle.

3. Which of the following malignancies are most


highly associated with HIV-positive patients?
a. Non-Hodgkin's lymphoma
b. Kaposi's sarcoma
c. Hodgkin's lymphoma
d. Squamous cell carcinoma of the head and
neck

13
C H A P T E R ELEVEN

SPECIAL C O N S I D E R A T I O N S I N
M A N A G I N G G E R I A T R I C PATIENTS

1. The most common manifestation of vestibular 4. Decrease i n fundamental frequency of voice i n


dysfunction i n the elderly is aging women most often is associated with
a. Loss of balance a. Edema caused by general endocrine changes
b. Oscillopsia b. History of previous intubation
c. Benign paroxysmal positional vertigo c. Benign essential tremor
d. Meniere's disease d. Cricoarytenoid joint fixation
e. Labyrinthine fistula e. Increase i n tissue hyaluronic acid content

2. The most significant complication of presbyasta- 5. Factors associated with presbydysphonia include
sis is all of the following except
a. Automobile accidents a. Diminished lung vital capacity
b. Hip fractures b. Use of antidepressants
c. Depression c. Prior intubation
d. Impaired activities of daily living d. Decreased salivary secretory rate
e. Benign paroxysmal positional vertigo e. Laryngeal elevation caused by calcification

3. Changes i n nasal physiology associated with


aging include
a. Reduction i n nasal cilia
b. Increase i n mucociliary clearance
c. Decrease i n olfactory epithelium
d. Decrease i n nasal airflow resistance
e. General decrease i n autonomic function

14
CHAPTER TWELVE

GENETICS A N D O T O L A R Y N G O L O G Y
m mm

1. Which of the following statements is true? 4. Which of the following statements regarding
a. Introns are noncoding parts of the gene that genetic expressivity is true?
are excised before transcription. a. X-linked genes tend to exhibit more variable
b. Genes are transcribed from the 3' to the 5' expressivity i n females than i n males.
end of DNA. b. I n males, variability i n gene expressivity is
c. Regulatory elements within the gene act p r i - partly due to a phenomenon known as Lyon's
marily to signal when translation into protein hypothesis.
begins and ends. c. The expressivity of recessive disorders is usu-
d. The wobble nucleotide refers to the third ally more variable than that of dominant dis-
nucleotide i n a codon, which can vary for orders.
most amino acids. d. A gene that is not penetrant can still have
e. The genetic distance between two genes is variable degrees of expressivity.
always directly proportional to its physical dis- e. Variability i n gene expression cannot be
tance. affected by other genes.

2. Which of the following genetic diseases is not 5. Which of the following regarding DNA testing is
correctly classified? false?
a. Turner's syndrome: aneuploidy a. DNA chips can be used to detect single-base
b. Treacher-Gollins syndrome: autosomal reces- differences i n DNA.
sive b. Polymerase chain reaction amplifies target
c. Branchiootorenal syndrome: autosomal domi- DNA sequences for further analysis.
nant c. A l l modern molecular genetic testing begins
d. Mohr-Tranebjaerg's syndrome: X-linked reces- with Southern hybridization of genomic DNA.
sive d. Heteroduplex testing is one method of screen-
e. Kearns-Sayre's syndrome: mitochondrial dis- ing DNA for mutations.
order e. Novel mutations cannot be detected by DNA
chips.
3. I n autosomal-recessive disorders
a. An affected man cannot transmit the gene to
his son.
b. When normal parents have an affected child,
the chance of the disorder affecting any other
children they have is 50%.
c. The abnormal gene is found i n higher fre-
quency than would be expected considering
the relative rarity of the disorder.
d. The mechanism of haploinsufficiency is
important i n influencing phenotype.
e. Twice as many females as males are affected.

15
CHAPTER THIRTEEN

F U N D A M E N T A L S OF M O L E C U L A R
BIOLOGY A N D GENE THERAPY

1. Which of the following vectors are not useful for 4. Gene therapy vectors can be used to
gene therapy? a. Deliver therapeutic genes
a. Adenovirus b. Decrease expression of targeted genes
b. Plasmids c. Change a cell's phenotype
c. Herpesvirus vectors d. A l l of the above
d. Coronavirus vectors e. None of the above

2. Gene therapy may be useful for the following 5. Which cell type should be excluded from stan-
applications except dard gene delivery protocols?
a. Treatment of head and neck cancer a. Neurons
b. Prevention of spiral ganglion degeneration b. Muscle
c. Treatment of obstructive sleep apnea c. Germ cells
d. Treatment of anosmia d. Lymphoid tissue

3. The three principal genetic components of retro-


viruses include all except
a. env
b. gag
c. lat
d. pol

16
CHAPTER FOURTEEN

M O L E C U L A R B I O L O G Y OF H E A D
A N D NECK CANCER

1. Head and neck malignancy arises as an alter- 4. Which of the following techniques is not used to
ation i n detect molecular genetic sequence alterations in
a. RNA head and neck cancer?
b. DNA a. Comparative genomic hybridization
c. Protein b. Microsatellite analysis
d. mRNA c. Promoter hypermethylation
e. tRNA d. Fluorescence i n situ hybridization
e. DNA sequencing
2. Malignancy can develop as a single genetic event
or as an accumulation of genetic alterations. 5. Theoretical advantages of immunotherapy and
What evidence supports the notion that head molecular-directed therapy include
and neck cancer occurs as a result of multiple a. Targeted therapy for tumor cells
genetic alterations? b. Decreased toxicity profile
a. A 20- to 25-year latency period c. The ability to combine with traditional surgi-
b. History of toxin exposure cal and medical therapies
c. Genetic alterations i n premalignant lesions d. Systemic effects of therapy
d. Statistical modeling e. A l l of the above
e. A l l of the above

3. The first study demonstrating the efficacy of


chemopreventive agents used which compound?
a. Vitamin D
b. Vitamin C
c. Vitamin B1 2

d. Vitamin A
e. Cyclooxygenase-2 inhibitors

17
CHAPTER FIFTEEN

O U T C O M E S RESEARCH

1. After an extensive review of the literature, an 4. With respect to appropriate measurement of out-
otolaryngologist finds that the best supporting comes i n a double-blind, randomized clinical
evidence for a new procedure is a case series of trial of hearing aids, which of the following state-
13 patients. This is an example of ment is most accurate?
a. Grade A, level 1 evidence a. Quality-of-life (QOL) scales are too soft for
b. Grade B, level 2 or 3 evidence controlled studies.
c. Grade G, level 4 evidence b. Physiologic measures such as pure tone
d. Grade D, level 5 evidence thresholds are the most reliable.
c. A generic outcome measure (e.g., the Medical
2. I n an observational cohort study of treatment of Outcomes Study Short Form-36 [SF-36]) is
sinusitis, patients are treated with either antibi- the most sensitive.
otics or surgery. Failure to recognize that the d. A disease-specific QOL scale facilitates com-
patients who receive surgical intervention gener- parisons w i t h other types of treatment.
ally have more severe symptoms represents a e. None of the above.
problem with
a. Selection bias 5. A n otolaryngologist wishes to determine whether
b. Intervention bias a history of childhood tonsillectomy affects rates
c. Comorbidity of adult atopic disease. He assembles a group of
d. Detection bias adults with and without atopic disease and then
compares the rates of tonsillectomy by reviewing
3. A double-blind, randomized clinical trial for their records. This is an example of a
allergic rhinitis demonstrates that rhinorrhea is a. Prospective observational study
reduced more often w i t h nasal steroids than with b. Retrospective observational study
antihistamines. Nasal steroids are c. Case-control study
a. More efficacious d. Case series study
b. More efficient e. Poorly designed study
c. More effervescent
d. More effective

18
C H A P T E R SIXTEEN

I N T E R P R E T I N G M E D I C A L DATA

1. A clinician reviews the medical records of all 4. Evidence-based medicine is defined as the j u d i -
patients who had tonsillectomy over the past 10 cious, explicit, and systematic use of current
years and records the frequency of primary hem- best evidence i n caring for individual patients.
orrhage. Follow-up is available for all subjects. Which statement is true concerning levels of evi-
The hemorrhage rate is reported using dence and corresponding grades of recommenda-
a. Prevalence, because the method of data collec- tion?
tion is retrospective a. Expert consensus can support only a grade G
b. Incidence, because the method of data collec- or D recommendation.
tion is prospective b. Expert consensus is unacceptable as the sole
c. Prevalence, because the direction of inquiry is criterion for a recommendation.
retrospective c. Grade A recommendations are required to jus-
d. Incidence, because the direction of inquiry is tify surgical therapy.
prospective d. Grade A or B recommendations are required
e. Survival analysis, because some observations to justify medical therapy.
are censored e. Levels of evidence differ for studies of therapy,
diagnosis, or prognosis.
2. When the change i n hearing levels is assessed
after ossicular reconstruction, a 95% confidence 5. The most important aspect of analyzing and
interval aids i n data interpretation because i t interpreting medical data is
a. Gives the range of results consistent with the a. Choosing the right statistical test for the right
data dataset
b. Estimates the accuracy of observed results b. Recognizing uncertainty and quantifying error
c. Adjusts for systematic error that may have rates
occurred when results were measured c. Avoiding the multiple P value problem by
d. Defines the variability of observed data rela- using multivariate techniques
tive to the mean d. Reporting statistical power whenever a signifi-
e. Ensures adequate statistical power cant P value is obtained
e. Reporting statistical power whenever a non-
3. A researcher reports that a new antibiotic is significant P value is obtained
more effective than an established standard for
bacterial sinusitis, P = .015. Which of the follow-
ing statements is true?
a. The new antibiotic increased relative efficacy
by 15%.
b. The new antibiotic increased absolute efficacy
by 15%.
c. The likelihood of a type I statistical error
(false positive) is 1.5%.
d. The likelihood of a type I I statistical error
(false negative) is 1.5%.
e. The likelihood of adequate statistical power is
98.5%.

19
CHAPTER SEVENTEEN

PAIN M A N A G E M E N T I N THE HEAD


A N D N E C K PATIENT

1. Which of these medical therapies is not effective 4. Which treatment is effective i n treating pain
in treating neuropathic pain? associated with whiplash?
a. Oxycodone a. Corticosteroid injections into the cervical
b. Ibuprofen zygapophyseal joint
c. Garbamazepine. b. Percutaneous radiofrequency neurotomy
d. Lidocaine c. Single sessions of extension-retraction exer-
e. Amitriptyline cises
d. Botulinum toxin A injections into certain trig-
2. Which specific nerve block can lead to the great- ger points
est morbidity after a misplaced injection?
a. Sphenopalatine ganglion 5. Which of the following statements is not true?
b. Maxillary nerve a. Intractable chronic cluster headaches can
c. Mandibular nerve resolve with nerve blockade of the sphenopala-
d. Glossopharyngeal nerve tine ganglion.
e. Stellate ganglion b. Overactive pericranial muscles may result i n a
constant bandlike pain i n the forehead.
3. Which of the following statements regarding c. Regular analgesic use by patients with a his-
migraines is not true? tory of migraine will likely decrease the devel-
a. Migraine is an undertreated or inadequately opment of chronic daily headaches.
treated syndrome, because patients generally d. Ocular and frontotemporal pain provoked by
rely on over-the-counter medications. certain neck movements or pressure i n the
b. Migraines may proceed through four phases: upper back are characteristics of paroxysmal
prodromal phase, aural phase, headache hemicrania.
phase, and resolution phase. e. Facet joint syndrome can be diagnosed by the
c. Migraines are the result of a cervical-trigemi- patient's response to a nerve block into the
nal-vascular disorder. zygapophyseal joints.
d. Migraine treatment can be prophylactic,
abortive, or acute.
e. Migraines are associated with lower levels of
serotonin centrally.

20
CHAPTER EIGHTEEN

I N T E G R A T I N G PALLIATIVE A N D
C U R A T I V E CARE STRATEGIES I N T H E
PRACTICE OF O T O L A R Y N G O L O G Y

Palliative care is a comprehensive approach to 5. Which of the following actions is least consistent
treating serious illness that focuses on patients' with effective negotiation strategies to persuade
a. Physical needs the patient to reconsider?
b. Psychological needs a. Promising the patient that the treatment will
c. Social needs cure the cancer
d. Spiritual needs b. Offering to respect the patient's choice
e. A l l of the above c. Allowing the patient time to voice concerns
d. Setting up another meeting at another time to
True/False. Palliative care and end-of-life care are talk again
synonymous. e. Assessing whether the patient has understood
enough about the condition and treatment
True/False. Palliative care and curative therapies options to make an informed refusal
are incompatible, and palliative care should be
pursued only when i t is clear that curative
strategies are futile.

A patient with advanced but treatable cancer


wants to refuse treatment—a choice that seems
irrational based on the current literature. Which
of the following is a potential reason for the
patient's refusal?
a. Fear of disfigurement
b. Fear of pain
c. Fear of death
d. Fear of isolation
e. A l l of the above

21
CHAPTER NINETEEN

GPAPHICS A N D DIGITAL I M A G I N G
FOR OTOLARYNGOLOGISTS

NO REVIEW QUESTIONS NECESSARY FOR THIS TOPIC

22
CHAPTER TWENTY

MEDICAL INFORMATICS
A N D TELEMEDICINE

1. Which of the following statements about litera- 4. Which of the following statements is true regard-
ture searches is false? ing telemedicine?
a. The quality of the literature search can be a. Telemedicine is already integrated into many
assessed by search precision and recall. facets of the practice of medicine.
b. You should take full advantage of the provided b. Telemedicine is known to be detrimental to
training materials to obtain the best possible doctor-patient interaction.
search results. c. I n a store-and-forward method, the referring
c. Thorough literature searches should be per- physician collects all relevant information and
formed only by a trained medical librarian. forwards i t to the remote specialist, which
d. Your topic should be systematically defined causes scheduling problems for programs that
and a specific aspect chosen, especially if the cover different time zones.
topic is broad. d. Telemedicine will be integrated into the prac-
e. You should take the time to become familiar tice of medicine only i n 5 to 10 years because
with the Medical Subject Headings terms of immature technology.
(MeSH). e. Telemedicine depends on live video teleconfer-
encing.
2. A l l of the following statements about information
technology systems are true except 5. Which statement about the electronic patient
a. Information technology systems are readily record (EPR) is false?
used at the institutional level by local area a. EPR is i n a structured format that allows
networks but have less value for individuals greater accessibility and understanding of
and society. health conditions and medications of one's
b. They are facilitated by physician-oriented web patients.
portals. b. A n EPR is simply an electronic version of the
c. They are facilitated by development of com- paper medical record that stores patient infor-
mon technology standards. mation i n a more efficient way.
d. They may lead to a reduction i n errors, poten- c. A n EPR adds information management tools to
tially saving patients' lives. provide reminders and alerts, knowledge
source linkages, specialized medical decision
3. I n describing patient use of the Internet, all of support, and analysis of aggregated data.
the following are true except d. Integrated access to all patients' data by legiti-
a. The Internet allows patients to get a virtual mate users is one of the primary purposes of
second opinion i n some cases. the EPR.
b. The Internet usurps the physician position as
the most authoritative source of medical infor-
mation.
c. The Internet allows patients to gather informa-
tion before visiting their physician.
d. The age of information has empowered the
patient, as well as the doctor.

23
PART T W O

FACE
CHAPTER TWENTY ONE

A E S T H E T I C FACIAL ANALYSIS

What are the anatomic landmarks that define the 4. The following descriptions of the nasolabial
Frankfort horizontal line? angles are correct except
a. Inferior wall of the external auditory canal to a. 95 to 110 degrees i n women
infraorbital r i m b. 90 to 95 degrees i n men
b. Root of the superior helical r i m to the infra- c. A measure of the nasal projection i n relation
orbital r i m to the upper lip
c. Superior tragal root to nasojugal fold d. A measure of nasal tip rotation
d. Superior wall of the external auditory canal to
infraorbital r i m 5. A l l of the following regarding the analysis of the
ears are correct except
When the face is analyzed, the width of one eye a. The ear protrudes from the skull at an angle
may be used as a basic unit of measurement that approximately 20 to 30 degrees.
is equal to the following except b. The helix of the ear lies 15 to 25 m m lateral to
a. Intercanthal distance the mastoid skin.
b. Alar base width c. The long axis of the ear is parallel to the long
c. One-fifth of the facial width axis of the nasal dorsum and is noted to have
d. One-half the nasal length a posterior rotation of approximately 15
degrees.
Which of the following is a facial landmark used d. The width of the ear is approximately
as a reference point when dividing the face into two-thirds its length.
thirds?
a. Rhinion
b. Nasion
c. Menton
d. Pogonion
e. Stomion

27
CHAPTER TWENTY TWO

RECOGNITION A N D TREATMENT
OF S K I N LESIONS

1. Studies indicate that up to what percent of 4. Treatment of hemangiomas may be indicated for
actinic keratoses can become squamous cell all circumstances except
carcinoma? a. Ocular involvement
a. 10% b. Airway involvement
b. 20% c. Rapidly growing
c. 30% d. Ulceration
d. 40% e. Beginning involution
e. 50%
5. A patient with bifid ribs, skin lesions, frontal
2. What is the treatment of choice for keratoacan- bossing, jaw cysts, and calcified cerebra has
thomas? a. Xeroderma pigmentosa
a. Retinoic acid b. Gardner's syndrome
b. Chemical peel c. Nevoid basal cell syndrome
c. Mohs' surgical excision d. Sturge-Weber syndrome
d. Laser resurfacing e. Kasabach-Merritt syndrome
e. Observation

3. What is the preferred biopsy technique for


melanoma?
a. Excisional biopsy
b. Shave biopsy
c. Cryotherapy
d. Curettage
e. Electrodesiccation

28
CHAPTER TWENTY THREE

M A N A G E M E N T OF H E A D
AND NECK MELANOMA

1. Desmoplastic melanoma 4. Which of the following criterion excludes a


a. Is most often black w i t h asymmetry and irreg- patient from consideration for sentinel lymph
ular borders and is larger than 6 m m i n diam- node mapping with biopsy?
eter a. Sentinel lymph node mapping with biopsy
b. Rarely arises on the head and neck region b. Computed tomography scan
c. Is associated with an increased incidence of c. Magnetic resonance imaging
cervical metastasis compared with other d. Positron emission tomography scan
melanoma variants e. Elective neck dissection
d. Often arises i n the setting of a lentigo malig-
nant melanoma histologic subtype and has a 5. Which of the following criterion excludes a
propensity for neurotropic spread patient from consideration for sentinel lymph
e. Has a worse prognosis even when corrected node mapping with biopsy?
for other risk factors a. Previous punch biopsy of primary lesion
b. Tumor ulceration
2. Adjuvant radiation therapy is considered i n all of c. Depth of invasion <1.0 m m
the following settings except d. Depth of invasion >4.0 m m
a. Neurotropism e. 1-cm cervical lymph node positive for
b. Extracapsular spread melanoma on fine-needle aspiration
c. Multiple lymph node involvement
d. Tumor ulceration
e. Recurrent melanoma

3. Which of the following criteria is not imple-


mented i n the staging of cutaneous melanoma
under the 2001 American Joint Committee on
Cancer guidelines?
a. Primary tumor thickness
b. Primary tumor ulceration
c. Number of metastatic lymph nodes
d. Size of metastatic lymph nodes
e. Microscopic vs macroscopic nodal disease

29
CHAPTER TWENTY FOUR

SCAR R E V I S I O N A N D C A M O U F L A G E

1. Which of the following statements regarding scar 4. Regarding dermabrasion, which is true?
re-excision is true? a. Wire fraises are ideal for the neophyte
a. Re-excision with primary closure is an irregu- surgeon.
larization procedure. b. Prophylaxis for herpetic outbreak is unneces-
b. Sharp excision of the scar should proceed with sary i n dermabraded patients.
the scalpel exactly parallel to the skin surface. c. Dermabrasion is performed best 6 to 8 weeks
c. Direct closure of the area should be performed after surgical scar revision.
without undermining. d. The dermabrasion bit should be moved paral-
d. Vertical mattress sutures are an effective way lel to the direction of rotation of the bit.
to properly evert the wound edges. e. Dermabrasion should be performed through
the layer of the reticular dermis.
2. Which of the following patients is most amenable
to surgical scar revision surgery? 5. Which of the following statements regarding scar
a. A 2-month-old w i t h a slightly depressed scar irregularization is true?
of the forehead 2 weeks after a fall that a. Irregularization is not an effective way to
resulted i n closure of a laceration i n the emer- camouflage a raised scar that falls between two
gency department facial aesthetic units.
b. A patient 6 weeks after a bilobed flap repair of b. A classic 30-degree Z-plasty will increase the
the nasal sidewall with pincushioning and edema length of the final scar by 25%.
c. A patient 6 months after a motor vehicle acci- c. The limbs of the triangles i n W-plasty should
dent w i t h depressed scar along the cheek and be between 2 and 4 m m i n length.
malar prominence area d. Geometric broken line closure is a poor choice
d. A patient with a red scar i n the preauricular of technique for long linear scars that do not
crease 2 months after harvest of a preauricular fall within relaxed skin tension lines.
full-thickness skin graft e. Dermabrasion is rarely used after scar irregu-
larization.
3. Which of the following statements regarding
tissue expansion is false?
a. Rectangular expanders provide the greatest
expansion.
b. I n general, the base of an expander should be
approximately 2.5 to 3.0 times as large as the
area to be reconstructed.
c. The effects of tissue expansion on skin include
epidermal thinning, decreasing melanin pro-
duction, and derma thickening.
d. Expansion can proceed until the skin blanches
or the patient complains of discomfort.
e. Tissue expanders can cause impressive disfig-
urement of the head and neck, which natu-
rally results i n significant emotional stress to
the patient.

30
C H A P T E R T W E N T Y FIVE

FACIAL T R A U M A : SOFT-TISSUE
L A C E R A T I O N S A N D BURNS

Which of the following statements regarding c. TAG (tetracaine [0.5%], adrenaline [1:2000],
facial laceration repair is true? cocaine [11.8%]) applied topically to a lacera-
a. Extensive soft-tissue debridement should be tion will minimize the pain of injected anes-
undertaken before repair of a simple laceration. thetics and may obviate the need for any
b. Sutures should be left i n place for 7 to 10 days injection of anesthetics.
to achieve optimal cosmetic results. d. Special consideration should be given to lacer-
c. Obtaining an accurate clinical history regard- ations that extend through an anatomic mar-
ing the facial injury is of little value i n making gin (i.e., nostril margin or vermillion border of
treatment decisions. the lip) for appropriate realignment.
d. Thorough removal of debris from a wound e. Neutralization of acidic injectable anesthetics
before primary closure will help prevent debris with sodium bicarbonate can decrease the dis-
tattooing. comfort associated with the injection.
e. Repair of facial lacerations is best accom-
plished with 3-0 or 4-0 sutures. 4. Which of the following statements regarding
facial burns is true?
Injury to Stensen's duct as a result of a deep lac- a. Early, aggressive soft-tissue debridement is
eration to the cheek recommended i n an oral commissure electri-
a. Is rarely associated w i t h an injury to the buc- cal burn.
cal branch of the facial nerve b. Tarsorrhaphy is required i n all thermal
b. Can often be confirmed by cannulating the injuries to the periorbital region.
duct intraorally with a lacrimal probe c. The sooner facial burns are sealed by sponta-
c. Is easier to repair i n a delayed fashion after neous re-epithelization or skin grafting, the
the wound has matured after primary closure better the ultimate cosmetic and functional
d. Never results in salivary fistula or sialocele, outcome will be.
even if the laceration is not repaired d. I n general, topical antimicrobial agents are
e. Is best repaired with 4-0 or 5-0 absorbable contraindicated i n facial burns.
sutures e. If a burn spontaneously heals at 6 to 8 weeks,
there will be little scarring, and skin quality
Which of the following statements regarding will be excellent.
facial laceration repair is false?
a. A soft-tissue avulsion should immediately be 5. A painful facial burn with significant erythema
covered with a full-thickness skin graft. and blistering is best classified as a
b. I n patients who will not likely return for a. Full-thickness burn
suture removal or i n children who may not be b. Third-degree burn
cooperative with suture removal, the use of c. First-degree burn
fast-absorbing plain gut is acceptable. d. Second-degree burn

31
C H A P T E R T W E N T Y SIX

MAXILLOFACIAL TRAUMA

The mesiobuccal cusp of the maxillary first 4. Preservation of what structure is important i n
molar sitting within the mesiobuccal groove of repairing the frontal sinus/anterior cranial fossa?
the mandibular first molar designates which type a. Supraorbital r i m
of occlusion? b. Temporalis fascia
a. Glass I c. Galea
b. Glass I I d. Pericranium
c. Glass I I I e. Periosteum
d. Glass IV
e. Glass V 5. To prevent postoperative enophthalmus, the sur-
geon must remember that the medial orbital wall
Lag screw fixation is recommended for which a. Should not be repaired
type of mandible fracture? b. Is concave
a. Nondisplaced c. Is convex
b. Comminuted d. Cannot be accessed through the approach
c. Oblique used to the orbital floor
d. Contaminated e. Lies i n a sagittal plane
e. Unfavorable

In panfacial fractures, what should be the first


and primary focus?
a. Orbital reconstruction
b. Establishing facial height
c. Occlusion
d. Repositioning the zygoma
e. Repairing the central face

32
CHAPTER T W E N T Y SEVEN

R E C O N S T R U C T I O N OF FACIAL DEFECTS

1. Which of the following is not true of rotational 4. A 57-year-old woman has a squamous cell cancer
flaps? removed from her upper lip, leaving a defect that
a. Rotational flaps are best used to close triangu- encompasses approximately two-thirds of her
lar defects. upper lip. Which of the following represents the
b. Rotational flaps usually have a random vascu- most appropriate choice for reconstruction?
lar supply. a. Microvascular tissue transfer
c. Rotational flaps are ideally superiorly based. b. Rotational flap
d. Rotational flaps are useful for posterior c. Advancement or transpositional flap from the
cheeks. cheek
e. Rotational flaps feature curvilinear shapes. d. Mucosal hinge flap
e. Primary closure
2. Which of the following is not true of the midfore-
head flap? 5. Which of the following is not a goal i n repairing
a. The midforehead flap has an excellent forehead defects?
color/texture match for midface defect repair. a. Preservation of frontalis muscle function
b. The axial flap is based on the supratrochlear b. Preservation of sensation of the forehead skin
artery. c. Maintenance of eyebrow symmetry
c. Frontalis muscle and fascia can be included d. Maintenance of natural-appearing temporal
when more depth is needed. and frontal hairlines
d. The midforehead flap represents one type of e. Creation of vertical instead of horizontal scars
rotational flap. whenever possible (except i n the midline fore-
e. Cartilage grafts can be included for nasal head)
reconstruction.

3. Which of the following is not true of advance-


ment flaps?
a. Advancement flaps have a linear configura-
tion.
b. Advancement flaps are moved into a defect by
being stretched forward.
c. Closure of advancement flaps depends on both
primary and secondary movement.
d. Advancement flaps are most commonly used
for very large facial defects.
e. Advancement flaps are categorized as pedicle,
bipedicle, and V-Y types.

33
CHAPTER TWENTY EIGHT

H A I R RESTORATION: M E D I C A L
A N D SURGICAL T E C H N I Q U E S

1. In androgenetic alopecia, hair growth of specific finasteride were rated as having no further
hair follicles on the scalp is sensitive to which of baseline hair loss.
the following androgens? d. Patients taking finasteride need to be on con-
a. Testosterone tinued therapy to preserve the renewed hair
b. Estradiol growth.
c. Estrone e. Patients taking finasteride for an extended
d. Dihydrotestosterone period of time report an increase i n the inci-
e. Dehydroepiandrosterone dence of adverse effects as high as 8%, includ-
ing decreased sexual libido and erectile
2. Telogen effluvium is a physiologic response to a dysfunction.
variety of stressors, including hormonal or sys-
temic conditions manifesting as a form of diffuse, 4. I n terms of follicular-unit hair transplantation,
nonscarring alopecia. Which of the following which of the following statements is false?
statements correctly characterizes this physio- a. Follicular-unit hair grafts consist of naturally
logic response? occurring groups of one to four hairs i n addi-
a. The hair follicle gradually shifts from the tion to the supporting structures, including
anagen phase to the catagen phase. sebaceous glands and a circumferential band
b. The hair follicle precipitously shifts from the of collagen.
anagen phase to the catagen phase. b. Ideal candidates for hair transplantation
c. The hair follicle gradually shifts from the have a significant contrast between hair
anagen phase to the telogen phase. color and skin color and are old enough
d. The hair follicle precipitously shifts from the that future hair loss is more likely to be
anagen phase to the telogen phase. predictable.
e. The hair follicle precipitously shifts from the c. I n planning the anterior hairline, considera-
catagen phase to the telogen phase. tion should be given to recreating a "feather-
ing" transition zone of approximately 0.5 to
3. I n terms of medical management of androgenetic 1.0 cm, as well as augmenting a dense frontal
alopecia, which of the following statements about forelock.
finasteride is false? d. The donor strip is harvested within the pre-
a. Finasteride is a 5-oc-reductase inhibitor that dicted hair fringe margin, which extends from
does not interfere with the actions or effects of each temporoparietal region to the midoccipi-
testosterone. tal scalp.
b. I n 5-year studies, 65% of men taking finasteride e. I n creating recipient incisions, specific
maintained or improved their hair counts, attention should be given to the surrounding
whereas men on placebo continued to lose hair. natural hair growth i n terms of proper
c. I n 5-year studies based on reviewing standard- hair direction and angulation from the
ized clinical photographs, 90% of men taking scalp.

34
Chapter 28 Hair Restoration: Medical and Surgical Techniques 35

5. I n the past, scalp reductions were used exten- c. I n terms of complications associated with
sively to minimize or eliminate areas of alopecia. scalp reductions, the most significant compli-
Which of the following statements about scalp cation is intraoperative bleeding associated
reductions is false? with extensive scalp dissection.
a. Excising a bald portion of scalp requires wide d. The vertical "slot" defect is created i n the pos-
subgaleal undermining. terior scalp and is related to misdirected hair
b. In patients with a high anterior hairline and a growth from repetitive scalp reductions.
prominent forehead, a hairline advancement can e. Modified scalp reductions still have a role i n
be performed through an irregular, trichophytic hair restoration surgery, often i n conjunction
incision and excision of pretrichial skin. with hair grafting.
CHAPTER TWENTY NINE

M A N A G E M E N T OF A G I N G S K I N

Which Fitzpatrick sun-reactive skin type does 3. What is the most important ingredient i n creat-
the following characterize: a fair-skinned individ- ing the depth of a Baker's solution peel?
ual with blond, red, or brown hair, usually burns a. Phenol 88%
and tans less than the average person? b. Septisol
c. Croton oil
Which of the following is not and absolute con- d. Distilled water
traindication to chemical peeling? e. Hydroxy-acid
a. Collagen vascular disease
b. Emotional instability 4. Describe the physiologic sequence seen with
c. Isotretinoin treatment 8 months ago phenol toxicity from a chemical peel, and
d. Immunosuppressed patient explain how such toxicity can be avoided.
e. Fitzpatrick type IV skin type
5. Describe the mechanism of action of retinoids
(i.e., tretinoin) and common side effects i n facial
rejuvenation.

36
CHAPTER THIRTY

RHYTIDECTOMY

1. Which is a disadvantage of a subperiosteal m i d - 4. A n SMAS rhytidectomy is a facelift i n which


face lift? a. A n extended supra-SMAS dissection is per-
a. It requires preauricular incision. formed.
b. I t impairs vascular supply of facial skin. b. The SMAS is tightened solely with suture sus-
c. I t weakens orbicularis occuli. pension.
d. I t risks injury to buccal branch of facial nerve. c. A flap consisting of a portion of the SMAS is
dissected.
2. Which surgical approach is not used to perform a d. A sub-SMAS dissection is performed to lift the
subperiosteal midface lift? midface.
a. Transtemporal e. An SMASectomy is performed over the parotid
b. Transfacial gland.
c. Transorbital
d. Transoral 5. The most effective method of managing anterior
e. Combined neck bands is
a. Suture suspension of the cervical platysma to
3. Which mimetic muscle is part of the superficial mastoid fascia
muscular aponeurotic system (SMAS)? b. Dissection of a platysmal flap below the angle
a. Zygomatic major of the mandible and advancing the flap pos-
b. Zygomatic minor terosuperiorly
c. Orbicularis occuli c. Horizontal division of the cervical platysma
d. Platysma and posterior advancement of the muscle flap
e. Orbicularis oris d. Submental fat excision and horizontal division
of the anterior border of the platysma
e. Excision of the anterior platysmal muscles and
midline suture apposition

37
CHAPTER THIRTY ONE

M A N A G E M E N T OF T H E A G I N G B R O W
A N D FOREHEAD

1. Which of the following statements regarding pre- 4. To achieve brow elevation, all of the following
trichal forehead lift is true? structures are "released" i n the endoscopic
a. Pretrichal forehead lift preserves the hairline. browlift except
b. Pretrichal forehead lift is indicated i n patients a. Supraorbital periosteum
with a low hairline. b. Temporal conjoint fascia
c. Pretrichal forehead lift has no risk of an c. Temporal conjoint tendon
exposed scar. d. Temporalis muscle
d. Pretrichal forehead lift does not treat all e. Brow depressor musculature
aspects of the aging forehead and brow.
e. The temporal incision i n the pretrichal fore- 5. Which nerve may be injured if cautery is per-
head lift is not similar to that i n the coronal formed on the undersurface of the temporopari-
lift. etal fascia i n the region of the "sentinel" vein?
a. Zygomaticotemporal nerve
2. Upper blepharoplasty performed before a b. Supratrochlear nerve
selected brow elevation procedure can be associ- c. Supraorbital nerve
ated with d. Frontal branch of the facial nerve
a. Minimal elevation of the brow-lid complex e. Auriculotemporal nerve
b. Lagophthalmos
c. Elevation of the lateral canthus
d. Upper eyelid ptosis
e. Injury to the supratrochlear nerve

3. A l l of the following are brow depressor muscles


except
a. Gorrugator
b. Frontalis
c. Procerus
d. Supraorbital orbicularis oculi
e. Depressor supercilii

38
CHAPTER THIRTY TWO

M A N A G E M E N T OF T H E A G I N G
PERIORBITAL AREA

1. A l l the following statements about the frontal 4. A l l of the following statements are true regarding
branch of the facial nerve are true except upper eyelid blepharoplasty except
a. It lies deep to the superficial musculoaponeu- a. The initial l i d marking is made at the natural
rotic system fascia. skin crease or 1 m m above the natural crease.
b. It enters the orbicularis oculi muscle and fronta- b. The medial end of the incision is carried to
lis muscle along the deep surface of the muscles. but not beyond the punctum of the medial
c. As i t crosses the zygomatic arch, i t lies deep can thus.
to the periosteum. c. If blepharoplasty is performed i n conjunction
d. I t supplies the muscles of the forehead and the with a browlift, the markings for the blepharo-
orbicularis oculi muscle. plasty incisions are made first.
e. I t courses anterior to the superficial temporal d. Orbital fat may be addressed by cauterizing
artery. through the orbital septum or by opening the
orbital septum to remove fat.
2. A l l of the following statements about ideal brow e. A variable amount of orbicularis oculi muscle
anatomy i n females are true except should be resected to create a distinct upper
SL. The medial end should lie i n a horizontal eyelid crease.
plane 2 m m inferior to the lateral end.
b. The medial end should have a club head 5. Compared with the transconjunctival approach
appearance. for lower eyelid blepharoplasty, the subciliary
c. The lateral end should gradually taper to a point. approach is useful for which of the following
d. The medial origin should be at the level of a group of patients?
vertical line drawn from the nasal alar facial a. Patients desiring associated midface lifting
junction. b. Patients primarily desiring treatment for fat
e. The brow should arch superiorly with the pseudoherniation
highest point at the lateral limbus. c. Patients with thick skin
d. Patients who smoke
3. The advantages of subcutaneous dissection for e. Patients with large amounts of excess skin
browlift and blepharoplasty compared with
deeper subgaleal and subperiosteal dissection
include all of the following except
a. Sensory nerve branches are spared with sub-
cutaneous dissection.
b. Subcutaneous dissection results i n a lower
incidence of skin slough and hair loss from
vascular compromise.
c. Subcutaneous dissection provides an effective
method for abolishing forehead rhytids and
moderate to severe brow ptosis.
d. Subcutaneous dissection protects the facial
nerve branches from injury.
e. Subcutaneous dissection may enable a greater
degree of accuracy for brow placement and
postoperative brow symmetry.

39
CHAPTER THIRTY THREE

SUCTION-ASSISTED LIPOCONTOURING

1. When considering whether a patient is a candi- 4. When ultrasonic liposuction is performed


date for lipocontouring, the surgeon need not be a. The risk of hematoma is greater than with
concerned with which of the following? standard liposuction.
a. The age of the patient b. Heat may be generated at the distal subdermis
b. The adjacent anatomic structures and skeletal and the cutaneous incision site, thereby lead-
framework ing to potential complications.
c. Whether the fat responds to weight loss c. The wound must be bathed i n ultrasound jelly.
d. The gender of the patient d. Fluctuations i n the weight gain and loss at the
treated area are less significant than those
2. The area of liposuction will that occur at areas treated with traditional
a. Have a greater tendency to gain or lose weight liposuction.
with fluctuation i n overall body mass
b. Have a response to changes i n weight that is 5. The ideal patient for liposuction
similar to other regions of the body a. Has loose skin and a weak jawline
c. Replace removed adipocytes with new ones b. Has young elastic skin, good bone structure,
during periods of weight gain and a low anterior hyoid
d. Never return to the preexisting contour c. Has young elastic skin, a strong jawline, a high
posterior hyoid, and fat that is not responsive
3. During liposuction to weight loss
a. Adipocytes are selectively removed because of d. Would like to have surgery that would help
their structural integrity. him or her attract a mate, get a raise, and look
b. Vessels, nerves, and muscles are protected like a model
because of their loose intercellular connec-
tions.
c. Maintaining bridges of uninterrupted tissue
between the deep and superficial tissue is
important.
d. Only hypertrophic adipocytes are removed.

40
CHAPTER THIRTY FOUR

M E N T O P L A S T Y A N D FACIAL I M P L A N T S

In contrast to alloplastic augmentation mento- 4. A disadvantage of the use of expanded polytetra-


plasty, the osseous genioplasty allows fluoroethylene (ePTFE) as a facial implant is
a. Correction i n the horizontal plane only a. Difficulty with removal
b. Correction i n the vertical plane only b. Capsule formation
c. Correction i n the transverse, horizontal, and c. Extrusion
vertical plane d. Instability
d. Correction i n the transverse plane only e. Difficulty contouring the implant
e. A reversible technique
5. Malar and/or submalar augmentation is indicated
If bony osteotomies are made, where should they for all of the following except
be i n relation to the mental foramen? a. Congenital defects
a. 2 m m above b. Traumatic deficiencies
b. 2 m m below c. Accentuating the nasolabial fold
c. 5 m m above d. Cheek soft-tissue ptosis
d. 5 m m below e. Submalar wasting
e. At the level of the foramen

When a chin implant is placed, all of the follow-


ing are true except
a. Supraperiosteal placement may cause bone
erosion.
b. Subperiosteal placement may cause bone ero-
sion.
c. The mentalis should be reapproximated.
d. Subperiosteal placement increases fixation of
the implant.
e. A n intraoral or extraoral approach may be
used.

41
C H A P T E R T H I R T Y FIVE

R E H A B I L I T A T I O N OF FACIAL PARALYSIS

1. Which branch of the facial nerve has the highest 4. Advantages of polytetrafluoroethylene (PTFE;
priority when reinnervation procedures are being Gore-Tex) for static suspension of the midface
considered? include all of the following except
a. Frontal a. No donor site morbidity
b. Buccal b. Shorter operative time
c. Mandibular c. Lower infection rate than w i t h autologous fas-
d. Cervical cia grafts
e. None. They are equally important. d. Less overcorrection necessary than autologous
fascia grafts
2. After transection of the facial nerve, the distal e. Immediate improvement of facial symmetry
branches retain their stimulability with a compared with reinnervation procedures
portable electrical stimulator for how long?
a. 1 day 5. A patient with complete facial paralysis for
b. 3 days YA years after parotidectomy is referred to you
c. 1 week for management. While reviewing the operative
d. 3 weeks report, you learn that the patient had transection
e. 12 months of the facial nerve during parotidectomy proxi-
mal to the pes anserinus. The patient has had no
3. A healthy 68-year-old patient develops House- discernible return of function but desires a rein-
Brackmann grade V I facial paralysis after resec- nervation procedure. Which diagnostic test is
tion of an acoustic neuroma. One year later, he most important to obtain before proceeding with
has no discernible return of function. The reinnervation?
patient desires the best possible outcome. Which a. Magnetic resonance imaging
treatment would provide optimal rehabilitation b. Audiogram
for this patient? c. Muscle biopsy
a. Reinnervation with sural nerve graft d. Schirmer test
b. Static sling, lateral canthoplasty, and gold e. An electromyogram
weight
c. XII-VII crossover graft
d. Temporalis transfer
e. Masseter transfer

42
C H A P T E R T H I R T Y SIX

OTOPLASTY

1. What is the embryologic basis for protruding 4. A 4-year-old child has bilateral protruding ears.
ears? During the initial consultation, i t is apparent that
a. Autosomal-dominant inheritance the mother has a similar affliction. The mother
b. Overgrowth of ectoderm from the first requests correction before the child enters
branchial arch kindergarten. What is the best plan of action?
c. Overgrowth of mesoderm from the third a. The parents should be advised to wait until
branchial arch the child is 5 or 6 years old and ear growth is
d. Hypertrophy of the otic placode nearly complete.
e. Maldevelopment of the forth hillock of His b. The parents should be advised to use octyl-
2-cyanoacrylate to hold the ears back until
2. What is the most appropriate indication for pur- surgical correction is achieved.
suing otoplasty? c. Postauricular fusiform skin excision should be
a. Age of patient performed.
b. Auriculocephalic angle d. Scoring of the anterior cartilage surface should
c. Cartilage proportions be performed followed by horizontal mattress
d. Distance of helical r i m from scalp sutures from the scapha to the concha.
e. Cartilage stiffness e. Scoring of the anterior cartilage surface alone
should be performed.
3. A 29-year-old man is initially seen with a unilat-
eral prominent ear. There is minimal furling of 5. A 32-year-old woman with Ehlers-Danlos syn-
the antihelix, yet the cartilage seems thick and drome undergoes bilateral otoplasty. At 3:00 A M ,
stiff. What is a practical treatment modality? she contacts you through the answering service
a. Octyl-2-cyanoacrylate to complain of pain. What is the appropriate
b. Postauricular fusiform skin excision action?
c. Horizontal mattress sutures from the conchal a. Instruct the patient to remove any dressings
wall to the mastoid periosteum or bolsters.
d. Scoring of the anterior cartilage surface fol- b. Instruct the patient to increase pain medica-
lowed by horizontal mattress sutures from the tions.
scapha to the concha c. See the patient and remove the dressing.
e. Scoring of the anterior cartilage surface alone d. See the patient and reassure h i m or her that
fullness under the skin will resolve.
e. See the patient and evaluate the ear under
dressing and drain any possible collection.

43
PART T H R E E

NOSE
CHAPTER T H I R T Y SEVEN

P H Y S I O L O G Y OF O L F A C T I O N

1. The primary neuron cell body for cranial nerve I 4. Of a random population of people, which person
is located i n the would do best on an olfactory identification test?
a. Olfactory bulb a. A 38-year-old man
b. Nasal mucosa b. A 40-year-old woman
c. Entorhinal cortex c. A 68-year-old woman
d. Cribriform plate d. A 67-year-old man
e. Prefrontal cortex e. A 5-year-old girl

2. A 38-year-old man with history of a sudden "bad 5. A 43-year-old woman comes to your office with a
cold" 4 months ago complains of bland taste of history of the inability to smell for the past 6
foods. On further questioning, he notes the years. She does not remember any previous head
inability to detect smells that are strong to his trauma but remembers an upper respiratory
wife. I n addition, he recalls burning the eggs last tract infection around the time of the loss of
week and had not noticed the fire until he saw smell. She denies current nasal obstruction,
the smoke. However, he does remember smelling recurrent sinus infections, headaches, and epis-
the ammonia 2 days ago that he was using to taxis. A neurologic examination is negative.
clean the bathroom. He can most likely detect Nasal endoscopy is normal, and smell identifica-
the ammonia because of a functioning tion testing is consistent with anosmia. The most
a. Cranial nerve I important next step is to
b. Cranial nerve V a. Obtain magnetic resonace imaging of the head
c. Cranial nerve V I I b. Obtain a computed tomography scan of the
d. Cranial nerve IX sinuses
e. Cranial nerve X c. Refer her for an electroolfactogram
d. Counsel her on hazards of anosmia
3. While undergoing olfactory identification testing, e. Admit her for high-dose intravenous steroids
it was noted that the patient was consistently
unable to identify the same specific odorant over
many trials; however, all other odorants were
identified correctly. The best possible explana-
tion is a mutation i n a gene encoding for a/an
a. Olfactory G-protein
b. Cyclic adenosine monophosphate
c. Inositol phosphate
d. Olfactory receptor protein
e. Calcium/sodium channel

47
CHAPTER THIRTY EIGHT

E V A L U A T I O N OF N A S A L B R E A T H I N G
F U N C T I O N W I T H OBJECTIVE A I R W A Y
TESTING

The nasal valve area consists of 4. Which of the following can be a source of vari-
a. The septum and distal end of the upper lateral ability i n objective airway testing?
cartilage a. The nasal cycle
b. The septum, piriform aperture, and head of b. Posture
the inferior turbinate c. Time of day
c. The septum, lower lateral cartilage, and floor d. Smoking
of nose e. A l l of the above
d. The distal end of the upper lateral cartilage,
head of the inferior turbinate, caudal end of 5. I n a patient with nasal obstruction, examination
the septum, and tissues surrounding the p i r i - reveals pathology on the same side as the
form aperture patient's symptoms. Objective testing confirms
e. The union of the lateral nasal wall with the the restriction, and the patient reports improve-
upper lateral cartilage forming an anatomic ment after decongestion. Which of the following
ridge is the best approach to treatment as outlined i n
the algorithm?
Acoustic rhinometry can be used to measure a. Surgery
which of the following? b. Medical management followed by surgical
a. Size of the minimal cross-sectional area intervention if no improvement
(MCA) c. Medical management
b. Distance to various cross-sectional areas i n d. Repeat examination and objective testing with
the nose imaging
c. Total volume of the nose e. Surgery with option for medical treatment if
d. Location of the MCA symptoms are still present
e. A l l of the above

Which of the following is true regarding rhino-


manometry?
a. Measurement of total nasal resistance is possi-
ble only by calculation from unilateral meas-
urements.
b. Inability to calculate a cross-sectional area
from rhinomanometric measurements is a l i m -
iting factor i n its use.
c. I n a more obstructed nose, the pressure-flow
curve is rotated toward the pressure axis.
d. The typical pressure-flow curve obtained i n a
normal nose is a linear relationship repre-
sented by a straight line.
e. Passive rhinomanometry requires the patient's
own respiratory effort as a source of pressure
and flow.

48
CHAPTER THIRTY NINE

M A N I F E S T A T I O N S OF SYSTEMIC
DISEASES OF T H E NOSE

1. With regard to anti-neutrophil cytoplasmic anti- 4. A l l of the following are true about atypical
body (ANGA) testing and Wegener's granulomato- mycobacterial infections except
sis (WG), which of the following statements is a. Purified protein derivative (PPD) skin testing
incorrect? is often negative.
a. A negative c-ANGA test does not exclude the b. Patients classically are initially seen with cer-
diagnosis of WG. vical adenopathy.
b. c-ANGA specificity is >90% during the sys- c. Causative organisms can inevitably be cul-
temic vasculitic phase of the disease. tured from biopsy specimens.
c. The characteristic c-ANGA pattern is caused d. Auramine-rhodamine staining is a useful i n i -
by antibodies against proteinase 3 (PR3). tial step i n the diagnosis.
d. Perinuclear-ANCA testing is superior to e. Nasal involvement is typified by anterior sep-
c-ANGA testing i n the diagnosis of WG. tal perforations.

2. I n patients with large, dry septal perforations 5. A 20-year-old patient is seen w i t h progressive
without evidence of ulceration and i n whom unilateral nasal destruction with involvement of
histopathologic findings are nonspecific and the adjacent maxillary sinus and early involve-
c-ANGA testing is negative, the most likely cause ment of the oral cavity. ANCA testing is negative.
of the nasal changes is Biopsy shows cells with angiocentric and
a. Wegener's granulomatosis angioinvasive features. The most likely diag-
b. T-cell lymphoma nosis is
c. Sarcoidosis a. Wegener's granulomatosis
d. Tuberculosis b. Sarcoidosis
e. Substance abuse c. T-cell lymphoma
d. Atypical mycobacterial infection
3. Which of the following laboratory results is e. Histiocytosis X
incorrect i n the diagnosis of patients suspected
of having sarcoidosis?
a. Hypocalcemia
b. Hypercalcuria
c. High number of T lymphocytes i n broncho-
alveolar lavage fluid
d. Elevated angiotensin-converting enzyme
e. Increased levels of sIL-2R serum

49
C H A P T E R FORTY

EPISTAXIS

Epistaxis presenting i n a delayed fashion after 4. Signs of fever, nausea, vomiting, and diarrhea i n
maxillofacial trauma should raise suspicion for the presence of nasal packing may indicate
a. Undiagnosed nasal fracture a. Toxic shock syndrome
b. Arterial aneurysm b. Streptococcal infection
c. Continued traumatic insult to nasal mucosa c. Viral infection
d. Altered nasal airflow d. Contamination of the packing material
e. Bacterial infection e. Anaphylactic reaction to packing material

The most common familial bleeding disorder to 5. The most effective laser for treatment of telang-
consider i n frequent, difficult-to-manage epis- iectasias i n Osier-Weber-Rendu disease is
taxis is a. Nd:YAG
a. Hemophilia A b. C 02

b. Hemophilia B c. KTP
c. von Willebrand's disease d. Argon
d. Thrombocytopenia e. Pulse dye laser
e. Osier-Weber-Rendu disease

In managing epistaxis, posterior nasal packing


a. Should be the first-line therapy
b. Should be avoided
c. Should not be performed with anterior nasal
packing
d. Requires the patient to be monitored for
hypoxia, apnea, and arrhythmias
e. May predispose to hypocarbia

50
CHAPTER FORTY O N E

NASAL FRACTURES

1. A 25-year-old man is seen i n the emergency 4. (Case) A 34-year-old woman is seen with a nasal
department 3 hours after being struck i n the deformity after a high-speed car accident.
nose with a basketball during a game. Examination reveals a flattened dorsum and
Examination reveals significant midfacial edema, widening between the inner canthi. What
ecchymosis, and crepitance of the nasal pyra- approach is best for repair of the injury?
mid. The remainder of the head and neck exami- a. Bicoronal scalp flap
nation is normal. What is the next best step i n b. Lateral rhinotomy
management? c. "Open sky" incision
a. Plain films of the nasal bones d. Intranasal, intercartilaginous incisions
b. Computed tomography scan of the maxillofa- e. Open rhinoplasty approach (bilateral marginal
cial skeleton and transcolumellar incisions)
c. Administration of local sedation followed by
closed nasal reduction 5. Which of the following statements regarding
d. Outpatient follow-up i n 48 to 72 hours to nasal septal hematomas is not true?
reassess nasal structure a. Children are more likely than adults to have
e. Open septorhinoplasty i n 6 months septal hematomas develop after nasal injury.
b. Hematoma collection results i n cartilage
2. What is the most common reason for failure after necrosis i n 3 days.
a closed nasal reduction (CNR) performed within c. Nasal septal hematomas often produce long-
1 week of injury? term complications such as saddle nose defor-
a. Nasal bone comminution with poor underlying mity, perforation, and columellar retraction.
support d. Nasal septal hematomas often appear as blue,
b. Fibrous tissue formation between bony frag- noncompressible intranasal masses.
ments e. An untreated nasal septal hematoma can lead
c. Nasal septal fracture to an intracranial infection.
d. Additional mid-facial fractures
e. Greenstick fracture of the nasal bones

3. Management of deviated nasal fractures i n chil-


dren should include all of the following except
a. Early closed nasal reduction (<4 days)
b. General anesthesia during closed nasal reduc-
tion
c. Immediate incision and drainage of septal
hematoma
d. Nasal stenting after closed nasal reduction for
infants less than 6 months of age
e. Open reduction for injuries with deviation
greater than one-half the width of the nasal
bridge

51
CHAPTER FORTY T W O

ALLERGIC R H I N I T I S

Which statement most accurately describes the 4. Which of the following statements is most accu-
importance of allergy i n the practice of otolaryn- rate regarding treatment of allergy?
gology—head and neck surgery? a. Most patients can be controlled with the use of
a. I t will be of no importance to the average oto- antihistamines, which are now available w i t h -
laryngologist. out a prescription.
b. I t will rarely be encountered i n a subspecialty b. Any patient with positive skin and/or i n vitro
practice such as otology or pediatric otolaryn- tests for allergen-specific IgE should receive
gology. allergen immunotherapy.
c. I t is responsible for a small fraction of health- c. Topical nasal corticosteroids should be used
care costs i n the United States. daily by patients with allergic rhinitis and are
d. I t may represent a primary or contributory safe for long-term use.
diagnosis i n up to half the patients seen by the d. The best method of managing inhalant allergy
general otolaryngologist. is environmental control, and the most avoid-
e. I t will frequently require referral to an aller- able allergens are the perennial group: dust
gist, because i t is outside the capabilities of a mite, mold, and animal danders.
surgical subspecialty. e. Leukotriene modifiers attack the allergic reac-
tion at its source and should be first-line ther-
The most important type of allergy encountered apy for patients with allergic rhinitis.
by the otolaryngologist, as classified by Gell and
Coombs, is 5. Which statement regarding immunotherapy is
a. Perennial most accurate?
b. Delayed a. Candidates for immunotherapy are coopera-
c. Immediate tive patients with proven IgE-mediated allergy,
d. Seasonal in whom pharmacotherapy and/or avoidance
e. Cytotoxic are ineffective or impractical, producing symp-
toms that are severe.
The most important tool i n making the diagnosis b. Monoclonal antibody (anti-IgE) therapy given
of allergy is as a 3- to 5-year therapy may produce the
a. A positive skin prick test same effects as conventional immunotherapy.
b. A positive intradermal skin test c. Inhalant immunotherapy is indicated i n
c. A positive history for symptoms associated patients with seasonal rather than perennial
with exposure allergy.
d. A total IgE >100 IU/mL d. The risk of anaphylaxis associated with
e. An mRAST value of class I I or higher immunotherapy on the basis of quantitative
testing is so negligible that i t may be adminis-
tered by a minimally trained assistant i n any
office setting.
e. The technique of immunotherapy involves
administering progressively smaller doses of
antigen to modify the patient's immune
response to antigen challenge.

52
CHAPTER FORTY T H R E E

NONALLERGIC RHINITIS

1. Nonallergic rhinitis usually presents with all of 4. Which of the following statements about atrophic
the following symptoms except rhinitis is false?
a. Rhinorrhea a. May be related to granulomatous diseases
b. Nasal congestion b. Aggressive nasal surgery
c. Nasal osbstruction c. Associated w i t h aging
d. Itching d. Presents with crusting and foul odor
e. Negative skin testing e. Demonstrates preservation of nasal airflow

2. The pathophysiology of nonallergic rhinitis may 5. Gustatory rhinitis appears related to


involve a. Food allergy
a. Underactivity of the afferent reflex pathway b. Excess sympathetic activity
b. Underactivity of the efferent reflex pathway c. Stimulation of afferent sensory nerves
c. Excess secretion of mucous and congestion by d. Positive skin testing
the sympathetic system e. Altered mucociliary clearance
d. Excess secretion of mucous and congestion by
the parasympathetic system
e. Inhibition of G-fibers

3. Rhinitis of pregnancy may be related to


increased vascular volume and vascular relax-
ation due to what hormone?
a. Estrogen
b. Progesterone
c. Placental growth factor
d. Human chorionic gonadotropin
e. Luteinizing hormone

53
CHAPTER FORTY FOUR

T H E N A S A L SEPTUM

During which week of embryonic development 4. Which two operative maneuvers during septo-
does the nasobuccal membrane rupture and by plasty decrease the risk of permanent perfora-
doing so allow communication of the nasal pas- tion?
sage and the nasopharynx?
5. To straighten the significantly bowed septum,
Describe the most accurate method(s) for assess- which operative maneuver is paramount i n this
ment of the nasal valve angle. effort?

Which septoplasty incision is the best for


addressing the deviated caudal septum and least
likely to disrupt tip support mechanisms?

54
C H A P T E R F O R T Y FIVE

RHINOPLASTY

The proper favorable tissue plane i n which dis- 4. The complete transfixion incision
section should be carried out when uncovering a. Promotes cephalic rotation
the nose is located b. Corrects a caudal septal deflection
a. Immediately subcutaneous c. Helps to narrow a wide tip
b. Within the superficial musculoaponeurotic tis- d. Is always combined with an intercartilaginous
sue layer incision
c. Beneath the periosteum e. Results i n tip retroprojection
d. Between the SMAS layer and the cartilaginous
structure of the nose 5. Which of the following is not true about microos-
e. I n the fatty tissue plane beneath the dermis teotomies?
a. Less trauma results from the use of 2- or
Wide dome angles and a broad interdomal dis- 3-mm osteotomies.
tance should be narrowed and refined during tip b. Microosteotomes should be used only for per-
rhinoplasty by cutaneous osteotomies.
a. Transdomal sutures and/or single dome c. Some periosteum is left intact after microost-
sutures eotomies.
b. Division of the dome angles with suture-repair d. Microosteotomes do not require guards.
c. Interdomal sutures only e. Lateral osteotomies with microosteotomes
d. Resection of the interdomal intermediate should begin at or just above the junction of
crura the inferior concha with the lateral nasal wall.
e. Removal of the upper half of the lateral crura

Preoperative evaluation of the surgical needs of


the nasal tip should include all but which of the
following
a. Need for cephalic rotation
b. Need for nasal spine reduction
c. Need for narrowing of the dome angle
d. Need for projection change
e. Need for interdomal distance change

55
C H A P T E R F O R T Y SIX

SPECIAL R H I N O P L A S T Y T E C H N I Q U E S

1. If a patient requires osteotomies, the order i n 4. Wier incisions are used to


which they should be performed is a. Narrow a wide alar base
a. Lateral, intermediate, medial b. Increase tip projection
b. Medial, intermediate, lateral c. Lengthen the columellar skin
c. Lateral, medial, intermediate d. Decrease the convexity of the lateral ala
d. Medial, lateral, intermediate e. Augment the nasal dorsum
e. Intermediate, medial, lateral
5. Anatomic characteristics of the unilateral cleft
2. I n the twisted nose, a C-shaped deformity can be lip-nasal deformity include
corrected by the use of a. Cleft ala displaced laterally, inferiorly, and
a. A spreader graft between the upper lateral car- posteriorly
tilage and the septum on the concave side b. Dislocation of the caudal end of septum
b. A spreader graft between the upper lateral car- toward the noncleft side
tilage and the septum on the convex side c. Tip deflected toward the noncleft side
c. Placement of an onlay graft on the concave d. Bony deficiency of the maxilla on the cleft
side side
d. a and b e. A l l of the above
e. a and c

3. In rhinoplasty, the DART graft is used to


a. Camouflage columellar asymmetries
b. Increase tip rotation and projection
c. Correct an acute nasolabial angle
d. Correct a shallow nasofrontal angle
e. Derotate the nose and increase tip projection

56
CHAPTER FORTY SEVEN

REVISION RHINOPLASTY

A patient is seen with supra-alar pinching and 4. When harvesting costal cartilage, the ribs most
alar retraction. A common cause of this defor- commonly used are the
mity is a. First and second
a. Inadequate osteotomies b. Third and fourth
b. Over-resected lower lateral crura c. Fifth and sixth
c. Improper graft placement d. Seventh and eighth
d. Excessive hump removal e. Eleventh and twelfth
e. Avulsion of the upper lateral cartilages
5. A n open roof deformity can occur after
The base view provides information about all of a. Bony hump removal with inadequate
the following except osteotomies
a. Shape of columella b. Excessive resection of lower lateral crura
b. Size of columella c. Avulsion of the upper lateral cartilages
c. Alar base d. Excessive soft tissue i n the supratip
d. Radix e. Over-resection of septal cartilage
e. Lobule

The nasolabial angle i n men should be


a. 65 to 75 degrees
b. 75 to 85 degrees
c. 90 to 95 degrees
d. 100 to 110 degrees
e. 110 to 120 degrees

57
CHAPTER FORTY E I G H T

RECONSTRUCTIVE RHINOPLASTY

1. I n selecting the donor site for a full-thickness 4. Which of the following statements regarding cal-
skin graft of a nasal tip defect, which of the fol- varial bone grafting is false?
lowing areas of skin matches most closely the a. Grafts are usually harvested from the parietal
thickness, color, and texture of nasal skin? region.
a. Nasolabial b. Grafts can be harvested w i t h lower donor site
b. Supraclavicular morbidity.
c. Postauricular c. Grafts are usually harvested from the outer
d. Upper eyelid cortex of the cranium.
e. Thigh d. Grafts resist resorption because of their endo-
chondral origin.
2. Which of the following statements regarding the
forehead flap is false? 5. Which of the following is the material of choice
a. The forehead flap is primarily based on the in reconstructing the nasal dorsum?
supraorbital artery. a. Calvarial bone
b. The forehead flap is an axial flap. b. Iliac bone
c. The donor site can usually be closed primarily c. Rib
when the defect is less than 3.5 cm wide. d. Irradiated cartilage
d. The forehead flap is elevated i n the supra- e. Alloplastic implants
galeal plane.
e. The distal end of the flap can usually be
thinned without injuring the pedicles.

3. Which of the following statements regarding


composite grafting is false?
a. Graft size is critical to survival.
b. Survival depends on transfer of fluids and
nutrients from graft edges immediately after
placement.
c. The auricular donor site is associated with
minimal morbidity.
d. Composite grafting should not be considered
when a graft larger than 1 cm is required.

58
PART F O U R

PARANASAL SINUSES
CHAPTER FORTY NINE

R A D I O L O G Y OF T H E N A S A L C A V I T Y
A N D PARANASAL SINUSES

1. Which of the following is false with regard to the 4. Radiographic signs of chronic sinusitis include
agger nasi cell? all of the following except
a. I t is an ethmoturbinal remnant. a. Drainage of intermediate attenuation on GT
b. I t is present i n about half of patients. b. Thickening of the bony walls
c. I t is usually aerated. c. Opacification of the middle meatus
d. Its roof usually borders the ostium or floor of d. Mucoperiosteal thickening of the maxillary
the frontal sinus. sinus
e. Its size directly influences the size of the e. Hyperintense drainage on Tl-weighted MRI
frontal sinus drainage tract. images

2. Which of the following is false with regard to the 5. Which of the following radiographic findings is
uncinate process? believed to be associated with inflammatory
a. I t is part of the ethmoid bone. sinusitis?
b. I t is part of the lateral nasal wall. a. Haller cells
c. I t contacts the agger nasi cell. b. Uncinate pneumatization
d. I t defines the infundibulum. c. Horizontal orientation of the uncinate process
e. I t is lateral to the hiatus semilunaris. d. Paradoxic turbinates
e. Concha bullosa
3. The sinus lateralis is the
a. Space between the uncinate and the ethmoid
bulla
b. Space between the posterior ethmoid and the
basal lamella
c. Space between the ethmoid bulla and the
basal lamella
d. Space between the agger nasi cell and the
middle turbinate when the turbinate inserts
on the cribriform plate

61
C H A P T E R FIFTY

I N F E C T I O U S CAUSES
OF R H I N O S I N U S I T I S

1. What fungus is most commonly responsible for 4. The most common bacterial causes of acute r h i -
invasive fungal sinusitis i n uncontrolled diabet- nosinusitis are
ics? a. Staphylococcus aureus and anaerobes
a. Rhizopus oryzae (mucormycosis) b. Streptococcus viridans, S. aureus, and anaer-
b. Aspergillus fumigatus obes
c. Aspergillus flavus c. Haemophilus influenza and Streptococcus
d. Candida albicans pneumoniae
e. Alternaria d. Pseudomonas and coagulase-negative staphy-
lococci
2. The patient with a sinus computed tomography
showing mucosal thickening occluding the 4. Complications of bacterial sinusitis include all of
osteomeatal complex the following except
a. Requires endoscopic sinus surgery a. Pseudotumor cerebri
b. May have a cold, a bacterial sinus infection, b. Subperiosteal orbital abscess
nasal polyps, or an irreversible obstruction of c. Cavernous sinus thrombosis
the maxillary infundibulum d. Epidural abscess
c. Should be treated w i t h a broad-spectrum
antibiotic
d. Requires a culture-directed antibiotic and
nasal steroid sprays

3. The histologic findings i n allergic fungal sinusitis


are
a. Allergic (eosinophilic) mucin with hyphae
present
b. Fungal invasion of the epithelium but not of
deeper structures
c. A tangled mass of hyphae
d. Eosinophilic mucin with Charcot-Leyden crys-
tals but no hyphae present

62
C H A P T E R FIFTY O N E

NEOPLASMS

Ohngren's line is an important delineator of 4. The imaging study most likely to provide accu-
prognosis i n the management of carcinoma of rate information regarding tumor extension
the maxillary sinus. This imaginary line is intracranially or intraorbitally is
described as a plane created by the intersection a. Ultrasonography
of a line drawn between b. Computed tomography scan
a. The tip of the nose and the tragus c. Magnetic resonance imaging
b. The medial can thus and the angle of the jaw d. Positron emission tomography scan
c. The nasal tip and the angle of the jaw
5. Which of the following is the most helpful i n
A maxillary sinus carcinoma involving the infe- controlling postoperative cerebrospinal fluid
rior lateral superior and medial walls of the max- leakage?
illary sinus and the anterior ethmoid would be a. Antibiotics
classified b. Postoperative radiation
a. T, c. Tissue glue
b. T2 d. Skull base reconstruction with soft tissue or
c. T3 bone
d. T ,

The first-echelon nodal drainage for tumors of


the nasal space and maxillary sinus is
a. Lateral retropharyngeal node
b. Facial node
c. Parotid node
d. Jugulodigastric node

63
C H A P T E R FIFTY T W O

MEDICAL MANAGEMENT
OF N A S O S I N U S I N F E C T I O U S
A N D I N F L A M M A T O R Y DISEASE

In the treatment of allergic rhinitis, cromolyn 4. Children may have a physiologic immunodefi-
preparations ciency predisposing them to chronic rhinosinusi-
a. Should be used after an attack occurs tis until what age?
b. Are effective for nasal congestion a. 6
c. Effectively treat sneezing and rhinorrhea b. 8
d. Should not be used with antihistamines c. 10
e. Have long duration of action d. 12
e. 14
All of the following are true about nasal steroids
except 5. Patients with allergic fungal sinusitis have all of
a. Are contraindicated i n infectious sinusitis. the following except
b. More potent nasal steroids like fluticasone a. Allergic mucin
may have some pituitary suppression. b. Nasal polyps
c. Should be used cautiously i n elderly patients c. Atopy
taking inhaled steroids. d. Fungal allergies
d. Septal perforation is a rare side effect. e. Immunodeficiency
e. Are effective against acute and late-phase
effects.

Which of the following about macrolide antibi-


otics is true?
a. Chronic use often leads to resistance.
b. Are recommended primarily for acute
sinusitis.
c. Should not be used i n conjunction with nasal
steroids.
d. Have important antiinflammatory effects
e. Have no role i n treating sinusitis.

64
CHAPTER FIFTY T H R E E

P R I M A R Y SINUS SURGERY

1. Anatomic variations of the paranasal sinuses that 4. The most common minor complication after
may predispose a surgeon to inadvertent pene- endoscopic sinus surgery is
tration of the orbit or the anterior cranial cavity a. Headache
include b. Hyposmia
a. Lamina papyracea lying medial to the maxil- c. Synechia
lary ostium d. Periorbital ecchymosis
b. Maxillary sinus hypoplasia e. Periorbital emphysema
c. Fovea ethmoidalis abnormalities such as low
or sloping fovea 5. The uncinate process can have all of the follow-
d. Lamina papyracea dehiscence ing superior attachments except
e. A l l of the above a. Superior turbinate
b. Lamina papyracea
2. Absolute indications for endoscopic sinus sur- c. Skull base
gery include all of the following except d. Middle turbinate
a. Mucoceles e. None of the above
b. Headaches
c. GSF rhinorrhea
d. Complications of rhinosinusitis
e. Tumors

3. Dividing the infundibulum into thirds, the natu-


ral ostium to the maxillary sinus will most com-
monly be found where?
a. Superior third
b. Middle third
c. Inferior third
d. Outside infundibulum
e. None of the above

65
C H A P T E R FIFTY F O U R

R E V I S I O N E N D O S C O P I C SINUS
SURGERY

1. Which of the following is not necessary to make 4. Important considerations i n preoperative man-
the diagnosis of chronic rhinosinusitis? agement of the chronic rhinosinusitis patient
a. CT scans with mucosal thickening include
b. Culture result for resistant bacteria a. Complete delineation of host and environmen-
c. History with length of symptoms greater than tal factors
3 months b. Judicious use of antiinflammatories, especially
d. Characteristic findings on nasal endoscopy corticosteroid nasal sprays and oral corticoste-
riods before surgery
2. Which of the following is not a common reason c. Empiric antimicrobials, especially broad-spec-
for failure of maxillary antrostomy? trum antibiotics to cover gram-negative organ-
a. Missed ostium sequence/recirculation phe- isms and anaerobes
nomenon d. Comprehensive history, including the symp-
b. Infected secretions dropping into maxillary toms that brought the patient to a primary
sinus "catch basin" surgery
c. Scarring of nasolacrimal duct from antrostomy
that is too anterior 5. MRI of the sinuses should be obtained i n all
d. Scarred over maxillary sinus ostium these situations except
e. Retained foreign body i n the sinus a. Tumor
b. Opacification against skull base
3. Important landmarks i n revision sinus surgery c. Opacified sphenoid sinus
include all the following except d. Dehiscent bone along skull base
a. Lateral nasal wall/lamina papyracea e. To evaluate mucosal disease
b. Posterior wall maxillary sinus
c. Superior turbinate
d. Anterior wall maxillary sinus

66
C H A P T E R F I F T Y FIVE

C E R E B R O S P I N A L F L U I D (CSF)
RHINORRHEA

1. Which of the following statements are false? 4. Which of the following statements are true?
a. The best classification system for CSF catego- a. Most instances of CSF rhinorrhea caused by
rizes CSF rhinorrhea as traumatic or nontrau- closed-head trauma resolve with conservative
matic. management.
b. Causes of traumatic CSF rhinorrhea include b. Most instances of nontraumatic CSF rhinor-
head injury and sinus surgery. rhea require operative repair.
c. Nontraumatic CSF rhinorrhea may also be c. Endoscopic repair of CSF rhinorrhea has
more appropriately termed "spontaneous" or emerged as the preferred method for surgical
"idiopathic" CSF rhinorrhea. closure of skull base defects when operative
d. Elevated intracranial pressure may occur i n closure is indicated.
nontraumatic CSF rhinorrhea. d. Only pedicled mucosal flaps can be reliably
e. Elevated intracranial pressure may result from used to reconstruct the site of a CSF leak.
intracranial tumors or abnormalities i n CSF e. A l l of the above
resorption and circulation.
5. I n the management of CSF rhinorrhea,
2. Which of the following statements are true? a. Prophylactic antibiotics should be routinely
a. All patients w i t h nontraumatic CSF rhinorrhea used.
are likely to have benign intracranial pressure. b. (3 -Transferrin testing and high-resolution
2

b. Nontraumatic CSF rhinorrhea has been associ- computed tomography may eliminate the need
ated with the empty sella syndrome and for CSF tracer studies.
benign intracranial pressure. c. Radionuclide tracer studies provide a sensitive
c. A n empty sella on MRI represents low and specific method for confirming and local-
intracranial pressure. izing a CSF leak.
d. Most nontraumatic CSF leaks occur i n d. Magnetic resonance cisternography requires
healthy, thin young men. the administration of intrathecal contrast.
e. None of the above e. CT cisternography can reliably identify more
than 95% of CSF leaks.
3. The p -transferrin assay
2

a. Serves as a specific marker for CSF


b. Requires large quantities of sample for reliable
result
c. Requires the administration of intrathecal
contrast
d. Provides information about the location of the
skull base defect
e. None of the above

67
PART FIVE

SALIVARY G L A N D S
C H A P T E R F I F T Y SIX

P H Y S I O L O G Y OF T H E SALIVARY G L A N D S

1. Which of the following statements regarding sali- 4. Which of the following statements regarding sali-
vary secretion is true? vary flow rates is not true?
a. The average daily volume of saliva produced is a. They are reduced during sleep.
between 500 and 750 mL. b. They are increased during exercise.
b. Sixty to seventy percent of the total daily c. They are increased by mastication.
saliva is produced by the parotid glands. d. They are increased before an episode of vomit-
c. Minor salivary glands are responsible for 20% ing
of the daily saliva produced. e. They steadily increase as the child grows and
d. Most of the unstimulated saliva is secreted by reach a maximum value by the age of 3 to 4
the submandibular gland. years.
e. Hyposalivation is defined as an unstimulated
salivary flow of less than 0.5 mL/min. 5. A l l of the following statements are true except
a. IgG is the predominant immunoglobulin.
2. Which of the following statements regarding the b. The relationship between IgA and the forma-
structure of a normal salivary gland secretory tion of dental plaque is unknown.
unit is not true? c. IgA i n saliva is i n the form of a dimer.
a. Salivary acini are classified as serous, muci- d. Lactoferrin scavenges free iron i n fluids and
nous, and mixed. inflamed areas so as to suppress free radi-
b. Serous cells are filled w i t h basophilic secretary cal-mediated damage and decrease the avail-
granules. ability of the metal to invading microbial and
c. Acini lead to intercalated ducts lined by a sin- neoplastic cells.
gle layer of cuboidal cells.
d. The intralobular ducts are commonly known
as striated ducts.
e. Myoepithelial cells are not seen i n normal sali-
vary gland acini.

3. Which of the following statements is not true


regarding salivary secretion?
a. I t is a two-stage process composed of primary
acinar secretions and ductal secretions.
b. I t is increased by the stimulation of the
parasympathetic nervous system.
c. I t is believed to occur by way of osmotic cou-
pling of transepithelial fluxes of sodium chlo-
ride and water.
d. I i is not dependent on the activity of Na -K -
+ +

ATPase.
e. Sympathetic stimulation leads to high fluid
output.

71
C H A P T E R FIFTY SEVEN

DIAGNOSTIC IMAGING
A N D FINE-NEEDLE ASPIRATION
OF T H E SALIVARY G L A N D S

1. The percentage of calculi i n the submandibular 4. Computed tomography scans are the best imag-
gland is ing study for detection of
a. 20% a. Neoplasms
b. 40% b. Calculi
c. 50% c. Abscesses
d. 60% d. Chronic inflammatory disease
e. 80% e. Parapharyngeal masses

2. The most common tumor to scan positively with 5. Magnetic resonance imaging is most useful i n
technetium is evaluating
a. Pleomorphic adenoma a. Parotid abscesses
b. Oncocytoma b. Calculi
c. Warthin's tumor c. Intrinsic versus extrinsic parotid masses
d. Mucoepidermoid carcinoma d. Malignant neoplasms at the skull base
e. Malignant mixed tumor e. Chronic inflammatory disease

3. Ultrasonography can be used to


a. Separate intrinsic from extrinsic parotid
masses
b. Separate super facial lobe from deep lobe
parotid masses
c. Distinguish facial nerve from tumor
d. Separate deep lobe from parapharyngeal
masses
e. Show where the marginal branch of the facial
nerve crosses the retromandibular vein

72
CHAPTER FIFTY E I G H T

I N F L A M M A T O R Y DISORDERS
OF T H E SALIVARY G L A N D S

1. Which salivary gland is most commonly affected 4. I n most cases involving cat-scratch disease, the
in acute suppurative sialadenitis? treatment includes
a. Submandibular gland a. Clindamycin
b. Parotid gland b. Augmentin
c. Sublingual gland c. No treatment
d. Minor salivary glands d. Doxycycline

2. Which would be the best antimicrobial for the 5. Which autoantibodies are tested for the diagnosis
treatment of acute suppurative sialadenitis? of Sjogren's syndrome?
a. Erythromycin a. Antimitochondrial antibodies
b. Tetracycline b. Rheumatoid factor
c. P-Lactamase-resistant penicillin c. Antimierosomal antibodies
d. Fluoroquinolone d. SS-A and SS-b autoantibodies

3. Which factors predispose the submandibular


gland to sialolithiasis?
a. Smaller-sized duct
b. Viscosity of saliva from the submandibular
gland
c. Larger-sized duct
d. Weight of the duct
e. b and c
f. b and d

73
CHAPTER FIFTY NINE

T R A U M A OF T H E SALIVARY G L A N D S

1. I n extensive proximal parotid duct injury, appro- 4. Which of the following is an advantage of using
priate management includes the great auricular nerve for facial nerve graft-
a. Duct ligation ing?
b. Superficial parotidectomy a. Minimal donor site morbidity
c. Pressure dressings b. Location within the operating field
d. Primary anastomosis c. Width
e. Observation d. Length
e. Branching pattern
2. Which statement about the use of electromyogra-
phy i n managing facial nerve injuries is true? 5. Treating parotid fistulas or sialoceles may
a. I t is most helpful w i t h i n the first 2 weeks of include all of the following except
injury. a. Repeat aspiration
b. I t has no role. b. Compression
c. I t provides prognostic information 3 weeks c. Tympanic neurectomy
after injury. d. Parotidectomy
d. I t has supplanted electroneuronography as a e. Ligating the chorda tympani
prognostic tool.
e. I t should not be used before 6 weeks after
injury.

3. Which statement about facial nerve injury is


true?
a. Repair should be delayed 7 days.
b. Best results are obtained when repair occurs
after 14 days.
c. I t should be performed on injuries medial to
the lateral can thus.
d. Best results are obtained the sooner repair is
performed.
e. Branches may be stimulated up to 96 hours
after injury.

74
C H A P T E R SIXTY

B E N I G N NEOPLASMS
OF T H E SALIVARY G L A N D S

1. Warthin's tumors are thought to arise from which 4. Which of the following is true of fine-needle aspi-
of the following cell types? ration (FNA) i n salivary gland neoplasms?
a. Acinar cells a. The sensitivity of FNA is <85%.
b. Intercalated duct cells b. The specificity of FNA is <95%.
c. Striated duct cells c. FNA rarely results i n a change i n manage-
d. Excretory duct cells ment.
e. Myoepithelial cells d. I t can be difficult to differentiate oncocytic
and adenoid cystic neoplasms by FNA.
2. Which of the following is not associated with the e. I t can be difficult to distinguish mucoepider-
development of salivary gland malignancy? moid carcinoma and sialolithiasis by FNA.
a. Smoking
b. Ionizing radiation 5. Which of the following is false with regard to sali-
c. Silica dust vary neoplasms?
d. Nitrosamines a. The presence of lymphoid tissue differentiates
e. Nulliparity Warthin's tumors from oncocytomas.
b. Pleomorphic adenomas may metastasize to
3. Which of the following statements is false? lymph nodes.
a. Pleomorphic adenomas are the most common c. Warthin's tumors may metastasize to lymph
salivary gland neoplasms. nodes.
b. Pleomorphic adenomas are the most common d. Pleomorphic adenomas are always well encap-
parotid deep lobe neoplasm. sulated.
c. Pleomorphic adenomas are the most common e. Arteriovenous fistulas typically result from
submandibular gland neoplasms. trauma
d. Pleomorphic adenomas are the most common
minor salivary gland neoplasms.
e. Papillary cystadenoma lymphomatosum is the
second most common benign salivary lesion.

75
C H A P T E R SIXTY O N E

M A L I G N A N T NEOPLASMS
OF T H E SALIVARY G L A N D S

Acinic cell carcinoma occurs most commonly i n 4. The most common site of distant failure i n
which gland? patients with parotid malignancy is
a. Parotid gland a. Brain
b. Submandibular gland b. Bone
c. Sublingual gland c. Lungs
d. Minor salivary glands d. Liver
e. None of the above e. Neck

The most common parotid malignancy is 5. Which statement regarding postoperative radia-
a. Mucoepidermoid carcinoma tion is true?
b. Adenoid cystic carcinoma a. Postoperative radiation is given for any parotid
c. Acinic cell carcinoma malignancy.
d. Squamous cell carcinoma b. Postoperative radiation improves overall sur-
e. Polymorphous low-grade adenocarcinoma vival and regional control.
c. Postoperative radiation is only given for posi-
Which tumor should routinely receive elective tive margins and unresectable disease.
neck dissection? d. Postoperative radiation improves regional con-
a. Polymorphous low-grade adenocarcinoma trol i n advanced stage tumors.
b. Adenoid cystic carcinoma e. Because neutron therapy has an improved
c. Acinic cell carcinoma radiobiologic effect on salivary cancers, i t has
d. Adenocarcinoma become the standard technique for postopera-
e. Lymphoma tive radiation.

76
PART SIX

ORAL C A V I T Y / P H A R Y N X /
ESOPHAGUS
C H A P T E R SIXTY T W O

P H Y S I O L O G Y OF T H E O R A L C A V I T Y

Which of the following statements is false? 3. The initiating cause of the dentinal hypersensi-
a. The lingual nerve is sensitive to chemical tivity caused by exposure of dentinal tubules,
stimulation of the tongue. which occurs, for example, with a cracked tooth
b. Loss of periodontal mechanoreceptors does or a cavity, can best be explained by
not eliminate intradental discrimination. a. The "hydrostatic" theory
c. Subnucleus caudalis is the only brain stem b. The "hydrodynamie" theory
trigeminal nucleus that mediates pain. c. Central sensitization
d. Stimulation of the hypoglossal nerve can d. Release of neuropeptides into the tooth pulp
result i n reflex action i n the trigeminal sys- e. Chemesthesia
tem.
e. The chorda tympani innervates fungiform 4. Gustatory transduction may involve
papillae on the front of the tongue. a. Direct entry of a stimulus into the receptor
cell
Which of the following is true? b. Activation of G-protein-coupled receptors
a. A jaw-opening reflex is mediated by muscle c. Changes i n the intracellular pH
spindle afferents i n the anterior digastric mus- d. A l l of the above
cle.
b. The jaw-closing reflex is a disynaptic reflex 5. The loss of the chorda tympani nerve after m i d -
through the spinal trigeminal complex. dle ear surgery i n humans
c. Cephalic-phase insulin release can be initiated a. Results i n loss of taste sensation from the back
by gustatory stimuli. of the mouth
d. The masticatory r h y t h m is generated by alter- b. Requires precise psychophysical procedures to
nating jaw-opening and jaw-closing reflexes. demonstrate any loss of function
e. Stimulation of the lingual and glossopharyn- c. Influences the sensation of thirst
geal nerves results primarily i n a protrusive d. Results i n a profound disruption i n salt intake
movement of the tongue. e. None of the above

79
C H A P T E R SIXTY T H R E E

M E C H A N I S M S OF N O R M A L
A N D ABNORMAL SWALLOWING

Two of the stages of swallow are under voluntary 4. Your patient has a suspected oral and tongue
control. They are base disorder. Which of the following assess-
a. Oral and pharyngeal ments do you recommend?
b. Pharyngeal and esophageal a. Scintigraphy
c. Oral and oral preparation b. Scintigraphy and endoscopy
d. Oral preparatory and pharyngeal c. Manometry and endoscopy
e. Oral and esophageal d. Videofluoroscopy and ultrasonography
e. Videofluoroscopy and endoscopy
Your patient has a suspected pharyngeal stage
swallowing disorder after chemoradiation. What 5. Patients who have undergone supraglottic laryn-
is your suggestion for an assessment to deter- gectomy may exhibit
mine the management plan? a. Reduced cricopharyngeal opening
a. Videofluoroscopy b. Reduced laryngeal elevation, laryngeal closure,
b. Manometry and pharyngeal contraction
c. Endoscopy c. Reduced tongue control
d. Scintigraphy d. Reduced cricopharyngeal opening and reduced
e. Ultrasonography laryngeal closure
e. Reduced laryngeal elevation, laryngeal closure,
You have a patient who aspirates the minute liq- and reduced tongue base movement
uid enters his mouth. You suspect two possible
physiologic reasons for the aspiration. What are
they?
a. Delayed pharyngeal swallow and reduced con-
traction of the pharyngeal constrictors
b. Delayed pharyngeal swallow and reduced air-
way closure
c. Delayed pharyngeal swallow and reduced con-
trol of the tongue
d. Reduced airway closure and reduced cricopha-
ryngeal opening
e. Reduced laryngeal closure and delayed pha-
ryngeal swallow

80
C H A P T E R SIXTY F O U R

O R A L M U C O S A L LESIONS

Aphthous ulceration of the oral mucosa is often 4. The entity, proliferative verrucous leukoplakia,
mistaken for recurrent intraoral herpes simplex may be separable from the common form of
infection but is distinguishable on the basis of all leukoplakia by virtue of its
of the following except a. Location
a. Location b. Relationship to smokeless tobacco
b. Vesicular phase c. High rate of cancer development
c. Viral cytopathic effect d. Relationship to use of certain mouthwashes
d. Duration
5. Oral lichen planus may present as a desquama-
Oral mucosal melanoma is not generally thought tive process involving the attached gingiva i n a
to parallel its cutaneous counterpart i n terms of manner similar to which of the following dis-
discrete preinvasive categories but may be best eases/conditions ?
considered to parallel which type of melanoma a. Contact mucositis
from a precursor lesion standpoint? b. Mucosal pemphigoid
a. Thin melanoma c. Nutritional deficiencies (vitamin C)
b. Acral lentiginous melanoma d. Leukemic infiltrate
c. Plantar melanoma
d. Cellular blue nevus

A diagnosis of "leukoplakia" effectively rules out


all of the following except
a. Lichen planus
b. White sponge nevus
c. Leukoedema
d. Epithelial dysplasia

81
C H A P T E R SIXTY FIVE

ORAL MANIFESTATIONS
OF SYSTEMIC DISEASE

Which of the following classifications of medica- 4. Antibiotic prophylaxis before dentoalveolar sur-
tions does not cause salivary hypofunction? gery is absolutely required for which of the fol-
a. Tricyclic antidepressants lowing conditions?
b. Antihistamines a. Pin placement after femur fracture
c. Gox-2 nonsteroidal antiinflammatories b. Mitral valve prolapse with regurgitation
d. Diuretics c. Three months after a GVA
d. Indwelling cardiac pacemaker
Which of the following diseases is most likely to
have oral mucocutaneous ulcers? 5. Squamous cell carcinoma of the tongue may
a. Pemphigus vulgaris manifest the following oral sequelae except
b. Parkinson's disease a. Nonhealing oral ulcer
c. Renal osteodystrophy b. Erythroplakic lesion
d. Down syndrome c. Exophytic erythroleukoplakic pustule
d. Mucocele of the lower lip
Oral manifestations of bleeding disorders include
all of the following except
a. Sublingual ecchymotic lesions
b. Hard palate petechiae
c. Fungiform papillae
d. Gingival hemorrhage

82
C H A P T E R S I X T Y SIX

ODONTOGENESIS A N D O D O N T O G E N I C
CYSTS A N D T U M O R S

1. Which is a radiographic feature often found i n 4. On a panoramic radiograph taken to evaluate


benign odontogenic cysts and tumors? a patient with a suspected mandible fracture, a
a. Radiopaque border molar with a large amalgam restoration and a
b. Blunted tooth roots 0.5-cm diameter radiolucent lesion associated
c. Cortical expansion with the apex of the root is seen. What is the
d. A l l of the above best course of action?
a. Dental evaluation and root canal
2. A patient with multiple odontogenic keratocysts b. Extraction of the tooth
should be evaluated for Gorlin's syndrome. This c. Incisional biopsy
diagnosis is important because d. Enucleation of the lesion
a. These patients must have genetic counseling.
b. These cysts become malignant. 5. After removal of a 2 x 2-cm radiolucent lesion of
c. These patients often have basal cell carcinoma the mandible, the hospital pathology report indi-
develop i n noncharacteristic areas. cates a diagnosis of "benign odontogenic cyst."
d. Keratocysts in these patients are more likely What is the next course of action?
to recur. a. Frequent radiographic evaluation for recur-
rence.
3. The acceptable surgical excision of a solid b. No other treatment is needed, because simple
ameloblastoma without cortical bone perfora- removal of this type of cyst is adequate.
tion is c. Histopathologic evaluation by an oral patholo-
a. Simple enucleation gist.
b. Enucleation and curettage
c. Excision with bony margins of 1 cm and
preservation of adjacent soft tissue
d. Composite resection of bone and soft tissue
with 1-cm margins

83
C H A P T E R SIXTY S E V E N

O D O N T O G E N I C INFECTIONS

1. Which of the following best describes a typical 4. If required i n the surgical management of
odontogenic infection? osteomyelitis of the jaws, skeletal stabilization is
a. Exclusively aerobic bacteria best accomplished by
b. Exclusively anaerobic bacteria a. External bandaging
c. Mixed aerobic and anaerobic bacteria b. External skeletal fixation
d. Nosocomial bacteria c. Internal skeletal fixation
e. Fungi and viruses d. Wire fixation

2. Which of the following signs and symptoms is 5. Diffuse sclerosing osteomyelitis of the facial
most commonly associated with odontogenic skeleton demands
infections? a. Long-term antibiotic maintenance and surgical
a. Constipation intervention as necessary
b. Diarrhea b. Treatment similar to florid osseous dysplasia
c. Productive cough c. Consideration for osteoradionecrosis as a
d. Mental confusion comorbidity
e. Facial swelling d. Comparison with primary chronic
osteomyelitis of childhood.
3. Which of the following fascial spaces is not p r i -
marily involved i n Ludwig's angina?
a. Submandibular space
b. Submental space
c. Sublingual space
d. Lateral pharyngeal space

84
C H A P T E R SIXTY E I G H T

TEMPOROMANDIBULAR JOINT
DISORDERS

1. Which of the following statements regarding the 4. The preferred treatment of a patient with an
changes produced i n the mandible by unilateral anteriorly displaced, nonreducing disk is
condylar hypoplasia is incorrect? a. The use of nonsteroidal anti-inflammatory
a. The mandibular body on the affected side is drugs and a bite appliance
shorter than the contralateral side. b. Surgical repositioning of the disk (discoplasty)
b. The chin is deviated to the affected side. c. Doing an arthrocentesis
c. There is decreased antegonial notching. d. Arthroscopic repositioning of the disk
d. The unaffected side is long and flat. f. Surgical removal of the disk (diskectomy)
e. The face appears fuller on the affected side.
5. Which of the following are the most important
2. Which of the following is not an indication for factors involved i n the cause of myofascial pain
open reduction and fixation of fractures of the dysfunction syndrome?
mandibular condyloid process? a. Muscular overextension
a. The presence of bilateral fractures i n a dentate b. Chronic clenching and grinding of the teeth
patient c. Psychological stress
b. The presence of bilateral fractures i n an eden- d. Malocclusion of the teeth
tulous patient e. Muscular overcontraction
c. The presence of interference of the fracture
with jaw movement
d. The presence of an intracapsular fracture
e. The presence of associated fractures of the
mandible

3. Which of the following is the least important


consideration i n the surgical management of
ankylosis of the temporomandibular joint?
a. The age of the patient
b. The amount of limitation of jaw movement
c. The use of postoperative physical therapy
d. The prevention of fusion of the new joint
e. The etiology of the problem

85
C H A P T E R SIXTY N I N E

BENIGN TUMORS A N D TUMOR-LIKE


LESIONS OF T H E O R A L C A V I T Y

1. Optimal treatment of nasoalveolar cysts requires 4. Which of the following lesions should be followed
a. Marsupialization closely for resolution?
b. Marsupialization w i t h curettage a. Pseudoepitheliomatous hyperplasia
c. Conservative surgical excision b. Pyogenic granuloma
d. Complete surgical excision c. Granular cell tumor
e. Complete surgical excision followed by radia- d. Lingual thyroid
tion therapy e. Necrotizing sialometaplasia

2. Which of the following lesions grows i n response 5. A woman is seen for denture placement w i t h a
to local trauma? bony lesion on the lingual surface of the
a. Nasoalveolar cyst mandible. Panorex imaging demonstrates this
b. Mandibular torus lesion to be multilobular with expansion of corti-
c. Fibroma cal bone. The best option for treatment of this
d. Choristoma lesion is
e. Parulis a. Curettage of the lesion with resurfacing of the
mandible for denture placement
3. Which of the following lesions is most easily mis- b. Simple excision of the lesion
taken for malignancy? c. Complete excision of the lesion
a. Fibrous histiocytoma d. Marginal mandibulectomy with 1-cm margins
b. Necrotizing sialometaplasia e. Segmental mandibulectomy with 2-cm mar-
c. Granular cell tumor gins and fibula free flap placement followed by
d. Squamous papilloma radiation therapy
e. Pyogenic granuloma

86
CHAPTER SEVENTY

M A L I G N A N T NEOPLASMS OF T H E O R A L
CAVITY

In patients with an oral tongue squamous cell 4. When considering elective treatment of the neck
carcinoma and a depth of invasion of 4 m m in a patient with a T lateral tongue squamous
3

a. Elective treatment of the neck is not neces- cell carcinoma


sary. a. A level I to IV neck dissection is advocated
b. The risk of perineural invasion within the p r i - because of the potential for skip metastases.
mary is significantly increased. b. Postoperative radiation therapy can include
c. Elective treatment of the N neck is advo-
0 the ipsilateral neck.
cated, because the risk of occult metastasis c. Radical neck dissection is required because of
exceeds 20%. the advanced T stage.
d. Resection of the primary tumor with a m i n i - d. A "watch and wait" philosophy can be used.
mum of 2-cm margins is required. e. Sentinel node localization should be per-
e. The likelihood of bilateral neck metastases is formed to determine the need for neck dissec-
greater than 30%. tion.

A 75-year-old edentulous patient is seen with an 5. I n a patient with a T oral tongue carcinoma
3

ulcerated 4-cm retromolar trigone lesion that with extension to the floor of mouth, for whom
appears to invade the ascending ramus of the free flap reconstruction is necessary and postop-
mandible. Appropriate care includes erative radiation likely, the best approach to the
a. External beam radiation to the primary site primary is
and the ipsilateral neck a. Midline mandibulotomy
b. Transoral resection of the primary with selec- b. Lateral mandibulotomy
tive neck dissection c. Transoral resection, if possible
c. Midline mandibulotomy with resection of the d. Paramedian mandibulotomy
primary and ipsilateral neck dissection e. Use of a pull-through technique
d. Composite resection of the primary tumor
after lip-splitting incision, ipsilateral neck dis-
section, and reconstruction with local or pedi-
cled soft tissue flap
e. Resection with fibular free flap reconstruction
of the lateral mandible

The best treatment option for a 3-cm lower lip


squamous cell carcinoma extending to the oral
commissure includes
a. Resection with primary closure
b. External beam radiation
c. Resection with a Bernard von Burow flap
reconstruction
d. Free flap reconstruction
e. Resection with reconstruction by Estlander
flap

87
CHAPTER SEVENTY ONE

R E C O N S T R U C T I O N OF T H E
MANDIBLE A N D MAXILLA

1. Autogenous bone graft sources used i n oro- 4. The main reason for hardware removal after a
mandibular reconstruction include all of the fol- bone grafting procedure is to
lowing except a. Prevent extrusion
a. Galvarium b. Prevent the long-term effects of stress shield-
b. Rib ing
c. Radium c. Prevent infection
d. Scapula d. Avoid effects on dosimetry i n postoperative
e. Femur radiation therapy
e. Improve the lower facial contour
2. The key substance that alone has been found to
induce differentiation of fibroblasts and mes- 5. Which of the following statements is not a desir-
enchymal bone cells into osteoblasts is able qualifier for bone used i n reconstruction of
a. Interferon the mandible?
b. Substance P a. I t has a natural shape or easy contourability to
c. Bone morphogenic protein conform to the missing mandible.
d. Cartilage-inducing factors b. I t is of sufficient length for reliable placement
e. Osteoinductive factor of endosteal dental implants.
c. I t is well vascularized.
3. The open anteromedial approach to the ilium d. Its vascular anatomy is easily preserved while
minimizes postoperative gait disturbance contouring the graft.
because e. There are no significant functional or aes-
a. The incision is smaller. thetic deficits at the donor site following
b. Bone is harvested i n a less traumatic fashion. harvest.
c. Attachment of the gluteal muscles is main-
tained.
d. The incidence of postoperative hematoma is
lower.
e. The incidence of postoperative hypoesthesias
is lower.

88
CHAPTER SEVENTY T W O

M A X I L L O F A C I A L PROSTHETICS
F O R H E A D A N D N E C K DEFECTS

1. A maxillary surgical prosthesis for a dentate 4. Resection of tumors of the soft palate may create
patient places emphasis on preservation of a. Paralysis of levator palatini muscle
a. The alveolar ridge b. Velopharyngeal incompetence
b. The nasopalatine papilla c. Velopharyngeal insufficiency
c. The hard palate d. Hyponasal speech
d. The molar teeth e. Chronic otitis media
e. The soft palate
5. Placement of a palatal lift prosthesis
2. Mandibular discontinuity is particularly problem- a. Can be successful without pharyngeal muscle
atic for functional disabilities of mobility
a. Speech b. Aids with masticatory ability
b. Swallowing c. Decreases the oral transit time
c. Mastication d. Aids the soft palate with stimulation and clos-
d. Respiration ing off the nasopharynx
e. Lip sealing with absence of the inferior alveo- e. Should be directed to the tubercle of the sec-
lar nerve ond cervical vertebrae

3. Resection of the tongue may indicate the use of a


palatal augmentation prosthesis, which has a p r i -
mary purpose of
a. Improving speech production
b. Lowering the vault of the palate for undulation
during deglutition
c. Creating an anterior barrier for food boluses
d. Compensating for lip incompetence
e. Decreasing resonance during speech

89
CHAPTER SEVENTY THREE

BENIGN A N D M A L I G N A N T TUMORS
OF T H E N A S O P H A R Y N X

1. Which of the following statements regarding the c. The 1992 UICC/AJCC staging classification
management of juvenile nasopharyngeal angiofi- accurately distinguished T from T disease
1 2

broma are true? and shows good correlation with survival.


a. Best results are obtained by the combination d. Both Ho's and the 1992 UICC/AJCC staging
of surgery and adjuvant radiotherapy. classification take into account lateral para-
b. The main consideration to the surgical pharyngeal spread.
approach is the age of the patient. e. The 1997 UICC/AJCC staging classification
c. Endoscopic resection is ideal for all tumors as has been prognostically validated both i n Asia
it avoids facial incisions and soft tissue and and the West.
bony disruption.
d. Locally advanced tumors with intracranial 4. Which of the following statements regarding the
extension can be effectively managed with management of NPC is true?
radiotherapy alone. a. Radiotherapy alone is seldom used i n the
e. Recurrence seldom occurs if the tumor is treatment of NPC.
completely removed. b. The use of chemotherapy i n advanced disease
is supported by level I data.
2. With regard to histopathology i n nasopharyngeal c. Neck irradiation need not be given if there is
carcinoma (NPC), no clinical evidence of neck disease.
a. Keratinizing squamous cell carcinoma of the d. Both three-dimensional conformal radiation
nasopharynx (type I) is the most common therapy and intensity modulated radiation
variety globally. therapy have been conclusively shown to
b. Types I I and I I I have distinct ultras true tural improve tumor coverage, locoregional control,
features that allows for a clear distinction and long-term complications.
between the two. e. Hyperfractionation and accelerated fractiona-
c. Type I is the most sensitive to radiotherapy tion have been shown to improve local control
and provides the best local control rates. rates without increase i n toxicity.
d. Epstein-Barr virus titer seems to correlate bet-
ter with types I I and I I I than with type I . 5. Surgery i n NPC
e. Open biopsy of a neck node is the simplest a. Is sometimes used initially i n the treatment of
and most expeditious means of obtaining neck disease
pathological diagnosis. b. Is used mainly i n the treatment of residual or
recurrent disease i n the primary site
3. With regard to staging i n NPG c. Must take into account tumor extent, expo-
a. Ho's classification closely resembles the TNM sure, and control of the internal carotid artery
staging system. d. Is not usually associated with significant mor-
b. Ho's classification is the most widely used i n bidity
Asia and has withstood the test of time, e. Does not provide good local control and sur-
despite not being validated. vival i n patients with recurrent disease

90
CHAPTER SEVENTY FOUR

PHARYNGITIS I N ADULTS

1. A 24-year old, otherwise healthy man is seen 4. A 25-year-old man with a history of intravenous
with a 3-day history of sore throat, low-grade drug use is seen with a 3-day history of sore
fever, nasal stuffiness, and a nonproductive throat, lethargy, high fevers, and headaches.
cough. The oropharynx has mild erythema on Which diagnostic test should be performed to
examination. Which of the following is the most evaluate for acute retroviral syndrome?
likely diagnosis? a. Enzyme-linked immunosorbent assay (ELISA)
a. Laryngopharyngeal reflux for HIV
b. Group A-a-hemolytic Streptococcus pyogenes b. CD4 count
pharyngitis c. Western blot for HIV
c. Fungal pharyngitis d. Throat culture
d. Allergy exacerbation e. Quantitative plasma HIV-1 RNA level
e. Viral pharyngitis
5. A 19-year-old man with a history of orogenital
2. Which of the following is not part of the Gentor contact presents with a 5-day history of a pain-
scoring system for predicting the diagnosis of less ulcer i n the left tonsil. What is the most
group A-a-hemolytic streptococcal pharyngitis? likely diagnosis?
a. History of fever a. Gonococcal pharyngitis
b. Anterior cervical adenopathy b. Primary syphilis
c. Odynophagia c. Secondary syphilis
d. Tonsillar exudates d. Chlamydial pharyngitis
e. Absence of cough e. Infectious mononucleosis

3. Which antibiotic is currently considered the first


choice by the Infectious Disease Society of
America, American College of Physicians, and
American Academy of Family Physicians for
treatment of acute pharyngitis caused by group
A-a-hemolytic Streptococcus?
a. Erythromycin
b. Clindamycin
c. Tetracycline
d. Penicillin
e. Azithromycin

91
C H A P T E R S E V E N T Y FIVE

SLEEP A P N E A A N D SLEEP-DISORDERED
BREATHING

1. What is the international classification of 4. What medical conditions are associated with
obstructive sleep apnea syndrome as a sleep dis- obstructive sleep apnea syndrome?
order? a. Depression
a. Intrinsic dyssomnia b. Congestive heart failure
b. Extrinsic dyssomnia c. Gastroesophageal reflux
c. Intrinsic parasomnia d. Diabetes mellitus
d. Extrinsic parasomnia
5. Perioperative airway obstruction may be m i n i -
2. What are the characteristics of upper airway mized by the following actions?
resistance syndrome? a. Do not give premedications.
a. Apnea-hypopnea index <5. b. Ensure the patient can be masked ventilated
b. Minimal number of arousals during sleep. before paralysis.
c. The chief complaint is daytime sleepiness. c. Use nasal continuous positive airway pressure
d. One-third of patients snore. postoperatively.
d. Use a patient-controlled anesthesia device
3. What is the association of the pathophysiology of (PCA) for pain control.
obstructive sleep apnea (OSA) i n children and
adults?
a. The cause of OSA i n children and adults is tis-
sue obstruction.
b. The cause of OSA i n children and adults is
primarily hypotonia.
c. Children have tissue obstruction and adults
have hypotonia.

92
C H A P T E R S E V E N T Y SIX

OROPHARYNGEAL M A L I G N A N C Y

1. Which of the following is not correctly associated 4. Which of the following statements regarding
with its embryologic structure of origin? treatment of oropharyngeal carcinoma is true?
a. Salivary glands: first pharyngeal arch a. For early tonsil cancer, there is no significant
b. Tonsillar crypts: second pharyngeal pouch difference i n survival between surgery and p r i -
c. Anterior tongue epithelium: second pharyn- mary radiotherapy.
geal arch b. Primary closure of a tongue base defect can be
d. Posterior tongue epithelium: third pharyngeal performed only if less than 25% of the tongue
arch base is removed.
e. Epiglottis: third and fourth pharyngeal arches c. Tumors of the upper pharyngeal wall are usu-
ally accessible through a transoral route.
2. Which of the following statements regarding d. For soft palate cancer, radiotherapy should be
squamous cell carcinoma (SGG) of the orophar- considered for lesions less than 2 cm i n dia-
ynx is true? meter.
a. Nonkeratinizing SGG is more common than e. Wide resection of the tonsil and surrounding
keratinizing SGG. soft tissues can result i n significant adverse
b. Verrucous carcinoma almost always presents effects on function.
with metastasis.
c. 80% of all malignant oropharyngeal neoplasms 5. Which of the following regarding microvascular
are SGG. free flaps is true?
d. Spindle cell carcinoma exhibits clinical behav- a. Advantages of the rectus abdominis free flap
ior similar to that of conventional SGG. include ease of harvest, length of the vascular
e. Basaloid SGG occurs most commonly i n the pedicle, and ease of providing a sensate flap.
tonsil. b. The forearm free flap uses the posterior cuta-
neous nerve of the arm to provide sensation.
3. Regarding neck metastases from oropharyngeal c. One advantage of the lateral arm free flap is
carcinoma, the large size of its feeding vessel.
a. Computed tomography is significantly more d. I n a fibula free flap, its muscle and soft tissue
accurate at detecting neck metastases than components cannot be epithelialized.
clinical evaluation alone. e. The scapular free flap provides two skin pad-
b. Supraclavicular and posterior triangle nodal dles for use i n reconstruction.
metastasis have the worst prognosis for sur-
vival.
c. Contralateral metastasis occurs i n 30% of
tongue base tumors.
d. Given the moderate morbidity of a staging
neck dissection for clinically N nodes, a selec-
0

tive neck dissection is only performed i n a


minority of patients initially seen with oropha-
ryngeal SGG.
e. A T primary tumor resected with negative
2

margins and no neck disease necessitates


postoperative radiation therapy.

93
CHAPTER SEVENTY SEVEN

RECONSTRUCTION
OF T H E O R O P H A R Y N X

1. Which of the following statements regarding 4. The platysma myocutaneous flap


microvascular reconstruction of advanced a. Has not been used successfully i n the tongue
oropharyngeal tumors is true? base
a. This form of reconstruction has improved b. Cannot be performed i n cases i n which the
prognosis and locoregional control. facial artery has been divided
b. Other forms of reconstruction are no longer c. Is not a reliable form of reconstructing defects
warranted. of the oropharynx
c. Quality of life and functional status are d. Flap survival rate i n the tongue base is i n the
restored at 6 months, and most are improved order of 65%
at 1 year. e. Provides a t h i n pliable skin paddle that is ideal
d. Fasciocutaneous flaps are preferred for all for oropharyngeal reconstruction
defects.
e. No reconstruction is needed because of the 5. With regard to the lateral arm flap,
poor prognosis of these patients. a. Elevation of the flap risks ischemia to the arm.
b. Neurotization of the flap is not possible.
2. The Gehanno technique of soft palate recon- c. I t is not regarded as a good choice for oropha-
struction is indicated i n cases i n which ryngeal defects.
a. Complete soft palatectomy has been per- d. It is regarded as a good choice for combined
formed defects of the pharyngeal wall and tongue
b. A 25% defect exists base.
c. A 75% defect exists e. The vascular pedicle is more robust than the
d. A 50% or greater defect is present radial forearm flap.
e. Any soft palate defect is present

3. The goals of tongue base reconstruction include


the following except
a. Articulation
b. Taste perception
c. Swallowing
d. Maintenance of the airway
e. To create a neotongue with the necessary
bulk to create a shelf above the laryngeal inlet

94
CHAPTER SEVENTY EIGHT

D I A G N O S T I C I M A G I N G OF T H E
P H A R Y N X A N D ESOPHAGUS

A patient receives surgery, radiation, and 4. Esophageal ulcers are seen i n all the following
chemotherapy for an oropharyngeal squamous except
cell carcinoma. One month after the completion a. Herpes esophagitis
of therapy, the most accurate way to assess for b. Acute radiation esophagitis
residual tumor is c. Intramural pseudodiverticulosis
a. Computed tomography (CT) d. Barrett's esophagus
b. Magnetic resonance imaging e. Crohn's disease
c. Endoscopy
d. Positron emission tomography (PET) 5. CT is superior to an esophagram i n the assess-
e. Combined PET/CT ment of
a. Prevertebral spread of a hypopharyngeal
In a patient with a suspected leak after supra- tumor
glottic laryngectomy, which is the most appropri- b. Staging of an esophageal carcinoma
ate initial oral contrast agent? c. Postsurgical leak
a. Thin barium suspension d. Fourth branchial cleft fistula
b. Thick barium suspension e. Recurrence i n a jejunal interposition graft
c. Gastrografin
d. Nonionic intravenous CT contrast agents
e. An esophagram should not be performed on
this patient

Esophagram is superior to endoscopy i n the


assessment of which disease?
a. Epidermolysis bullosa
b. Nutcracker esophagus
c. Mallory-Weiss tear
d. Barrett's esophagus
e. Laryngeal carcinoma

95
CHAPTER SEVENTY NINE

E N D O S C O P Y OF T H E P H A R Y N X
A N D ESOPHAGUS

1. The following are not well tolerated by the awake 4. Indications for the use of pharyngoscopy include
or nonsedated patient. a. Assessment of velopharyngeal insufficiency
a. Rigid esophagoscopy b. Evaluation of a patient with obstructive sleep
b. Flexible esophagoscopy apnea
c. Rigid pharyngoscopy c. Tumor surveillance
d. Miiller maneuver d. Evaluation of a patient with dysphagia
e. A l l of the above
2. The role of functional endoscopic evaluation of
swallowing does not include 5. Preoperative assessment of the patient undergo-
a. Assessment of delay i n swallowing ing an esophagoscopy does not routinely include
b. Assessment of laryngeal penetration a. History and physical examination
c. Assessment of esophageal motility b. Radiologic evaluation
d. Assessment of vocal cord mobility c. Dental x-rays
d. Evaluation for cervical spine instability
3. Complications of esophagoscopy include
a. Esophageal perforation
b. Dental trauma
c. Bleeding
d. Cardiac arrhythmia
e. A l l of the above

96
CHAPTER EIGHTY

T H E ESOPHAGUS: A N A T O M Y ,
PHYSIOLOGY, A N D DISEASES

1. A l l of the following are true regarding ambulatory b. Laryngeal signs related to GERD include
24-hour esophageal pH monitoring except hoarseness, throat clearing, dysphagia,
a. The single most important parameter to meas- increased phlegm, and globus sensation.
ure is the amount of time that the pH is less c. Approximately 10% to 20% of patients with
than 6.0. asthma have GERD.
b. The primary indications for this procedure are d. GERD is the third most common cause of
to document excessive acid reflux i n patients chronic cough.
with expected gastroesophageal reflux disease e. Patients with extraesophageal GERD do not
(GERD) and to evaluate the efficacy of med- typically demonstrate esophagitis
ical or surgical therapy.
c. Standard pH monitoring measures distal 4. A white man with a history of GERD and
esophageal acid exposure by use of a single pH Barrett's esophagus is initially seen with rapidly
electrode catheter positioned 5 cm about the progressive solid food dysphagia. He is found to
superior margin of the LES. have a neoplastic lesion i n the distal esophagus
d. One advantage of this method is the ability to at the gastroesophageal junction. Biopsy of this
correlate symptoms w i t h reflux episodes. lesion is most likely to reveal which of the fol-
e. Less than 20% of reflux episodes i n patients lowing
with well-documented GERD are associated a. Leiomyoma
with symptoms. b. Adenocarcinoma
c. Squamous cell carcinoma
2. A patient with oropharyngeal dysphagia is likely d. Lymphoma
to demonstrate all of the following except
a. A n improvement i n symptoms when swallow- 5. A l l of the following are true regarding a patient
ing liquids who is seen i n the emergency department
b. Difficulty with initiating a swallow after ingestion of a strong alkali chemical
c. Localization of symptoms to the cervical or except
throat region a. Upper endoscopy should be performed during
d. A disruption i n the finely coordinated act of the first 24 to 48 hours after ingestion.
swallowing secondary to neuromuscular dys- b. The patient will have an estimated thousand-
function fold increase i n the risk of squamous cell car-
e. Diseases that affect nerves or muscles, as well cinoma of the esophagus.
as structural abnormalities c. The esophageal injury is the result of a coagu-
lative necrosis.
3. A l l of the following statements regarding extrae- d. The patient may complain of oropharyngeal,
sophageal GERD are true except retrosternal, or epigastric pain.
a. The primary esophageal cause of noncardiac e. The patient should be examined for evidence
chest pain is GERD i n 40% to 60% of patients. of oropharyngeal injury.

97
CHAPTER EIGHTY ONE

ZENKER'S D I V E R T I C U L U M

1. Where is Killian-Jamieson's region located? 4. Which statement(s) is/are false?


a. Between the cricopharyngeal and inferior con- a. No cricopharyngeal myotomy is performed
strictor muscle with endoscopic techniques.
b. Between the oblique and transverse fibers of b. ESD can be performed more quickly than
the cricopharyngeal muscle external techniques.
c. Between the cricopharyngeal and most supe- c. There is no need for perioperative antibiotics
rior esophageal circular muscle for the electrostatic discharges (ESD) proce-
d. Between the fibers of the inferior constrictor dure.
muscles d. Zenker's diverticulum is thought to be due to
e. None of the above discoordination of the cricopharyngeal muscle
during glutition.
2. What diagnostic test(s) should be obtained to e. There is no role for external techniques with
evaluate a patient suspected of having an the development of endoscopic techniques.
esophageal diverticulum?
a. Chest x-ray 5. What are some methods to help completely
b. Barium swallow divide the common wall between the esophagus
c. Computed tomography scan and diverticulum during ESD?
d. Esophagogastroduodenoscopy (EGD) a. Retraction sutures
e. Rigid endoscopy b. Placing the longer stapling blade containing
the cartridge into the esophagus
3. What are the advantages of the endoscopic sta- c. Sawing off the distal part of the stapler anvil
pling method over other endoscopic techniques? d. Use of multiple stapler cartridges
a. Faster operative times e. Placing the blades of the Weerda laryngoscope
b. No thermal injury to the recurrent laryngeal directly into the diverticulum and esophagus
nerve as distally as possible
c. The incised mucosa is sealed mechanically
d. Lower complication rate
e. No external scar is produced

98
CHAPTER EIGHTY TWO

NEOPLASMS OF T H E H Y P O P H A R Y N X
A N D C E R V I C A L ESOPHAGUS

1. Which of the following statements is false? 4. A 65-year-old man is seen with a neck mass and
a. Hypopharyngeal cancer is more common i n otalgia on the right. Laryngoscopy demonstrates
men. a squamous cell carcinoma of approximately 2.5
b. Hypopharyngeal cancer is more common i n cm i n the lateral wall of the pyriform sinus
black men. extending to the apex, but without involvement
c. The most common presenting complaint of of the cervical esophagus. The most conservative
hypopharyngeal cancer is otalgia. surgical option for this patient is likely to be
d. Most hypopharyngeal cancers present w i t h a. Lateral pharyngectomy and primary closure
associated lymphadenopathy. b. Lateral pharyngectomy and pectoralis flap
e. In total, 5-year survival is less than 35% i n reconstruction
patients with hypopharyngeal cancer. c. Lateral pharyngectomy and radial forearm
free-flap reconstruction
2. A 62-year-old smoker and former alcoholic is d. Partial laryngopharyngectomy
seen with a 3.5-cm left-sided neck mass that is e. Total laryngectomy and partial pharyngectomy
firm, minimally mobile, and nontender. Clinical
examination fails to identify a primary tumor. 5. A n MRI scan of the preceding patient demon-
The best initial imaging study to evaluate for strates a 3-cm necrotic node on the side of the
hypopharyngeal cancer is tumor without any additional lymphadenopathy.
a. Barium swallow Management of the neck i n the patient described
b. Noncontrast enhanced high-resolution com- in the preceding should include
puted tomography (CT) scan of the neck a. Bilateral I to V neck dissection
c. Contrast-enhanced high resolution CT scan of b. Bilateral I I to V neck dissection
the neck c. Ipsilateral I to V neck dissection and con-
d. Magnetic resonance imaging (MRI) with tralateral I I to IV neck dissection
gadolinium enhancement of the neck d. Ipsilateral I I to IV neck dissection
e. Whole body positron emission tomography e. Ipsilateral I to I I I neck dissection
(PET) scan

3. Which of the following tumor types is treated p r i -


marily with radiation therapy i n the hypophar-
ynx?
a. Squamous cell carcinoma
b. Lymphoepithelioma
c. Adenosquamous carcinoma
d. Basosquamous carcinoma
e. Synovial cell sarcoma

99
CHAPTER EIGHTY THREE

RADIOTHERAPY A N D CHEMOTHERAPY
OF S Q U A M O U S CELL C A R C I N O M A S OF
T H E H Y P O P H A R Y N X A N D ESOPHAGUS

Which of the following statements regarding the 3. Which of the following statements regarding lar-
treatment of advanced hypopharynx cancer is ynx preservation with induction chemotherapy is
true? true?
a. Conventional external beam irradiation is the a. After chemotherapy, a subsequent irradiation
treatment of choice for T hypopharyngeal
4 is the treatment of choice whatever the
cancer. response to chemotherapy.
b. When treated by radical surgery and postoper- b. Apart from its ability to allow preservation of
ative irradiation, these cancers usually recur some of the larynx, induction chemotherapy
in half the cases at the primary site. has significantly improved overall survival.
c. The most frequent evolution after radical sur- c. Apart from the ability to allow to preservation
gery and postoperative irradiation is the of some of the larynx, induction chemother-
appearance of distant metastases. apy has definitively suppressed distant fail-
d. Large randomized trials have concluded i n a ures.
similar outcome either after radical surgery d. Induction chemotherapy has decreased the
and postoperative irradiation or after defini- incidence of metachronous cancers.
tive irradiation alone. e. This strategy assessed i n a randomized trial
e. Adjuvant chemotherapy has improved the out- has allowed preservation of the larynx i n half
come after either radical surgery and postoper- the survivors at 3 and 5 years.
ative irradiation or definitive irradiation alone.
4. Which of the following statements regarding the
Which of the following statements regarding treatment of squamous cell esophageal cancer is
locoregional control after total laryngectomy true?
with partial laryngectomy and radical neck dis- a. Postoperative radiation therapy improves sur-
section for a preliminary untreated pyriform vival i n localized cancer.
sinus cancer is true? b. Postoperative chemotherapy therapy improves
a. Preoperative radiation therapy has been survival i n node-positive cancer.
proven to be able to improve locoregional c. Increasing radiation dose from 50.4 to
control. 64.8 Gy did not translate into enhanced sur-
b. Postoperative radiation therapy has been vival i n cancer treated by definitive chemora-
proven to be able to improve locoregional diation.
control. d. Split-course should be preferred as conven-
c. Preoperative chemotherapy has been proven tional protracted radiation therapy i n
to be able to improve locoregional control. esophageal cancer treated by definitive
d. Perioperative chemotherapy has been proven chemoradiation.
to be able to improve locoregional control. e. A Cochrane systematic review supports the
e. Postoperative chemotherapy has been proven need of preoperative chemotherapy i n local-
to be able to improve locoregional control. ized cancer.

100
Chapter 83 Radiotherapy and Chemotherapy of Squamous Cell Carcinomas of the Hypopharynx and Esophagus 101

5. Which of the following statements regarding the c. Preoperative chemoradiation improves the dis-
treatment of squamous cell esophageal cancer is ease-free survival i n operable cancer.
true? d. I n metastatic disease, chemotherapy provides
a. I n locally advanced operable cancer respond- a slight but significant increase i n survival vs
ing to chemoradiation, the continuation of best supportive care.
chemoradiation is an alternative to surgery. e. Definitive chemoradiation increases survival
b. Preoperative chemoradiation improves sur- compared with radiation therapy alone.
vival i n operable cancer.
CHAPTER EIGHTY FOUR

RECONSTRUCTION OF HYPOPHARYNX
A N D ESOPHAGUS

1. Which of the following was not a reason for 4. Which of the following reconstruction methods
abandoning the deltopectoral flap for reconstruc- gives the highest rate of return to oral feeding?
tion of the pharynx following laryngopharyngec- a. Tubed pectoralis major myocutaneous flap
tomy? b. Gastric pull-up
a. High fistula rates c. Colonic transposition flap
b. Delay i n postoperative chemoradiation ther- d. Free radial forearm fasciocutaneous graft
apy e. Free jejunum interposition graft
c. Stenosis of the flap
d. High donor site morbidity 5. Which of the following flaps is the most suscepti-
e. Need for multiple procedures ble to ischemic injury?
a. Deltopectoral flap
2. Which of the following is not an advantage of b. Pectoralis major myocutaneous flap
pectoralis major myocutaneous flap over del- c. Radial forearm free flap
topectoral flap reconstruction? d. Lateral thigh free flap
a. Lower flap failure rates e. Jejunal free flap
b. Single-stage reconstruction possible
c. Lower mortality rate
d. More versatile for range of pharyngectomy
defects
e. Shorter delay for initiation of radiation ther-
apy

3. Which of the following procedures has the high-


est morbidity and mortality rates?
a. Tubed pectoralis major myocutaneous flap
b. Gastric pull-up
c. Colon transposition graft
d. Free radial forearm fasciocutaneous graft
e. Free jejunum interposition graft

102
PART S E V E N

LARYNX/TRACHEA/BRONCHUS
C H A P T E R E I G H T Y FIVE A

LARYNGEAL A N D PHARYNGEAL
FUNCTION

1. Which muscle opens the larynx? 4. Vocal pitch drops with which pattern of laryngeal
a. Cricothyroid muscle activity?
b. Thyroarytenoid a. Isolated thyroarytenoid (TA) muscle contraction
c. Lateral cricoarytenoid b. Isolated cricothyroid (CT) muscle contraction
d. Posterior cricoarytenoid c. Isolated PCA
e. Interarytenoid d. Cocontraction of the TA and CT
e. Cocontraction of the PCA and lateral cricoary-
2. Laryngospasm is more likely to occur i n a tenoid (LCA)
patient who is
a. Hypoxic 5. The shaping of vocal sound into words is termed
b. Hypercarbic a. Phonation
c. Acidotic b. Resonance
d. Lightly anesthetized c. Articulation
e. Deeply anesthetized d. Intonation
e. Transduction
3. Strong respiratory demand alters activity of the
posterior cricoarytenoid (PCA) muscle so that
the PCA
a. Mirrors activity i n the diaphragm
b. Mirrors activity i n the cricothyroid muscle
c. Begins to contract before onset of inspiration
d. Stops contracting before the inspiration ends
e. Continues to contract during expiration

105
C H A P T E R E I G H T Y FIVE B

EVALUATION A N D M A N A G E M E N T
OF H Y P E R F U N C T I O N A L DISORDERS

Rhythmic movements of the tongue, jaw, and 3. Which of the following is not a feature of Meige's
lips may be associated with which of the follow- syndrome?
ing drugs? a. Blepharospasm
a. Diazepam b. Adductor spasmodic dysphonia
b. Diphenhydramine c. Oromandibular dystonia
c. Gyclobenzaprine d. Writer's cramp
d. Ghlorpromazine e. Lingual dystonia
e. Clozapine
4. Features that support a diagnosis of spasmodic
Which of the following statements is true about dysphonia include all of the following except
dystonias? a. Voice improvement with singing
a. Most patients w i t h idiopathic dystonia show b. Abnormal response to stress on personality
an autosomal-dominant inheritance pattern testing
with reduced penetrance. c. Voice improvement with alcohol ingestion
b. Most patients with primary laryngeal dystonia d. Worsening of voice on the telephone
will have abnormal movements develop i n e. Voice improvement with pinching of the nares
another body part.
c. X-linked torsion dystonia is associated with 5. The mechanism of action of botulinum toxin is
parkinsonism. a. Blockade of muscarinic acetylcholine recep-
d. Most cases of laryngeal dystonia are associated tors at the neuromuscular junction
with other neurologic conditions. b. Blockade of nicotinic acetylcholine receptors
e. Older age is associated with a higher probabil- at the neuromuscular junction
ity of spread to another body part. c. Inhibition of acetylcholine reuptake at the
neuromuscular junction
d. Inhibition of acetylcholine release into the
neuromuscular junction
e. Inhibition of intracellular acetylcholine forma-
tion

106
C H A P T E R E I G H T Y SIX

VISUAL D O C U M E N T A T I O N
OF T H E L A R Y N X

1. What is the best way to minimize a moire pat- 4. What conditions would not cause decreased
tern during laryngeal examinations? mucosal wave?
a. Record the examination with National a. Scarring
Television Standards Committee (NTSC) b. Incomplete closure with large glottic gap
format instead of Phase Alternating Line c. Cyst
(PAL) format. d. Increased pitch
b. Focus the camera. e. Sulcus vocalis
c. Defocus the camera
d. Use a flexible laryngoscope instead of a rigid 5. Supraglottic constriction during phonation can be
telescope. a. A sign of muscle tension dysphonia
e. Use a digital format for recording the images. b. A sign of dehydration
c. Masking an underlying lesion or bowing
2. The following parameter(s) can be accurately d. A normal variant
assessed only with stroboscopic lighting: e. A sign of an autoimmune disorder
a. Vocal fold closure
b. Mucosal wave
c. Vocal fold edges
d. Supraglottic constriction
e. Vibration symmetry

3. Which closure patterns are sometimes found


in subjects without voice disorders?
a. Spindle-shaped gap
b. Small posterior gap
c. Irregular gap
d. Anterior gap
e. Incomplete

107
C H A P T E R E I G H T Y SEVEN

V O I C E ANALYSIS

1. Patient scales 4. Which statement is true?


a. Are less important than the results of the a. Narrow-band spectrograms can highlight a loss
objective evaluation of harmonic structure i n the voice signal.
b. Can measure satisfaction, quality of life, hand- b. Jitter and shimmer are the best measures of
icap, or a particular aspect of voice production hoarseness.
c. Add nothing to a good case history c. Cepstral peak prominence (CPP) is an excel-
d. Are not reliable lent measure of breathiness that is based on
e. A l l of the above frequency analysis.
d. Mean nasalance above 50% for sustained Iml
2. Which feature is not important to voice produc- generally corresponds to hyponasality.
tion? e. A l l of the above
a. Hypernasality
b. Clenched jaw 5. Which of the following statements is false?
c. Neck extension a. Maximum phonation time (MPT) can be influ-
d. Decreased thyrohyoid space enced by respiratory function, laryngeal valv-
e. None of the above ing, velopharyngeal closure, practice,
frequency, intensity, vowel, and instructions.
3. Which statement is true? b. Laryngeal airway resistance (RLar) is a ratio
a. Frequency is the same as pitch. of translaryngeal air pressure to translaryngeal
b. Loudness is the perceptual correlate of fre- airflow.
quency. c. Electroglottographic (EGG) traces show degree
c. Frequency and intensity are independent. of vocal fold closure.
d. Frequency range can be stated in semitones. d. High mean airflow is common with glottic
e. a and c incompetence, such as unilateral vocal fold
motion impairment.
e. None of the above

108
CHAPTER EIGHTY EIGHT

D I A G N O S T I C I M A G I N G OF T H E L A R Y N X

1. Imaging signs of retropharyngeal abscess include 4. Which of the following statements regarding
a. Thickening of the retropharyngeal soft tissues imaging of glottic carcinoma is true?
on lateral soft-tissue radiography of the neck a. GT reliably distinguishes between benign cord
b. Hypodense fluid collection i n the retropharyn- paralysis and direct involvement with tumor.
geal space on computed tomography (GT) b. MRI may demonstrate tumor infiltration
c. Hyperintense fluid collection i n the retropha- within the paraglottic and preepiglottic spaces.
ryngeal space on T2-weighted magnetic reso- c. Phases of respiration have little impact on
nance imaging (MRI) cord appearance.
d. Ring-enhanced pattern i n the retropharyngeal d. Soft tissue thickening of the anterior commis-
space on GT or MRI sures up to 5 m m may be considered normal.
e. A l l of the above e. Soft tissue plain film radiography is the best
means for detecting cartilage invasion.
2. Regarding the imaging evaluation of vocal cord
paralysis, which of the following statements is 5. Regarding imaging of the postoperative neck,
true? which of the following statements is true?
a. Most causes can be detected with MRI of the a. MRI is the preferred modality for imaging the
brain. posttherapy neck.
b. Imaging evidence of perineural spread of the b. Expanding nodular soft tissue masses within
vagus nerve caused by remote carcinoma an irradiated field usually herald tumor recur-
metastasis is commonly seen. rence.
c. Imaging from the skull base to the pulmonary c. Scar within an operated field tends to remain
hila should be performed. stable over time.
d. Imaging signs include widely abducted vocal d. Hemorrhage or edema within a surgical bed
cords and ipsilateral collapsed pyriform sinus. may persist for 4 to 6 weeks.
e. GT tends to be superior for posterior fossa e. A l l of the above
evaluation compared w i t h MRI.

3. Features of MRI include all of the following


except
a. Acquisition of images is generally slower than
GT.
b. Cortical bone and calcifications do not pro-
duce a signal.
c. MRI is contraindicated i n patients with car-
diac pacemakers, metallic cochlear implants,
and cerebral aneurysm clips.
d. MRI is limited to the transverse plane.
e. Motion artifacts may severely degrade images.

109
CHAPTER EIGHTY NINE

NEUROLOGIC EVALUATION
OF T H E L A R Y N X A N D T H E P H A R Y N X

1. Isolated superior laryngeal injury results i n 4. Which of the following statements regarding
a. Rotation of the glottis to the side of the injury laryngeal electromyography (EMG) is false?
b. Rotation of the glottis to the side opposite the a. I t is essential before reinnervation procedures.
injury b. Both superior and recurrent laryngeal nerve
c. Prolapse of the arytenoid function can be tested.
d. Rowing of the vocal fold c. Demonstration of polyphasic action potentials
e. No appreciable change to the glottis means that function will be restored.
d. Fibrillation potentials are noted about 2 weeks
2. The most common cause of unilateral vocal fold after nerve transaction.
paresis is e. I t is useful i n localizing muscles for botulinum
a. Thyroid surgery injection.
b. Thoracic surgery
c. Stroke 5. Edrophonium is used to test for
d. Idiopathic a. Multiple sclerosis
e. Arnold-Chiari malformation b. Amyotrophic lateral sclerosis (ALS)
c. Syringomyelia
3. Laryngeal and speech abnormalities found i n d. Shy-Drager syndrome
patients with Parkinson's disease include all the e. Myasthenia gravis
following except
a. Dysarthria secondary to hypokinetic lingual
function
b. Dysphagia secondary to decreased laryngeal
sensation
c. Decreased loudness secondary to bowing of
the vocal folds
d. Decreased loudness secondary to hypokinetic
bellows function
e. Dysphonia secondary to vocal tremor

110
CHAPTER NINETY

LARYNGEAL A N D TRACHEAL
M A N I F E S T A T I O N S OF SYSTEMIC DISEASE

Which of the following has not been shown to be 4. Which of the following is commonly seen i n
of significant benefit i n the treatment of croup? patients with laryngeal tuberculosis?
a. Humidified air a. chest radiograph positive for tuberculosis
b. Racemic epinephrine b. History of tobacco and alcohol use
c. Nebulized budesonide c. Bloody sputum
d. Oral dexamethasone d. Negative purified protein derivative skin test
e. Intravenous dexamethasone e. History of HIV

Adult patients with epiglottitis are more likely to 5. Pseudoepitheliomatous hyperplasia can be seen
require intubation if they are initially seen with in all of the following except
a. Involvement of other supraglottic structures a. Blastomycosis
b. Symptoms for more than 5 days b. Histoplasmosis
c. Tachycardia c. Cryptococcosis
d. Positive soft-tissue neck radiograph d. Actinomycosis
e. Blood cultures positive for Streptococcus e. A l l of the above

3. Which of the following statements are true


regarding pertussis?
a. The incidence is rising i n infants but not
adults.
b. The cause is known to be failure of the vac-
cine i n some children.
c. Once a patient has contracted pertussis, they
have natural i m m u n i t y for life.
d. The classic "whooping" cough associated with
pertussis is more common i n infants than chil-
dren.
e. None of the above

111
CHAPTER NINETY ONE

C H R O N I C ASPIRATION

1. Which of the following statements regarding 4. Which of the following statements regarding aspi-
laryngotracheal separation is true? ration is false?
a. I t is irreversible. a. Normal, healthy patients never aspirate.
b. I t may be performed at the bedside. b. Chronic aspiration may have severe, long-term
c. The stasis of secretions i n the laryngeal pouch pulmonary consequences.
results i n chronic, symptomatic irritation. c. Most patients with chronic aspiration have
d. I t is not recommended for children. severe underlying medical conditions.
e. Patient's who have undergone previous tra- d. Cerebrovascular accidents are the most com-
cheotomy are not candidates for this proce- mon underlying medical condition i n adults
dure. with chronic aspiration.
e. The volume and character of the aspirated
2. Which of the following statements regarding tra- material has a marked impact on the clinical
cheotomy and aspiration is true? impact of aspiration.
a. A properly placed tracheotomy tube with an
inflated cuff effectively prevents aspiration. 5. Nonsurgical management of chronic aspiration
b. Tracheotomy has been definitively shown to includes all of the following except
cause aspiration. a. Discontinuation of all oral intake
c. Tracheotomy should be the first surgical pro- b. Initiation of parenteral antibiotics to cover
cedure for all patients with chronic aspiration. impending pulmonary infections
d. A causal relationship between tracheotomy c. Institution of an alternative route of alimenta-
and aspiration has yet to be demonstrated. tion
e. Tracheotomy has no role i n patients with d. Swallowing therapy including chin-tuck and
chronic aspiration. multiple swallow techniques

3. Which of the following statements regarding


videofluoroscopic swallowing studies and func-
tional endoscopic evaluation of swallowing
(FEES) is true?
a. Videofluoroscopic examination is preferred
because i t is more sensitive.
b. Videofluoroscopic examination is preferred
because i t is an easier bedside test.
c. FEES is preferred because i t is more sensitive.
d. FEES or videofluoroscopic examination have
similar sensitivity and specificity and may be
used according to hospital availability and
preference.
e. FEES should only be performed after a video-
fluoroscopic swallowing examination.

112
CHAPTER NINETY TWO

LARYNGEAL A N D ESOPHAGEAL T R A U M A

1. Which of the following statements best describes 4. Which of the following statements best describes
the role of computed tomography of the larynx the use and characteristics of laryngeal stenting?
after blunt trauma? a. A l l patients undergoing open reduction and
a. A l l patients with suspected laryngeal injuries internal fixation of laryngeal fractures should
should undergo computed tomography to doc- undergo stenting.
ument the extent of their injury. b. Laryngeal stents are problematic and can be
b. Computed tomography should be used only replaced by stabilization of the laryngeal
when the results of the study will influence skeleton with such fixation devices as m i n i -
the course of treatment. plates or microplates.
c. Suspected fractures of the laryngeal skeleton c. Laryngeal stents are necessary to stabilize
that are difficult to document by physical complex laryngeal skeletal fractures and
examination should be viewed by computed should be used for a m i n i m u m of 3 months to
tomography. permit significant wound healing.
d. b and c d. Laryngeal stents are necessary to stabilize
e. All of the above complex laryngeal skeletal fractures and there-
fore should be fixated with wire to ensure
2. Understanding the mechanism of trauma i n their prolonged positioning within the larynx.
blunt laryngeal injuries is important because e. Laryngeal stents are selectively indicated, and
a. Physical examination of the patient is often prolonged stenting may give rise to further
unreliable. injury of the larynx.
b. Patients are frequently unable to give an ade-
quate history. 5. The timing of surgical treatment for laryngeal
c. The degree of wounding or energy imparted to fractures or significant mucosal injuries should
the anterior neck may provide useful informa- be
tion about the severity of injury. a. Delayed for 3 or more days while the patient
d. Early examination of the larynx may be mis- is placed on high-dose corticosteroids to m i n i -
leading because the injury may still be mize edema and to permit more careful
evolving. restoration of the soft and hard tissue of the
e. a and b larynx
f. c and d b. Performed as soon as possible to permit more
careful restoration of the soft and hard tissue
3. Endotracheal intubation of patients with laryn- of the larynx
geal injuries is c. Irrelevant because essentially all patients with
a. Problematic and should be avoided significant injuries do poorly, and repair of
b. Beneficial i n all patients and avoids the need these injuries seems to be of minimal value
for tracheotomy d. Inconsequential given the lack of sufficient
c. Permissible and frequently safely performed i n studies evaluating the early management of
the emergency room setting such injuries
d. Permissible when the airway appears intact
and skilled personnel are available.
e. Cricothyrotomy or tracheotomy are the only
options

113
CHAPTER NINETY THREE

SURGICAL M A N A G E M E N T
OF UPPER A I R W A Y STENOSIS

1. The characteristics of an ideal mesodermal graft 4. Which of the following statements is correct?
include all of the following except a. The use of topical mitomycin-G is a proven
a. Rapid healing with minimal long-term graft time tested treatment for the prevention of
resorption subsequent restenosis.
b. Adequate strength, size, and pliability of graft b. Segmental resection and primary anastomosis
to contour i t to the defect provide optimal results for complete tracheal
c. Minimal donor site morbidity stenosis.
d. Absence of an accompanying epithelial lining c. Tracheal mobilization and laryngeal release
e. A donor site within the same operative field techniques are not required when a gap
greater than 6 cm i n tracheal continuity
2. Which sentence about laryngeal stenting is occurs.
correct? d. Suprahyoid laryngeal release usually requires
a. A soft stent is required when the cartilagenous transecting only the insertion of the digastric
framework is disrupted. muscles.
b. A hollow stent is preferable to a solid stent
for minimizing aspiration. 5. Requirement for successful repair of laryngotra-
c. A finger cot type stent will minimize pressure cheal stenosis include which of the following?
on mucosal surfaces. a. Establishment of an intact, reasonably shaped
d. A Montgomery T-tube is used to prevent skeletal framework to provide a scaffold for
phonation. the airway
b. Establishment of a completely epithelialized
3. The principles i n laryngeal keels usage include lumen of reasonably normal size and shape.
which of the following? c. Primary closure of mucosal lacerations after
a. The material used should be inert. minimal debridement of nonviable tissue
b. The length should be sufficient to extend is preferable i n acute case.
from the cricothyroid membrane to at least d. A l l of the above are correct.
2 to 3 m m above the anterior commissure.
c. The posterior wing of the keel should lie at
the vocal processes and should not touch the
posterior commissure.
d. A l l of the above are correct.

114
CHAPTER NINETY FOUR

T H E PROFESSIONAL V O I C E

1. Optimal results after vocal fold surgery may be 4. Laryngeal examination of a professional voice
best achieved by including user with the flexible fiberoptic laryngoscope has
a. Postoperative absolute voice rest what significant advantage compared with the
b. Smoking cessation rigid telescope of mirror?
c. Antireflux therapy a. Superior light intensity
d. Preoperative and postoperative voice therapy b. Ability to evaluate laryngeal biomechanics
e. Perioperative steroids c. Superior magnification
d. Higher quality laryngeal photographs
2. Extraesophageal reflux i n a singer may be mani- e. Provides more natural color
fested as
a. Decreased vocal range 5. Vocal nodules
b. Vocal fatigue a. Often require surgical therapy
c. Frequent throat clearing b. Always result i n dysphonia
d. Cough c. Are congenital
e. A l l of the above d. Are synonymous with vocal cord cysts
e. Usually respond to medical and behavioral
3. Which of the following is considered an absolute therapy
indication for the cancellation of a performance?
a. Upper respiratory infection 6. Which of the following statements regarding
b. Vocal process granuloma laryngovideostroboscopy is false?
c. Vocal fold hemorrhage a. I t is an objective examination.
d. Vocal fold varix b. I t allows observation of vocal fold
e. Muscle tension dysphonia vibration.
c. I t assists i n differentiating vocal fold cysts
from polyps.
d. Evaluation criteria include symmetry and
enclosure.
e. Stroboscopy was first reported by Oertel.

115
C H A P T E R N I N E T Y FIVE

BENIGN VOCAL FOLD MUCOSAL


DISORDERS

Which of the following is true regarding vocal 3. Use of aspirin may predispose one to have which
fold nodules? of the following lesions develop
a. They are often seen i n men. a. Vocal nodules
b. They are rarely unilateral. b. Sulcus vocalis
c. They result from trauma to capillaries. c. Intracordal cyst
d. They are most often treated surgically. d. Vocal fold polyp
e. They are rarely associated with vocal abuse. e. Capillary ectasia

A professional vocalist notes a sudden breathy 4. Which of the following lesions is most commonly
hoarseness and inability to maintain high notes associated with smoking?
during a performance. The best initial manage- a. Vocal fold nodules
ment is likely to be b. Vocal fold polyps
a. Voice rest for 2 weeks with oral corticosteroids c. Vocal fold granuloma
b. Voice rest for 2 weeks with proton-pump d. Vocal fold cysts
inhibitor e. Reinke's edema
c. Careful continuation of performance schedule
with aggressive singing voice therapy, 5. A smooth 7-mm lesion at the vocal process of a
increased hydration, and proton-pump nonsmoker is best treated with
inhibitor a. Voice therapy and increased hydration
d. Careful continuation of performance schedule b. Oral corticosteroids and voice rest
with singing voice therapy, increased hydra- c. Aggressive surgical excision followed by radia-
tion, and steroid injection into vocal cords tion therapy if incompletely excised
e. Microlaryngoscopy w i t h removal of hyaline d. Limited surgical excision, steroid injection
polyp and aggressive voice therapy
e. Limited surgical excision and intralesional
Cidofovir injection

116
C H A P T E R N I N E T Y SIX

M E D I A L I Z A T I O N THYROPLASTY

1. Which of the following materials predictably pro- 4. The optimal position of a medialization pros-
vides the longest duration of function when used thesis is
for injection laryngoplasty? a. A t the anterior commissure
a. Gelfoam b. At the mid-cord level
b. Autologous fat c. A t the posterior cord level
c. Hydroxyapatite d. At the anterolateral aspect of the vocal process
d. Teflon e. At a point posterior to the vocal process
e. Micronized human collagen
5. Six months after thyroidectomy, a patient has
2. Which of the following materials would be best persistent left vocal fold motion impairment
suited for injection laryngoplasty for unilateral an hoarseness. Palate and tongue function are
vocal fold paresis after thyroidectomy, knowing normal. Laryngeal EMG is performed demon-
the recurrent laryngeal nerve is intact? strating fibrillation potentials only i n the left
a. Bovine collagen thyroarytenoid muscle. The most appropriate
b. Teflon recommendation would be
c. Micronized human collagen a. Teflon injection
d. Autologous fat b. Gelfoam injection
e. Hydroxyapatite c. Wait for spontaneous recovery to occur
d. Medialization thyroplasty
3. During a medialization thyroplasty procedure, e. MRI of the neck and chest to rule out an occult
as the inner perichondrium is elevated the lesion
patient coughs violently and complains of pain
i n the throat and ear. When the patient is
asked to phonate air freely bubbles from
the thyroplasty window. The appropriate next
step is
a. Fill the window with tissue glue
b. Pack the window w i t h gauze and leave the
wound open to heal secondarily
c. Terminate the procedure and close the wound
with a drain i n place
d. Place an implant large enough to fill the hole
e. Perform a reinnervation procedure instead

117
CHAPTER NINETY SEVEN

ARYTENOID A D D U C T I O N

1. I n the paralyzed vocal cord, the purpose of the of the paralyzed hemiglottis, the most likely
arytenoid adduction suture is to mimic the cause is
action of which muscle? a. Oversizing of the implant, resulting i n overcor-
a. Thyroarytenoid rection of the anterior glottic gap
b. Posterior cricoarytenoid b. Excessive tension on the arytenoid adduction
c. Lateral cricoarytenoid suture, resulting i n overclosure of the poste-
d. Interarytenoid rior glottic gap
e. Cricothyroid c. Paresis of the cricothyroid muscle caused by
local anesthetic block of the superior laryngeal
2. Tension on the muscular process of the ary- nerve
tenoid results i n d. Edema of the paraglottic space after implant
a. External rotation of the arytenoid w i t h medial- placement
ization and downward displacement of the e. Persistent hyperfunction of the contralateral
vocal cord vocal fold
b. External rotation of the arytenoid with medial-
ization and upward displacement of the vocal 4. Which of the following is the least likely compli-
cord cation to be associated with arytenoid adduction?
c. External rotation of the arytenoid with lateral- a. Dysphagia
ization and upward displacement of the vocal b. Airway obstruction
cord c. Worsening of vocal quality
d. Internal rotation of the arytenoid with medial- d. Salivary fistula
ization and downward displacement of the e. Carotid artery injury
vocal cord
e. Internal rotation of the arytenoid with medial- 5. Relative to the cadaveric position of the dener-
ization and upward displacement of the vocal vated vocal cord, the reinnervated vocal cord is
cord likely to lie
a. Lateral and superior to the cadaveric cord
3. During an awake Type I thyroplasty w i t h ary- b. Lateral and inferior to the cadaveric cord
tenoid adduction and endoscopic monitoring, c. Medial and superior to the cadaveric cord
the patient's voicing is noted to be harsh, tight, d. Medial and inferior to the cadaveric cord
and choppy. With good endoscopic appearance e. I n the same position as the cadaveric cord

118
CHAPTER NINETY EIGHT

LARYNGEAL R E I N N E R V A T I O N

1. Performing an anastomosis of a divided recurrent 4. The ansa cervicalis-to-recurrent laryngeal nerve


laryngeal nerve at the level of the main trunk is anastomosis is indicated for
most likely to produce a. Unilateral vocal fold paralysis
a. Vocal fold motion symmetric with the nor- b. Bilateral vocal fold paralysis
mally innervated side c. Glottic stenosis
b. Normal abduction of the reinnervated vocal d. A l l of the above
fold but little or no adduction
c. Normal adduction of the reinnervated vocal 5. Which of the following is not an advantage of the
fold but little or no abduction ansa cervicalis-to-recurrent laryngeal nerve
d. Minimal purposeful movement of the reinner- (RLN)?
vated vocal fold a. No permanent implant material is used.
b. Ansa-RLN anastomosis does not interfere with
2. The neuromuscular pedicle technique and the the spontaneous recovery of vocal fold motion.
ansa cervicalis to recurrent laryngeal nerve anas- c. Ansa-RLN anastomosis does not interfere with
tomosis are similar i n that they both the subsequent use of thyroplasty or vocal fold
a. Rely on the transfer of intact neuromuscular injection.
units d. The sacrifice of the sternothyroid muscle is
b. Reinnervate multiple muscles with a single clinically insignificant.
pedicle or anastomosis
c. Use a branch of the ansa cervicalis as a donor
nerve
d. Require a midline thyrotomy approach to the
muscle to be reinnervated

3. Which of the following muscles has the fastest


contraction time?
a. The posterior cricoarytenoid
b. The thyroarytenoid
c. The soleus
d. The thyrohyoid

119
CHAPTER NINETY NINE

M A L I G N A N T T U M O R S OF T H E L A R Y N X
A N D HYPOPHARYNX

1. Laryngeal organ preservation is based on the 4. Which one of the following principles apply to
premise that the addition of chemotherapy will the management of the neck i n patients with
a. Reduce the incidence of distant metastatic dis- supraglottic squamous cell carcinoma?
ease a. Level I should be dissected i n patients with
b. Provide an increased 5-year survival positive adenopathy i n level I I or I I I .
c. Improve the quality of life b. Radiation therapy is as effective as surgery for
d. Select individuals that will have a favorable control of the contralateral neck.
response to radiation therapy c. Recurrence i n the previously modified neck
e. Extend the interval u n t i l a laryngectomy is dissection and postoperative irradiated neck
required can be controlled by a radical neck dissection.
d. I n an N neck, a radical neck dissection is
1 +

2. A residual mass i n the neck i n a patient treated required.


with a larynx organ preservation protocol who e. The submandibular gland should never be
initially had a T N carcinoma of the larynx
2 + resected i n a modified neck dissection.
should
a. Be observed for 3 months or longer for resolu- 5. Which of the following findings is the most impor-
tion tant i n the decision as to which treatment should
b. Undergo a laryngectomy with neck dissection be selected for an early vocal cord cancer?
regardless of response at the primary site a. Tumor differentiation
c. Should undergo a neck dissection as soon as b. Limited subglottic extension
feasible c. Involvement of the anterior commissure
d. Should undergo a neck dissection only if the d. Arytenoid cartilage involvement
initial tumor was an N 3 e. Vocal cord mobility
e. Does not need a neck dissection if the initial
lesion was staged N x

3. Overall 5-year survival rates for hypopharyngeal


carcinoma rates are poor because
a. Of the high incidence of comorbid conditions
b. Of the high incidence of distant metastases
c. Of the high incidence of second primary
malignancies
d. More than 60% of patients are initially seen
with positive cervical adenopathy
e. A l l of the above

120
CHAPTER ONE HUNDRED

M A N A G E M E N T OF EARLY G L O T T I C
CANCER

Which of the following risk factors are associated 4. Which of the following statements is true regard-
with laryngeal cancer? ing reconstruction of the neocord after partial
a. Gastroesophageal reflux laryngectomy?
b. Human papillomavirus a. Supraglottic structures do not contribute to
c. Alcohol use postoperative phonation.
d. Second-hand tobacco smoke b. Forearm fascial flap with buccal mucosal graft
e. A l l of the above is ideal i n the setting of arytenoid fixation.
c. Injection laryngoplasty is hindered by para-
According to the sixth edition of the American glottic space scarring.
Joint Committee on Cancer (AJCC) staging man- d. Closure of the posterior glottic gap by partial
ual, a transglottic tumor involving the right true cricoid resection poses no risk of diminishing
vocal fold with impaired vocal fold motion and the airway.
invasion limited to the paraglottic space is classi- e. None of the above
fied as
a- T2 a 5. Which of the following is true regarding external
b. T2 b beam radiation therapy?
a. Compared with lower dose regimens, acceler-
d. T4 ated fractionation of external beam radiation
e. None of the above therapy may offer improved control.
b. Compared with lower dose regimens, acceler-
Excision of the vocal cord and a segment of ated fractionation of external beam radiation
underlying thyroid cartilage with or without the therapy causes less dysphagia and mucositis i n
ipsilateral arytenoid is classified as a the first 2 months of therapy.
a. Vertical partial laryngectomy c. Duration of treatment does not predict survival.
b. Laryngofissure with cordectomy d. Radiation failures usually manifest as distant
c. Vestibulectomy metastases.
d. Supracricoid laryngectomy e. Radiation therapy precludes conservation
e. None of the above laryngeal surgery.

121
CHAPTER ONE HUNDRED AND ONE

T R A N S O R A L LASER M I C R O R E S E C T I O N
OF A D V A N C E D LARYNGEAL T U M O R S

1. Which one of the following statements about 4. When TLM is performed i n the patient with a
transoral laser microresection (TLM) is true? high-risk N neck, which of the following is not a
0

a. TLM is a surgical treatment strategy for p r i - logical reasons to perform the neck dissection
mary cancers of the larynx but not the phar- at a separate time?
ynx or mouth. a. Micrometastases " i n transit" at the time of the
b. I n aggregate, the resected tissue volume paral- TLM will have time to lodge i n the nodes.
lels that of an open operation. b. A patient with serious comorbidities may have
c. Radiotherapy is offered after TLM to finesse recovered from the primary resection.
margins but not to treat the N neck.
0 c. A n elderly patient may have regained swallow-
d. TLM has never been successfully used for the ing after a laser supraglottic laryngectomy.
treatment of a T laryngeal cancer.
3 d. The neck is already violated to access the p r i -
e. A negative second look guarantees no local mary tumor by laser endoscopic surgery.
recurrence will happen. e. Staging the primary tumor and the neck sur-
gery at separate sittings reduces the chance of
2. A l l of the following are possible reasons a laser a pharyngocutaneous fistula to zero.
tumor transection might be considered safe
except 5. Contraindications to TLM will include all of the
a. Cancer cells do not adhere to a beam of light following except
so the cutting instrument does not provide a a. Extensive tumor spread to the great vessels,
physical carrier to transplant tumor. the esophagus, or the thyroid gland
b. Cancer cells revealed by laser energy should b. Inability to expose the larynx or tumor
be thermocoagulated, hence not viable. c. Recurrent cancer i n an irradiated "bed"
c. Cancer cells falling unseen into a laser wound d. Advanced cancer needing reconstruction
light on a thin layer of coagulum that, left e. Patients with functional disorders after exten-
open, is superficially sloughed. sive partial resections (like severe persistent
d. Suctions and forceps could never transport aspiration or secondary stenosis)
viable cancer cells exposed by tearing of the
specimen
e. C 0 laser incisions seal lymphatic vessels in the
2

wound margin immediately, and lymphatic ves-


sels remain sealed for approximately 10 days.

3. A l l of the following improves access to the ante-


rior commissure except
a. A laryngoscope w i t h a wide body to spread out
the tongue
b. Tape across the cricoid to tip the larynx
c. A laryngoscope w i t h less bevel at the tip
d. A laryngoscope w i t h a narrow vertically oval
body to sink into the tongue

122
CHAPTER ONE H U N D R E D AND TWO

C O N S E R V A T I O N L A R Y N G E A L SURGERY

The cricoarytenoid unit consists of b. I t is contraindicated i n tumors that extend to


a. The recurrent laryngeal nerve, the associated the preepiglottic space.
musculature, the cricoid and arytenoid c. The reconstruction is performed by pexy of
b. The superior laryngeal nerve, the associated the hyoid to cricoid with or without the
musculature, the cricoid and arytenoid epiglottis.
c. The recurrent laryngeal and superior laryngeal d. The superior laryngeal nerves are preserved
nerve, the cricoid and arytenoid whenever possible.
d. The recurrent laryngeal and superior laryngeal e. Interarytenoid involvement w i t h tumor is a
nerve, the associated musculature, the cricoid contraindication.
and arytenoid
e. The recurrent laryngeal and superior laryngeal 4. Which of the following statements regarding
nerve, the vocal folds, the associated muscula- supraglottic laryngectomy is true?
ture, the cricoid and arytenoid a. Involvement of the preepiglottic space necessi-
tates hyoid bone resection.
Which of the following is not a principle of organ b. I t is contraindicated i n tumors that extend to
preservation surgery? the vallecula.
a. Local control is of paramount importance c. I t is contraindicated i n tumors that extend
b. Resection of normal tissue is necessary to into the ventricle.
allow for functional reconstruction. d. Vocal fold fixation is not a contraindication.
c. Cricoarytenoid unit is the essential functional e. Interarytenoid involvement with tumor is not
unit of the larynx. a contraindication.
d. Accurately assessing the three dimensional
extent of the tumor 5. Which of the following lesions is not suitable for
e. The T-stage indicates the conservation laryn- supracricoid laryngectomy?
geal procedure to be performed. a. T glottic lesion with arytenoid mobility
3

b. Anterior commissure lesion with involvement


Which of the following statements regarding of the thyroid cartilage not extending through i t
supracricoid laryngectomy is false? c. T glottic lesion with 2 cm of subglottic extent
2

a. The resection includes the entire thyroid car- d. A transglottic lesion with a fixed vocal cord
tilage, false cords, true cords with or without and epiglottic involvement
the epiglottis sparing at least one arytenoids. e. T glottic lesion w i t h impaired mobility
2

123
CHAPTER ONE HUNDRED AND THREE

TOTAL LARYNGECTOMY A N D
LARYNGOPHARYNGECTOMY

1. A patient underwent a laryngectomy for glottic d. Concurrent chemotherapy with radiotherapy


carcinoma that previously failed radiotherapy provides better local control than surgery plus
6 months earlier. He complains of dysphagia to radiotherapy.
both liquids and solids. A barium swallow is e. The rate of pharyngocutaneous fistula is
obtained and reveals a structure i n the higher after salvage surgery for patients after
hypopharynx that seems to be an epiglottis. The sequential chemotherapy and radiation com-
most likely reason for this problems is pared with after concurrent radiation.
a. Vertical closure of the pharynx
b. T-shaped closure of the pharynx 4. A patient who is not a suitable candidate for organ
c. Lack of flap reconstruction preservation surgery opts for surgical therapy of a
d. Pharyngeal stricture caused by closure of supraglottic cancer that involved the epiglottis,
muscle left false vocal cord, arytenoid, and the anterior
e. A remnant of epiglottis that was not commissure. He has bilateral adenopathy. Which
resected of the following is true about performing thy-
roidectomy at the time of total laryngectomy?
2. A patient is treated w i t h concurrent chemother- a. Total thyroidectomy may be indicated because
apy and radiotherapy for T N glottic cancer. The
3 0 of paratracheal lymph node involvement.
patient has persistent disease, and a laryngec- b. Total thyroidectomy should be performed
tomy is performed. What is the risk of pharyngo- regardless of the tumor extent if tumor inva-
cutaneous fistula postoperatively? sion is suspected.
a. 30% c. Left thyroid lobectomy should be performed i n
b. <10% continuity with the larynx, because the tumor
c. 15% is transglottic.
d. 45% d. Preservation of both thyroid lobes will nearly
e. 60% eliminate the risk of hypothyroidism after
treatment.
3. Which is true of organ preservation strategies for e. If there is 5 m m of subglottic extension,
laryngeal and hypopharyngeal carcinomas? thyroid invasion is likely, so thyroidectomy is
a. Concurrent chemotherapy with radiotherapy indicated.
provides superior local control and organ preser-
vation compared with sequential chemotherapy 5. After entry into the vallecula for an endolaryn-
with radiotherapy for laryngeal carcinoma. geal tumor, the most logical next step i n total
b. The rate of organ preservation is consistently laryngectomy is to
higher i n hypopharyngeal carcinomas com- a. Cut along the aryepiglottic fold from vallecula
pared with laryngeal carcinomas. to pyriform sinus
c. Chemotherapy combined with radiotherapy b. Elevate the pyriform sinus mucosa
improves survival over radiotherapy alone or c. Divide the constrictor muscles at the lateral
sequential chemotherapy and radiation. border of the thyroid cartilage
d. Transect the trachea
e. Divide the sternohyoid muscle and the thyroid
isthmus

124
CHAPTER ONE HUNDRED AND FOUR

R A D I A T I O N THERAPY FOR T H E LARYNX


A N D HYPOPHARYNX

1. Which of the following regarding lymphatic 4. I n the management of advanced laryngeal carci-
drainage is true? noma, the Radiation Therapy Oncology Group
a. The supraglottis has a more extensive l y m - protocol 9111 and Head and Neck Intergroup
phatic network than the glottis. 3-arm phase I I I trial concluded that
b. The subglottic lymphatics drain exclusively to a. Survival at 2 years for radiation therapy alone
the paratracheal nodes. was superior to concurrent chemoradiation.
c. The supraglottic lymphatics only drain to the b. The rate of laryngeal preservation was higher
jugulodigastric nodes. with concurrent chemoradiation than
d. The false vocal folds are devoid of lymphatics. chemotherapy alone.
e. None of the above c. The rate of laryngeal preservation was higher
with radiation alone than chemoradiation.
2. Which of the following is a potential contraindi- d. Radiation therapy alone has higher rates of
cation for supraglottic laryngectomy? toxicity than chemoradiation.
a. Bilateral arytenoid disease e. None of the above
b. Postcricoid disease
c. Thyroid cartilage involvement 5. I n external beam radiation given before total
d. True vocal fold fixation laryngectomy, the typical total radiation dose is
e. All of the above a. 10 to 20 Gy
b. 30 to 40 Gy
3. Which of the following is true? c. 50 to 60 Gy
a. Neck lymph nodes should be treated with radi- d. 70 to 80 Gy
ation i n T disease to prevent occult metas-
a e. 90 to 100 Gy
tases.
b. Local control for T disease with radiation
1

therapy is 68% to 88%.


c. All patients with T N disease should undergo
2 0

neck radiation treatment.


d. The radiation field should be limited to the
nodal region involved w i t h gross disease.
e. None of the above

125
CHAPTER ONE H U N D R E D AND FIVE

VOCAL REHABILITATION FOLLOWING


TOTAL LARYNGECTOMY

1. Which of the following is not a primary reason 4. What is the most appropriate first step i n the
for the failure of early surgical shunt procedures management of leakage after primary TE
for the rehabilitation of voice following laryngec- puncture?
tomy? a. Removal of the prosthesis with a red rubber
a. Aspiration catheter placement to allow the site to narrow
b. Pharyngocutaneous fistula formation b. Removal of the prosthesis to allow the site to
c. Stenosis close for revision puncture
d. Need for multiple procedures c. Initiation of antifungal therapy
e. Failure to attain adequate voicing d. Replacement of the prosthesis with a larger
device
2. The most widely practiced method of surgical e. Replacement of the prosthesis with a shorter
voice rehabilitation after laryngectomy is device
a. Esophageal speech
b. Tracheoesophageal (TE) fistula speech without 5. Management of premature device failure includes
prosthesis a. Systemic antifungal therapy
c. TE fistula speech w i t h a prosthesis b. Topical antifungal therapy
d. The Asai procedure c. Device replacement with another brand
e. Electrolaryngeal speech d. Gleaning of the device with chlorhexidine
e. Abandoning TE fistula speech
3. Which of the following is not a common reason
for the failure of TE fistula speech acquisition?
a. Pharyngeal hypertonia
b. Pharyngeal hypotonia
c. Failure of neoglottic mucosa vibration
d. Fungal colonization of the prosthesis
e. Poor stoma and fistula design

126
C H A P T E R O N E H U N D R E D A N D SIX

M A N A G E M E N T OF T H E I M P A I R E D
AIRWAY I N THE ADULT

1. I n a modified Mallampati class I I I airway, which 4. Which of the following is true about percuta-
of the following structures are visible? neous dilatational tracheotomy (PDT)?
a. Uvula, faucial pillars a. PDT should be avoided i n obese patients and
b. Uvula, faucial pillars, soft palate visible children.
c. Soft palate only b. Conventional open tracheotomy requires more
d. Hard palate only time than PDT.
c. Endoscopic guidance during PDT may result
2. The most common complication after tracheo- in hypercarbia i n patients with head injuries.
tomy is d. Cadaver studies show that puncture site i n the
a. Pneumomediastinum trachea vary greatly during PDT.
b. Tracheal stenosis e. A l l of the above
c. Tube displacement
d. Tube obstruction 5. Which of the following is an absolute contraindi-
e. Hemorrhage cation for PDT?
a. Limited cervical spine extension
3. Which of the following statements is true? b. Positive end-expiratory pressure of greater
a. A tracheoesophageal fistula usually occurs than 15 cm H 0
2

when the tracheotomy tube is tipped anteri- c. Need for emergent airway access
orly. d. History of difficult intubation
b. Chronic cough with eating and recurrent aspi- e. Acute infection over tracheotomy site
ration are common signs of a tracheoinnomi-
nate fistula.
c. A low-volume, high-pressure tracheotomy tube
cuff decreases the risk of tracheal stenosis.
d. Sentinel bleeding may occur 3 days to 3 weeks
before a tracheoinnominate blowout.
e. None of the above

127
CHAPTER O N E H U N D R E D AND SEVEN

E N D O S C O P Y OF T H E
T R A C H E O B R O N C H I A L TREE

What are the limitations of fiberoptic bron- 3. Potential risks and complications that should be
choscopy (FOB)? discussed with patients include which of the fol-
a. Inability to visualize beyond the seventh-gen- lowing?
eration airways a. Fever is an unexpected sequelae of a diagnos-
b. Inability to visualize extrabronchial structures tic bronchoscopy and should always prompt
such as lymph nodes and or blood vessels culture and initiation of antibiotic treatment.
c. Inability to sample peripheral lung nodules b. Pneumothorax can occur only with the trans-
smaller than 2 cm bronchial use of needles, forceps, and brushes.
d. Inability to sample bronchoscopically low c. Airway perforation during tumor or granula-
paraesophageal lymph nodes tion debridement is more likely with heat
e. Inability to sample the left atrial pressure therapies such as argon plasma coagulation
than with cryotherapy.
Which of the following statements regarding d. Airway edema and obstruction is equally likely
preparation and anesthesia for bronchoscopy is after laser, electrocautery, cryotherapy, and
true? photodynamic therapy (PDT).
a. There are no required standard preprocedure e. After PDT or endobronchial brachytherapy,
laboratory studies such as platelet counts and there is a 3% to 25% risk of eventual fatal
coagulation parameters, and i t is acceptable to hemoptysis.
perform biopsies with a platelet count of less
than 50,000. 4. Which of the following is true about stents that
b. As an alternative to general anesthesia i n the are available for tracheobronchial airways use?
operating room, intravenous conscious seda- a. Stents are made of either metal or silastic
tion with midazolam, fentanyl, and Diprivan (silicone and plastic).
can be safely given i n an off-site endoscopy b. Silicone stents are removable, but they are
suite with a lower level of monitoring. also more prone to migration.
c. Because there is no respiratory or hemody- c. The only self-expanding stents are metallic,
namics compromise, lidocaine is safe to use i n and they can all be placed without the need
nonallergic patients. for rigid bronchoscopy or direct suspension
d. The choice of bronchoscope size and site of laryngoscopy.
passage is not a major consideration i n a d. A l l metal stents are self-expanding and have
patient with normal trachea and vocal cords. the benefit of not requiring balloon expansion.
e. General anesthesia w i t h paralytic agents need e. Only the covered silastic stents are usable for
only be used i n rigid bronchoscopies. covering tracheobronchial-esophageal fistulas.

128
Chapter 107 Endoscopy of the Trancheobronchial Tree 129

5. Which of the following is true regarding innova- erate "virtual bronchoscopy" fly-throughs i n
tions i n bronchoscopy? the airway and obviate the need for many
a. Autofluorescence bronchoscopy requires the diagnostic bronchoscopies.
use of special bronchoscopes and a photosen- d. Endoscopic lung volume reduction uses valves
sitizes and tissue glue to cause regional atelectasis of
b. Endobronchial ultrasound aids i n the localiza- hyperinflated lung segments.
tion of all the regional hilar and mediastinal e. High-dose local therapy with drug-eluting
lymph nodes for transbronchial needle aspira- stents and bronchoscopic direct intralesional
tion. injection of drugs do not require additional
c. Three-dimensional reconstruction of m u l t i - FDA approval as long as they use existing
slice detector computed tomography can gen- approved FDA medications.
CHAPTER ONE HUNDRED AND EIGHT

DIAGNOSIS A N D MANAGEMENT
OF T R A C H E A L NEOPLASMS

1. Which two of the following tracheal tumors com- 4. Which of the following treatments provide pallia-
prise most primary tracheal neoplasms i n tion to patients w i t h extrinsic tracheal compres-
adults? sion?
a. Carcinoid and adenoid cystic carcinoma a. Photodynamic therapy
b. Adenoid cystic carcinoma and squamous cell b. Rigid bronchoscopy with core-out techniques
carcinoma c. Stent placement
c. Granular cell tumor and squamous cell carci- d. Cryosurgery
noma e. Laser therapy
d. Papilloma and granular cell tumor
e. Carcinoid and leiomyoma 5. Major disadvantages of expandable metal stents
include
2. Which of the following techniques is the pre- a. Requirement of rigid bronchoscopy to place
ferred treatment for primary malignant tracheal b. Stent migration, erosion, ingrowth of granula-
tumors? tion tumor, and inability to effectively clear
a. Radiation and chemotherapy secretions
b. Chemotherapy alone c. Cannot be used if tumor involves the carina
c. Photodynamic therapy d. Cannot be used if tumor has an luminal com-
d. Stent placement ponent
e. Complete surgical resection and consideration e. Need to be replaced every 6 months
of radiation therapy

3. Which of the following treatments is the pre-


ferred management of invasive well-differentiated
thyroid carcinoma that invades the trachea?
a. Surgical resection of involved tracheal seg-
ment and primary anastomosis
b. Radiation therapy
c. Shave the tumor off the trachea then radiation
therapy
d. Chemotherapy and radiation therapy
e. Stent placement

130
CHAPTER ONE HUNDRED AND NINE

UPPER A I R W A Y M A N I F E S T A T I O N S OF
GASTROESOPHAGEAL REFLUX DISEASE

1. Which of the following regarding the upper 4. Which of the following regarding manifestations
esophageal sphincter (UES) is true? of extraesophageal reflux (EER) is true?
a. The upper esophageal sphincter is a complete a. Pseudosulcus involves the free edge of the
muscular circle. vocal fold and ends at the vocal process.
b. The cricopharyngeus receives input from cen- b. Pachydermia laryngeus refers to thickening of
ters i n the nucleus ambiguous and nucleus the anterior larynx.
solitarius. c. Stimulation of the larynx by aspirated secre-
c. During UES contraction, the cricoid is dis- tions causes reflexive vocal cord abduction.
placed inferiorly and posteriorly. d. Granuloma formation may indicate severe
d. During sleep, UES pressure is increased. EER.
e. The UES is innervated solely by the vagus e. None of the above
nerve.
5. Which of the following is true regarding GERD
2. Which of the following substances decreases management?
lower esophageal sphincter tone? a. Chewing gum 1 hour after meals increases
a. Secretin acid contact time i n patients with reflux.
b. Cholecystokinin b. Antacids decrease LES resting pressure.
c. Glucagon c. I n a small double-blind placebo-controlled
d. Vasoactive intestinal peptide trial, short-term omeprazole was equivalent to
e. A l l of the above placebo i n controlling patient symptoms.
d. Long-term H -blocker therapy is as effective as
2

3. Which of the following conditions is associated proton pump inhibitors i n treating esophagitis.
with increased lower esophageal sphincter rest- e. Metoclopramide improves gastric motility and
ing pressure? decreases LES pressure.
a. Hiatal hernia
b. CREST syndrome
c. Scleroderma
d. Isolated Raynaud's phenomenon
e. None of the above

131
PART E I G H T

NECK
CHAPTER ONE HUNDRED AND TEN

DEEP N E C K I N F E C T I O N

A 48-year-old woman who had a small cutaneous 3. A patient has a buccal space infection develop
nevus removed from the lateral side of her neck that is not eliminated by an initial course of
is seen 2 days later w i t h high fever, confusion, broad-spectrum antiobiotic therapy. What is the
and a 5-cm area of erythema surrounding the next step i n this patient's management?
wound. The area has indistinct borders, and a. Intraoral incision and drainage
crepitance is noted. What is the appropriate i n i - b. Extraoral incision and drainage
tial management of this patient? c. Second course of a different antibiotic
a. Antistaphylococcal antibiotics and overnight d. Short-term course of steroids
observation e. Antifungal therapy
b. Fine-needle aspiration and oral antibiotics
c. Hyperbaric oxygen therapy and broad-spec- 4. Which bacteria are most commonly cultured
t r u m antibiotics from deep space neck abscesses?
d. Emergent surgical exploration/debridement a. Aerobic gram-negative bacilli
and broad-spectrum antibiotics b. Actinomyces israelii
e. Bedside incision and drainage and discharge c. Streptococci species
on oral antibiotics d. Staphylococci species
e. Pseudomonas species
A 7-year-old child is initially seen with fever and
firm, painful swelling of the bilateral floor of 5. What percentage of retropharyngeal abscesses
mouth 1 day after frenulectomy. Her tongue is occur i n children younger than 6 years of age?
elevated, and she speaks with a characteristic a. 5%
breathy voice. Which of the following is an b. 10%
important component i n the initial evaluation of c. 30%
this patient? d. 50%
a. Fiberoptic laryngoscopy to evaluate the airway e. 90%
b. Fine-needle aspiration
c. Ultrasound evaluation of the floor of mouth
d. Both computed tomography and magnetic res-
onance imaging of the neck
e. Panorex film

135
CHAPTER ONE HUNDRED AND ELEVEN

BLUNT A N D PENETRATING T R A U M A
TO THE NECK

1. A 40-year-old man sustained a stab wound to 4. The anatomic boundaries of zone III are from
zone I I I . He is hemodynamieally stable but has a. The hyoid to base of skull
an acute hypoglossal nerve paralysis. What is the b. The superior border of the thyroid cartilage to
next step? the base of skull
a. Four-vessel angiogram c. The hyoid to the mandible angle
b. Computed tomography scan d. The mandible angle to the skull base
c. Magnetic resonance imaging e. The clavicle to the cricoid
d. Lateral soft tissue of the neck
e. Direct laryngoscopy 5. During endoscopy, what injured area is most
commonly missed i n evaluating a penetrating
2. What region of the neck has the most difficult neck-injury?
surgical access for exploration? a. Esophageal inlet
a. Base of skull region b. Distal cervical esophagus
b. Midcervical region c. Nasopharynx
c. Lower cervical region d. Vallecula
d. Posterior neck triangle e. Supraglottic area
e. Anterior-cervical triangle

3. The best incision to explore the carotid sheath


for a unilateral, penetrating, neck injury is
a. A modified Conley incision
b. Diagonal cervical incision along the anterior
sternocleidomastoid muscle
c. A MacFee incision
d. A n H incision
e. A Schobinger incision

136
CHAPTER O N E H U N D R E D AND TWELVE

DIFFERENTIAL DIAGNOSIS
OF N E C K MASSES

1. When taking random guided biopsies to look for 4. A 13-year-old child is seen with a fever and
an occult primary tumor, which is not one of the painful swelling i n the area of the angle of the
most likely sites? left mandible. Last week, she had an upper respi-
a. Nasopharynx around Rosenmuller's fossa ratory tract infection. Ultrasonography reveals
b. Tonsil the area to be cystic. What would be expected on
c. Base of the tongue aspiration of this lesion?
d. Pyriform sinus a. Serosanguineous fluid with abundant mono-
e. Buccal mucosa cytes
b. Milky brown fluid that contains cholesterol
2. Which of the following is not an indication for crystals
biopsy of a neck mass i n a child? c. Clear fluid with many lymphocytes
a. Progressively enlarging nodes d. Inflammatory fluid with abundant neutrophils
b. Single asymmetric nodal mass e. Serous fluid with many bacteria
c. Stable bilateral, symmetric masses
d. Persistent nodal mass without antecedent 5. A 36-year-old man is seen with a right parotid
signs of infection mass. Which of the following characteristics sug-
e. Actively infectious conditions that do not gests malignancy?
respond to conventional antibiotics a. Size >2 cm
b. Previous history of parotid mass
3. You are called to see a 3-day-old, full-term infant c. Pain
who was delivered with the assistance of forceps. d. Intact cranial nerve exam
The child has a palpable mass i n the anterior e. Family history of lymphoma
neck i n the region of the sternocleidomastoid
muscle. What is the appropriate management?
a. Heat, massage, and observation
b. Fine-needle aspiration
c. Open drainage
d. Two-week course of antistreptococcal antibi-
otics
e. Surgical exploration of the neck

137
CHAPTER ONE HUNDRED AND THIRTEEN

P R I M A R Y NEOPLASMS OF T H E N E C K

1. A l l of the following are potential etiologic factors 4. After a complete history and comprehensive
in the development of paragangliomas except physical examination of the head and neck
a. Familial syndromes, such as multiple region, the next diagnostic step for a neck mass
endocrine neoplasia types IIA and IIB suspicious for a cervical lymph node with squa-
b. Autosomal-dominant inheritance pattern mod- mous cell carcinoma metastatic from an
ified by genomic imprinting unknown primary tumor is
c. History of previous radiation exposure a. Incisional biopsy
d. Living at elevated altitudes b. Excisional biopsy
e. Conditions of chronic arterial hypoxemia, c. Fine-needle aspiration biopsy
such as cyanotic heart disease d. Neck dissection
e. Close observation
2. Symptoms or signs attributable to a carotid para-
ganglioma may include all of the following except 5. The most common soft tissue sarcoma of the
a. Pulsatile tinnitus head and neck i n children is
b. Dysphagia a. Angiosarcoma
c. Hoarseness b. Chondrosarcoma
d. Palpitations c. Osteosarcoma
e. Flushing d. Rhabdomyosarcoma
e. Ewing's sarcoma
3. The cellular pattern of alternating regions of
compact, spindle cells, called Antoni type A
areas, and more loosely arranged, hypocellular
zones, called Antoni type B areas, is characteris-
tic of
a. Paragangliomas
b. Schwannomas
c. Neurofibromas
d. Fibrosarcomas
e. Synovial sarcomas

138
CHAPTER ONE HUNDRED AND FOURTEEN

LYMPHOMAS PRESENTING
I N THE HEAD A N D NECK

1. Which of the following represents a viral-associ- 4. A patient with stage I I diffuse large B-cell l y m -
ated lymphoma? phoma is treated with six cycles of GHOP-based
a. Tonsillar lymphoma and parvovirus chemotherapy followed by radiation. What is the
b. African Burkitt's lymphoma and Epstein-Barr expected percentage of freedom from disease
virus progression?
c. Hodgkin's lymphoma and HIV a. 80%
d. B-cell lymphoma and parainfluenza virus b. 60%
e. Sinonasal lymphoma and adenovirus c. 40%
d. 20%
2. A patient has lymphoma of the right tonsil. This e. 10%
patient has a 20% to 30% chance of having a syn-
chronous or metachronous involvement of what 5. A n 18-year-old woman is seen with a rapidly
other organ? enlarging neck mass that is shown on biopsy to
a. Brain be Burkitt's lymphoma. What is the initial ther-
b. Spleen apy for this patient?
c. Thyroid a. Chemotherapy alone
d. Gastrointestinal tract b. Induction chemotherapy followed by low-dose
e. Kidney radiation therapy
c. Induction chemotherapy followed by
3. A patient is seen with a stage I I , low-grade l y m - high-dose radiation therapy
phoma of the lingual tonsil. What is the initial d. Radiation therapy alone
therapy for this patient? e. Surgical resection followed by radiation
a. Partial glossectomy w i t h bilateral neck dissec- therapy
tion
b. Three to six cycles of cyclophosphamide, dox-
orubicin, vincristine, and prednisone (CHOP)-
based chemotherapy, then radiation
c. Radiation therapy alone
d. Total glossectomy w i t h bilateral radical neck
dissection
e. Six cycles of GHOP-based chemotherapy alone

139
CHAPTER ONE H U N D R E D AND FIFTEEN

R A D I A T I O N THERAPY A N D
M A N A G E M E N T OF T H E
CERVICAL LYMPH NODES

Which of the following sites of squamous cell car- 4. Which of the following is not true of hyperfrac-
cinoma is most likely to present with positive tionation?
neck nodes? a. The total dose of radiation is increased.
a. Tonsillar fossa b. The dose per fraction is decreased.
b. Lip c. A single dose is given each day.
c. Subglottic larynx d. Overall treatment time is the same as conven-
d. Nasal cavity tional therapy.
e. Glottic larynx e. Benefit is greater i n primary tumor control
than nodal control.
Although the ideal dose regimen for elective
neck irradiation (ENI) has yet to be clearly 5. Which of the following statements regarding
established, current studies support which of the combined chemotherapy and radiation therapy
following regimens? in head and neck cancer is true?
a. 2000 to 2500 cGy i n 3 to 4 weeks a. Overall 5-year survival is >75% with concur-
b. 7000 to 8000 cGy i n 4 to 5 weeks rent therapy.
c. 4500 to 5000 cGy i n 4.5 to 5.5 weeks b. Concurrent therapy increases survival by 8%
d. 1500 to 2000 cGy i n 4.5 to 5.5 weeks to 12% i n advanced cancers.
e. 1500 to 2000 cGy i n 2.5 to 3 weeks c. Adjuvant chemotherapy before radiation
increases survival by >20%.
Which of the following is a true statement d. Neoadjuvant chemotherapy followed by radia-
regarding combined radiation and surgical ther- tion increases survival by 33%.
apy for head and neck squamous cell cancer? e. Paclitaxel is the most commonly used
a. Preoperative doses of radiation are lower than chemotherapeutic agent.
postoperative doses.
b. Postoperative radiation is only directed at the
bed of the primary tumor.
c. Sensitive structures such as the larynx and
spinal cord are more easily protected with pre-
operative versus postoperative radiation.
d. Radiation therapy is not of use with extracap-
sular spread.
e. High-dose preoperative radiation decreases the
morbidity of surgical resection.

140
CHAPTER O N E H U N D R E D A N D SIXTEEN

NECK DISSECTION

1. Which of the following is not of prognostic signif- 3. Which of the following is true of selective neck
icance for recurrence after radical neck dissec- dissection for oral cavity cancer?
tion? a. Includes levels I to I I I
a. Presence of microscopic extracapsular exten- b. Also called infrahyoid neck dissection
sion c. The posterior border of the dissection is the
b. Presence of macroscopic extracapsular exten- anterior border of the sternocleidomastoid.
sion d. If the oral tongue is involved, level V should be
c. Number of involved nodes included i n the specimen.
d. Level of nodal involvement e. Contralateral neck dissection is indicated for
e. Presence of bilateral nodal disease N disease.
2 a

2. On postoperative day 1 after neck dissection, a 4. Which of the following is not true of radical neck
patient has increasing fullness of the ipsilateral dissection?
neck develop, and her wound drains put out a. The spinal accessory nerve is sacrificed.
700 m L of yellowish fluid. Which of the following b. Levels I to V are resected with the specimen.
is true? c. The internal jugular vein is preserved.
a. This complication occurs i n 10% of neck dis- d. Does not include postauricular or suboccipital
sections. nodes
b. Conservative management with drainage and e. The sternocleidomastoid muscle is sacrificed.
pressure dressings should control the condi-
tion. 5. Which of the following structures is contained i n
c. Early surgical exploration is indicated. level V?
d. High-fat diet will help control the fluid accu- a. Precricoid (Delphian) node
mulation. b. Sentinel (Virchow's) node
e. Enteral feedings should be immediately dis- c. Carotid bifurcation
continued. d. Submandibular gland
e. Phrenic nerve

141
CHAPTER ONE HUNDRED AND SEVENTEEN

SURGICAL C O M P L I C A T I O N S
OF T H E N E C K

1. When planning a neck dissection incision, which 3. Which of the following are true about radiation
is the best choice i n a patient who has under- and chemotherapy?
gone previous radiation therapy? a. Both treatments affect wound healing.
a. A modified MacFee incision because of the b. The only effects of these treatments that are
decreased risk of wound dehiscence relevant when planning surgery are the effects
b. A triradiate incision because of the excellent on wound healing.
exposure i n a neck with postradiation fibrosis c. Timing of surgery after radiation and/or
and difficult landmarks chemotherapy is an important consideration.
c. A n apron flap because it is cosmetically most d. Treatment time is the important factor affect-
appealing ing injury i n late-responding tissues, whereas
d. A triradiate incision because of the decreased fraction dose is the most important factor
vascularity after radiation therapy affecting injury i n early responding tissues.
e. A n apron incision because of the decreased e. Two of the above are correct.
risk of wound dehiscence f. Three of the above are correct.
g. A l l are correct.
2. Which of the following is not a reason to use
24 hours of perioperative antibiotics when per- 4. Chylous fistula after neck dissection can be
forming a neck dissection? treated i n all the following ways except
a. Studies have shown that longer courses of a. Head elevation
antibiotics do not affect the incidence of post- b. Pressure dressings
operative wound infection. c. Total parenteral nutrition
b. The incidence of wound infection i n neck dis- d. Instillation of doxycycline
section is significantly higher than the inci- e. Prevention
dences of wound infection i n other clean head f. Subcutaneous somatostatin injections
and neck cases. g. Modified chain triglyceride enteral diet
c. Prospective, randomized trials have found h. Reoperation
a decreased incidence of wound infections
in patients who undergo neck dissection 5. 11th nerve syndrome
and are treated w i t h 24 hours of a. Always occurs after sacrifice of the spinal
antibiotics. accessory nerve
d. The cost of treatment of postoperative b. Consists of a constellation of symptoms,
wound infections outweighs the cost of pro- including limited active shoulder abduction
phylaxis. and a constant dull ache, stiffness, or soreness
e. The neck dissection may be combined with a c. Does not occur with neck dissections that pre-
procedure that leads to spillage of oral flora serve the 11th nerve
into the wound. d. A l l of the above

142
PART N I N E

T H Y R O I D / PARAT H Y R O I D
CHAPTER ONE HUNDRED AND EIGHTEEN

DISORDERS OF T H E T H Y R O I D G L A N D

1. Otolaryngologic manifestations of hypothy- 4. A l l of the following should be administered to a


roidism include all of the following except patient with thyroid storm except
a. Sensorineural hearing loss and tinnitus a. Supportive measures to maintain adequate
b. Sinus congestion central nervous system and cardiovascular
c. Enlargement of the tongue function
d. Husky, raspy voice b. Propranolol
c. High doses of glucocorticoids
2. Thyrotropin (TSH) levels are useful i n all of the d. Purified thyroglobulin to bind excess free T4

following clinical scenarios except e. Inorganic iodine, orally or by nasogastric tube


a. A 58-year-old women previously diagnosed
with hypothyroidism, to establish the severity 5. You have just completed a total thyroidectomy
of her disease and selective neck dissection for a 48-year-old
b. Screening for hypothyroidism i n a 24 year-old woman with papillary thyroid carcinoma and
symptomatic woman metastatic disease i n her neck. She is scheduled
c. Guidance of thyroid replacement regimen i n to undergo I treatment i n 6 to 8 weeks. I n the
1 3 1

an elderly gentlemen with longstanding interim, you prescribe


hypothyroidism a. Synthroid
d. Establishing diagnosis i n a 38 year-old man b. Gytomel
with palpitations, weight loss, and increasing c. Synthroid and Gytomel
diaphoresis, who has normal T and T levels.
4 3 d. Nothing

3. Methimazole and propylthiouracil may lead to


remission of Graves' disease by
a. Decreasing the release of thyroid hormone
b. An immunosuppressive action
c. Decreasing thyroid hormone biosynthesis
d. Toxic effects on thyroid follicular cells

145
CHAPTER ONE HUNDRED AND NINETEEN

M A N A G E M E N T OF T H Y R O I D
NEOPLASMS

1. The primary blood supply for the superior 4. The primary method of spread for follicular car-
parathyroid glands is cinoma beyond the initial disease site is
a. Superior thyroid artery a. Regional metastases
b. Inferior thyroid artery b. Local extension
c. Thyrocervical trunk c. Distant metastases
d. Carotid artery
5. Where does the recurrent laryngeal nerve (RLN)
2. Unlike other malignancies, which factor has an enter the laryngeal framework?
important prognostic significance i n patients a. Deep to the inferior thyroid artery
with thyroid cancer b. Lateral to the inferior constrictor muscles
a. Gender c. Between the arch of the cricoid cartilage and
b. Regional metastases the inferior cornu of the thyroid cartilage
c. Age d. Through the cricothyroid muscle
d. Histologic type

3. The most common form of thyroid carcinoma is


a. Follicular carcinoma
b. Medullary carcinoma
c. Papillary carcinoma
d. Anaplastic carcinoma

146
CHAPTER ONE HUNDRED AND TWENTY

SURGICAL M A N A G E M E N T
OF P A R A T H Y R O I D DISORDERS

1. Most patients with primary hyperparathyroidism 4. Which of the following is the most reliable preop-
present with which of the following? erative localizing study used to predict the loca-
a. Osteitis fibrosa cystica tion of a single parathyroid adenoma?
b. Renal lithiasis a. Magnetic resonance imaging
c. Neuromuscular syndrome b. Technetium 99m sestamibi scintigraphy
d. Osteoporosis c. Computed tomography
e. Minimally symptomatic hypercalcemia d. High-resolution ultrasonography
e. Technetium 99m-thallium 201 subtraction
2. The most reliable determinant for the presence imaging
of parathyroid carcinoma is
a. Fibrotic, septate glands 5. Which of the following anatomic regions is most
b. Periglandular fibrosis likely to harbor an ectopic superior parathyroid
c. Cervical soft tissue invasion adenoma?
d. Lymphatic metastasis a. Intrathyroid
e. Cellular atypia with mitosis b. Intrathymic
c. Carotid sheath
3. Which of the following biochemical profiles is d. Anterior mediastinum
found in patients with primary hyperparathy- e. Retroesophageal
roidism?
a. Increased serum calcium, increased serum
vitamin D, decreased urinary calcium
b. Increased serum calcium, increased serum
phosphate, decreased serum vitamin D
c. Decreased serum phosphate, increased urinary
calcium, normal serum vitamin D
d. Increased serum phosphate, increased urinary
calcium, decreased vitamin D
e. Increased serum calcium, increased serum
phosphate, normal serum vitamin D

147
CHAPTER ONE HUNDRED AND TWENTY ONE

PARANASAL SINUSES: M A N A G E M E N T
OF T H Y R O I D EYE DISEASE (GRAVES'
OPHTHALMOLOGY)

Recent evidence suggests that Graves' ophthal- b. There is a positive association between pre tib-
mopathy is a result of ial myxedema and ophthalmopathy i n patients
a. Autoimmune response directed at the extraoc- with Graves' disease.
ular muscle fibers c. Thyrotoxicosis aggravates eye disease i n
b. Immunomodulation mediated by the retrobul- patients with Graves' disease.
bar fibroblasts d. Ophthalmoplegia is a temporary finding i n
c. Toxicity caused by fibroblast by-products of Graves' disease, present only i n the rapid pro-
the altered metabolic pathways i n Graves' dis- gression stage of the disease (after complete
ease and spontaneous return of eye movement
d. Intravascular plaque deposits impairing the function).
blood supply to the extraocular muscles,
resulting i n ischemic damage and fibrosis 4. When planning surgical decompression of the
orbit, the most appropriate imaging study to
Which of the following is true according to obtain is
recent evidence regarding the epidemiology of a. Thin-cut computed tomography scan of orbits
Graves' ophthalmopathy? b. Orbital echography
a. Tobacco use increases the risk for goiter but c. Magnetic resonance imaging of orbits
has not been linked to Graves' ophthalmopa- d. Nuclear imaging using single photon emission-
thy. computed tomography with 99mTc-DTPA and
b. Men with Graves' disease have a lower rate of gallium-67
ophthalmopathy.
c. Women are three times more likely than men 5. Orbital decompression, independent of the surgi-
to have Graves' disease. cal method used, can offer patients all of the fol-
d. Patients of Asian descent with Graves' disease lowing except
are more likely to have ophthalmopathy a. Reduction of proptosis
develop than those of European descent. b. Improvement of vision from reduced optic
neuropathy
Which of the following is true regarding the natu- c. Resolution of diplopia
ral history and clinical presentation of Graves' d. Improved ocular motility
disease?
a. I n most patients w i t h undiagnosed Graves' dis-
ease, ophthalmopathy is the presenting symp-
tom.

148
PART T E N

GENERAL
CHAPTER ONE HUNDRED AND TWENTY TWO

A N A T O M Y OF T H E SKULL BASE,
T E M P O R A L B O N E , EXTERNAL EAR,
A N D M I D D L E EAR

The tympanic bone contributes to the formation 4. Which artery travels i n the fallopian canal?
of all of the following except a. Anterior tympanic artery
a. External auditory meatus b. Superior petrosal artery
b. Foramen lacerum c. Inferior tympanic artery
c. Styloid process d. Superior tympanic artery
d. Eustachian tube e. Jacobson's artery
e. Glenoid fossa
5. I n a middle cranial fossa procedure, which struc-
Within the middle cranial fossa, the arcuate emi- ture is considered the principal surgical land-
nence of the superior surface of the temporal mark?
bone corresponds to a. Middle meningeal artery
a. Cochlea b. Greater superficial petrosal nerve
b. Superior semicircular canal c. Foramen lacerum
c. Tegmen tympani d. Arcuate eminence
d. Geniculate ganglion e. Bill's bar

All of the following contribute to the sensory


innervation of the external ear except
a. Jacobsen's nerve
b. Great auricular nerve
c. Auriculotemporal nerve
d. Facial nerve
e. Vagus nerve

151
1. When considering the neural projection system b. After partial cochlear deafferentation i n the
from the cochlea to cortex, the terms "tono- adult subject, reorganization of tonotopic
topic" and "cochleotopic" are often used inter- maps rapidly occurs i n the cochlear nucleus.
changeably because c. I n studies that use the aminoglycoside drug
a. The only information to reach auditory cortex amikacin to damage the cochlea, a typical
relates to tone frequency. result is a degeneration of hair cells at the
b. Processing of sound intensity and sound fre- base of the cochlea.
quency are carried out independently at the d. Plastic change after neonatal ablation of one
level of the cochlea. cochlea results i n a symmetric ascending audi-
c. There is an analogy w i t h how light spectrum tory pathway.
information is coded i n the retinotopic organi- e. Lesions to primary auditory cortex can lead to
zation of the visual system. hair cell damage i n the corresponding
d. The cochlea performs a place coding of sound cochlear areas.
frequency.
e. Cochlear hair cell damage can initiate a tonal 4. The cochleotopic (or tonotopic) projection sys-
tinnitus. tem up to the cortex can be considered the
"main-line organization" of the auditory system
2. Which statement about cellular mechanisms of because
plasticity is correct? a. I t functions to transfer to sensory cortex, as
a. The N-methyl-D-aspartate (NMDA) receptor is directly and efficiently as possible, the
mainly associated w i t h adrenergic synapses. cochlear pattern of neural activity that is
b. Long-term potentiation (LTP) of synapses has caused by acoustic stimulation.
only been experimentally observed i n neurons b. Retinotopic pathways do not have such a
of hippocampus. clearly structured organization.
c. Hebbian strengthening of synapses requires c. Such a system allows information transfer
the activation of cholinergic neurons of the between multiple sensory modalities (e.g.,
nucleus basalis. touch, vision, hearing) at subcortical levels.
d. LTP can result from alterations i n both presy- d. The sensory transduction of acoustic signals is
naptic and postsynaptic mechanisms. carried out by cochlear hair cells.
e. The NMDA receptor is slowly activated, e. There is little or no processing of sound infor-
because i t is of the metabotropic type. mation until signals reach auditory cortex.

3. Which of these comments concerning physio-


logic studies on auditory system plasticity is
correct?
a. High-frequency sensorineural hearing loss is
always the result of apical hair cell lesions i n
the cochlea

152
Chapter 123 Neural Plasticity in Otology 153

5. Which of the following statements about age- plastic (as judged by neural reorganization)
related plasticity is accurate? that i n the adult subject.
a. Experimental studies have revealed that there d. I n the adult animal, tonotopic map reorganiza-
is no significant different between plasticity i n tion has been experimentally demonstrated at
the developing auditory system compared with all levels of the auditory system up to the infe-
the mature system. rior colliculus.
b. The long-term performance of congenitally e. Age-related plasticity has been clearly demon-
deaf subjects with a cochlear prosthesis is strated i n the auditory system but does not
independent of their age at implantation. seem to be the case i n the visual system.
c. I n early development, the subcortical area of
the auditory system seems to be much more
CHAPTER ONE HUNDRED AND TWENTY FOUR

T I N N I T U S A N D HYPERACUSIS

1. Pulsatile tinnitus is uncommon i n which clinical 4. Clinicians should tell patients with chronic
condition? tinnitus
a. Pseudotumor cerebri a. Because there is no cure for tinnitus, nothing
b. Otosclerosis can be done for them.
c. Acoustic neuroma b. Because nothing can be done for tinnitus, the
d. Dural arteriovenous fistula patient should just learn to live with i t .
c. There is a good chance that their tinnitus is
2. Radiographic studies are helpful i n diagnosing all the result of a brain tumor.
of the following conditions except d. Even though there often is no cure for chronic
a. Pseudotumor cerebri tinnitus, effective management strategies are
b. Carotid artery stenosis available.
c. Glomus jugulare
d. Dural arteriovenous fistula 5. A l l of the following are common elements of
effective tinnitus management programs except
3. Postoperative tinnitus after acoustic neuroma a. In-the-ear sound generators
resection is most likely i n which scenario? b. Hearing aids
a. Preoperative tinnitus present; hearing pre- c. Stress reduction/relaxation therapy
served after surgery d. Spending 15 to 20 minutes with each patient
b. Preoperative tinnitus present; hearing not pre-
served 6. Tinnitus severity
c. Preoperative tinnitus absent; hearing pre- a. Is correlated with the matched loudness of the
served sound
d. Preoperative tinnitus absent; hearing not pre- b. Is correlated with the matched pitch of the
served sound
c. Is correlated with the patient's degree of sleep
interference
d. Is the same for most patients

154
C H A P T E R O N E H U N D R E D A N D T W E N T Y FIVE

M A N A G E M E N T OF T E M P O R A L
BONE T R A U M A

All of the following are true of otic-capsule-dis- 4. A l l of the following are acceptable methods i n
rupting temporal bone fractures except evaluating suspected CNS fistula after tympa-
a. They almost always result i n sensorineural nomastoidectomy except
hearing loss. a. Evaluate draining fluid for (3-2 transferrin
b. They have a much higher incidence of facial content
nerve palsies. b. High resolution computed tomography
c. Recent studies show that only 10% to 15% of (HRGT) with intrathecal contrast
temporal bone fractures are otic-capsule- c. Intrathecal fluorescein injection followed by
disrupting. examination of draining fluid under a Wood's
d. There is a two to four times increase i n the lamp
risk of cerebrospinal fluid fistula. d. Skull base ultrasonography

After re-examining published rates of facial nerve 5. If a suspected fistula is confirmed, the most
palsies associated with temporal bone fractures appropriate next step should be
and removing the associated sampling errors, the a. Repeat mastoidectomy with craniotomy and
true percentage of facial nerve palsies has been fistula repair
calculated to be b. Placement of a lumbar drain
a. 3% c. Prophylactic 14-day course of antibiotics with
b. 5% streptococcal///, influenzae coverage
c. 7% d. Conservative measures including bedrest with
d. 10% head elevation and stool softeners
e. 15%

The most important factor i n predicting facial


nerve recovery after temporal bone fracture is
a. Severity of injury
b. Timing of onset
c. Presence of associated infection
d. Presence of sensorineural hearing loss

155
C H A P T E R O N E H U N D R E D A N D T W E N T Y SIX

O T O L O G I C SYMPTOMS
A N D SYNDROMES

1. Which of the following statements about bullous 4. Ramsay-Hunt syndrome does not usually include
myringitis is true? a. Hearing loss
a. The infection is usually painless. b. Otalgia
b. The etiology may be viral. c. Vertigo
c. The presentation involves an sensorineural d. Facial paralysis
hearing loss (SNHL) alone. e. Dysarthria
d. The hearing loss is usually permanent.
e. The drainage is typically thick and mucopuru- 5. The different diagnosis for sudden SNHL would
lent. not include
a. Cerebellopontine angle tumor
2. Clear otorrhea from a ventilation tube may arise b. Immune mediated
from c. Barotrauma
a. Bullous myringitis d. Head trauma
b. Gustatory otorrhea e. Graves' disease
c. Spontaneous cerebrospinal fluid leak
d. Atypical tuberculosis
e. Chloroma formation

3. Aural fullness is usually not a symptom associ-


ated with
a. Eagle's syndrome
b. Patent eustachian tube
c. Perilymphatic fistula
d. Otitis media
e. Meniere's disease

156
CHAPTER O N E H U N D R E D AND T W E N T Y SEVEN

OTOLOGIC MANIFESTATIONS
OF SYSTEMIC DISEASE

1. Which of the following disorders can mimic the 4. Which of the following clinical findings would
symptoms and signs of chronic otitis media? suggest a diagnosis of otosyphilis?
a. Langerhans cell histiocytosis a. Positive Schwartze's sign (reddish hue to t y m -
b. Tuberculosis panic membrane on otoscopy)
c. Wegener's granulomatosis b. Positive Brown's sign (blanching of tympanic
d. All of the above membrane on pneumatic otoscopy)
e. None of the above c. Positive Hennebert's sign (ocular deviation
with pneumatic otoscopy)
2. What is the most common cause for the air-bone d. A l l of the above
gap that is seen on audiometric evaluation of e. None of the above
patients with Paget's disease affecting the tempo-
ral bone? 5. The histopathologic report of granulation tissue
a. Malleus fixation removed at tympanomastoidectomy indicates the
b. Stapes fixation presence of "chronic inflammation, necrosis,
c. Obliteration of the round window granulomas with multinucleated giant cells, vas-
d. Resorption of the incus culitis, and microabscesses." What is your diag-
e. All of above may occur i n different patients nosis?
f. None of the above a. Tuberculosis
b. Wegener's granulomatosis
3. What is the most common otologic manifestation c. Langerhans' cell histiocytosis
of fibrous dysplasia affecting the temporal bone? d. Sarcoidosis
a. Progressive narrowing of the external auditory e. Syphilis
canal
b. Obstruction of the eustachian tube with con-
ductive hearing loss
c. Reddish mass behind an intact tympanic
membrane
d. Facial nerve paralysis
e. Sensorineural hearing loss and vertigo caused
by inner ear involvement

157
CHAPTER ONE HUNDRED AND TWENTY EIGHT

N O I S E - I N D U C E D H E A R I N G LOSS

1. Which one of the following has not been pro- 4. Which one of the following nonauditory condi-
posed as an anatomic mechanism of noise dam- tions has not be shown to be associated with
age? chronic exposure to noise?
a. Mechanical injury a. Vertigo
b. Metabolic exhaustion b. Biologic stress
c. Ischemia c. Emotional unrest
d. Ionic poisoning d. Hypertension
e. Chronic bleeding e. Gastrointestinal disease

2. Otoacoustic emissions can be used clinically i n 5. Identify which one of the following is not a cru-
patients with noise-induced hearing loss (NIHL) cial aspect i n the regulatory control of sound lev-
to accomplish all but which one of the following? els i n the workplace.
a. To detect the initial stages of NIHL a. A hearing-conservation program if employees
b. To specify damage to the inner hair cell are exposed to sounds >85 dBA
system b. Sound levels measured on the dB SPL scale
c. To estimate the configuration of the audio- c. The equivalent continuous sound level ( L ) eq

gram principle
d. To objectively monitor the progression of d. The time-weighted average
NIHL i n instances of continued exposure e. The equal-energy principle
e. To determine the frequency/amplifying char-
acteristics of a prescribed digital hearing aid

3. Synergistic effects are likely with chronic expo-


sure to noise and all but which one of the
following?
a. Cisplatin
b. Carbon monoxide
c. Microwaves
d. Aminoglycosides
e. Whole-body or segmental vibration

158
CHAPTER ONE HUNDRED AND TWENTY NINE

A U T O I M M U N E I N N E R EAR DISEASE

Which of the following autoimmune diseases 3. Autoimmune processes may involve which of the
involves the middle ear more frequently than i t following structures?
affects the inner ear? a. The cochlear nerve
a. Systemic lupus erythematosus b. The vestibular labyrinth
b. Gogan's disease c. The cochlea
c. Wegener's granulomatosis d. The labyrinthine vasculature
d. Polyarteritis nodosa e. A l l of the above
e. Behcet's disease
4. Patients with suspected autoimmune inner ear
Autoimmune inner ear disease is best diagnosed disease (AIED) require an initial trial of pred-
in a patient with rapidly progressive hearing nisone for
loss by a. 2 weeks
a. A documented clinical response to corticos- b. 4 weeks
teroids c. 6 weeks
b. A family history of autoimmune disease d. 8 weeks
c. Positive human leukocyte antigen (HLA) e. 10 weeks
markers
d. Detection of specific circulating antibodies
e. The configuration of the audiogram

159
CHAPTER ONE HUNDRED AND THIRTY

VESTIBULAR A N D A U D I T O R Y
TOXICITY

1. Which of the following ototoxic agents does not 4. Which of the following factors has the lowest pre-
affect the basal turn of the cochlea predomi- dictive value for hearing loss i n patients undergo-
nantly? ing therapy with aminoglycoside antibiotics?
a. Gentamicin a. Mutations involving connexin 26
b. Gisplatin b. Mutations of mitochondrial RNA
c. Arsenic c. Renal insufficiency
d. Neomycin d. Combined therapy with cisplatin
e. Amikacin e. Septicemia

2. Which of the following drugs is most likely to 5. Which of the following drugs is most likely to
cause permanent sensorineural hearing loss i n damage the inner hair cells of the organ of
patients? Gorti?
a. Erythromycin a. Gentamicin
b. Vancomycin b. Furosemide
c. Furosemide c. Gisplatin
d. Gisplatin d. Carboplatin
e. Torsemide e. Vancomycin

3. Which of the following statements about amino-


glycoside antibiotics is false?
a. Aminoglycoside antibiotics may cause perma-
nent sensorineural hearing loss.
b. Permanent vestibular damage may result from
aminoglycoside therapy.
c. Serum levels are predictive of aminoglycoside
vestibular toxicity.
d. Oscillopsia is a symptom of severe vestibular
damage from aminoglycoside antibiotic treat-
ment.
e. Intratympanic therapy with gentamicin carries
a risk of permanent sensorineural hearing loss.

160
CHAPTER ONE HUNDRED AND THIRTY ONE

PHARMACOLOGIC TREATMENT
OF T H E C O C H L E A A N D L A B Y R I N T H

1. Ten days after using intratympanic gentamicin d. Patients with Meniere's disease may have
injections to treat intractable vertigo i n a 65- bilateral disease develop, precluding the ability
year-old patient with Meniere's disease, you get a to treat both ears with the same strategy.
call that your patient is i n the emergency depart- e. Researchers have not identified the exact
ment complaining of severe nausea and vomit- mechanism of action explaining how steroids
ing. What is your reaction? produce their beneficial response.
a. The patient is having severe intracerebral drug
reaction to the gentamicin and needs immedi- 3. The proven way to avoid anacusis when adminis-
ate intravenous steroids and fluid bolus to tering intratympanic gentamicin treatment is
maintain hemodynamic stability. a. To use the titration strategy of dosing, so that
b. The patient is beyond the window of adverse if a patient starts to experience hearing loss,
drug reactions for intratympanically delivered the protocol can be immediately stopped
gentamicin. The triage team should be notified b. To administer intratympanic steroids at the
that this is malingering behavior and a psychi- first signs of hearing loss
atry consult should be ordered. c. To use the low-dose microcatheter perfusion
c. The patient is experiencing acute vestibular systems as the delivery method
deafferentation syndrome. She should be reas- d. To use frequent caloric testing to determine the
sured and offered vestibular rehabilitation as moment that vestibular ablation is achieved and
an outpatient if symptoms do not resolve i n to then promptly halt further treatment
the next 2 to 3 days. e. None of the above
d. The patient is experiencing visual-vestibular
mismatch. Caloric testing should be performed 4. A patient is seen by you with severe tinnitus
to document the severity of the reaction and after going to a rock concert. He is concerned
to guide future treatment. because several family members have had a his-
e. The patient is having acute otitis media with tory of hearing loss after severe noise exposure.
central complications, a known consequence He begs you for some kind of treatment. Which
of disturbing the middle ear resident flora with of the following compounds could you use on an
the introduction of intratympanic gentamicin. off-label basis to t r y to prevent permanent noise-
Start antibiotics immediately. induced hearing loss?
a. Aspirin
2. The primary problem w i t h the use of intratym- b. Alcar
panic steroids for Meniere's disease is that c. Riluzole
a. There are no statistical data i n randomized d. D-Methionine
controlled clinical trials that show that e. A l l of the above
steroids are effective for any otologic indica-
tion related to Meniere's disease. 5. Neurotrophins have been shown to
b. Current steroid formulations are not suitable a. Cause an acceleration of the apoptotic process
to intratympanic use, because the acidity of b. To effect a tonic suppression of the apoptotic
the preparations induces stinging on injection. process
c. Steroids decrease the host response to local c. To stimulate free radical production
pathogens, resulting i n an unacceptably high d. To suppress free radical production
rate of postinjection otitis media. e. A l l of the above

161
PART E L E V E N

I N F E C T I O U S PROCESSES
CHAPTER ONE HUNDRED AND THIRTY TWO

I N F E C T I O N S OF T H E EXTERNAL EAR

1. A 36-year-old man is seen by you with a 3-day b. Place an earwick and prescribe 14 days of
history of progressive pain and itching i n his Gortisporin
right ear. This morning he found drainage from c. Topical clotrimazole cream
that ear on his pillow. Your examination con- d. Topical corticosteroid cream
firms your suspicion of otitis externa with mild
amount of creamy otorrhea. What is the best 4. You are seeing a 52-year-old diabetic man with
treatment option? possible malignant otitis externa. Your history
a. Oral antibiotics and physical examination confirm your suspi-
b. Debridement of canal and oral antibiotics cion. Your next steps include all of the following
c. Debridement of canal and o to topical anti- except
biotics a. Normalization of any hyperglycemia
d. Debridement of canal, oral antibiotics, and b. Culture of the EAG and frequent debridements
ototopical antibiotics c. Technetium-99m and gallium-67 scans of the
temporal bones
2. A 42-year-old woman is seen with 6 days of ear d. Initiation of IV antibiotics and obtain
pain and otorrhea. Over the past 24 hours, her Infectious Disease consult
ipsilateral cheek has become swollen and red. e. Proceed straight to mastoidectomy and t y m -
On examination, you see otitis externa with a panoplasty to remove granulation tissue and
moderate amount of creamy otorrhea and a mild necrotic bone
amount of erythema and edema to her ipsilateral
cheek. There is one enlarged preauricular lymph 5. While evaluating a patient for right-sided hearing
node. What is the best treatment option? loss, you discover a yellow-white mass that
a. Oral antibiotics occludes the EAG. I t does not appear to be
b. Debridement of canal and oral antibiotics invading the canal walls. You see a similar find-
c. Debridement of canal and ototopical anti- ing on the right, but to a lesser degree. What is
biotics the diagnosis?
d. Debridement of canal, oral antibiotics, and a. External auditory canal cholesteatoma
ototopical antibiotics b. Exostoses
c. Keratosis obliterans
3. A 56-year-old man is seen with a history of 3 d. Furunculosis
months of itching and pain i n his right ear. He e. Cerumen impaction
has been prescribed several otic drops for otitis
externa, but none have resolved his condition.
On examination, you see a red, mildly thickened
EAG and conchal bowl with no otorrhea or
debris i n canal. The skin i n these areas has a
slight amount of desquamation. How will you
treat him?
a. Oral antibiotics because patient has had oto-
topical therapy fail

165
CHAPTER ONE HUNDRED AND THIRTY THREE

C H R O N I C OTITIS MEDIA, MASTOIDITIS,


A N D PETROSITIS

1. Which of the following theories on the pathogen- 4. Tympanosclerosis is associated with


esis of acquired aural cholesteatoma does not a. Atherosclerosis of the internal carotid artery
exist? b. Necrosis of the tympanic membrane
a. Invagination of the tympanic membrane c. Cholesteatoma
b. Transdifferentiation d. History of otosclerosis
c. Basal cell hyperplasia e. Recurrent bouts of acute otitis media
d. Epithelial ingrowth through a perforation
e. Squamous metaplasia of middle ear epithelium 5. I t has been observed that patients with a history
of chronic otitis media with effusion have
2. Which following statements regarding infectious a. More sclerotic mastoids with decreased
and noninfectious complications of otitis media pneumatization compared with healthy sub-
is true? jects
a. Complications are usually seen as a bullous b. Less sclerotic mastoids with decreased
myringitis. pneumatization compared with healthy sub-
b. They are characterized by a perforation of the jects
lateral squamous layer. c. More sclerotic mastoids with increased
c. Chronic perforation of the tympanic mem- pneumatization compared with healthy sub-
brane, ossicular erosion, labyrinthine erosion, jects
and tympanosclerosis are major causes of d. Less sclerotic mastoids with increased
hearing loss. pneumatization compared with healthy sub-
d. On physical examination there is usually a jects.
Schwartze's sign seen on the tympanic mem- e. More sclerotic mastoids with absent pneumati-
brane. zation compared with healthy subjects
e. Chronic otitis media is a frequent sequela to
cerumen impaction.

3. The diagnosis of petrous apicitis is suspected by


a. Scintigraphy
b. Plain x-ray
c. Surgical exploration
d. Clinical grounds and computed tomography
e. Tympanometry

166
CHAPTER ONE HUNDRED AND THIRTY FOUR

C O M P L I C A T I O N S OF T E M P O R A L
BONE INFECTIONS

1. A previously healthy 15-month-old girl is seen 4. Masked mastoiditis


with a 1-day history of high fevers, irritability, a. Is often associated with a sterile, serous otor-
and left-sided facial paralysis. On examination, rhea caused by the usual history of multiple
you see a bulging tympanic membrane (TM) and antibiotic therapies
diagnose acute otitis media (AOM). What is caus- b. Can most often be treated medically
ing the facial paralysis? c. Often occurs i n patients who have not yet had
a. Acquired cholesteatoma erosion of the fallop- antibiotic therapy
ian canal d. Is a disease entity i n which patients experi-
b. Congenital dehiscent fallopian canal ence chronic but not severe postauricular pain
c. Encephalitis from intracranial extension of after multiple courses of antibiotics
infection e. Is most often associated with a retracted or
d. Viral infection causing Bell's palsy and edema perforated TM
of eustachian tube mucosa with subsequent
bacterial AOM 5. A l l of the following describe labyrinthine fistulas
except
2. I n performing a mastoidectomy for chronic otitis a. Most fistulas involve the lateral semicircular
media complications, the surgeon would benefit canal
by b. The damaging effects are most often a result of
a. Directing the mastoidectomy directly toward a cholesteatoma
the area of complication first, then completing c. Patients experience whirling vertigo, diaphore-
the remainder of the mastoidectomy sis, and extreme nausea
b. Using a diamond bur instead of a cutting bur d. Principally diagnosed with history and fistula
c. Always performing a canal-wall-down mas- test
toidectomy
d. Avoid drilling too close to the tegmen, because
this bone is extremely t h i n and friable from
the chronic infection

3. A l l of the following are characteristic of coales-


cent mastoiditis except
a. It usually evolves from acute otitis media and
mastoiditis that persists for 2 to 4 weeks.
b. It is more common i n children younger than
4 years old.
c. It tends to occur i n patients with well-devel-
oped mastoid air cell systems.
d. It occurs i n children who tend to have recur-
rent ear problems.

167
C H A P T E R O N E H U N D R E D A N D T H I R T Y FIVE

I N F E C T I O N S OF T H E L A B Y R I N T H

The most important common cause of congenital 4. Hearing loss from congenital rubella is most
hearing loss i n the United States is commonly
a. Treponema pallidum (congenital syphilis) a. Bilateral involving all frequencies
b. Rubella virus b. Bilateral involving low frequencies
c. Rubeola virus (measles) c. Bilateral involving high frequencies
d. Cytomegalovirus d. Unilateral involving all frequencies
e. Mumps virus e. Unilateral involving low frequencies

Congenital cytomegalovirus most commonly 5. Hearing loss from mumps is most commonly
causes a. Bilateral and congenital
a. Bilateral deafness b. Unilateral and congenital
b. Unilateral deafness c. Bilateral and acquired i n childhood
c. Asymptomatic infection d. Unilateral and acquired i n childhood
d. Bilateral deafness and balance problems e. Unilateral and acquired i n adulthood
e. Blindness but not hearing loss

The pathology of hearing loss from congenital


viral infections occurs primarily i n the
a. Perilymphatic structures
b. Endolymphatic structures
c. Spiral ganglia
d. Vestibular ganglia
e. A l l of the above

168
C H A P T E R O N E H U N D R E D A N D T H I R T Y SIX

T Y M P A N O P L A S T Y A N D OSSICULOPLASTY

1. Which of the following graft materials has been 4. Bony fixation of the ossicles from infection is
shown to have similar success rates with tempo- most commonly seen at
ralis fascia, but with significantly less surgical a. Stapes footplate
time and without external incision? b. Incudostapedial joint
a. Perichondrium c. Head of the malleus
b. Vein d. Incus i n the attic
c. Alloderm
d. Autologous fat 5. When desiring to place a partial ossicular recon-
e. Cartilage struction prosthesis (PORP) i n an ear that has a
chronically retracted umbo, one can
2. I n a lateral graft tympanoplasty, i t is very impor- a. Excise the malleus and stapes superstructure
tant to do all of the following except and proceed with a total ossicular reconstruc-
a. Completely remove all of the squamous tion prosthesis (TORP) placement
epithelium from the lateral surface of the t y m - b. Conduct a cochleostomy closer to the umbo
panic membrane (TM) and place a TORP to achieve a more favorable
b. Be certain to avoid overlay of the graft onto prosthesis position
the posterior canal wall c. Sever the tensor tympani tendon
c. Position the graft to have the malleus lateral to d. Trim the length of the PORP and insert into
the graft the ear as is
d. Maintain the anterior tympanomeatal angle
less than 90 degrees

3. When using a medial graft technique for repair of


a total TM perforation, the most common area
for graft failure is
a. Anterosuperior
b. Anteroinferior
c. Posterosuperior
d. Posteroinferior
e. All areas with same risk

169
C H A P T E R ONE H U N D R E D A N D T H I R T Y SEVEN

MASTOIDECTOMY

Proper placement of the C-shaped incision used 4. Which of the following are advantages of the
in simple mastoidectomy is intact canal wall mastoidectomy?
a. 1 cm anterior to the postauricular crease a. More rapid healing postoperatively
b. 1 cm posterior to the postauricular crease b. Preservation of a self-cleaning ear
c. Directly i n the postauricular crease c. In-the-canal hearing aids are well tolerated.
d. 3 cm posterior to the postauricular crease d. No limitations on water activities
e. 5 cm posterior to the postauricular crease e. A l l of these

Which structure can be found along the floor of 5. During mastoidectomy, the sigmoid sinus is
the antrum? injured. The bleeding is ultimately controlled,
a. Superior semicircular canal and the procedure is completed. After awaken-
b. Horizontal semicircular canal ing, the patient reports visual changes and per-
c. Posterior semicircular canal sistent headaches. Which of the following is not
d. Spine of Henle indicated?
e. Temporal line a. Magnetic resonance imaging
b. Magnetic resonance venography
Which of the following is best describes the facial c. Ophthalmology consultation
recess? d. Immediate surgical exploration
a. The space between the incus and malleus e. A l l of these are indicated.
b. The space between the tympanic membrane
and the chorda tympani
c. The junction of the middle fossa dura and the
sigmoid sinus
d. The space between the facial nerve and the
chorda tympani
e. A brief pause made by the surgeon to prepare
for delicate surgery around the facial nerve

170
PART T W E L V E

V E S T I B U L A R SYSTEM
CHAPTER ONE HUNDRED AND THIRTY EIGHT

A N A T O M Y OF V E S T I B U L A R E N D
O R G A N S A N D N E U R A L PATHWAYS

Which of the following statements regarding the 3. Which of the following statements regarding
efferent innervation to the vestibular periphery inner ear fluids is true?
is true? a. Endolymph resembles extracellular fluid by
a. Vestibular efferents originate i n a small having high Na and low K content.
+ +

nucleus lateral to the facial genu. b. Perilymph is high i n amino acids, especially
b. Some of these 200 neurons project both ipsi- glycine, compared with blood.
laterally and contralaterally. c. Vestibular endolymph is produced by cochlear
c. They make contact w i t h both hair cells and stria vascularis cells that are mitochondria poor.
afferents. d. Endolymph is resorbed in the endolymphatic
d. They are joined by cochlear efferents sac.
arising from the lateral superior olivary e. Perilymph leaves the inner ear by drainage
nucleus. through the eustachian tube.
e. A l l of the above
4. Which of the following depends on the integrity
Which of the following statements regarding of the visual system?
development of the human vestibular system is a. Vestibuloocular reflex
false? B. Vestibulo colic reflex
a. The otic placode develops at the seven-somite c. Optokinetic reflex
stage. d. Vestibulospinal reflex
b. Hair cells differentiate before afferent nerve e. None of the above
fibers arrive at the sensory epithelium.
c. Neural crest migrates at approximately 4 5. The vestibular nuclei do not project to which of
weeks to form the acousticofacial ganglion. the following targets?
d. Semicircular canals form from three divertic- a. Vestibular labyrinth
ula whose centers break down at approxi- b. Thalamus and cortex
mately 35 days. c. Cerebellum and precerebellar nuclei
e. The organ of Gorti is the last portion of the d. Extraocular motor nuclei and nucleus pre-
labyrinth to reach adult form at approximately positus
25 weeks. e. Reticular nuclei

173
CHAPTER ONE HUNDRED AND THIRTY NINE

PRINCIPLES OF A P P L I E D V E S T I B U L A R
PHYSIOLOGY

1. Ewald's first law is most accurately represented 4. A patient with unilateral vestibular impairment
by which statement? has left-beating nystagmus (fast phase to the left)
a. Stimulation of a semicircular canal produces following the head-shake nystagmus test. This
eye movements i n the plane of that canal. most likely suggests a defect i n which location?
b. Stimulation of a semicircular canal produces a. Left vestibular apparatus
eye movements i n a plane orthogonal to that b. Right vestibular apparatus
canal. c. Bilateral vestibular disease
c. Stimulation of a semicircular canal produces d. Central nervous system
eye movements that are unpredictable. e. None of these are correct
d. Stimulation of a semicircular canal does not
produce eye movements under normal condi- 5. Isolated loss of utricular nerve activity may
tions. result i n which of the following findings?
e. Stimulation of a semicircular canal always a. Head tilt toward the lesioned side
produces horizontal eye movements. b. Disconjugate deviation eyes such that one
pupil is elevated and one is depressed
2. If the head is pitched nose up while rolling i t c. A static conjugate counter-roll of the eyes
toward the right i n a plane 45 degrees off the rolling the superior pole of each away from the
midsagittal plane, which semicircular canal is intact utricle
most likely to be excited? d. None of these findings
a. Right anterior canal e. A l l of these findings
b. Right posterior canal
c. Left anterior canal
d. Left posterior canal
e. Left horizontal canal

3. A young woman complains that exposure of the


left ear to loud sound "makes the world flutter
up and down." She most likely has
a. Left horizontal canal dehiscence
b. Right horizontal canal dehiscence
c. Left superior canal dehiscence
d. Right superior canal dehiscence
e. Left posterior canal dehiscence

174
CHAPTER ONE HUNDRED AND FORTY

E V A L U A T I O N OF T H E P A T I E N T
W I T H DIZZINESS

1. Why does nystagmus change its direction after 4. Stereotypic eye movements for the most com-
head shaking? mon form of benign positional vertigo with the
a. Nystagmus is generally unstable and can patient i n the Dix-Hallpike position include
change direction without warning. a. Downbeat vertical nystagmus with fast tor-
b. The neural integrator only has an effect for a sional movements toward the lower ear
limited period of time. b. Upbeat vertical nystagmus w i t h fast torsional
c. Position of the head can cause nystagmus to movements toward the lower ear
change direction. c. Downbeat vertical nystagmus with fast tor-
d. Nystagmus never changes its direction after sional movements toward the upper ear
head shaking. d. Upbeat vertical nystagmus with fast torsional
e. Adaptation of the vestibular pathways movements toward the upper ear
e. Horizontal nystagmus alone, with the direc-
2. After unilateral deafferentation, which is not tion depending on the cause
true?
a. Patients tend to exhibit a head tilt to the 5. What is the mechanism of caloric stimulation of
weakened side. the labyrinth?
b. Patients tend to exhibit ocular counter-rolling. a. Heating of the horizontal canal
c. The effect of the velocity storage mechanism b. Heating of the anterior canal
is prolonged. c. Direct stimulation of vestibular afferents
d. Patients exhibit a combined horizontal/tor- d. Heating of the horizontal canal and direct
sional nystagmus. stimulation of afferents
e. Patients have increased nystagmus i n the e. None of the above
dark.

3. Appropriate uses of posturography include which


of the following (choose all that apply)?
a. Detection of malingering patients
b. Determining duration of gentamicin therapy
for Meniere's disease
c. Screening for most patients complaining of
dizziness
d. Confirming a diagnosis of perilymphatic fistula
e. Following a course of physical therapy

175
CHAPTER ONE H U N D R E D AND FORTY ONE

I M B A L A N C E A N D FALLS
I N T H E ELDERLY

1. In which room of the home do elderly individu- 5. Which of these tests is the least useful i n assess-
als fall the most often? ing balance and/or gait i n an aging patient?
a. Bathroom a. Clinical Test of Sensory Integration i n Balance
b. Kitchen (CTSIB)
c. Bedroom b. Fast Evaluation of Mobility, Balance and Fear
d. Living room (FEMBF)
e. Utility room c. Activity-Specific Balance Confidence Test
(ABC)
2. What is the most primary body balance strategy d. Auditory Brainstem Response (ABR)
that is frequently lost first i n aging patients who e. Modified Falls Efficiency Scale (MFES)
fall?
a. Ankle strategy 6. Which test i n the ENG battery is the most sensi-
b. Head strategy tive to age related changes?
c. Hip strategy a. Optokinetic
d. Shoulder strategy b. Caloric
e. Stepping strategy c. Pursuit
d. Head autorotation
3. What is the most common balance complaint of e. Positional/positioning
elderly patients with presbyastasis when seeing a
physician?
a. Vertigo
b. Disequilibrium
c. Rapid heartbeat
d. Headache
e. Vision loss

4. Which of these senses does not play a role i n bal-


ance i n the elderly?
a. Vision
b. Vestibular
c. Proprioceptive
d. Auditory
e. Gustatory

176
CHAPTER O N E H U N D R E D AND FORTY T W O

MENIERE'S DISEASE A N D O T H E R
PERIPHERAL V E S T I B U L A R DISORDERS

1. W i t h regard to transtympanic administration of 4. Which of the following statements is most likely


gentamicin i n the treatment of Meniere's disease, to be true?
which of the following is most accurate? a. Positive middle ear pressure may cause nys-
a. Hearing is always preserved. tagmus toward the ear with the higher middle
b. Complete ablation of vestibular function is not ear pressure.
always required for control of symptoms. b. Positive middle ear pressure may cause nys-
c. Single transtympanic injections of gentamicin tagmus toward the ear with the lower middle
rarely result in demonstrable vestibular ear pressure.
changes. c. Positive middle ear pressure i n one ear may
d. The preferred administration technique is cause nystagmus toward either ear.
direct injection into the peri/endolymph by d. Positive middle ear pressure will most likely
way of the horizontal canal. cause vertical, up-beating nystagmus.
e. Injections should always be done before insti- e. None of these are correct.
tuting any medical management, because the
medication use will not allow the clinician to 5. Which of these should be avoided i n treatment
monitor the response to gentamicin. of atmospheric inner ear barotrauma?
a. Bed rest
2. Which is the test of choice for the detection of b. Head elevation
otologic syphilis? c. Surgical exploration for selected cases with
a. Venereal Disease Research Laboratory (VDRL) progressive symptoms within 3 to 5 days
b. Rapid plasma reagin d. Recompression
c. Fluorescent treponemal antibody e. Close monitoring of hearing and balance
d. Lumbar puncture
e. Pathogen isolation from a perilymph sample

3. Treatment of patients w i t h suspected inner ear


fistulae should consist of the following except
a. Bed rest
b. Head elevation
c. Laxatives
d. Transtympanic steroid infusion
e. Monitoring of hearing and vestibular function

177
C H A P T E R O N E H U N D R E D A N D FORTY T H R E E

C E N T R A L V E S T I B U L A R DISORDERS

1. For a patient i n an attack of acute vertigo, which 4. A 56-year-old man is being evaluated for progres-
of the following is most suggestive of a central sive hearing loss and imbalance over the past
rather than peripheral cause? 3 years without any similar family history.
a. Mixed horizontal-torsional nystagmus Examination reveals severe bilateral sen-
b. Nystagmus that increases with removal of sorineural hearing loss, downbeat and gaze-
visual fixation evoked nystagmus, anosmia, and gait ataxia.
c. Unidirectional nystagmus that increases with Taking an extensive history reveals no other
rightward gaze and decreases with leftward diagnostic clues. To arrive at the correct diagno-
gaze sis, you should next
d. Absence of a head thrust sign a. Perform an MRI of the head looking for iron
accumulation around the brainstem and cere-
2. An elderly man awoke with vertigo and vomiting bellum
and is being evaluated i n the emergency depart- b. Perform an MRI of the head with gadolinium
ment. Examination reveals direction-changing looking for enhancement along the eighth
nystagmus and profound gait imbalance. After nerves
determining that the patient is otherwise stable, c. Perform brainstem auditory evoked responses
the next step i n management is to (BAERs) looking for slowing along the conduc-
a. Administer vestibular suppressants and admit tion pathways
the patient for observation d. Perform a GT scan of the head with contrast
b. Begin a course of oral steroids and acyclovir looking for a posterior fossa tumor
and schedule outpatient follow-up i n 1 week e. Perform genetic testing for spinocerebellar
c. Prescribe vestibular suppressants and arrange ataxias
for outpatient vestibular rehabilitation
d. Obtain an emergent head computed tomogra- 5. A 45-year-old pilot is seen with recurrent attacks
phy (GT) or magnetic resonance imaging (MRI) of vertigo associated with aural fullness and fluc-
e. Perform a lumbar puncture and obtain an MR tuating unilateral low-frequency hearing loss. He
angiogram. has been forced to stop working and now rarely
leaves the house, has lost interest i n his hobbies,
3. You are evaluating a 42-year-old woman with has difficulty sleeping, and has had some crying
recurrent spontaneous attacks of vertigo for the spells. Appropriate management is to
past 4 years that generally last several hours, are a. Place h i m on a salt-restricted diet and start a
associated with light sensitivity, improve with diuretic
sleep, and occasionally are associated with aural b. Perform intratympanic gentamicin therapy
fullness or headache. There is a history of c. Begin a migraine prophylactic medication
motion sickness. Physical examination, audio- d. Refer h i m for psychotherapy
gram, and vestibular testing are normal. The e. Aggressively treat both his Meniere's disease
most likely diagnosis is and his depression
a. Benign paroxysmal positioning vertigo (BPPV)
b. Vestibular migraine
c. Meniere's disease
d. Multiple sclerosis
e. Transient ischemic attacks

178
CHAPTER ONE H U N D R E D A N D FORTY FOUR

SURGERY F O R V E S T I B U L A R DISORDERS

1. Which of the following statements about surgery 4. Which of these clinical scenarios is not compati-
for benign paroxysmal positional vertigo is not ble with a fluctuating peripheral vestibular disor-
true? der and thus better treated with vestibular
a. Singular neurectomy is only appropriate for rehabilitation rather than surgery?
posterior semicircular canal disease. a. Persistent disabling motion-provoked vertigo
b. Singular neurectomy is not effective if the and chronic disequilibrium after a significant
condition is due to cupulolithiasis as opposed vestibular crisis
to canalithiasis. b. Episodic spells of spontaneous vertigo associ-
c. Posterior semicircular canal occlusion has ated with subjectively fluctuating hearing loss,
widely replaced singular neurectomy, because tinnitus, and fullness i n one ear
it is technically simpler and safer. c. Episodic spells of spontaneous vertigo i n a
d. Surgery for BPPV is rarely required, because patient who has normal caloric responses and
particle-repositioning maneuvers are effective a profound hearing loss after a failed stapedec-
in most cases. tomy 10 years ago
e. Although generally safe, posterior semicircular d. Intermittent disequilibrium, aural fullness, and
canal occlusion may result i n sensorineural a mixed hearing loss after penetrating trauma
hearing loss i n the operated ear. to one ear
e. Persistent positional vertigo unresponsive to
2. Any of the following options could be undertaken particle repositioning maneuvers
in an effort to relieve ongoing episodic spells of
vertigo after a retrolabyrinthine vestibular 5. When performing an endolymphatic sac opera-
neurectomy except tion, the surgeon should not
a. Middle fossa vestibular neurectomy a. Counsel the patient that further procedures
b. Transmastoid labyrinthectomy may be required to control the vertigo
c. Transcanal labyrinthectomy b. Decompress the sigmoid sinus
d. Vestibular rehabilitation c. Violate the medial wall of the endolymphatic
e. Intratympanic gentamicin injections sac
d. Close the wound unless absolutely certain that
3. When attempting to identify the offending ear the sac is properly identified and fully decom-
that is causing episodic vertigo of peripheral ori- pressed
gin, which is the most reliable clinical symptom e. Skeletonize the facial nerve
or finding?
a. Unilateral tinnitus
b. Subjective aural fullness
c. A unilateral weakness of caloric response on
vestibular testing
d. Asymmetric rotary chair responses
e. A long-standing profound sensorineural hear-
ing loss i n one ear

179
C H A P T E R O N E H U N D R E D A N D F O R T Y FIVE

•jj VESTIBULAR A N D BALANCE


,M R E H A B I L I T A T I O N THERAPY:
P R O G R A M ESSENTIALS

One of the two principle goals of vestibular and 4. The following statements about the techniques
balance rehabilitation therapy (VBRT) is commonly used i n VBRT are all true except
a. Advance the central vestibular compensation a. Habituation and adaptation both rely on
process repeated head movements to give the desired
b. Eliminate symptoms associated with head and effect.
eye movements b. The outcome of a VBRT program is equally as
c. Serve as an exposure therapy technique for effective with both individually customized
patients with anxiety disorders exercises and generic exercises as long as the
d. Improve the control of eye movements during patient is active.
head movement by use of central preprogram- c. Central preprogramming plays a role i n the
ming use of substitution exercises.
e. Increase the speed of the eye movements dur- d. The basic goal of adaptation exercises is to
ing head rotation improve the functionality of the vestibuloocu-
lar reflex.
After a sudden, stable vestibular insult, the initial e. Maintenance activities are important w i t h all
symptoms are significantly reduced with i n 72 the patients i n a VBRT program, but this is
hours as a result of especially true for those with cerebellar
a. The patient remaining still and not aggravating involvement.
the injury
b. Combination of the suppressive medication 5. Which of the following patient groups is likely to
and the patient at bed rest benefit from a VBRT program, but have a limited
c. The naturally occurring process of tonic rebal- outcome?
ancing at the level of the vestibular nuclei a. Those with stable peripheral lesions reporting
d. The combined process of adaptation and head movement-provoked symptoms
habituation b. Patients with purely spontaneous events of
e. Use of significant and early substitution exer- vertigo lasting several hours per event
cises c. Patients with classic posterior canal benign
paroxysmal positional vertigo
One of the primary reasons for determining d. Patients with bilateral vestibular involvement
whether a patient's symptoms occur sponta- with the primary goal of improvement i n gait
neously or are provoked by head or eye move- e. Those who are post head injury with periph-
ment is eral and central involvement
a. To assist i n the determination of the medica-
tion to be used i n a suppression format
b. As the first point of decision as to whether
VBRT would be likely as a primary manage-
ment technique
c. As an indication that the lesion is unstable i n
nature
d. Both b and c
e. As the main indicator of central nervous sys-
tem involvement

180
PART T H I R T E E N

FACIAL NERVE
C H A P T E R O N E H U N D R E D A N D F O R T Y SIX

TESTS OF FACIAL NERVE F U N C T I O N

1. A Sunderland class I injury is also called 4. I n electromyographic facial nerve monitoring i n


a. Axonotmesis acoustic tumor surgery, which of the following
b. Axonotomy statements is not true?
c. Neuropraxia a. Spontaneous " t r a i n " activity may follow ther-
d. Neurotomy mal stimulation.
e. Neurotmesis b. The lower the threshold for stimulation at the
brainstem at the end of the procedure, the
2. A nerve that has suffered a Sunderland class I I better the prognosis.
injury can still produce muscle contraction if c. There is professional consensus that monitor-
electrically stimulated ing is beneficial.
a. Proximal to the lesion, within 3 days of the d. Monitoring cannot be performed if the anes-
injury thesiologist uses paralytic agents.
b. Distal to the lesion, w i t h i n 3 days of the injury e. Stimulus artifact sounds are useful i n confirm-
c. Proximal to the lesion, w i t h i n 24 hours of the ing proper equipment function.
injury
d. Distal to the lesion, w i t h i n 24 hours of the 5. The topognostic test most likely to be abnormal
injury in the early phase of Bell's palsy is
e. Proximal to the lesion, within 1 week of the a. Schirmer's test
injury b. Stapedius reflex
c. Electrogustometry
3. Which of the following statements about facial d. Salivary flow
nerve testing i n Bell's palsy is true? e. Salivary pH
a. Electroneurography is useless after more than
6 weeks have elapsed.
b. Electroneurography predicts outcome better
than nerve excitability testing.
c. Electromyographic fibrillation potentials are a
good prognostic sign.
d. Maximum stimulation testing yields useful
prognostic information i n cases of partial
paralysis.
e. Patients whose nerves become totally inex-
citable may still recover completely.

183
CHAPTER O N E H U N D R E D AND FORTY SEVEN

C L I N I C A L DISORDERS OF T H E FACIAL
NERVE

The most likely pathogenesis for Bell's palsy is 3. The most common cause of facial paresis and
a. Epstein-Barr virus paralysis i n infants is
b. Autoimmune ischemic neuropathy a. Hemifacial microsomia
c. Herpes simplex virus (HSV) b. Subarachnoid hemorrhage
d. Varicella-zoster virus c. Congenital absence of depressor labii muscle
e. Heterotopic viruses e. Mobius syndrome
d. Birth trauma
In the interpretation of electrical testing of the
facial nerve, i t is important to perform both elec- 4. Facial paralysis of pregnancy is associated with
troneuronography (ENoG) and electromyography a. Preterm labor
(EMG) because b. Low birth weight
a. Desynchronization of motor units can cause c. Perinatal abnormalities
artifactual decrease i n the compound action d. Preeclampsia
potential, but voluntary motor responses are e. First trimester pregnancy
preserved.
b. Desynchronization of motor units can cause a 5. The defining symptom of Melkersson-Rosenthal
normal compound action potential but syndrome is
decreased voluntary motor responses. a. Fissured tongue
c. Recruitment of motor units can cause artifac- b. Bilateral facial paralysis
tual increase i n the compound action poten- c. Recurrent facial paralysis
tial, but voluntary motor responses are d. Rapid onset facial swelling
preserved. e. Orofacial edema
d. Recruitment of motor units can cause a nor-
mal compound action potential but decreased
voluntary motor responses.
e. EMG can detect injury of the nerve proximal
to the testing site, whereas ENoG cannot.

184
CHAPTER ONE H U N D R E D AND FORTY EIGHT

I N T R A T E M P O R A L FACIAL NERVE
SURGERY

1. A 45-year-old man with a 30-dB conductive left 4. A 35-year-old man is kicked by a horse and has a
hearing loss and recurrent facial paralysis seems temporal bone fracture with direct immediate
to have a mass extending from the geniculate facial nerve trauma restricted to the area of the
ganglion to the mid-stapes region. Which surgical geniculate ganglion. Three weeks later pure-tone
approach is best? thresholds are 85 dB with 8% word understand-
a. Canal-wall-down mastoidectomy ing, and there is marked vestibular paresis on
b. Canal-up mastoidectomy the affected side. The opposite side is normal. To
c. Translabyrinthine explore and manage the facial nerve, what is the
d. Middle cranial fossa best approach?
e. Retrolabyrinthine a. Translabyrinthine
b. Middle cranial fossa
2. What is the principal advantage of the middle c. Transmastoid
cranial fossa approach to the facial nerve com- d. Transotic
pared with the retrolabyrinthine or retrosigmoid e. Retrolabyrinthine
approach?
a. Lower incidence of sensorineural hearing loss 5. I n performing end-to-end anastomosis of the
b. Lower incidence of postoperative infection facial nerve after resecting the area of the genic-
c. Access to the labyrinthine segment without ulate ganglion for direct trauma, the greatest risk
impairing hearing to hearing loss occurs during
d. Less operative time a. Resection of the damaged geniculate ganglion
e. Less brain retraction b. Mobilization of the tympanic segment
c. Mobilization of the labyrinthine segment
3. The major drawback of the transmastoid d. The anastomosis
approach to the facial nerve is the e. The application of the tissue glue
a. Difficult exposure of the stylomastoid foramen
region
b. Incidence of postoperative conductive hearing
loss
c. Incidence of postoperative sensorineural hear-
ing loss
d. Limited access to the geniculate ganglion
e. Limited access to the middle-ear segment

185
PART F O U R T E E N

A U D I T O R Y SYSTEM
CHAPTER ONE H U N D R E D AND FORTY NINE

COCHLEAR A N A T O M Y A N D CENTRAL
A U D I T O R Y PATHWAYS

1. Which statement is true regarding the cochlear their afferent nerve fibers rather than on their
endolymph and perilymph? cell body.
a. Perilymph is contained within the scala media c. IHGs receive 90% of the afferent innervation,
and exhibits a high K and a low Na ion con-
+ +
and their efferent nerve fibers synapse on
centration. their cell body.
b. Endolymph is contained within the scala d. OHGs receive 90% of the afferent innervation,
media and exhibits a high K and a low Na ion
+ +
and their efferent nerve fibers synapse on
concentration. their cell body.
c. Endolymph exhibits a negative electrical e. IHGs and OHGs are richly innervated by auto-
potential relative to perilymph. nomic nerve fibers.
d. Perilymph is contained within the scala t y m -
pani and exhibits a positive electrical potential 4. The first obligatory nerve relay center for GN
relative to endolymph. V I I I nerve afferent fibers is which nucleus i n the
e. Endolymph and perilymph communicate by GNS?
way of the helicotrema. a. Scarpa
b. Rosenthal
2. Which cell type of the stria vascularis exhibits c. Spiral
convoluted basolateral cell membranes that con- d. Olivary
tain ion transporting enzymes? e. Cochlear
a. Intermediate
b. Basal 5. Which of the following statements is true for the
c. Melanocyte hair cell stereocilia?
d. Marginal a. Stereocilia are true cilia-like structures,
e. Interdental decrease i n length toward the cochlear apex,
and do not contain mechanoelectrical trans-
3. Inner hair cells (IHG) and outer hair cells (OHG) duction channels.
show a different type of nerve innervation. b. Stereocilia are microvilli-like structures, increase
Which is the most correct description of their in length toward the cochlear apex, and contain
innervation? mechanoelectrical transduction channels.
a. IHGs receive 90% of the afferent innervation, c. Individual stereocilia are not connected to one
and their efferent nerve fibers synapse on another within the bundle.
their afferent nerve fibers rather than on their d. Auditory hair cells i n the adult mammal
cell body. cochlea contain stereocilia and a kinocilium.
b. OHGs receive 90% of the afferent innervation, e. Stereocilia are motile, because they contain
and their efferent nerve fibers synapse on an actin and myosin cytoskeleton.

189
CHAPTER ONE HUNDRED AND FIFTY

M O L E C U L A R BASIS OF A U D I T O R Y
PATHOLOGY

1. Which of these statements regarding the catego- 4. Which of the following statements regarding the
rization of nonsyndromic deafness is correct? homozygous Myo7a mutant mouse is false?
a. DFNA designates autosomal-recessive nonsyn- a. I t is frequently thought of as a model of Usher
dromic deafness. syndrome type I B .
b. DFNB designates autosomal-dominant nonsyn- b. I t often develops degenerative disease of the
dromic deafness. retina.
c. DFN designates X-linked nonsyndromic deaf- c. I t is often found to exhibit inner ear abnor-
ness. malities.
d. DFNMt designates mitochondrial nonsyn- d. I t lacks melanosomes i n cells of RPE.
dromic deafness. e. A l l of the above statements are true.
e. A l l of these statements are correct.
5. Mutation of which gene may be associated with
2. Which of the following statements about outer perilymphatic gusher during stapes surgery?
hair cells is true? a. Connexin 26
a. There are roughly three times as many outer b. Pou3f4
hair cells as inner hair cells. c. KNCQ1
b. Only - 5 % of auditory nerve fibers innervate d. KCNE1
outer hair cells. e. BSND
c. Outer hair cells are more susceptible to noise
damage than inner hair cells.
d. Outer hair cells are more susceptible to dam-
age by aminoglycoside use than inner hair
cells.
e. A l l of these are true.

3. Which of the following is not commonly associ-


ated with Usher syndrome type I?
a. Congenital deafness
b. Vestibular dysfunction
c. Congenital aural atresia
d. Retinitis pigmentosa
e. Genetic mutations

190
C H A P T E R O N E H U N D R E D A N D FIFTY O N E

E L E C T R O P H Y S I O L O G I C ASSESSMENT
OF H E A R I N G

1. Absence of transient evoked otoacoustic emis- 4. Although the click is the stimulus used most fre-
sions (TEOAEs) i n a child is most consistent quently to evoke the ABR, this stimulus has a
with which of the following conditions? broad acoustic spectrum. Despite the relative
a. Normal hearing lack of frequency specificity, click-evoked ABR
b. Sensorineural hearing loss thresholds correlate most strongly with audio-
c. Conductive hearing loss metric thresholds i n which of the following fre-
d. Auditory neuropathy quency regions?
e. b and c a. 250 to 500 Hz
f. b and d b. 1000 Hz
c. 2000 to 4000 Hz
2. Historically, electrocochleography (ECoG) has d. 4000 to 8000 Hz
been used to help with the diagnosis of Meniere's e. None of the above
disease. Which of the following measures were
considered consistent w i t h the diagnosis of this 5. How does the electrically evoked auditory brain-
disorder? stem response (EABR) as recorded from adult
a. An abnormally large negative peak ( N l ) cochlear implant users differ from the acousti-
b. Absent ECoG i n the face of normal hearing cally evoked version of this response (ABR)
sensitivity when i t is recorded from adult subjects with mild
c. An enlarged SP:AP ratio to moderate amounts of hearing loss?
d. A reduced SP:AP ratio a. The peaks of the EABR are typically larger i n
e. None of the above amplitude and shorter i n latency than the cor-
responding peaks of the ABR.
3. The auditory brainstem response is a measure of b. The peaks of the EABR are typically smaller i n
the synchronized firing of neurons within the amplitude and longer i n latency than the cor-
auditory pathways of the low brainstem. Wave V responding peaks of the ABR.
of the auditory brainstem response (ABR) prima- c. The latency of wave V of the EABR
rily reflects neural activity from which of the fol- changes significantly with stimulation level,
lowing structures? whereas the latency of wave V of the ABR does
a. The auditory nerve not.
b. The cochlear nucleus d. The latency of wave V of the EABR does not
c. The superior olivary complex change significantly with stimulation level,
d. The lateral lemniscal track whereas the latency of wave V of the ABR
e. The inferior colliculus does.
f. The auditory m i d brain e. a and d
g. The auditory cortex f. b and d

191
CHAPTER ONE HUNDRED AND FIFTY T W O

D I A G N O S T I C A N D REHABILITATIVE
AUDIOLOGY

1. The difference between speech detection thresh- 3. Which of the following is most correct regarding
old (SDT) and speech reception threshold the use of the stapedial reflex i n the evaluation
(SRT) is of facial paralysis?
a. SRT requires the listener to repeat the pre- a. I t helps distinguish between neoplastic and
sented word. nonneoplastic causes of facial paralysis.
b. SDT is usually 8 to 9 dB higher than the pure b. I t helps distinguish between viral and nonviral
tone average (PTA). causes of facial paralysis.
c. SRT usually coincides with the PTA. c. I t helps distinguish between a proximal and
d. SDT can only be obtained with air distal lesion of the facial nerve.
conduction. d. I t is an important prognostic tool i n the evalu-
e. There is no difference between the terms. ation of iatrogenic facial nerve injury.
e. I t is not useful i n the evaluation of facial paral-
2. Which of the following statements about masking ysis.
is true?
a. Masking refers to the removal of all visual 4. Which component of the ABR is the most robust
clues when evaluating speech discrimination. and persists with significant degrees of hearing
b. Masking should be used routinely with bone- loss?
conduction testing when threshold levels a. Wave I
between ears are asymmetric. b. Wave I I
c. A masking dilemma frequently occurs when a c. Wave I I I
patient has very severe SNHL i n one ear and d. Wave IV
normal hearing i n the other ear. e. Wave V
d. Overmasking is a potential complication of
audiometry that results when a patient has 5. Which of the following is a simple and straightfor-
tinnitus develop after exposure to a loud ward audiometric indicator of pseudohypoacusis?
masking signal. a. Disagreement between the 500-, 1000-, and
e. A l l of these are true. 2000-Hz PTA threshold and SRT
b. Bilateral absence of the stapedial reflex
c. Absence of evoked otoacoustic emissions
d. A normal ABR
e. An abnormal ABR

192
CHAPTER O N E H U N D R E D AND FIFTY T H R E E

AUDITORY NEUROPATHY

1. Each of the following are characteristic findings b. Hearing aids generally offer long-term success-
of auditory neuropathy except ful auditory rehabilitation for most patients
a. Normal otoacoustic emissions and/or normal with auditory neuropathy.
cochlear microphonics c. Hearing aids may damage the cochlea i n
b. Abnormal or absent auditory brainstem patients with auditory neuropathy.
responses (ABRs) d. Cochlear implantation restores neural syn-
c. Enhancement of the auditory nerve on post- chrony for patients with auditory neuropathy.
gadolinium contrast T l magnetic resonance e. The complication rates of cochlear implanta-
imaging scan tion i n auditory neuropathy are comparable to
d. Absent stapedial reflexes standard cochlear implantation complication
e. Poor speech recognition scores on audiogram rates.

2. Which of the following is true regarding auditory 4. The human inner ear has some unique features
neuropathy i n children? in terms of the inner hair cells (IHC) and the
a. Auditory neuropathy is often diagnosed by outer hair cells (OHC). The afferent enervation
current newborn hearing screening protocols. to the OHC vs IHC can best be described as fol-
b. Most children with auditory neuropathy are lows
initially seen with an associated peripheral a. IHC, 50%;OHC, 50%
neuropathy. b. IHC, 75%; OHC, 25%
c. Currently, cochlear implantation is contraindi- c. IHC, 25%; OHC 75%
cated i n the auditory rehabilitation of children d. IHC, 5%; OHC 95%
with auditory neuropathy. e. IHC, 95%; OHC, 5%
d. Neonatal hypoxia and hyperbilirubinemia are
risk factors associated with the development 5. There are characteristic evoked potential
of auditory neuropathy. responses (cochlear microphonic, CM; auditory
e. Auditory neuropathy is transmitted to children brainstem response, ABR) for patients with audi-
by way of an X-linked inheritance pattern. tory neuropathy that are independent of the
degree of hearing loss. Which item is most likely
3. I n considering auditory rehabilitation for to be observed i n a child with auditory neuro-
patients with auditory neuropathy, which of the pathy?
following statements is false? a. Phase-reversing CM absent and ABR normal
a. When conducting a hearing aid trial, maximal b. Phase-reversing CM present and ABR normal
benefit should be attempted by decreasing c. Phase-reversing CM present and ABR abnor-
background noise and improving the mal
signal-to-noise ratio. d. Phase-reversing CM absent and ABR abnormal

193
C H A P T E R O N E H U N D R E D A N D FIFTY F O U R

EVALUATION A N D SURGICAL
M A N A G E M E N T OF C O N D U C T I V E
H E A R I N G LOSS

1. Maximum conductive hearing loss occurs when 4. I n a 7-year-old girl, stapes fixation is encoun-
a. The incudostapedial j o i n t is eroded behind an tered during a mastoidectomy for cholesteatoma.
intact tympanic membrane. Stapes manipulation should be done
b. The middle ear is filled with a thick effusion. a. Immediately with soft tissue sealing of the
c. The tympanic membrane is completely perfo- opening to the inner ear.
rated. b. Staged after the ear heals (6-9 months)
d. The round and oval windows are obliterated c. Once the child is old enough to participate i n
with otosclerosis. the decision for surgery
e. The external canal is blocked by cerumen. d. Never. A hearing aid or bone-anchored hearing
aid (BAHA) is a better solution.
2. To optimize hearing w i t h an ossicular prosthesis
a. Use a Cervital prosthesis 5. Ossiculoplasty should be considered when
b. Use titanium prosthesis a. The preoperative speech reception threshold
c. Stage ossiculoplasty 6 to 9 months after (SRT) is greater than 30 dB or when the dam-
removing cholesteatoma aged ear is more than 15 dB less than the con-
d. Use cartilage interposed between the tympanic tralateral ear.
membrane and the prosthesis b. The preoperative SRT is less than 30 dB and
e. Place the prosthesis perpendicular to the t y m - the opposite ear is greater than 15 dB less
panic membrane and under minimal tension than the contralateral ear.
beneath the drum c. The external auditory canal is occluded by
large osteomas resulting i n a 15-dB conductive
3. The most commonly encountered ossicular hearing loss
abnormality i n chronic otitis media is d. The tympanic membrane has failed a medial
a. Malleus head fixation grafting.
b. Erosion of the head of the incus
c. Erosion of the lenticular process of the incus
d. Stapes superstructure erosion
e. Calcification of the lateral mallear ligament

194
CHAPTER ONE HUNDRED AND F I F T Y FIVE

S E N S O R I N E U R A L H E A R I N G LOSS:
EVALUATION A N D M A N A G E M E N T
I N ADULTS

Well-defined risks that enhance the likelihood of 4. Which of the following statements regarding sud-
aminoglycoside ototoxicity include all the follow- den sensorineural hearing loss and acoustic neu-
ing except roma is a false statement?
a. Presence of renal disease a. Approximately 10% of patients with acoustic
b. Increased duration of therapy neuroma are initially seen w i t h sudden sen-
c. Increased age sorineural hearing loss.
d. Malnutrition b. Approximately 1% of patients with sudden sen-
e. Concomitant administration of loop diuretics sorineural hearing loss have acoustic neuroma.
c. Recovery of hearing after steroid therapy indi-
Factors influencing the development of noise- cates that acoustic neuroma is not the etiology
induced hearing loss include all the following of the sudden hearing loss.
except d. There is no relationship between tumor size
a. Intensity of offending sound and sudden sensorineural hearing loss.
b. Duration of offending sound e. Gadolinium-enhanced magnetic resonance
c. Frequency of offending sound imaging is a more sensitive test than auditory
d. Age of patient brainstem response for small acoustic neuro-
e. Continuous versus intermittent sound mas.

Which of the following treatment options for sud- 5. Low-frequency sensorineural hearing loss is fre-
den sensorineural hearing loss is most widely quently seen i n all of the following disorders
accepted and most likely to be effective? except
a. Antiviral drugs a. Benign intracranial hypertension
b. Anticoagulation b. Endolymphatic hydrops
c. Steroid therapy c. Presbycusis
d. Carbogen therapy d. Basilar migraine
e. Hypaque administration e. Syphilis

195
C H A P T E R O N E H U N D R E D A N D F I F T Y SIX

OTOSCLEROSIS

1. Otosclerosis is a disease that 4. On physical examination


a. Is common to all a. A Schwartze sign is present i n most patients.
b. Occurs only i n females b. The tympanic membrane is opaque.
c. Is unique to the otic capsule c. Tuning forks are important i n establishing the
d. Is found at birth conductive component of the hearing loss.
e. Occurs mostly i n males d. Patients may have blue sclera.
e. A white forelock is common.
2. Otosclerosis causes which type of hearing loss?
a. A purely conductive hearing loss 5. The first widely used surgery to correct conduc-
b. A progressive hearing loss tive hearing loss was
c. A purely sensorineural hearing loss a. Stapedectomy by Shae
d. A profound hearing loss b. Stapes mobilization by Rosen
e. All of the above c. Small fenestra stapedectomy by Lempert
d. One stage tympanoplasty by Schuknecht
3. Otosclerosis typically e. One stage fenestration by Lempert
a. Has its onset of hearing loss i n the fifth decade
b. Is more common i n females than males by a
ratio of 2:1
c. Occurs equally i n males and females
d. Usually presents as a sudden hearing loss
e. Is associated with vertigo

196
CHAPTER ONE HUNDRED AND FIFTY SEVEN

SURGICALLY I M P L A N T A B L E H E A R I N G
AIDS

List at least five limitations of traditional hearing 4. For what patients is an osseointegrated titanium
aids that can theoretically be overcome (or fixture bone-conduction aid (e.g., the Entific
improved on) by implantable hearing aids. Bone-Anchored Hearing Aid [BAHA]) appro-
priate?
Why can an implanted hearing aid that directly
drives the incus and/or stapes generate louder 5. What are (relative) contraindications for implan-
perceived sound with less distortion using less tation and use of a BAHA?
battery power than is possible with a traditional
aid?

Why is the malleus neck cut during implantation


of a Totally Integrated Cochlear Amplifier (TICA)
implantable hearing aid?

197
PART F I F T E E N

COCHLEAR IMPLANTS
CHAPTER ONE HUNDRED AND FIFTY E I G H T

•{J PATIENT EVALUATION A N D DEVICE


7M SELECTION FOR C O C H L E A R
IMPLANTATION

Which of the following statements is not true? 4. Which of the following is a reasonable expecta-
a. Up to 50% of all nonsyndromic sensorineural tion for a child with the onset of deafness of less
hearing loss can be attributed to a mutation i n than 1 year who is implanted between the ages
a gap junction protein. of 4 and 5 years?
b. Genetic syndromal deafness is the leading a. Communication skill development at rates
cause of sensorineural hearing loss. similar to normal-hearing peers
c. Auditory neuropathy is a hearing disorder i n b. Attendance at a school with minimal support
which normal cochlear outer hair cell function services
is present i n conjunction with abnormal audi- c. Improvement i n speech perception with good
tory neural responses, resulting i n poor neural closed-set performance but limited open-set
synchrony. abilities
d. Prenatal infection w i t h TORCH organisms can d. Reduced dependence on visual cues for com-
result i n reduced ganglion cell counts and munication
abnormal positions of the facial nerve.
e. Bilateral temporal bone fractures resulting i n 5. Factors that affect cochlear implant performance
deafness can be rehabilitated with cochlear in children include all of the following except
implantation. a. Age at implantation
b. Hearing experience
Current adult selection criteria for cochlear c. Presence of other disabilities
implantation include all of the following except d. Parent and family support
a. Severe or profound hearing loss with a pure- e. Motivation to hear
tone average (PTA) of 50 dB
b. Aided scores on open-set sentence tests of 6. Which of the following current speech-processing
<50% strategies is unique to the MED-EL system?
c. No evidence of central auditory lesions a. Advanced combination encoder strategy
d. One- to three-month preoperative trial of b. Spectral peak extraction
hearing aid use c. N-of-m pulsatile strategy
d. Continuous interleaved sampling
Which of the following is a contraindication to e. Pulses with the high-resolution strategy.
cochlear implantation?
a. An adult with prelingual onset of severe-
to-profound hearing loss
b. Age >65 years
c. A narrow internal auditory canal
d. A child w i t h chronic suppurative otitis media
e. Auditory neuropathy

201
H
C H A P T E R O N E H U N D R E D A N D FIFTY NINE

M E D I C A L A N D SURGICAL
CONSIDERATIONS I N COCHLEAR
IMPLANTS

1. The medical evaluation of a cochlear implant 4. Recurrent otitis media i n a young child
(GI) candidate includes a. Is an absolute contraindication to GI
a. General health b. Frequently leads to meningitis i n implanted
b. Imaging studies (computed tomography or children
magnetic resonance imaging) c. Must be controlled at the time of surgery
c. Determination of appropriate expectations d. Cannot be treated with ventilation tubes if GI
d. Degree and duration of hearing loss is being considered
e. A l l of the above e. Becomes more frequent after GI

2. The most frequent complication of GI surgery 5. Most cochlear ossification


involves a. Precludes GI
a. Skin flap b. Is associated with bad results i n GI
b. Mastoid cavity infection c. Fills the basal turn of the cochlea
c. Facial nerve paresis d. Is limited to the round window area
d. Perilymph fistula e. Is associated w i t h bent or broken GI elec-
e. Meningitis trodes

3. Selection criteria for pediatric GI candidates


include all of the following except
a. Family support and appropriate expectations
b. Oral habilitation opportunities
c. Medical clearance for general anesthesia
d. No benefit from hearing aids
e. Physiologic age >12 months i n most cases

202
CHAPTER ONE HUNDRED AND SIXTY

C O C H L E A R I M P L A N T S : RESULTS,
OUTCOMES, A N D REHABILITATION

1. Compared with cochlear implantation of the bet- 4. I n terms of cost-effectiveness, cochlear implanta-
ter-hearing ear, performance after implantation tion is
of the poorer hearing ear shows what differ- a. Not cost-effective but covered by insurance
ences? companies
a. Implantation of the better-hearing ear leads to b. Not cost-effective but too beneficial to deny
better outcomes. c. Highly cost-effective with a cost per quality-
b. Implantation of the worse-hearing ear leads to adjusted life-year analysis
better outcomes. d. Only cost-effective i n adults with post-lingual
c. Implant performance w i t h either ear is statisti- deafness
cally equivalent.
d. Implantation of the poorer hearing ear should 5. Cochlear implantation i n the elderly
be performed only i n cases of deafness caused a. Should not be performed because of the
by meningitis. increased risk of morbidity
b. Should only be performed before the age of 65
2. Which of the following is most important vari- years
able for implant performance i n children? c. Is not cost-effective and should not be per-
a. Type of multichannel device chosen formed
b. Implantation of the better-hearing ear over the d. Is cost-effective based on large gains i n health-
worse-hearing ear related quality of life
c. Intelligence quotient (IQ) of child
d. Duration of deafness

3. After cochlear implantation i n children, the rate


of language development
a. Is slower i n implanted children than normal-
hearing children
b. Is equal i n implanted children than normal-
hearing children
c. Is faster i n implanted children than normal-
hearing children
d. Is dependent on age at implantation

203
PART S I X T E E N

SKULL BASE
C H A P T E R O N E H U N D R E D A N D SIXTY O N E

DIAGNOSTIC A N D INTERVENTIONAL
NEURORADIOLOGY

1. For which condition is magnetic resonance imag- 4. One or more of the following are true of the
ing (MRI) not the imaging modality of choice? imaging of a glomus jugulare tumor
a. Conductive hearing loss a. The tumor produces bone destruction simulat-
b. Sensorineural hearing loss ing a malignant tumor.
c. Intracranial meningeal disease b. I t is always a very vascular tumor.
d. Perineural spread of tumor c. A large tumor may have feeding arteries from
multiple major intracranial and extracranial
2. The ideal embolic agent for embolizing a vessels.
chemodectoma is d. Preoperative embolization may be valuable to
a. Gelfoam decrease blood loss at surgery.
b. A tissue adhesive ("glue") e. Multiple chemodectomas may be detected on
c. Polyvinyl alcohol foam (PVA) MRI studies.
d. Metallic coils
5. One or more of the following statements regard-
3. One or more of the following rationales make ing intracranial meningiomas is true
valuable the addition of cerebral blood flow (CBF) a. Fifty percent of meningiomas are located over
studies to the balloon occlusion test (BOT) the convexities and i n the parasagittal and
a. It is important to define areas of ischemia and parafalcine regions; 40% originate from sites
infarction preoperatively. along the skull base or tentorium; 10% are i n
b. The BOT is only "positive" when CBF drops to other locations.
less than 20 mL/100 g brain tissue per minute b. Meningiomas are always hypervascular at
during temporary carotid occlusion. angiography, although they may appear to be
c. CBF studies can determine the potential for less vascular on computed tomography and
clot propagation and embolization after vascu- MRI.
lar occlusion. c. Meningiomas should be embolized preopera-
d. CBF studies may define the risk of cerebral tively to decrease blood loss at surgery.
ischemia from decreased cardiac output or d. A provocative injection i n a feeding artery
hypotension following permanent vascular with 1% lidocaine may avoid inadvertent cra-
occlusion. nial nerve palsy with embolization.

207
C H A P T E R O N E H U N D R E D A N D SIXTY T W O

T E M P O R A L B O N E NEOPLASMS A N D
LATERAL C R A N I A L BASE SURGERY

1. The smallest acceptable procedure to remove a 4. High-grade neoplasms extending from the exter-
squamous cell carcinoma localized to the nal auditory canal to involve the medial meso-
osseous external auditory canal is tympanum are best managed w i t h
a. Localized resection of the skin of the external a. Resection of the external auditory canal with
auditory canal with frozen-section margins mastoidectomy to remove tumor i n the meso-
b. Sleeve resection of the external auditory tympanum, followed by radiotherapy
canal, including the tympanic membrane b. Subtotal temporal bone resection followed by
c. Lateral temporal bone resection radiotherapy
d. Subtotal temporal bone resection c. Concomitant cisplatin/5-fluorouracil
e. Total temporal bone resection chemotherapy and electron beam radio-
therapy
2. The Fisch type A infratemporal fossa dissection d. Extended temporal bone resection with sacri-
is designed to permit fice of the carotid artery and facial nerve fol-
a. Exploration and resection of the petrous apex, lowed by radiation therapy
clivus, and superior infratemporal fossa e. Stereotactic radiosurgery
b. Resection of squamous cell carcinomas of the
external auditory canal invading the mesotym- 5. Which surgical approach is usually most appro-
panum priate to remove glomus tympanicum tumors
c. Access i n resection of lesions involving the (paragangliomas) whose borders are not entirely
jugular bulb and vertical petrous carotid and visible through the tympanic membrane?
posterior infratemporal fossae a. Transtympanic
d. A l l of the above b. Transcanal
e. None of the above c. Fisch type A
d. Lateral temporal bone resection
3. Rhabdomyosarcoma of the temporal bone is opti- e. Extended facial recess
mally treated with
a. Lateral temporal bone resection
b. Total temporal bone resection with complete
tumor removal
c. Subtotal temporal bone resection alone for
surgical debulking
d. Chemotherapy and radiotherapy with surgi-
cal debulking or total tumor removal when
possible
e. Chemotherapy and radiotherapy without
surgery

208
C H A P T E R O N E H U N D R E D A N D SIXTY T H R E E

LBJ E X T R A - A X I A L NEOPLASMS I N V O L V I N G
,M THE ANTERIOR A N D MIDDLE CRANIAL
FOSSA

The 5-year survival for both squamous cell carci- 4. Which American Joint Commission of Cancer
noma and adenocarcinoma is (AJCC) 2002 stage of maxillary sinus cancer is
a. 15% to 25% described as: Tumor invasion of posterior maxil-
b. 30% to 40% lary sinus, subcutaneous tissue, floor or medial
c. 40% to 50% wall of orbit, pterygoid fossa, and ethmoid
d. 50% to 70% sinuses?
e. 70% to 85% a. T\
b. T 2

Perineural spread is found i n two-thirds of which c. T 3

histologic subtype of anterior skull base tumor? d- T


4 a

a. Squamous cell carcinoma e- T


4 b

b. Adenoid cystic
c. Esthesioneuroblastoma 5. Which AJCC 2002 stage of nasal cavity and
d. Sinonasal undifferentiated carcinoma ethmoid sinus is described as: Tumor with
e. Rhabdomyosarcoma extension into the anterior orbital contents,
minimal extension into anterior cranial
Which tumor type has the highest 5-year sur- fossa, pterygoid plates, and sphenoid, frontal
vival percentage? sinus?
a. Squamous cell carcinoma a. T x

b. Sarcoma b. T 2

c. Sinonasal undifferentiated sinonasal carcinoma c T 3

d. Mucosal melanoma d- T 4 a

e. Adenocarcinoma e. T dh

209
C H A P T E R O N E H U N D R E D A N D SIXTY F O U R

SURGERY OF T H E A N T E R I O R
A N D M I D D L E C R A N I A L BASE

1. Which cranial nerves pass through the superior 4. I n the early postoperative period after a com-
orbital fissure? bined transfacial/frontal craniotomy approach,
a. I l l , IV, V - l , and V I the patient experiences progressive neurologic
b. I I , I I I , IV, and V I deterioration, confusion, and obtundation after
c. I l l , IV, V - l , and V2 blowing his nose. The next course of action
d. I l l , IV, V-2, and V I should be
e. I I , I I I , IV, and V-2 a. Reduce inhaled oxygen concentration
b. Increase CSF drainage rate
2. The maxillary branch of the trigeminal nerve c. Stat head computed tomography scan
(V2) travels through which foramen i n the floor d. Administer a benzodiazepine
of the middle cranial fossa? e. A l l of the above
a. Superior orbital fissure
b. Foramen rotundum 5. The frontal branch of the facial nerve runs deep
c. Foramen ovale to which of the following structures?
d. Foramen spinosum a. Temporal parietal fascia
e. Foramen lacerum b. Superficial layer of the deep temporal fascia
c. Deep layer of the deep temporal fascia
3. Brain relaxation techniques that may be used d. Periosteum of the zygomatic arch
during anterior skull base surgery include e. Temporal fat pad
a. Mannitol
b. Cerebral spinal fluid (CSF) drain
c. Hyperventilation
d. Corticosteroids
e. A l l of the above

210
C H A P T E R O N E H U N D R E D A N D SIXTY FIVE

EXTRA-AXIAL N E O P L A S M
OF T H E P O S T E R I O R FOSSA

Which of the following statements regarding 3. Which of the following statements regarding the
neurofibromatosis and acoustic tumors is translabyrinthine approach is true?
true? a. Large tumors are inadequately exposed with
a. Bilateral acoustic tumors are diagnostic of this approach.
NF-1. b. The facial nerve i n the internal auditory canal
b. Bilateral acoustic tumors are diagnostic of (IAG) fundus is typically obscured i n this
NF-2. approach.
c. Bilateral optic meningiomas are diagnostic of c. Bill's bar helps localize the facial nerve i n the
NF-1. porous acusticus.
d. Cafe-au-lait spots are characteristic of NF-2. d. The dura is best opened after exposing the dis-
e. Acoustic tumors cannot occur i n NF-1. tal IAG but before uncovering the posterior
fossa dura.
Which of the following statements regarding e. The superior vestibular nerve is posterior to
cerebellopontine angle meningiomas is true? the facial nerve i n the IAG.
a. They are usually centered on the porous acus-
ticus. 4. The characteristic symptoms of cyclophos-
b. They are usually eccentric to the porous acus- phamide (CPA) malignancies are
ticus. a. Rapid symptom progression i n multiple CPA
c. Hyperostosis is uncommon. nerves
d. They typically arise from the meninges cover- b. Sudden sensorineural hearing loss
ing V I I I . c. Disequilibrium
e. They typically arise from the dural-glial junc- d. Retroorbital pain
tion. e. Retroauricular pain

211
CHAPTER ONE H U N D R E D AND SIXTY SIX

AUDITORY BRAINSTEM IMPLANTS

1. The target region for the auditory brainstem 4. Users of the multichannel auditory brainstem
implant is the implant have experienced all of the following
a. Interstitial nucleus of Gajal except
b. Dorsal and ventral cochlear nuclei a. Variations i n pitch sensations across elec-
c. Zona inserta of the cochlear nerve trodes
d. Roof of the fourth ventricle b. Open-set speech discrimination
e. Superior and medial vestibular nuclei c. Auditory sensations
d. Extra-auditory sensations
2. The optimum surgical approach for the auditory e. A l l have been experienced
brainstem implant is the
a. Middle cranial fossa 5. Which of the following statements regarding
b. Retrosigmoid open-set speech discrimination after multichan-
c. Suboccipital nel auditory brainstem implantation is true?
d. Transcochlear a. Average auditory brainstem implantation per-
e. Translabyrinthine formance equals that of multichannel cochlear
implants.
3. I n contrast to traditional multichannel cochlear b. Only closed-set discrimination has been
implants, the multichannel auditory brainstem achieved.
implant c. When open-set speech discrimination occurs,
a. Has a substantially higher complication rate it is evident within the first 3 months after
b. Cannot realize significant speech discrimina- hook-up.
tion d. Ultimate performance also depends on learn-
c. Achieves maximum results more quickly ing or brain accommodation.
d. Is under investigational FDA protocol e. I t correlates positively with nonauditory
e. Causes no nonauditory sensations effects.

212
C H A P T E R O N E H U N D R E D A N D SIXTY SEVEN

TRANSNASAL ENDOSCOPIOASSISTED
SURGERY OF T H E SKULL BASE

1. For masses involving midline skull base struc- 4. Profuse bleeding is encountered when the exter-
tures such as the clivus and sella regions, the nal clival dura is incised during a surgery when a
most ideal endoscopic approach for resection is transnasal endoscopically assisted approach is
which of the following? used to access the clivus. The source of this
a. Transmaxillary bleeding is most likely
b. Transseptal a. The cavernous sinus
c. Transnasal direct b. The basilar venous plexus
d. Transethmoidal c. The internal carotid artery
e. Transpalatal d. The vertebral artery

2. A l l of the following structures are located within 5. A patient undergoes resection of a skull base
the pterygopalatine fossa except mass with a transnasal endoscopically assisted
a. Pterygopalatine ganglion approach and complains of visual changes i n the
b. Vidian nerve recovery room and is ultimately found to have
c. Internal maxillary artery optic nerve damage. The injury to the optic
d. The maxillary nerve nerve most likely accompanied injury i n which
e. The anterior ethmoidal artery of the following areas
a. The optic foramen
3. A l l of the following statements regarding b. The anterior ethmoid air cells
transnasal endoscopically assisted approaches for c. The lamina papyracea
repair of cerebral spinal fluid (CSF) fistulas are d. The superior-lateral sphenoid sinus wall
true except e. The inferior-lateral sphenoid sinus wall
a. They allow for precise localization of bony and
tissue defects.
b. They allow for direct repair of bony and tissue
defects.
c. They allow for visualization of intranasal fluo-
rescein after intrathecal injection.
d. They may involve repair with free mucope-
riosteal grafts from the inferior or middle
turbinate or the nasal septum.
e. They require total middle turbinate resection
for access.

213
C H A P T E R O N E H U N D R E D A N D SIXTY E I G H T

I N T R A O P E R A T I V E M O N I T O R I N G OF
C R A N I A L NERVES I N N E U R O T O L O G I C
SURGERY

1. When using auditory brainstem response (ABR) 4. Methods based on recording compound nerve
monitoring i n the operating room, what should action potentials (CNAP) have what potential
be done when at the time of the craniotomy? advantage over EMG monitoring?
a. Switch to the use of analog rather than digital a. They are more sensitive.
filtering b. They can be used even if the patient is para-
b. Change the filter settings to a wider frequency lyzed.
range c. They are insensitive to artifact from bipolar
c. Increase the rate of stimuli for eliciting the cautery.
ABR d. They are insensitive to artifact from anes-
d. Obtain a control ABR from the contralateral thetic agents.
ear e. They can more accurately detect manipulation
e. Obtain a new intraoperative baseline of the nerve.

2. One of the electrodes for facial nerve monitoring 5. Surgical manipulation near which of the follow-
is typically placed i n the orbicularis oculi near ing nerves has the potential to cause brady-
the lateral can thus. How can a response from CN cardia?
V I be distinguished from CN VII? a. CN V
a. CN V I will have a longer latency and higher b. CN V I
amplitude. c. CN V I I
b. CN V I will have a longer latency and lower d. CN V I I I
amplitude. e. CN IX
c. CN V I will have a shorter latency and higher
amplitude.
d. CN V I will have a shorter latency and lower
amplitude.
e. If placed correctly, the electrode should only
record signals from CN V I I .

3. I n large vestibular schwannomas, which of the


following was found to be related to poor postop-
erative facial nerve outcome?
a. Large-amplitude spontaneous activity
b. Change i n pattern of electromyography (EMG)
from responsive to silent
c. Occurrence of "burst" activity
d. Low-amplitude train activity
e. "Popcorn" activity

214
C H A P T E R O N E H U N D R E D A N D SIXTY N I N E

R A D I A T I O N THERAPY
OF T H E C R A N I A L (SKULL) BASE

The ability to change the target of a beam 4. What is the best treatment option for a patient
instantly is a characteristic of which method of with early-stage nasopharyngeal carcinoma?
radiation therapy? a. Radiation therapy only
a. Linear-accelerator (LINAG)-based b. Complete surgical resection only
b. Gamma knife unit c. Complete surgical resection followed by radia-
c. Proton beam radiotherapy tion therapy
d. Cyberknife (photon beam) d. Preoperative chemotherapy followed by com-
e. Intensity-modulated radiation therapy (IMRT) plete surgical resection
e. Chemotherapy and radiation therapy
Which type of radiosurgery is based on the
underlying principle that each broad radiation 5. High-dose conformal radiotherapy to the skull
beam is divided into a number of smaller "beam- base, as may be required to treat skull base chor-
lets," which are then added to form a dose distri- domas, can result i n which of the following com-
bution that is tailored to the shape of the target? plications?
a. Linear-accelerator (LINAG)-based a. Optic neuropathy
b. Gamma knife unit b. Memory loss
c. Proton beam radiotherapy c. Hypoadrenalism
d. Cyberknife (photon beam) d. Right-sided hemiparesis
e. IMRT e. Left-sided hemiparesis

For which of the following tumors should com-


plete surgical resection always be followed by
adjuvant radiation therapy?
a. Juvenile angiofibroma
b. Esthesioneuroblastoma
c. Nasopharyngeal carcinoma
d. Temporal bone paraganglioma
e. Meningioma

215
PART S E V E N T E E N

PEDIATRIC OTOLARYNGOLOGY
CHAPTER O N E H U N D R E D AND SEVENTY

GENERAL C O N S I D E R A T I O N S

1. Airway obstruction i n newborns may cause rapid 4. Often the first sign of hypoxia i n a neonate is
ventilatory fatigue because a. Tachycardia
a. Their diaphragm is low i n type I muscle fibers. b. Bradycardia
b. Their diaphragm is low i n type I I muscle c. Hypotension
fibers. d. Hypertension
c. Of relative low compliance of the chest wall e. Apnea
d. Of their relatively low basal metabolic rate
e. They have a low rest tone while sleeping. 5. Of the following physical signs, which is the best
estimate that a young infants blood volume is
2. The laryngeal chemoreflex (LGR) causing laryn- adequate?
gospasm is most sensitive to a. Heart rate
a. Water b. Mean arterial blood pressure
b. Saline c. Color
c. Acid d. Temperature
d. Base e. Percent hemoglobin saturation
e. Pressure

3. Because newborn cardiac muscle has fewer con-


tractile fibers and more connective tissue, car-
diac output is most dependent on which of the
following
a. Preload
b. Afterload
c. Rate
d. Systolic pressure
e. Diastolic pressure

219
CHAPTER ONE H U N D R E D AND SEVENTY ONE

DEVELOPMENTAL A N A T O M Y

1. What are the contents of the carotid sheath? 4. Which portions of the ossicular chain derive
a. The common carotid artery (including the from the first branchial arch?
internal and external carotid arteries); cranial a. Stapes
nerves IX, X, X I , and X I I ; and the ansa cervi- b. Short processes of the malleus and incus
calis c. Long processes of the malleus and incus
b. The internal jugular vein; the common carotid d. Short process of the malleus and long process
artery (including the internal and external of the incus
carotid arteries); and cranial nerves X, X I , and
XII 5. Which portions of the ossicular chain derive
c. The common carotid artery (including the from the second branchial arch?
internal and external carotid arteries) and cra- a. Stapes
nial nerves IX, X, and X I b. Stapes suprastructure and long processes of
d. The common carotid artery (including the the malleus and incus
internal and external carotid arteries); the c. Stapes suprastructure and short processes of
internal jugular vein; and cranial nerves IX, X, the malleus and incus
and X I d. Malleus and incus

2. What is the most reliable way to differentiate the 6. What are the embryologic origins of the laryngeal
internal from the external carotid artery i n the cartilages?
neck? a. First branchial arch
a. The internal carotid artery has no branches i n b. Second branchial arch
the neck. c. Third branchial arch
b. The internal carotid artery lies anterior to the d. Fourth, fifth, and sixth branchial arches
external carotid artery.
c. The external carotid artery has no branches i n 7. What are the clinical features of hemifacial
the neck. microsomia?
d. The external carotid artery lies anterior to the a. Mandibular hypoplasia, microtia, and aural
internal carotid artery. atresia
b. Zygomatic and mandibular hypoplasia
3. What are the branches of the thyrocervical c. Zygomatic hypoplasia, microtia, and aural
trunk? atresia
a. Superior thyroid, inferior thyroid, and supras- d. Orbital and zygomatic hypoplasia
capular arteries
b. Superior and inferior thyroid arteries and cer- 8. What is the pathogenesis of hemifacial micro-
vical artery somia?
c. Inferior thyroid, ascending cervical, a. Positional plagiocephaly
transverse cervical, and suprascapular b. In utero vascular injury
arteries c. Abnormalities of fusion of the branchial
d. Ascending, transverse, and descending cervi- arches
cal arteries d. Unknown

220
Chapter 171 Developmental Anatomy 221

9. Complete injury of the accessory nerve i n the 10. Which of the following masses present as midline
right supraclavicular fossa results i n masses of the neck?
a. Inability to turn the head to right a. Branchial cyst and carotid body tumor
b. Inability to turn the head to the left b. Branchial cyst and thyroglossal duct cyst
c. Inability to turn the head to the left and shrug c. Thyroglossal duct cyst and dermoid cyst
the right shoulder d. Pharyngocele and laryngocele
d. Inability to shrug the right shoulder
CHAPTER ONE H U N D R E D AND SEVENTY T W O

ANESTHESIA

The premedication drug of choice for children 4. What receptor type has been implicated i n post-
ages 8 months to 8 years is operative nausea and vomiting?
a. IV midazolam a. Dopamine
b. IV diazepam b. Acetylcholine
c. IM ketamine c. Histamine
d. Rectal methohexital d. Serotonin
e. Oral midazolam e. A l l of the above

During tympanoplasty, which inhalational agent 5. I n premature infants, elective procedures should
should be turned off before closure of the middle be delayed until 55 weeks to lower the risk of
ear or even avoided entirely? which complication of anesthesia?
a. Nitrous oxide a. Cardiopulmonary arrest
b. SevofTurane b. Malignant hyperthermia
c. Halothane c. Apnea
d. Isoflurane d. Bronchospasm
e. Desflurane e. A l l of the above

Which inhalational agent has an irritant effect on


the airway, making i t contraindicated for airway
procedures such as bronchoscopy?
a. Nitrous oxide
b. Sevoflurane
c. Halothane
d. Isoflurane
e. Desflurane

222
CHAPTER O N E H U N D R E D AND SEVENTY THREE

C H A R A C T E R I S T I C S OF N O R M A L A N D
A B N O R M A L POSTNATAL C R A N I O F A C I A L
G R O W T H A N D DEVELOPMENT

Craniofacial growth is believed to be 4. Early sphenooccipital closure resulting i n a short


a. Genetically programmed cranial base, small foramen magnum and micro-
b. Mediated primarily by cartilage cephaly with frontal bossing, sunken bridge of
c. Controlled entirely by a feedback system the nose, and midface hypoplasia is seen i n
between the frontal matrix and bone and car- which of the following entities?
tilage a. Down syndrome
d. Concentrated i n growth centers b. Treacher-Collins syndrome
e. A complex multifactorial phenomenon c. Isolated Pierre Robin sequence
d. Stickler syndrome
Which synchondrosis is the principle growth car- e. Achondroplasia
tilage of the basicranium during childhood?
a. Sphenooccipital 5. Which of the following statements about open-
b. Sphenoethmoidal mouth posture (OMP) i n children is correct?
c. Intersphenoidal a. I t alters dentofacial growth.
d. Sphenofrontal b. I t is an indication for adenoidectomy.
e. Frontoethmoidal c. I t is associated with a posterior crossbite.
d. I t indicates nasal airway blockage.
Craniofacial abnormalities associated with Down e. I t may be a normal part of development.
syndrome include all of the following except
a. Brachycephaly
b. Midface hypoplasia
c. Wide and long palate
d. Reduced interorbital distance
e. Small ears

223
H
CHAPTER ONE H U N D R E D AND SEVENTY FOUR

VASCULAR TUMORS
A N D MALFORMATIONS
OF T H E H E A D A N D N E C K

1. Which of the following statements regarding 4. Which of the following statements regarding vas-
hemangioma is true? cular malformation is true?
a. Hemangioma is always present at birth. a. Vascular malformation often present by 6
b. Hemangioma will grow with the child. months of age.
c. There is an equal distribution of hemangioma b. There is a rapid growth followed by slow reso-
between boys and girls. lution i n the life cycle of vascular malforma-
d. Hemangioma is more common i n African tion.
Americans. c. Vascular malformations are more common i n
e. Hemangioma grows rapidly during the first 6 girls.
to 8 months of life. d. Vascular malformations are divided into slow-
flow or fast-flow lesions.
2. Which of the following statements regarding the e. Systemic steroid is the first line of medical
management of hemangioma is true? treatment.
a. A l l hemangiomas respond to corticosteroid
treatment. 5. Which of the following statements regarding vas-
b. Systemic interferon is the first line of medical cular malformation is incorrect?
treatment for hemangioma. a. Lymphatic malformation grow commensu-
c. Combined treatment with corticosteroid and rately with the child.
interferon is recommended i n the manage- b. The two strategies for the management of l y m -
ment cervicofacial hemangioma. phatic malformations are sclerotherapy and
d. The usual dosage of corticosteroid is surgical resection.
1 mg/kg/day. c. The 19th century term for "capillary malfor-
e. The empiric dose for IFN is 2 to 3 million mation" is "port-wine stain."
units/m .
2
d. Ligation or proximal embolization of feeding
vessels is the treatment choice for AVM.
3. Which of the following statements regarding sub- e. Venous malformations are easily compressible
glottic hemangioma is incorrect? and expand when the affected area is depend-
a. Subglottic hemangioma usually presents i n the ent or after a Valsalva maneuver.
first 6 months of life.
b. Diagnosis of subglottic hemangioma is based
on findings at the time of laryngoscopy, and
often biopsy is not required.
c. The most common presentation is a left-sided
subglottic hemangioma.
d. There is a higher risk of having a subglottic
hemangioma i n children with cervicofacial
hemangioma i n the "beard" distribution.
e. A l l children with subglottic hemangiomas will
response to systemic corticosteroids if treated
long enough.

224
C H A P T E R O N E H U N D R E D A N D S E V E N T Y FIVE

C R A N I O F A C I A L SURGERY
FOR C O N G E N I T A L A N D
A C Q U I R E D DEFORMITIES

Premature fusion of the sagittal suture results i n 4. Delaying surgical intervention is acceptable i n
which craniofacial abnormality? which of the following cases?
a. Brachycephaly a. Corneal exposure
b. Acrocephaly b. Increased intracranial pressure
c. Trigonocephaly c. Kleeblattschadel (clover-leaf skull)
d. Scaphocephaly d. Maxillary hypoplasia
e. Plagiocephaly e. None of the above

Which of the following syndromes exhibits auto- 5. Distraction osteogenesis


somal-dominant inheritance and often results i n a. Refers to creating a controlled, rapid fracture
brachycephaly, midface hypoplasia, class I I I mal- between bony segments
occlusion, and normal intelligence? b. Induces the formation of new bone between
a. Apert's syndrome distracted segments
b. Pfeiffer's syndrome c. Is a new technique i n orthopedics that seems
c. Grouzon's syndrome promising for adaptation to craniofacial sur-
d. Jackson-Weiss syndrome gery
e. Saethre-Chotzen syndrome d. Requires external hardware to execute the
fracture
Which of the following is caused by mutations e. Allows for two-dimensional skeletal lengthening
in genes that encode type I I and/or type X I
collagen?
a. Carpenter syndrome
b. Stickler syndrome
c. Velocardiofacial syndrome
d. Treacher-Collins syndrome
e. Facio-oculo-auriculo-vertebral spectrum

225
C H A P T E R O N E H U N D R E D A N D S E V E N T Y SIX

CLEFT LIP A N D PALATE

1. What forms the primary palate? 4. The most common complication after repair of a
a. I t forms as an outgrowth of the incisive fora- cleft palate is
men. a. Postoperative bleeding
b. Fusion of the palatine shelves b. Oronasal fistula
c. Fusion of the medial nasal prominences c. Velopharyngeal insufficiency
d. Fusion of the lateral nasal prominences d. Wound infection
e. Fusion of the maxillary prominences e. Airway obstruction

2. The velopharyngeal sphincter is composed of all 5. Some centers advocate a staged approach to cleft
of the following except palate repair to improve outcomes of which goal
a. Levator veli palatine of palatoplasty?
b. Palatopharyngeus a. Development of functional occlusion
c. Superior pharyngeal constrictor b. Preservation of midface growth
d. Middle pharyngeal constrictor c. Creation of competent velopharyngeal valve
e. Muscularis uvulae for swallowing
d. Creation of competent velopharyngeal valve
3. According to the author, the most important for speech
aspect of cleft lip repair is e. All of the above
a. Reorientation and reconstitution of orbicularis
oris around the entire oral cavity
b. Creating a philtral ridge height of at least
12 m m
c. Using a lip adhesion preliminary procedure to
provide sufficient tissue for reconstruction
d. Complete correction of all nasal deformity
during the initial procedure
e. None of the above

226
CHAPTER O N E H U N D R E D A N D SEVENTY SEVEN

VELOPHARYNGEAL D Y S F U N C T I O N

Which of the following statements regarding velo- 4. Which of the following statements is false?
cardiofacial syndrome is false? a. Hypernasality may be the result of mislearning
a. The internal carotid arteries may be found i n and not true velopharyngeal insufficiency.
an aberrant medial position i n the oro- b. Nasal emissions may occur with select
pharynx. phonemes.
b. A l l patients with velocardiofacial syndrome c. Palatal lifts elevate the soft palate and can be
have cardiac anomalies. used when palate length is adequate.
c. Pharyngeal hypotonia is a feature of velocar- d. The conditions referred to i n a and b are best
diofacial syndrome. treated with speech therapy.
d. Microdeletions of the q l l region of chromo- e. Nasometers measure airflow orally versus
some 22 have been found i n patients with nasally and display a ratio of the two.
velocardiofacial syndrome.
e. Microdeletions for velocardiofacial syndrome 5. Which of the following statements is false?
may overlap with those of DiGeorge syn- a. The sphincter pharyngoplasty involves the
drome. palatopharyngeus posterior tonsillar pillar
muscles.
Which of the following statements regarding b. Pharyngeal flaps are effective i n reducing
velopharyngeal closure patterns is true? hypernasality, have been used for decades to
a. Maximum lateral wall motion usually occurs correct velopharyngeal insufficiency, and are
above the levator eminence often tailored to match the gap size.
b. The vertical pattern is the most common. c. Posterior-wall augmentation has many theo-
c. Closure patterns are described by the shape of retic advantages; i n practice, i t is useful for
the gap while closing and the relative contri- small, central gaps of 3 m m or less.
bution of the pharyngeal walls. d. Apnea is most common i n sphincter pharyn-
d. Studies have shown that vertical closure goplasty surgery.
length is short. e. Superiorly based pharyngeal flaps involve a
e. Pharyngeal wall motion is rated by its move- myomucosal flap using the constrictor muscle.
ment to the midline.

Which of the following statements regarding sub-


mucous clefts is true?
a. A bifid uvula is diagnostic of a submucous cleft.
b. The presence of a submucous cleft palate is a
strict contraindication to adenoidectomy.
c. A submucous cleft has a bifid uvula, muscular
diastasis of the soft palate, and loss of the
anterior nasal spine.
d. Almost all children w i t h submucous cleft are
symptomatic for velopharyngeal insufficiency.
e. A flat soft palate or absence of the bulge from
uvula contraction during palatal elevation is a
finding of occult submucous clefts.

227
CHAPTER ONE H U N D R E D A N D SEVENTY EIGHT

CONGENITAL MALFORMATIONS
OF T H E NOSE

1. The embryogenesis of choanal atresia generally 3. Which of the following is not an anatomic defor-
is believed to be caused by mity i n choanal atresia?
a. Failure of nasal canalization i n the 10th a. Narrow nasal cavity
intrauterine week b. Medial obstruction caused by thickening of the
b. Failure of rupture of the nasobuccal vomer
membrane i n the fifth to sixth intrauterine c. Lateral bony obstruction i n the medial ptery-
week goid plate
c. Nasal placode invagination i n the third to d. Membranous obstruction
fourth intrauterine week e. Bony obstruction
d. Neural crest maldevelopment i n the 12th
intrauterine week 4. Pyriform aperture stenosis typically presents
e. Failure of ectoderm migration i n the eighth a. Immediately after birth
week b. With the first feeding
c. During the first few months of life
2. A patient with a single upper incisor and narrow d. When the first teeth erupt
bony nasal pyriform aperture most likely has a e. During adolescence
form of
a. CHARGE association 5. Dermoids are composed of which germinal layers?
b. Holoprosencephaly a. Mesoderm and ectoderm
c. Down syndrome b. Endoderm and mesoderm
d. Hydrocephalus c. Endoderm, mesoderm, and ectoderm
e. Goldenhar syndrome d. Mesoderm only
e. Ectoderm only

228
CHAPTER ONE H U N D R E D AND SEVENTY NINE

PEDIATRIC C H R O N I C SINUSITIS

1. Children are more likely to have sinusitis than 4. Absolute indications for endoscopic sinus sur-
adults because gery include all of the following except
a. They have an immature immune system. a. Complete nasal airway obstruction from
b. Have more upper respiratory tract viral infec- polyps
tions b. Antrochoanal polyps
c. Smaller ostia to the sinuses c. Chronic rhinosinusitis that persists despite
d. A l l of the above maximum medical management
e. None of the above d. Mucoceles
e. Orbital abscess
2. Which of the following is true regarding imaging
of sinuses i n children? 5. Which of the following is true regarding facial
a. Plane films are valid form of imaging. growth i n children after endoscopic sinus sur-
b. Plane films are required to make the diagnosis gery?
of sinusitis. a. Endoscopic sinus surgery does not alter facial
c. Computed tomography (CT) scans are growth.
required to make the diagnosis of sinusitis i n b. Endoscopic sinus surgery results i n retarded
children. midface growth.
d. The diagnosis of sinusitis is clinical and does c. The effects of facial growth is not known and
not require radiologic confirmation. surgery should not be performed.
e. CT scans are best obtained during an acute d. Piglet models show retarded facial growth, and
infection. it therefore is assumed that the same will be
true i n children
3. Medical management is e. Facial plastic surgeons have shown there m i d -
a. Not frequently effective face growth retardation.
b. Directed toward more resistant bacteria
c. Usually targeted toward specific bacteria, and
broad-spectrum coverage is not warranted
d. Universally effective
e. Best provided with IV therapy

229
CHAPTER ONE HUNDRED AND EIGHTY

SALIVARY G L A N D DISEASE

Which of the following inflammatory processes of 4. The most common malignant salivary gland neo-
the salivary glands is least likely to occur? plasm i n children is
a. Gat scratch disease a. Lymphoma
b. Atypical mycobacterial infections b. Mucoepidermoid carcinoma
c. Mumps c. Squamous cell carcinoma
d. Reactive lymph node hyperplasia d. Warthin's tumor
e. Suppurative bacterial sialadenitis e. Rhabdomyosarcoma

The most common organism responsible for sup- 5. Treatment options for excessive
purative bacterial sialadenitis is salivation/chronic drooling include all of the fol-
a. Pseudomonas aeruginosa lowing except
b. Streptococcus pneumoniae a. Bilateral submandibular duct rerouting
c. Escherichia coli b. Bilateral submandibular gland excision with
d. Bacteroides melaninogenicus parotid duct ligation
e. Staphylococcus aureus c. Sublingual gland excision
d. Bilateral parotid duct and submandibular duct
The most common salivary gland neoplasm i n ligation
young children is e. Use of glycopyrrolate
a. Mucoepidermoid carcinoma
b. Lymphangioma
c. Lymphoma
d. Acinic cell carcinoma
e. Hemangioma

230
CHAPTER ONE HUNDRED AND EIGHTY ONE

PHARYNGITIS A N D ADENOTONSILLAR
DISEASE

1. Cultures taken from deep neck abscesses reveal 4. Postoperative admission after adenotonsillec-
a. Staphylococcus aureus tomy is indicated i n all patients except those
b. Streptococcus pyogenes a. Younger than 3 years of age
c. Bacteroides b. With a history of snoring
d. Polymicrobial infections (aerobic and anaero- c. Who live more than 90 minutes from the hos-
bic) pital
e. Pseudomonas aeruginosa d. With a history of asthma
e. With a history of an underlying bleeding disor-
2. Which of the following statements regarding der
chronic adenotonsillar hypertrophy is false?
a. Adenotonsillar hypertrophy has been associ- 5. Before undergoing adenotonsillectomy, all
ated with second-hand smoke exposure. patients with Down syndrome should undergo
b. Adenotonsillar hypertrophy has a significant which of the following tests?
association with inhalant allergies i n children a. Pulmonary function
younger than 3 years of age. b. Magnetic resonance imaging of the head and
c. (J-lactamase-producing bacteria play a signifi- neck
cant role i n pathologically enlarged tonsils and c. flexion and extension radiographs of the cervi-
adenoids. cal spine
d. When studying the bacteriology of adenoton- d. Immunoglobulin levels, including IgG sub-
sillar hypertrophy, core samples tend to be classes
more accurate than surface cultures. e. Rhinomanometry
e. Adenotonsillar hypertrophy with airway
obstruction has significantly increased as an
indication for adenotonsillar surgery i n recent
years.

3. Performance of adenoidectomy i n the presence


of an occult submucous cleft palate may result i n
a. Chronic eustachian tube dysfunction
b. The development of dental malocclusion
c. Speech articulation problems, primarily
related to the nasal sounds [M], [N], and [NG]
d. Velopharyngeal insufficiency
e. Chronic rhinosinusitis

231
CHAPTER ONE HUNDRED AND EIGHTY TWO

O B S T R U C T I V E SLEEP A P N E A
IN CHILDREN

1. The diagnosis of obstructive sleep apnea i n chil- 4. A l l of the following statements about primary
dren is best made by snoring are true except
a. Careful observation of sleep pattern by the a. Risk factors include adenotonsillar hypertro-
caregivers phy, obesity, decreased nasal patency, and
b. Polysomnography i n a pediatric sleep lab- passive smoke exposure.
oratory b. Primary snoring does not seem to progress to
c. Assessment of tonsil and adenoid size OSAS.
d. Home sleep monitoring tests c. Primary snoring can be distinguished from
e. The presence of daytime somnolence OSAS by a careful history and physical exami-
nation.
2. Which of the following is a clinical predictor of d. Currently, treatment is not recommended for
high risk for respiratory compromise after adeno- primary snoring.
tonsillectomy for obstructive sleep apnea syn-
drome (OSAS)? 5. The most common first-line treatment for pedi-
a. Severe OSAS on polysomnography atric OSAS is
b. Young age a. Continuous positive airway pressure (CPAP)
c. Cerebral palsy b. Adenotonsillectomy
d. Down syndrome c. Tracheotomy
e. A l l of the above d. Supplemental oxygen

3. A child with severe snoring is observed by skilled


health providers to have no apneic pauses during
sleep. This observation
a. Effectively rules out OSAS i n this child
b. Cannot exclude the diagnosis of OSAS,
because obstructive hypoventilation may be
present
c. Is evidence that adenotonsillectomy should be
performed
d. Is helpful only i n children older than age 5
e. Combined with clinical measures of tonsil size
can predict the need for adenotonsillectomy

232
CHAPTER ONE HUNDRED AND EIGHTY THREE

PEDIATRIC H E A D A N D N E C K
MALIGNANCIES

1. Certain populations of children are at increased 4. Class I monostotic Langerhans' cell histiocytosis
risk for malignancies. These include (LCH) can be treated with
a. Down syndrome a. Low-dose radiotherapy
b. Patients who have received radiotherapy b. Curettage
c. Bloom syndrome c. Steroid injection
d. Hemihypertrophy d. Biopsy
e. A l l of the above e. A l l of the above

2. Tissue from a biopsy of a suspected childhood 5. A post-liver transplant pediatric patient is seen
tumor is best handled by with new enlargement of the tonsils and/or ade-
a. Placing the tissue i n formalin noids. The otolaryngologist should
b. Placing the tissue i n Karnofsky's solution/EM a. Schedule the patient for T & A
fixative b. Observe the patient for 6 weeks
c. Snap freezing c. Start decreasing the immunosuppressive drugs
d. Placing the tissue i n a sterile container and d. Perform an FNA
delivering i t fresh to the pathologist

3. Routine elective neck dissection for pediatric


patients with salivary gland malignancies is
a. Always recommended
b. Recommended only if there are suspicious
nodes on computed tomography (CT)
c. Not recommended
d. Recommended depending on tumor subtype

233
CHAPTER ONE HUNDRED AND EIGHTY FOUR

DIFFERENTIAL DIAGNOSIS
OF N E C K MASSES

Second branchial cleft cysts are typically found 6. The diagnosis of cat-scratch disease is best made
a. Near the angle of the mandible with which of the following?
b. I n the posterior cervical triangle a. Culture of infected tissue for the bacterium,
c. High i n the neck and deep to the anterior bor- Bartonella henselae
der of the sternocleidomastoid muscle b. History of superficial scratch by a cat
d. Near the upper pole of the thyroid gland c. Serologic testing for Bartonella henselae
d. Biopsy of infected nodes looking for viral
Which radiologic study is the most efficient for inclusions
identifying a normal thyroid gland before per-
forming excision of the thyroglossal duct cyst? 7. The best treatment of neck lymph nodes infected
a. Technetium-99 scan by Mycobacterium tuberculosis is
b. Iodine-131 scan a. Surgical excision
c. Magnetic resonance imaging with gadolinium b. Antituberculous chemotherapy with two drugs
d. Ultrasonography of the neck c. Antituberculous chemotherapy with one drug
d. Incision of the infected nodes and curettage
In most cases of cutaneous hemangiomas of the
neck, the following are acceptable forms of treat- 8. A l l of the following clinical signs are seen acutely
ment except with Kawasaki's disease except
a. Laser therapy a. Coronary artery aneurysms
b. Radiation therapy b. Erythema, edema, and desquamation of hands
c. Watchful waiting and feet
d. Surgical excision c. Nonpurulent cervical adenopathy
d. Thrombocytosis
Thymic cysts develop as a derivative of which
embryologic structure 9. A l l of the following have been reported to cause
a. Second branchial arch drug-induced lymphadenopathy except
b. Fourth pharyngeal pouch a. Allopurinol
c. Third pharyngeal pouch b. Pyrimethamine
d. Third branchial arch c. Phenytoin
d. Chlorpromazine
After the neonatal period, suppurative l y m -
phadenopathy may occur as the result of 10. A l l of the following may be seen with nasopha-
infection with any of the following organisms ryngeal carcinoma except
except a. Unilateral otitis media with effusion
a. Pseudomonas species b. Neck mass
b. Staphylococcus aureus c. Positive mono spot test
c. Haemophilus influenzae d. Elevated titers of Epstein-Barr virus types 2
d. Group A P-streptococcus and 3

234
C H A P T E R O N E H U N D R E D A N D E I G H T Y FIVE

C O N G E N I T A L DISORDERS
OF T H E L A R Y N X

A child who has been identified with a laryngeal 4. Which congenital laryngeal anomaly typically
web should undergo which type of evaluation presents with symptoms and signs of aspiration?
prior to surgical repair? a. Laryngomalacia
a. Hearing evaluation b. Laryngeal web
b. Coagulation studies c. Saccular cyst
c. Renal ultrasound d. Bifid epiglottis
d. Cardiac evaluation e. Laryngeal cleft
e. Flexion and extension neck x-rays
5. Which congenital anomaly has been associated
At the time of endoscopy, a 2-month-old infant with sudden infant death?
with stridor and aspiration symptoms is found a. Thyroglossal duct cyst (in vallecula)
to have a laryngeal cleft that extends through b. Laryngeal cleft
the cricoid cartilage but not into the cervical c. Laryngomalacia
trachea. What is the proper staging of this d. Bilateral vocal cord paralysis
anomaly? e. Unilateral vocal cord paralysis
a. Armitage type 1C
b. Evans type I I
c. Renjamin/Inglis type I I
d. Myer/Cotton type L I I I
e. All of the above

Which of the following is frequently associated


with laryngomalacia?
a. Nasal septal deviation
b. Developmental delay
c. Gastroesophageal reflux disease
d. Auricular deformities
e. Disorders of the pituitary gland

235
C H A P T E R O N E H U N D R E D A N D E I G H T Y SIX

M A N A G I N G T H E STRIDULOUS C H I L D

1. Management of the difficult to extubate neonate 4. Vocal cord paralysis is more often associated
would include with
a. Antireflux treatment a. Birth trauma
b. Corticosteroids b. Malignant disease
c. Cricoid split c. Arnold-Chiari and hydrocephalus
d. Nebulized racemic epinephrine d. Familial disease

2. Tracheotomy complications may be prevented by 5. Subglottic hemangiomas


a. Stay sutures i n the trachea a. Are more common i n males
b. Antibiotic coverage at surgery b. Are associated with cervicofacial heman-
c. Proper trach tube fitting giomas
d. Inserting an extra long tube initially c. Are normally present at birth
d. May be treated with steroids
3. Vocal cord paralysis
a. Is more often bilateral
b. Is more commonly found i n males
c. May be associated w i t h paradoxical movement
of the vocal cords
d. May be diagnosed w i t h ultrasonography

236
C H A P T E R O N E H U N D R E D A N D E I G H T Y SEVEN

G L O T T I C A N D S U B G L O T T I C STENOSIS

Laryngeal stenosis 4. Repair of laryngeal stenosis


a. Is most commonly associated with iatrogenic a. Should be attempted through endoscopic
injury means as a first alternative
b. Is most often found i n the subglottis b. Is individualized according to site, severity,
c. Is associated with necrosis, ulceration, and and circumstances of the individual patient
perichondritis c. Is performed to achieve early decannulation
d. Always occurs with prolonged intubation with a good voice
d. Should be performed i n all individuals
Important factors i n the development of laryn-
geal stenosis include 5. The type of open repair performed depends on
a. Duration of intubation a. The degree of cartilaginous support remaining
b. Tube size b. The length of the stenosis
c. Patient age c. The degree of glottic involvement
d. Tube movement d. Grade of stenosis

Symptoms of laryngeal stenosis


a. Relate to airway, voice, and feedings
b. Are not progressive
c. Are best evaluated w i t h rigid endoscopy
d. Are best evaluated w i t h flexible endoscopy

237
CHAPTER ONE HUNDRED AND EIGHTY EIGHT

GASTROESOPHAGEAL REFLUX
A N D L A R Y N G E A L DISEASE

Which of the following statements regarding GER 3. The management of GER always includes
in children is true? a. Healthy diet and lifestyle
a. GER disease is rare i n children. b. Small meals at regular times
b. Postprandial GER is physiologic. c. Raising the head of the bed
c. Silent GER can induce extra-gastrointestinal d. Prokinetic drugs
symptoms. e. Proton pump inhibitors
d. Heartburn is infrequent i n children.
e. Major complications of GER are exceptional i n
children.

2. Which of the following laryngeal pathologic con-


ditions is frequently associated with GER?
a. Laryngomalacia
b. Posterior laryngeal cleft
c. Congenital subglottic stenosis
d. Acquired subglottic stenosis
e. Vallecular cyst

238
CHAPTER ONE HUNDRED AND EIGHTY NINE

ASPIRATION A N D SWALLOWING
DISORDERS

Normal swallowing includes which of the follow- Causes of swallowing disorders include
ing stages? a. Esophageal atresia
a. The oral stage is the automatic sucking reflex b. Posterior laryngeal cleft
in infants. c. Unilateral laryngeal palsy
b. The sucking reflex disappears after 1 month. d. Tracheoesophageal fistula
c. A pharyngeal stage e. Laryngomalacia
d. A n esophageal stage taking about 0.5 seconds
e. An esophageal stage taking about 5 seconds

239
CHAPTER ONE HUNDRED AND NINETY

VOICE DISORDERS

1. Unilateral vocal cord paralysis i n children with a 4. Vocal cord granulomas are usually
weak, breathy, and hoarse voice is initially man- a. Secondary to external cervical trauma
aged by b. Can be caused by gastroesophageal reflux dis-
a. Gelfoam injection ease (GERD)
b. Fat injection c. Treated surgically using G 0 laser
2

c. Isshiki thyroplasty d. Operated on to improve the voice quality


d. Speech therapy
5. Epidermoid cysts of vocal cord
2. Posterior glottic stenosis and cricoarytenoid fixa- a. Are usually bilateral
tion b. Improve with speech therapy
a. Are usually due to airway trauma c. Are often mistaken for vocal cord nodules
b. Usually results i n persistent voice disorder d. Stroboscopy shows decreased vibrating ampli-
even after surgical correction tude and complete glottic closure
c. Are easily diagnosed by flexible fiberoptic
laryngoscopy
d. Are treated by arytenoidectomy to improve
the airway and voice

3. Recurrent respiratory papillomatosis usually


a. Presents with hoarseness and/or airway
obstruction
b. Is usually diagnosed after age 5
c. Is associated mainly with human papillo-
mavirus types 9 and 16
d. Has good long-term prognosis for voice

240
CHAPTER ONE HUNDRED AND NINETY ONE

C O N G E N I T A L DISORDERS
OF T H E T R A C H E A

1. What is the classic presentation of an infant with 4. Which of the following condition(s) is/are often
tracheal stenosis? seen after repair of tracheoesophageal fistula?
a. Inspiratory stridor a. Tracheomalacia
b. Expiratory stridor b. Esophageal dysmotility
c. Cough and feeding difficulty c. Gastroesophageal reflux
d. Biphasic stridor with a marked expiratory d. None
component e. a, b, c
e. Hoarseness
5. Which of the following is the most common form
2. Primary tracheomalacia may present i n which of vascular compression of the tracheobronchial
group(s) of patient(s)? tree?
a. Premature infants a. Innominate artery compression
b. Infants with connective tissue disorders b. Double aortic arch
c. Healthy full-term infants c. Right aortic arch
d. a and b d. Anomalous subclavian artery
e. A l l of the above e. Pulmonary artery sling

3. Which of the following is the most common pres-


entation of tracheoesophageal fistula?
a. Pure atresia
b. Esophageal atresia w i t h proximal esophagotra-
cheal fistula
c. Esophageal atresia w i t h distal esophago tra-
cheal fistula
d. Esophageal atresia w i t h double fistula
e. H-type fistula without true esophageal atresia

241
CHAPTER ONE HUNDRED AND NINETY TWO

T R A C H E A L STENOSIS

1. The trachea develops from 4. According to the author, resection of greater


a. An evagination of foregut mesenchyme than three to five cartilaginous rings i n children
b. An invagination of embryonic ectoderm necessitates
c. Direct extension of midgut endoderm a. Wedge resection
d. The fourth pharyngeal arch b. Slide tracheoplasty
e. The sixth pharyngeal arch c. Laryngeal release
d. Perihilar release
2. All of the following factors are thought to con- e. Dissection of pulmonary vasculature
tribute to laser therapy failure except
a. Circumferential scarring 5. A l l of these methods are acceptable for dealing
b. Mature fibrosis w i t h long segment tracheal stenosis i n children
c. Fibrosis greater than 1 cm i n vertical length except
d. Tracheomalacia a. Slide tracheoplasty
e. Severe bacterial infection i n the presence of a b. Augmentation with autologous graft
tracheostomy c. Cadaveric human tracheal homograft
d. Free tracheal autograft
3. If rib cartilage is unavailable to be used as a tra- e. Synthetic tracheal prostheses
cheal graft material, what other autologous mate-
rial may be used?
a. Periosteum
b. Esophagus
c. Pericardium
d. Dura
e. A l l of the above

242
CHAPTER ONE HUNDRED AND NINETY THREE

CAUSTIC I N G E S T I O N

1. Which of the following is most likely to lead to 4. Esophagoscopy of a young child witnessed to
severe esophageal injury i n children even when have swallowed liquid sodium hydroxide reveals
swallowed i n small amounts? a circumferential exudative mucosal burn at the
a. Crystalline NaOH drain cleaner midesophageal level. What is the most accurate
b. Hydrochloric acid drain cleaner stage of this lesion?
c. Liquid NaOH drain cleaner a. First degree
d. Sulfuric acid b. Second degree
e. Concentrated ammonia c. Third degree
d. Fourth degree
2. Which of the following signs is most likely to be
associated with severe esophageal injury after 5. Esophagoscopy of a teenager who drank a small
ingestion of liquid NaOH? amount of liquid drain cleaner reveals a mides-
a. Drooling ophageal third-degree burn. What is probably the
b. Fever most important pharmacologic therapy for this
c. Oral mucosal burns injury?
d. Stridor a. Broad-spectrum antibiotics
e. Cough b. Pharmacologic therapy has not been proven
effective
3. A n 18-month-old child is seen with dysphagia i n c. 60 mg or oral prednisone for 2 weeks and then
the emergency department at 10:00 P M . He taper
appears i n no distress, but according to his par- d. Aggressive antireflux medications (e.g.,
ents has refused all food and drink for the past omeprazole)
14 hours. Results of his examination are normal, e. Penicillamine
and you decide to order a chest radiograph to
check for a foreign body. A small metallic disk i n 6. Esophagoscopy of a lye burn reveals blackened,
the midesophagus looks suspiciously like a bat- necrotic mucosa through the full length of the
tery. What is the best plan of action? esophagus. What additional procedures should be
a. Observation with recheck i n the morning after performed?
another chest radiograph a. Gastroscopy with a fiberoptic scope to see
b. Removal by endoscopy i n the morning as an whether debridement of necrotic stomach tis-
elective procedure sue is necessary
c. Immediate removal i n the operating room that b. Placement of a gastrostomy tube and trans-
evening by rigid endoscopy esophageal/gastric string for future retrograde
d. Attempted removal with a small transoral dilations
Foley catheter placed distal to the disk and c. Strong consideration of a thoracotomy to rule
then slowly pulled out out transmural necrosis of the esophagus
e. Immediate fiberoptic esophagoscopy with the d. A l l of the above
patient under sedation e. a and b

243
244 Part Seventeen PEDIATRIC OTOLARYNGOLOGY

7. A t approximately what pH is the threshold for 8. What type of injury is characteristic of alkaline
alkalinity severe enough to cause rapid mucosal burns?
necrosis? a. Granulomatous inflammatory necrosis
a. 13 b. Liquefactive necrosis
b. 10 c. Coagulative necrosis
e. 12.5 d. Ischemic necrosis
d. 13.5
C H A P T E R ONE HUNDRED A N D NINETY F O U R

F O R E I G N BODIES OF T H E A I R W A Y
A N D ESOPHAGUS

Which of the following statements regarding 4. Disk batteries


bronchial foreign bodies is true? a. May cause injury to the esophageal mucosa i n
a. Bronchial foreign bodies are more prevalent as little as 1 hour
than esophageal foreign bodies. b. Can be removed within 24 to 48 hours after
b. The three phases of foreign body aspiration ingestion
are the initial, asymptomatic, and complica- c. Are commonly aspirated into the bronchi
tion phases. d. Cannot be localized with radiographs in sus-
c. Radiographs are essential i n ruling out a pected ingestions
bronchial foreign body. e. Greater than 15 m m have a greater chance to
d. Hyperinflation of the affected lung from a pass through the gastrointestinal tract
check-valve effect is a late finding.
e. Optical forceps aid the endoscopist by increas- 5. Which of the following regarding foreign body
ing the ventilatory capacity of the broncho- emergencies of the airway and esophagus are
scope. true?
a. Impaction of a disk battery within the esopha-
Which of the following statements regarding gus is not a true emergency.
esophageal foreign bodies is false? b. The Heimlich maneuver should only be per-
a. Coins are the most common esophageal for- formed on an esophageal foreign body
eign body. impaction where the airway is not obstructed.
b. There is an increased probability of an c. Public education regarding choking emergen-
esophageal anomaly i n a patient w i t h multiple cies has not decreased mortality of acute air-
foreign bodies of the esophagus. way obstruction.
c. Most esophageal foreign bodies lodge at the d. Food objects do not cause acute airway
gastroesophageal junction. obstruction.
d. An esophageal foreign body may cause an e. The Heimlich maneuver should be performed
acute airway obstruction. with back blows and chest thrusts i n children
e. An esophageal foreign body may migrate to an younger than 1 year of age.
extraluminal position.

Regarding esophageal perforations, which of the


following statements is false?
a. Esophageal perforation may occur from pill
ingestion.
b. Esophageal perforations may occur from the
length of time an object is lodged.
c. Perforation of the esophagus is a risk of
esophageal foreign body removal.
d. Fever, tachycardia, and pain may be signs of
an early perforation.
e. A l l esophageal perforations require open
drainage to manage adequately.

245
C H A P T E R O N E H U N D R E D A N D N I N E T Y FIVE

I N F E C T I O N S OF T H E A I R W A Y

1. What is the narrowest portion of the upper respi- 4. If supraglottitis is suspected, what should be the
ratory tract i n a child? first course of action?
a. Glottis a. Establish IV access
b. Subglottis b. Perform nasopharyngolaryngoscopy to confirm
c. Supraglottis diagnosis
d. Trachea c. Culture the surface of the epiglottis
e. Oropharynx d. Establish airway through endotracheal or
nasotracheal intubation
2. Inspiratory stridor, hoarseness, and barking e. Perform emergency tracheotomy
cough are the cardinal symptoms of an illness
most often caused by which organism? 5. A recent report found Moraxella catarrhalis to
a. Influenza A and B be the most frequently implicated pathogen i n
b. Haemophilus influenzae type B which of the following disease entities?
c. Parainfluenza virus 1 and 2 a. Epiglottitis
d. Moraxella catarrhalis b. Bacterial tracheitis
e. Respiratory syncytial virus c. Spasmodic croup
d. Retropharyngeal abscess
3. The "steeple sign" seen the AP view of radi- e. Laryngotracheitis
ographs of the upper airway is associated with
which illness?
a. Epiglottitis
b. Supraglottitis
c. Peritonsillar abscess
d. Retropharyngeal abscess
e. Group

246
C H A P T E R O N E H U N D R E D A N D N I N E T Y SIX

R E C U R R E N T RESPIRATORY
PAPILLOMATOSIS

Although most of the human papillomavirus 3. I n which of the following anatomic regions does
(HPV) genetic types can infect respiratory RRP pose the worst prognosis?
mucosa, which of the following pairs of HPV a. Pulmonary
types is most closely associated with recurrent b. Larynx
respiratory papillomatosis? c. Soft palate
a. HPV 3 1 , HPV 33 d. Oral vestibule
b. HPV 6, HPV 11 e. Nasal vestibule
c. HPV 16, HPV 18
d. HPV 5, HPV 10 4. Which of the following therapeutic modalities has
e. HPV 79, HPV 84 consistently been shown to eradicate RRP-HPV?
a. C 0 laser
2

Which of the following statements concerning b. Microdebrider


recurrent respiratory papillomatosis is true? c. Photodynamic therapy
a. Once RRP goes into remission, the patient is d. No single modality has been consistently
cured for life. shown to efficiently eradicate RRP-HPV.
b. Cesarean section delivery always prevents e. Cidofovir
maternal-fetal transmission. f. Interferon
c. RRP is a benign disease and poses no malig-
nant risk. 5. A l l of the following signs and/or symptoms are
d. RRP of adults is generally more aggressive often associated with the new diagnosis of laryn-
than RRP i n children. geal papillomatosis i n a child except
e. There is a close association between maternal a. Dysphonia
condylomata and the development of recur- b. Stridor
rent respiratory papillomatosis (RRP) i n her c. Respiratory distress
offspring. d. Cough
e. Snoring

247
CHAPTER O N E H U N D R E D AND NINETY SEVEN

EARLY D E T E C T I O N A N D D I A G N O S I S
OF I N F A N T H E A R I N G I M P A I R M E N T

All of the following statements supporting the 3. Which of the following correctly describes a rea-
use of universal newborn hearing screening are son why ABR might be preferred over otoa-
true except coustic emission (OAE)?
a. Fifty percent of children with moderate to pro- a. OAE frequently misses hearing losses i n which
found hearing loss exhibit no risk factors for hearing is normal at some frequency.
hearing loss. b. OAE may miss inner hair cell and eighth
b. Infants with hearing loss enrolled i n appropri- nerve hearing losses.
ate early intervention have significantly fewer c. ABR takes less time to perform.
developmental delays. d. ABR is unaffected by neurologic pathology.
c. There is a significant delay i n identification of e. ABR provides information over a broader fre-
deaf infants without risk factors for hearing quency range.
loss.
d. Congenital hearing loss accounts for more 4. Most cases of genetic deafness exhibit which
than half the cases of significant hearing loss inheritance pattern?
in children 3 to 17 years of age. a. Autosomal recessive
e. Reliance on physician observation and/or b. Autosomal dominant
parental recognition has not been successful c. X-linked
in the past in detecting significant hearing loss d. Mitochondrial
in the first year of life. e. Paternal

Auditory brain stem response (ABR) morphology 5. Which of the following is a considered a con-
is affected by which of the following? traindication to cochlear implantation?
a. Sleep a. Age <1 year
b. Attention b. Vestibular nerve aplasia
c. Sedation c. Dilated vestibular aqueduct
d. Age d. A l l of the above
e. Gender of child e. None of the above

248
CHAPTER ONE HUNDRED AND NINETY EIGHT

CONGENITAL MALFORMATIONS
OF T H E I N N E R EAR

When do most malformations of the inner ear 4. What is the most common radiographically
occur? detectable malformation of the inner ear?
a. When formation of the membranous labyrinth a. Complete absence of the cochlea
is interrupted during the first trimester b. Enlargement of the vestibular aqueduct
b. When formation of the organ of Gorti is inter- c. Aberrant semicircular canals
rupted during the second trimester d. Congenitally large internal auditory canal
c. When maturation of the sensory epithelium is (IAG)
interrupted during the third trimester e. Cochlear aqueduct dilation
d. When formation of the otic capsule is inter-
rupted during the first trimester 5. Which procedure is indicated as a first attempt
e. When ossification of the otic capsule is inter- in transotic cerebrospinal fluid (CSF) leakage
rupted during the second trimester when hearing is poor?
a. Tympanotomy with overlaying of connective
Teratogenic influences known to affect inner ear tissue graft at the site of leakage
organogenesis include all of the following except b. Posterior fossa craniotomy with placement of
a. Rubella muscle plug i n the IAC
b. Cytomegalovirus c. Direct approach with tympanotomy, removal
c. Thalidomide of footplate, and obliteration of vestibule
d. Radiation exposure d. Hypo tympanic approach to ablation of
e. A l l of the above are known to affect inner ear cochlear aqueduct
organogenesis. e. Closure of eustachian tube

What is the most frequent histopathologic find-


ing i n congenital deafness?
a. Cochlear aplasia
b. Complete labyrinthine aplasia
c. Complete membranous labyrinthine dysplasia
d. A deformed inner ear i n which the cochlea
and vestibule are confluent, forming an ovoid
cystic space without internal architecture
e. Incomplete development of the pars inferior

249
CHAPTER ONE HUNDRED AND NINETY NINE A

R E C O N S T R U C T I O N SURGERY OF T H E
EAR: M I C R O T I A R E C O N S T R U C T I O N

1. Which of the following statements regarding 4. I n classic microtia reconstruction, which of the
microtia is false? following describes the correct staging order?
a. Approximately half of patients have an associ- a. Cartilage implantation, posterior skin graft,
ated congenital syndrome. lobule transfer, tragus reconstruction
b. Unilateral cases outnumber bilateral cases. b. Tragus reconstruction, cartilage implantation,
c. There is a left ear predominance, and girls are posterior skin graft, lobule transfer
affected more often than boys at roughly a c. Lobule transfer, tragus reconstruction, carti-
2.5:1 ratio. lage implantation, posterior skin graft
d. Certain populations, such as Navaho Indians d. Lobule transfer, cartilage implantation, poste-
and Japanese, may have a higher incidence. rior skin graft, tragus reconstruction
e. Incidence is 1 i n 10,000 to 20,000 live births. e. Cartilage implantation, lobule transfer, poste-
rior skin graft, tragus reconstruction
2. Which of the following ear embryology state-
ments is true? 5. Which of the following is true for stage 1 micro-
a. Most of the central ear is derived from hillocks tia repair?
4 and 5. a. Donor site incision is made on the ipsilateral
b. The external ear begins to form during the chest.
eighth week of gestation. b. Cartilage from a floating rib is used to create
c. The lobule seems to be one of the first parts of the helix.
the ear to develop. c. The perichondrium is removed from the carti-
d. The auricle begins as six small buds of mes- lage framework to improve helical and antihe-
enchyme surrounding the dorsal end of the lical details.
second pharyngeal cleft. d. A thick recipient pocket flap is desirable to
e. The first pharyngeal arch forms about 85% of cover and protect the cartilage framework.
the auricle. e. The lobule is elevated and trimmed until i t is
symmetric with the contralateral ear.
3. Which of the following anatomic statements is
false?
a. Ear protrusion from the mastoid is 1.5 to 2.0
cm, creating an angle of 15 to 20 degrees.
b. The ear is i n a slightly more vertical orienta-
tion than the nasal dorsum.
c. Normal ear height at maturity is 5.5 to 6.5 cm
and is attained at ages 15 for boys and 13 for
girls.
d. The horizontal width of the ear is achieved at
a later age than the ventral height.
e. The distance from the lateral palpebral fissure
to the root of the helix should approximate
the length of the normal ear at maturity

250
CHAPTER ONE HUNDRED AND NINETY NINE B

R E C O N S T R U C T I O N SURGERY
OF T H E EAR: A U D I T O R Y C A N A L
AND TYMPANUM

1. Which of the following is not a major malforma- 4. The stapes footplate is derived from which
tion that would preclude atresiaplasty? embryologic structure (s)?
a. Poor pneumatization a. Meckel's cartilage (first branchial arch)
b. Malleus/incus complex fixation b. Reichert's cartilage (second branchial arch)
c. Abnormal course of the facial nerve c. Otic capsule
d. Inner ear deformity d. a and b
e. b and c
2. I n atresiaplasty surgery, landmarks routinely
identified for drilling the new ear canal include 5. The most common cause of inoperability i n con-
all of the following except genital aural atresia is
a. The glenoid fossa a. Poor pneumatization
b. The tegmen b. Abnormal facial nerve
c. Lateral semicircular canal c. Inner ear deformity
d. Malleus/incus complex d. Absence of the oval window

3. The most common complication following atresi-


aplasty is
a. Facial nerve injury
b. Sensorineural hearing loss
c. Devascularization of cartilage graft used for
microtia repair
d. Tympanic membrane graft lateralization

251
CHAPTER TWO HUNDRED

ACUTE OTITIS M E D I A A N D OTITIS


M E D I A W I T H EFFUSION

1. Which of the following statements regarding the b. Children younger than 2 years old are more
epidemiology of OM is false? likely to benefit from antibiotic therapy than
a. The highest prevalence of all forms of OM older children.
occurs in the first 2 years of life. c. The use of broad-spectrum antibiotics clearly
b. In the United States, a large majority of children improves the response rates when treating
will have at least one bout of AOM i n their child- AOM.
hood, but less than 10% will be otitis-prone. d. Response rates are improved with the use of
c. Day-care attendance is the most important longer courses of antibiotics.
risk factor for OM. e. The risk of withholding antibiotics clearly out-
d. There are racial differences i n the incidence of weigh the risks of administering antibiotics in
OM. the treatment of AOM.
e. OME is uncommon i n newborns.
4. Regarding tympanostomy tubes, which of the fol-
2. With regard to the sequelae of chronic OME, lowing statements are false?
which of the following are true? a. The rate of recurrent AOM is roughly halved
a. Studies have clearly shown that the hearing for the duration the tubes remain in situ.
loss associated w i t h chronic OME has adverse b. The hearing loss produced by OME is effec-
effects on speech, language, and cognitive tively reversed by the insertion of tubes i n
development. most instances.
b. Studies have clearly shown that interventions c. The benefit to hearing and reduced rate of
with hearing-sparing therapies such as tympa- infection may extend for a period of months
nostomy tubes greatly mitigate adverse effects after extrusion of the tubes.
of chronic OME-related hearing loss on d. Nearly half of all children who undergo t y m -
speech, language, and cognitive development. panostomy tube placement will have at least
c. Children with frequent upper respiratory one bout of otorrhea at some time after the
infections not complicated by OME show less postsurgical period while the tubes are i n
cognitive delays than similarly afflicted chil- place.
dren with OME. e. The perforation rate for grommet-style tubes
d. There is a clear-cut relationship between is less than 5%, and greater with T-shaped
socioeconomic status and the effect of chronic tubes.
OME on cognitive development.
e. None of the above 5. Which of the following are relative indications
for tympanostomy tube placement?
3. With regard to antibiotic therapy and AOM, a. Cleft palate
which are true? b. Eustachian tube dysfunction and underlying
a. With a mixed age group and a diagnosis of sensorineural hearing loss
AOM made on clinical grounds, approximately c. AOM complicated by facial paralysis
80% of patients not treated with antibiotics d. Severe tympanic membrane retraction
and 94% of patients treated with antibiotics e. Hyperbaric therapy i n patients not requiring
will be symptom free at 7 to 14 days. airway support

252
CHAPTER TWO HUNDRED AND ONE

GENETIC SENSORINEURAL
H E A R I N G LOSS

1. With mitochondrial inheritance 4. I n evaluating an individual for genetic hearing


a. Transmission from father to daughter is com- loss, which of the following is not true:
mon. a. An electrocardiogram (ECG) is necessary to
b. The mtDNA is present i n the nuclear envelope rule out long-QT intervals i n Jervell and
of the cell. Lange-Nielsen syndrome.
c. Only maternal transmission is possible. b. Genetic counseling affords the opportunity to
d. Homoplasmy accounts for variability i n provide risk estimates and appropriate inter-
expression. pretation to the family and patient.
e. Congenital hearing impairment is never seen. c. Computed tomography of the temporal bones
should be performed if GJB2-related deafness
2. I n branchiootorenal syndrome, otologic abnor- is diagnosed.
malities include d. Consultation with an ophthalmologist should
a. Purely conductive hearing loss i n most cases be obtained i n patients suspected of having
b. Rarely preauricular pits Usher syndrome.
c. Enlarged vestibular aqueducts e. Urinalysis can be useful to screen for
d. Thyroglossal duct cysts hematuric syndromes associated with hearing
e. Michel aplasia loss.

3. Waardenburg syndrome type I is characterized 5. Nonsyndromic hearing impairment:


by all of the following except a. Accounts for 70% of genetic deafness
a. Heterochromic irides b. Can be due to mutations i n GJB2 that result
b. An association with Hirschsprung's disease in loss of hair cell stability
c. A white forelock c. Can be caused by aminoglycosides i n persons
d. Hearing loss that is often profound, bilateral, with specific GJB2 mutations
and stable over time d. Is never congenital
e. Synophrys e. Is rarely a cause of late-onset deafness

253
CHAPTER TWO HUNDRED AND TWO

P E D I A T R I C FACIAL FRACTURES

1. Which is most important when operating on 4. Which of the following statements is true regard-
nasoethmoid fractures i n children? ing orbital roof fractures i n children?
a. The use of absorbable plates and screws a. Orbital roof fractures occur primarily i n
b. Undercorrection of the fracture core younger children as a consequence of the pro-
c. Overcorrection of the fracture core portionally larger cranium and the lack of
d. Never to use preexisting lacerations for expo- frontal sinus pneumatization.
sure b. Are rarely associated with concomitant
e. Setting the intercanthal distance wider than intracranial injuries
anticipated c. Are unsafe to manage nonsurgically
d. If left unprepared have a high rate of orbital
2. The best imaging study for nasoethmoid complex mucocele
fractures is e. None of the above
a. Axial computed tomography (GT) scan with
1.5- to 3.0-mm cuts 5. Which of the following statements regarding
b. Panorex maxillomandibular fixation i n children is
c. Coronal CT scan w i t h 1.5- to 3.0-mm cuts false?
d. Townes view a. Two to three weeks of mandibular immobiliza-
e. Both a and c tion i n children younger than 12 is adequate.
b. Deciduous molars should be used for arch bars
3. Absorbable plating systems i n children are cur- or capping when present.
rently not indicated for c. I n general after age 10, the development of
a. Frontal sinus fractures permanent teeth provides for safe anchors.
b. Currently contraindicated i n the pediatric d. I n children younger than 2 and between 5 and
population 9, immobilization requires unconventional fix-
c. Load-bearing fractures ation techniques, because the dentition will
d. Zygomatic fractures not support arch bars.
e. Infraorbital r i m fractures e. Should not be used i n condylar fractures

254
ANSWERS
! ( ANSWERS KEY

CHAPTER 1 3. e used i n therapy and are produced


History, Physical Examination, 5 b by the same linear accelerators
and the Preoperative Evaluation 6. d used to produce x-rays. They have
the same radiobiologic properties
1 b CHAPTER 3 as the x-rays but are advantageous
By definition, i n the latest neck clas- Biophysiology and Clinical in the treatment of lymph nodes i n
sification system developed by the Considerations in Radiotherapy the neck that overlie the spinal
American Head and Neck Society cord because of their shorter pene-
and the AAO-HNS, the Virchow 1 b tration distance.
node is part of level IV, not sublevel The primary interaction between a
VB, which includes the transverse megavoltage x-ray and matter typi- 2. c
cervical and supraclavicular nodes. cally produces a high-energy x-ray LET refers to the energy distrib-
that causes a chain of ionization uted by the particle along its path.
2. e events as i t goes along its path. High LET radiation produces a
Patients treated for more than 3 These ionization events break dense ionization chain as i t goes
weeks with exogenous glucocorti- molecular bonds, and this causes through tissue, which i n turn
coids should be assumed to have the biologic damage and not simple causes a high percentage of double-
suppression of their adrenal-pitu- thermal heating. Electromagnetic stranded breaks i n the DNA that
itary axis and should receive radiation has a dual nature and can are not readily repairable. This
stress-dose steroids. be described as either quanta of gives rise to small shoulders on
energy or waves. With the wave radiation cell survival curves and a
3 b picture, the energy goes inversely steeper slope to the curves. Hence,
Latex allergies may result i n seri- as the wavelength, with shorter the RBE of high LET radiation is
ous, life-threatening anaphylactic wavelengths corresponding to higher than standard forms of radi-
reactions. Proper planning requires higher energies. A 1-MeV photon, ation. Direct interaction with the
avoidance of latex products i n the which is the order of magnitude of cellular DNA means that there is
operating room. the x-rays used i n treating head less dependence on a free radi-
and neck cancer, has a wavelength cal-mediated mechanism, which i n
4 b of approximately 10~ angstrom,
2
turn means that high LET radia-
Mithramycin treats hypercalcemia which is considerably smaller than tion is more effective i n killing
by inhibiting parathyroid hormone- a cell. The nature lof the primary hypoxic cells (e.g., the OER is low).
induced osteocytoclastic activity. interaction between x-rays and Most of the clinical work on high
matter depends on the energy of LET radiation is with fast neutrons.
5 b the x-ray. X-rays used i n therapy After a large number of
Age older than 70 years is a risk have much higher energy than the clinical trials, only a few selected
factor. x-rays used for diagnostic purposes types of tumors have been
and primarily interact by way of shown to respond better to fast
CHAPTER 2 the Compton effect. This means neutron radiation. I n the head and
Overview of Diagnostic Imaging that bone and soft tissues show up neck region, i t is salivary gland
of the Head and Neck about the same, and one does not tumors and not the more common
see the distinction between the two squamous cell tumors that are
1. e as readily as i n diagnostic films. better treated with high LET
2 d Megavoltage electrons can also be radiation.

257
258 ANSWERS

3. c cells i n a portion of the cell cycle sis. Eventually, thrombosis leads to


The whole rationale behind frac- that is more radiation sensitive. loss of nutritive flow.
tionation is to allow normal tissue Traditional chemotherapy agents
time to repair and thus cause less have always been cytotoxic. 2. False
long-term damage. Some tumor However, i t may be possible to also The paramedian forehead flap is
cells may repair, but most tumor increase the therapeutic ratio by based on the distribution of the
cells have defective repair mecha- protecting normal tissues. Because supratrochlear artery. This makes
nisms and are thus sensitive to the radiation will only kill tumor cells the flap an arterial or axial pattern
radiation. Hyperfractionated that are targeted, concurrent sys- flap. Random pattern flaps are per-
schemes are the same treatment temic therapy may be able to clear fused by no identifiable artery but
length as standard fractionation. micrometastatic disease that is not rather by the subdermal plexus.
Accelerated hyperfractionation in the radiation field. Because Axial pattern flaps are more robust
escalates the dose and decreases many chemotherapy agents also and generally can be tolerated to
the treatment time. Repopulation affect rapidly proliferating tissue much greater lengths without
is a significant problem i n tumors (much like radiotherapy does), the ischemia.
that seem less sensitive to radio- acute effects of combined modality
therapy. Altered fractionation therapy can be very intense. 3. c
schemes are designed to deliver an The surviving length of the random
increased effective dose to the CHAPTER 4 portion of the flap depends on the
tumor so that the tumor cannot Chemotherapy for Head and Neck physical properties of the supplying
repopulate, while still allowing Cancer vessels (intravascular resistance)
normal tissues time to repair. and the perfusion pressure. When
This is the reason that the acute 1 b the perfusion pressure decreases
effects of hyperfractionated ther- 2. a below the pressure i n the intersti-
apy are more intense, but the late 3 b tial space, capillary blood flow
effects are similar to standard 4 b ceases. The pressure at which
radiotherapy. 5. e there is no longer enough intravas-
cular blood pressure to maintain
4. a CHAPTER 5 capillary blood flow is called the
Trials to date with IMRT have, i n Skin Flap Physiology and Wound critical closing pressure.
general, the same number of treat- Healing
ment days as standard therapy. 4. True
However, because the isodose gra- 1. e Impairment of lymphatic drainage
dients can be very steep and the The zones of perfusion may be clas- with flap elevation also occurs.
dose to normal tissue can be sified according to the anatomic Reduction of the cutaneous l y m -
reduced, future studies may have and physiologic components of vas- phatic drainage results i n an
IMRT treatments taking longer to cular supply to the soft tissue. Zone increase i n interstitial fluid pres-
allow for dose escalation. The I refers to the macrocirculation, the sure that is compounded by
steeper isodose gradients with arterial conduits, and venous increased leakage of intravascular
IMRT also mean that the treat- drainage. In free tissue transfer, the protein associated with inflamma-
ments are more conformal than microvascular anastomoses create tion. The resulting edema leads to
traditional three-dimensional con- an alternative zone I perfusion to increased interstitial pressure,
formal therapy. The dose i n three- the transferred tissue. Thromboses which decreases capillary perfusion
dimensional therapy is more at the anastomosis represents zone by increasing the critical closing
homogeneous throughout the field I failure. Zone I I is the capillary cir- pressure. Alterations i n Starling's
than i t is for IMRT. culation, and zone I I I is the inter- forces result i n further ischemic
stitium. The "no-reflow" swelling of cells and the interstitial
5 f phenomenon is the failure of flap space, setting a positive feedback
Chemotherapy with cytotoxic survival despite adequate zone I cycle i n motion. This can threaten
agents is thought to make the circulation. I t is believed to be the flap survival by limiting the perfu-
tumor cells more sensitive to radio- result of the accumulation of free sion pressure.
therapy. The exact mechanisms are radicals with extended periods of
unknown, but there is some ischemia. This leads to swelling of 5. a
thought that certain chemotherapy the endothelial and parenchymal In surviving flaps, blood flow gradu-
agents might temporarily arrest cells coupled with intravascular sta- ally increases. If the flap is i n a
Answers Key 259

favorable recipient site, a fibrin 4. e 5. e


layer forms within the first 2 days. Donor site selection is influenced For benign lesions of the vocal
Neovascularization of the flap by many factors, including all folds involving the lamina propria
begins 3 or 4 days after flap trans- those listed i n Question 4. such as cysts, nodules, polyps, and
position. Revascularization ade- Additional factors include surgeon sulcus vocalis, the best surgical
quate for division of the flap preference and experience and technique would be mucosal spar-
pedicle has been shown as early as patient anatomy and vascular sta- ing excision. The C 0 laser would
2

7 days i n animal models and tus, which may require additional be most appropriate for RRP,
humans. During revascularization, preoperative testing to verify. because i t is an epithelial disease,
vascular endothelial cells play a and the risk of thermal damage to
major role i n the formation of new 5 a otherwise normal tissue is less-
vessels. Normally, endothelial cells The direct observation of a portion ened.
are i n a quiescent state, although of the flap with pin prick to assess
when stimulated by angiogenic bleeding remains the most reliable CHAPTER 8
growth factors, these cells can dra- method of flap monitoring. This Difficult Airway/Intubation:
matically proliferate. approach, however, is very labor Implications for Anesthesia
intensive. As a result, there are
CHAPTER 6 ongoing efforts to develop less 1 b
Free Tissue Transfer manpower-intensive approaches for Appropriate immediate manage-
monitoring. To date, none have ment for this patient with partial
1. b proven reliable enough to replace upper airway obstruction is basic
The first free tissue transfer was close direct tissue monitoring. noninvasive airway adjuncts such
reported i n 1959; however, the first as chin lift and jaw thrust.
report for oral cavity reconstruc- CHAPTER 7 Placement of a nasal trumpet or
tion was not until 1973 by Kaplan Laser Surgery: Basic Principles oral 100% oxygen with FM-Ambu
and others using a free groin flap. and Safety Considerations bag and airway, if tolerated, could
be an appropriate next step. If
2. e 1. c these simple maneuvers failed to
The multiple advantages of free tis- The Nd:YAG laser has the deepest relieve the obstruction i n this
sue transfer for head and neck thermal penetration of the listed patient with known sleep apnea,
reconstruction over all other tech- lasers. Thermal injury can occur then diagnostic procedures such as
niques are outlined i n Table 2. The 4 m m deep to the ablation crater. naloxone and/or nasopharyn-
numerous available donor sites goscopy could be performed. If
provide tremendous versatility of 2 d indicated by the clinical scenario,
tissues (bone, skin, and muscle) The above lasers cause tissue re-intubation through an orotra-
from nonirradiated regions of the effects by absorption of the light cheal route would be preferable to
body. This allows the surgeon to energy and conversion to heat. immediate cricothyrotomy.
address all components of the
required reconstruction i n a single 3. a 2 b
procedure, replacing "like tissues The retina is most at risk with Midazolam is a benzodiazepine
with like tissues." wavelengths in the visible and with anxiolytic properties that is a
near-infrared range of the electro- useful adjunct for awake intubation
3 b magnetic spectrum. Corneal in agitated patients. Unlike
Forearm skin is available i n large injury can occur with lasers i n the diazepam, the anxiolytic properties
quantities, is thin and pliable, and ultraviolet or infrared range of are not dose-dependent, and mida-
has excellent sensory capability, the spectrum. zolam can be titrated safely (in
which is ideal for oral cavity recon- small doses) to facilitate awake
struction. The vascular pedicle is 4. c intubations. Labetalol is a combined
long, vessel-caliber favorable, con- Knowledge of laser-tissue interac- a- and P-blocker that is an excellent
current harvest easily performed, tions is essential for the surgeon antihypertensive but has little role
and donor site functional morbidity to safely apply laser technology to in the acute setting in which any
acceptable. These attributes have tissue. Several parameters are hypertension is most likely second-
made the radial forearm free flap important i n laser use, including ary to agitation. Succinylcholine is
the "workhorse flap" for head and power, density, and fluence (see a fast-acting depolarizing paralytic
neck reconstructions. pp. 9, 10). that is contraindicated i n this
260 ANSWERS

patient who is spontaneously venti- not intubate, cannot ventilate" sce- 8. c


lating and awake. Propofol is an nario. I n a patient with known The use of any cautery within the
induction agent that could be used obstructive sleep apnea, difficulty airway poses a risk for airway fire.
for sedation i n smaller doses, but a with mask ventilation is to be Both bipolar and unipolar cautery
full induction dose causes apnea, expected. I n addition, this patient produce sparks. Minimizing the
and, therefore, propofol would not has a Mallampati class IV oral view. oxygen concentration by reducing
typically be a first-line agent for Paralysis is best avoided i n this both the inspired oxygen and
sedation during an awake intubation. patient until ability to mask venti- nitrous oxide fractions can reduce
late is verified, i n case this patient the risk of fire. Special endotra-
3 d is also difficult to intubate. Unless cheal tubes are manufactured or
The lateral approach to glossopha- this patient has a documented his- can be jury-rigged that reduce the
ryngeal nerve block brings the path tory of recent successful airway risk of heat from laser surgery
of the needle dangerously close to management with conventional igniting the oxygen-rich atmos-
the carotid artery. Injection of even laryngoscopes (Mac/Miller), consid- phere within the tube.
small amounts of local anesthetic erations for awake techniques
into the artery delivers a bolus of and/or the immediate availability of 9. c
local anesthetic to the brain and the OLHN rigid. The patient with Malignant hyperthermia can be trig-
may result i n seizure. Local tissue gastroparesis, the trauma patient gered by succinylcholine or volatile
swelling and total overdose are also (considered to have a full stom- anesthetics. Therefore, these agents
relative concerns but not as clini- ach), and the actively vomiting should be avoided i n patients at risk
cally important with this nerve patient all present significantly for MH. Once MH is suspected, dis-
block. Toxic absorption of local increased risk of aspiration, and, continuing the volatile anesthetic,
anesthetic is manifested by therefore, weighing risks and bene- immediate administration of dantro-
arrhythmia before GNS changes. fits would most likely favor rapid lene, and symptomatic cooling are
Therefore, dysrhythmias are a con- sequence induction/intubation all part of the treatment for this
cern, but compared with intrac- unless there was reason to antici- potential catastrophe.
arotid injections, a larger dose of pate difficulty w i t h their airways.
anesthetic would need to be CHAPTER 9
injected intravenously before see- 6. b Allergy and Immunology
ing these side effects. The ASA Difficult Airway of the Upper Airway
Algorithm does ask the practitioner
4. a to consider broad points, such as 1. c
This is a patient who is known to awake vs asleep techniques, surgi- The essence of specific immunity
be hypertensive and is also at high cal vs nonsurgical techniques, and is the ability to discriminate at the
risk of coronary artery disease. paralyzed vs spontaneously venti- molecular level between self and
Therefore, exacerbations of hyper- lating patients. I t does not offer a nonself. This ability allows the
tension and tachycardia should be checklist of specific intubating immune system to attack and
avoided. Cocaine can cause these tools such as Miller vs Macintosh destroy potentially harmful
sympathetic effects. Lidocaine blades for laryngoscopy. microorganisms without simultane-
mixed with phenylephrine can be ously destroying the individual
used as a combination anesthetic 7 b infected by these agents. The mole-
and vasoconstrictor with less risk of The LMA does not isolate the cules determined by the human
systemic adverse effects. Etomidate patient's esophagus and trachea, as leukocyte antigen (HLA) complex
would be an appropriate induction does an endotracheal tube. The mediate this crucial function. The
agent i n this patient with poten- LMA is usually fast and easy to generic term "major histocompati-
tially impaired cardiac function, place and can be used for positive bility complex" (MHC) has been
and p-blockade with metoprolol pressure ventilation if necessary. coined for the HLA complex and its
would be a good technique to When used for positive pressure homologues i n other species. I n
reduce risk of myocardial ischemia. ventilation, the LMA can also force man, the MHC occupies -4000 kb
air into the stomach and increase of DNA on the short arm of chro-
5 b the risk for aspiration. Maintaining mosome 6 and contains a large
The cardinal principle is that airway pressures <20 cm H 0 is2 number of genes encoding mole-
rapid-sequence induction/intuba- advised, so that the amount of air cules that serve a variety of func-
tion has both potential risks and pushed past the esophageal sphinc- tions. Among these molecules, a
benefits. The main risk is the "can- ter is lessened. group of glycoproteins belonging to
Answers Key 261

the immunoglobulin supergene small protein hormones that func- system. IgM is the predominant
family are present on the cell sur- tion i n controlling the growth and class formed on initial contact with
face and play a major role i n allow- differentiation of cells i n the antigen (primary immune
ing the immune system to microenvironment. The pattern of response). I t is confined mostly to
distinguish between self and non- cytokine secretion of T h cells the intravascular compartment and
self. These are MHC class I mole- allows their further subdivision can efficiently bind antigen and
cules (HLA-A, HLA-B, and HLA-G) into T h l and Th2 cells. T h l cells activate complement. The synthe-
and class I I molecules (HLA-DR, elaborate inflammatory cytokines sis of IgM is much less dependent
HLA-DQ, and HLA-DP). involved i n effector functions of than that of other isotypes on the
cell-mediated immunity, such as activity of T lymphocytes. Certain
2. f IL-2 and IFN-a, whereas Th2 cells antigens are capable of stimulating
Antigen presentation is carried out elaborate cytokines such as IL-4, IgM production by B cells i n a
by specialized cells referred to as IL-5, and IL-13 that control and T-cell-independent fashion, and
antigen-presenting cells, and these regulate antibody responses. Some the resultant immune response is
include a diverse group of leuko- GD4+ cells, capable of secreting usually restricted to the IgM iso-
cytes such as monocytes, both T h l - and Th2-type cytokines, type and does not exhibit immuno-
macrophages, dendritic cells, and B are sometimes designated ThO and logic memory. IgG is the most
cells. These cells are found prima- may be the precursors of fully dif- abundant immunoglobulin i n the
rily i n the solid lymphoid organs ferentiated T h l and Th2 cells. serum and the principal antibody
and the skin. Follicular dendritic Differentiation into T h l vs Th2 generated during the secondary
cells are specialized antigen-pre- cells is regulated by positive feed- immune response. Because of its
senting cells i n the B-cell areas of back loops promoted primarily by capacity to activate complement
lymph nodes and the spleen. IL-12 i n the case of T h l cells and and the expression on phagocytes
Peripheral-tissue dendritic cells IL-4 i n the case of Th2 cells. I n of FC receptors, IgG is regarded as
engulf and process antigen and addition to their central role i n i n i - the most important antibody of
then leave the tissues and home to tiating and regulating immune memory immune responses.
T-cell areas i n draining l y m p h responses, GD4 T lymphocytes are
+
Furthermore, IgG is the only iso-
nodes or the spleen. The predomi- important effectors of cell-medi- type that is actively transported
nant antigen-presenting cells of the ated immunity by virtue of the across the placenta, providing new-
skin are Langerhans cells, which cytokines that they elaborate. borns with a full repertoire of
are found i n the epidermis and These cytokines, particularly IFN- maternal IgG antibodies. These
deliver antigens entering the skin a, are essential contributors to the maternal antibodies provide the
to the effector cells of the lymph generation of chronic inflammatory neonate with antibody protection
nodes. I n the l y m p h nodes, these responses characterized by during the early months of life. IgA
antigen-presenting cells can mononuclear cellular infiltration is present as a dimer i n tears,
directly present processed antigens and activated macrophages. The saliva, and the secretions of the
to resting T cells to induce their cytokine profile observed after respiratory, gastrointestinal, and
proliferation and differentiation. allergen provocation of allergic genitourinary systems and is rela-
Monocytes-macrophages exist as individuals supports the involve- tively resistant to enzymatic diges-
monocytes i n blood and as ment of Th2-type lymphocytes i n tion. I t is also abundant i n
macrophages (a more differentiated the allergic reaction. Because IL-5 colostrum and provides passive
form) i n various tissues such as the promotes the differentiation, vas- immunity to the gastrointestinal
lungs, liver, and brain. I n addition cular adhesion, and i n vitro sur- system of nursing newborns. I t
to phagocytic and cytotoxic func- vival of eosinophils, as well as does not fix complement by the
tions, these cells have receptors for enhances histamine release from antibody-dependent pathway and
various cytokines (IL-4, IFN a) basophils, and because IL-4 is a does not promote phagocytosis. IgA
that can serve to regulate their mast cell growth factor and also contributes to the defensive func-
function. A l l antigen-presenting promotes the switching of B cells tions of the immune system by
cells have MHC class I I surface to the production of IgE, Th2-like preventing a breach of the mucous
molecules. T cells are thought to be particu- membrane surface by microbes
larly important i n allergic disease. and their toxic products. Finally,
IgE is important i n immediate-type
3. b, e, e
hypersensitivity reactions and i n
The activities of GD4 cells are
+
4. a
host defenses against parasitic
largely mediated by way of the Each of the antibodies contributes
infestation. The latter role is
secretion of cytokines, which are differently to the human defense
262 ANSWERS

accomplished both by the direct is highly toxic to a variety of u n i - the setting of peripheral general-
toxic effects of mast cell and cellular, multicellular, and other ized lymphadenopathy (PGL). FNA
basophil mediators and by the targets, including viruses, should be the first line of tissue
potent stimulatory effects of T cells mycoplasma, bacteria, fungi, and sampling, and open biopsy should
and mast cell products such as IL-5 parasites. EGP, like MBP, has be considered i n cases i n which an
in promoting eosinophilia and marked toxicity for helminth para- FNA is nondiagnostic. The poste-
attracting eosinophils to the local sites, blood hemoflagellates, bacte- rior triangle is the most common
environment. These, i n t u r n , con- ria, and mammalian cells and location of HIV-associated cervical
tribute to the eradication of para- tissues. Purified EGP has been used adenopathy.
sitic infestation by releasing in a number of studies i n which
mediators with parasite-toxic prop- respiratory epithelial damage 3. a
erties. (epithelial stripping, mucus plug- Increased incidence of Kaposi's sar-
ging) similar to that seen i n severe coma and both types of lymphoma
5. e asthma has been reproduced. EDN have been shown i n multivariate
Eosinophils secrete cationic gran- has been shown to induce a syn- analysis to be correlated with
ule proteins that include major drome of muscle rigidity, ataxia, HIV infection. Although squamous
basic protein (MBP), eosinophil eventual paralysis, widespread loss cell carcinoma seems to have a
peroxidase (EPO), eosinophil of Purkinje cells, and spongiform more aggressive course i n
cationic protein (EGP), and degeneration of the white matter of HIV-positive patients, an increased
eosinophil-derived neurotoxin the cerebellum, brainstem, and incidence was not seen i n m u l t i -
(EDN). Another prominent con- spinal cord when injected intrathe- variate analysis.
stituent protein of the eosinophil is cally or intracerebrally into experi-
the Charcot-Leyden crystal (GLG) mental rabbits or guinea pigs. 4 d
protein, which constitutes an esti- Histamine, a prominent mediator Surgical intervention should be
mated 7% to 10% of total cellular in allergic diseases, is secreted by considered i n HIV-positive patients
protein, possesses lysophospholi- mast cells and basophils but not who have symptoms refractory to
pase activity, and forms the dis- eosinophils. medical management, not just i n
tinctive hexagonal bipyramidal those with complications or life-
crystals that are the hallmark of CHAPTER 10 threatening illness. Although HIV-
eosinophil-associated inflamma- Head and Neck Manifestations positive patients seem to have a
tion. MBP is a potent cy to toxin and of Human Immunodeficiency Virus similar rate of sinonasal complaints
helminthotoxin i n vitro. I t is capa- Infection compared with the general popula-
ble of killing bacteria and many tion, HIV-positive patients with
types of normal and neoplastic 1. b sinusitis have an increased rate of
mammalian cells, stimulating hista- HIV infection requires the virus to sphenoid involvement.
mine release from basophils and bind to a GD4 receptor and thus Pseudomonas and fungi are par-
mast cells, activating neutrophils most frequently infects GD4 +
ticularly aggressive pathogens i n
and platelets, and augmenting T-lymphocytes and macrophages. HIV-related sinusitis.
superoxide generation by alveolar The viral proteases and the reverse
macrophages. I t has also been transcriptase enzyme are unique 5. c
shown to induce bronchoconstric- from human enzymes and critical Inexperienced surgeons such as
tion and transient airway hyperre- for viral replication and thus have medical students and j u n i o r resi-
activity when instilled into the been targeted for antiretroviral dents are more likely to have
monkey trachea. As for MBP and therapy. Transcription errors and a sharp injuries than more experi-
EGP, EPO is highly cationic and prolific rate of replication create a enced surgeons. Gases of serocon-
exerts some cytotoxic effects on vast pool of genetic diversity that version have been documented
parasites and mammalian cells i n allows the virus to evade the despite the use of postexposure
the absence of hydrogen peroxide. immune system and develop resist- prophylaxis. Some studies suggest
However, i t is highly effective i n ance to antiretroviral medications. that only 16% of surgeons follow
combination with hydrogen perox- universal precautions. The rate of
ide and a halide cofactor (iodide, 2. c seroconversion after a needle stick
bromide, or chloride) from which Idiopathic follicular hyperplasia is is estimated at 0.3%. The risk is
EPO catalyzes the production of the most common cause of cervical increased w i t h hollow-bore nee-
the toxic hypohalous acid. I n the adenopathy i n HIV-positive dles or devices that are visibly
presence of these compounds, EPO patients. This is typically seen i n bloody.
Answers Key 263

CHAPTER 11 the abnormal gene is seen i n and 3' ends of the desired DNA frag-
Special Considerations higher frequency i n the population, ment. Southern hybridization, i n
in Managing Geriatric Patients residing mostly with asymptomatic which radiolabeled DNA probes are
carriers. Haploinsufficiency, i n hybridized with DNA on a stable
1. a which the inactivation of one gene membrane support (such as filter
2 b results i n an insufficient level of paper), has limited use i n modern
3. c gene product to maintain normal molecular genetics testing, but i t is
4. a cellular function, influences pheno- still useful for analysis of large DNA
5. e type i n autosomal-dominant disor- fragments. Heterozygous mutations
ders. I n many autosomal-recessive in DNA can be detected by the for-
CHAPTER 12 disorders, heterozygous carriers mation of a heteroduplex-two
are asymptomatic, and the mecha- strands of DNA with mismatched
Genetics and Otolaryngology nism of haploinsufficiency does not bases-when the DNA fragment i n
influence phenotype. Females and question is amplified, heated, and
1. d males are equally affected i n auto- allowed to anneal with itself.
Introns are noncoding DNA within somal-recessive disorders.
a gene that are excised from the CHAPTER 13
genetic message after transcription 4. a Fundamentals of Molecular
has occurred. Genes are tran- Expressivity refers to the severity Biology and Gene Therapy
scribed from the 5' to the 3' end. of the phenotype seen i n genetic
Regulatory elements within the diseases. X-linked disorders tend to 1 d
gene influence the rate of tran- have more variable expression i n 2. c
scription and cellular specificity of females than i n males. One reason 3. c
gene action. Most amino acids are for this increased variability of 4 d
associated with more than one expressivity i n females is due to 5. c
codon, and because i t can vary for the random inactivation of one X
most amino acids, the third chromosome early during develop- CHAPTER 14
nucleotide is referred to as the ment, a phenomenon known as Molecular Biology of Head
wobble nucleotide. The genetic dis- Lyon's hypothesis. The expressivity and Neck Cancer
tance between two genes reflects of recessive disorders is more con-
the frequency of observed combi- sistent than the expressivity of 1. b
nations between them and is only dominant ones. A gene that has no The basis for the known mecha-
imperfectly correlated with its penetrance has, by definition, zero nisms behind the development of
physical distance or the number of expression. Variability i n gene head and neck cancer has been
bases between two genes. expression implies the existence of shown to be genetic i n origin.
2 b mechanisms by which the severity DNA is the code from which mRNA
Treacher-Collins syndrome is a of the disorder can be influenced; protein products arise and, there-
monogenic disorder inherited i n an such mechanisms may include fore, is the foundation for genetic
autosomal-dominant fashion. The expression of background genes or alterations that can lead to carcino-
other disorders listed are correctly environmental effects. genesis. Ultimately, these genetic
classified with its mode of inheri- alterations lead to a malignant phe-
tance. 5. c notype that can include altered cell
DNA chips have oligonucleotides of proliferation, invasion, metastasis,
3. c known sequences arrayed on a chip, altered immunogenicity, resistance
In autosomal-recessive disorders, such that homologous RNA or DNA to therapy, genetic instability, as
two abnormal copies of the same can be detected. DNA chips can be well as other phenotypic character-
gene are required for an individual used both to analyze patterns of istics common to malignancy.
to be affected. A n affected man can gene expression and to detect sin-
transmit the gene to his son i n gle-base changes i n DNA. DNA chip 2. e
autosomal-recessive disorders, but technology is limited i n that i t can- For all of these reasons, head and
not i n X-linked recessive disorders. not be used to detect a novel muta- neck cancer is thought to arise as a
The chance that two heterozygous tion. Polymerase chain reaction result of a series of genetic alter-
parents will have an affected child (PCR) amplifies a targeted sequence ations, the sum of which leads to
is 25%. Although autosomal-reces- of DNA by use of oligonucleotide malignancy. Rennan and others
sive disorders are relatively rare, primers complementary to the 5' (1993) suggested that between
264 ANSWERS

6 and 10 genetic alterations were 3. a effect size not statistical signifi-


required for the development of 4. e cance. Power is related to type I I
head and neck carcinoma. The 5. c error, not to P values.
detection of microsatellite alter-
ations i n premalignant lesions i n a CHAPTER 16 4. e
progression model by Califano and Interpreting Medical Data Evidence-based medicine is based
others (1996) also indicates that on a hierarchy of research evi-
there are early, predictable 1. d dence, with different classification
changes that occur i n the pathway Retrospective and prospective refer schemes for studies of therapy,
to malignant transformation. to directions of study inquiry, not diagnosis, or prognosis. The level of
to methods of data collection. A evidence is then used to derive a
3 d prospective study records new grade of recommendation, which is
Chemoprevention was first studied events (incidence) and may be the same for medical and surgical
and demonstrated efficacy i n conducted by record review or by interventions. Randomized trials
patients with oral leukoplakias i n a observing future events. Survival and prospective controlled studies
study by Hong and others (1986) analysis, which adjusts for cen- yield the highest level of evidence
using vitamin A derivatives. Since sored observations, is unnecessary (and recommendation grades), but
then, many studies have examined because follow-up is available for expert consensus yields only the
the possibility of using other all subjects. lowest level of evidence (grade D
chemopreventive agents to reduce recommendation). Expert consen-
the incidence of tumor develop- 2. a sus, however, is an acceptable basis
ment, tumor recurrence, and A 95% confidence interval aids i n for recommendations if higher
tumor progression, including data interpretation, because i t esti- quality studies are unavailable,
answers a, b, and e. mates the range of results consis- unfeasible, or unethical.
tent with the observed data. This
4. c, a, b, d, e permits extrapolation of results 5 b
These are just some of the tech- beyond the study (inference) Effective interpretation of medical
niques researchers use to detect based on the single set of measure- data is a systematic process of
genetic sequence alterations i n ments made on a limited number moving from observations to gener-
head and neck cancer cells. of study subjects. Conversely, alizations with predicable degrees
Promoter hypermethylation is an accuracy reflects nearness to the of certainty (and uncertainty). The
epigenetic event that does not t r u t h (bias) and has nothing to do most important part of the process
involve DNA sequence change, or with precision or confidence inter- is recognizing the inherent vari-
loss or gain of DNA. vals. Similarly, statistical power is ability i n all biologic systems and
an unrelated concept. Variability the inevitable uncertainties i n
5. e of observed data relative to the related measurements and observa-
Translating the known molecular mean is described by standard tions. Because error can never be
alterations of tumors into practical deviation, not by confidence avoided, it is estimated with confi-
therapy modalities has long been the intervals. dence intervals, P values (rates of
goal of many researchers. With these type I error), and power calcula-
very specific alterations, one could 3. c tions (rates of type I I error).
theoretically devise a treatment plan When study results are statistically Choosing the right statistical test
that would target only the cells with significant, the P value is the prob- and avoiding multiple P values are
abnormalities and preserve the nor- ability of making a type I error: of secondary importance.
mal cells. This has proven to be very concluding the drugs have differing Statistical power is irrelevant when
difficult, but strides are underway to efficacy when i n fact they are a significant P value is obtained.
create therapies that can be used really comparable. Alternatively,
alone or i n conjunction with con- we could state that there is only a CHAPTER 17
ventional treatment. 1.5% chance that the differences Pain Management i n the Head
observed by the investigators are and Neck Patient
CHAPTER 15 strictly fortuitous (e.g., explainable
Outcomes Research by random error). Relative and 1. b
absolute efficacy is assessed by the Neuropathic pain can be treated
1. c relative risk and the rate differ- with anticonvulsant agents such as
2. a ence, respectively, and measures carbamazepine, local anesthetics
Answers Key 265

such as lidocaine, tricyclic antide- Overactive pericranial muscles helps to standardize patient photo-
pressants that decrease the emo- may play a role i n the pathophysi- graphs to achieve consistency and
tional depression that amplifies ology of chronic tension headaches eliminate variability that may exist
pain, and opioids. Nonsteroidal that consist of a constant bandlike from day-to-day with the same
antiinflammatory drugs such as pain that is bilateral and contained patient, from patient to patient, and
ibuprofen are used to treat mild i n the forehead. Paroxysmal hemi- between different photographers.
pain associated with inflammation. crania is a unilateral headache
NSAIDs alter the inflammatory characterized by excruciating pain 2 d
process by blocking expression of in the ocular and frontotemporal After evaluating facial symmetry,
the cyclooxygenase (COX) area that is provoked by certain the face may be divided into fifths.
enzymes that mediate production neck movements and pressure i n The basic unit for dividing the face
of the prostaglandins that sensitize the upper back. Facet j o i n t syn- vertically is the width of the eye.
pain afferents. drome can be differentiated by the Each eye is one-fifth of the total
response to radiographically guided facial width. The intercanthal dis-
2 d injections of local anesthetics into tance approximates the width of
The proximity of the glossopharyn- the zygapophyseal joints or around one eye. Moreover, a line dropped
geal nerve to the carotid artery dic- the dorsal medial branches of the from each medial canthus approxi-
tates extreme care when posterior primary rami. mates the side of the ala of the
performing a nerve block to avoid nose, making the nasal base one-
profound toxicity from a misplaced CHAPTER 18 fifth of the facial width.
injection. Integrating Palliative and Curative
Care Strategies in the Practice 3. c
3. e of Otolaryngology When assessing facial height, the
Several migraine triggers include face is divided into thirds. The land-
alcohol, certain foods, changes in 1. e marks are the trichion to glabella,
hormonal levels, stress, and sleep 2 b from glabella to the subnasale, and
patterns. Serotonin levels have been 3 b from the subnasale to the menton.
found to be higher centrally and 4. e A second method of assessing the
lower peripherally during migraines. 5 a facial height disregards the upper
third of the face because of the vari-
4 b CHAPTER 20 ability of the hairline.
Multiple radiofrequency lesioning Medical Informatics Measurements are made from the
of target nerves has been found to and Telemedicine nasion to subnasale and from the
reduce pain i n patients w i t h cervi- subnasale to menton representing
cal zygapophyseal joint pain or 1. c the midface or nasal height and the
whiplash. No evidence supports the 2 a lower facial height, respectively.
effectiveness of single sessions of 3 b
extension-retraction exercises or 4. a 4. c
corticosteroid injections i n reliev- 5 b The nasolabial angle defines the
ing pain. Botulinum toxin A injec- angular inclination of the columella
tions have been shown to lead to a CHAPTER 21 with the upper lip. I n the female,
trend toward improved function, Aesthetic Facial Analysis the ideal angle ranges from 95 to
but not i n treating pain. 110 degrees, and i n the male from
1. d 90 to 95 degrees. A nasolabial
5. c The Frankfort horizontal is the angle less than that of the ideal is
Regular analgesic use has been standard reference point i n which described as under-rotated, and an
implicated as a cause of chronic to position the patients' head and angle greater than that of the ideal
headache. Regular analgesic use gaze. The patient is positioned is described as over-rotated.
will likely lead to chronic daily standing or sitting upright with the
headaches i n patients with a his- legs uncrossed, hair tucked behind 5 d
tory of migraine. Intractable the ears, and jewelry removed. The The width of the ear is approxi-
chronic cluster headaches can head is positioned adjusting the mately one-half its length. The
resolve with blockade of the chin to achieve a Frankfort line superior and inferior aspect of the
trigeminal ganglion or the parallel to the ground and eyes i n ear should approximate the level of
sphenopalatine ganglion. forward gaze. This reference point the brow and the ala, respectively.
266 ANSWERS

CHAPTER 22 (macroscopic) nodal metastasis is not limit the ability to perform


Recognition and Treatment also made within the N category. I n SLNB.
of Skin Lesions doing so, the AJCC has now incor-
porated the technique of sentinel CHAPTER 24
1 b lymph node mapping with biopsy Scar Revision and Camouflage
2. c (SLNB) into the new staging
3. a system. 1. d
4. e When re-excision with direct p r i -
5. c 4. a mary closure is undertaken for an
Sentinel lymph node mapping with unsightly scar, the following princi-
CHAPTER 23 biopsy (SLNB) is considered the ples should be followed. This is not
Management of Head and Neck method of choice for staging of an irregularization procedure such
Melanoma regional nodal basins. Both the as geometric broken line closure or
sensitivity and specificity for running W-plasty. The purpose is
1. c detecting micrometastasis is to close a wound using soft tissue
Desmoplastic melanoma (DM) higher with SLNB than radi- principles that will help form an
accounts for 1% of all cutaneous ographic studies such as GT scan, optimal scar. This includes bevel-
melanoma cases. Up to 75% of MRI, and PET scan. The SLNB ing of the scalpel slightly outward
lesions arise i n the head and neck technique provides the pathologist to prepare for wound edge ever-
region, often i n the setting of with a limited number of nodes to sion, undermining of 1 to 2 cm of
lentigo maligna and lentigo maligna thoroughly evaluate with serial surrounding skin, and closure with
melanoma. DM is a diagnostic chal- sectioning, hematoxylin and eosin interrupted vertical mattress
lenge, because many lesions lack staining, and melanoma-specific sutures to evert the wound edges.
the typical ABGD warning signs immunohistochemistry if war-
associated with melanoma, and up ranted. Therefore, the histologic 2. c
to 75% of cases are amelanotic. DM analysis of sentinel lymph nodes is A depressed scar along the
has a propensity for neurotropic more thorough and complete than cheek/malar prominence area is a
spread. The 12.5% rate of regional traditional evaluation scar favorable for improvement
metastasis i n the setting of DM is of the entire lymphadenectomy through surgical scar revision. If
lower than other melanoma sub- specimen from an elective neck the scar's main unfavorable charac-
types. dissection. teristic is that i t is depressed, then
re-excision w i t h primary closure is
2 d 5. e indicated. If the scar is depressed
Most authors advocate the use of Patients with primary melanoma and also not i n alignment with
radiation as adjuvant therapy for tumors measuring >1 m m i n thick- relaxed skin tension lines, then re-
patients with adverse prognostic ness should be considered for excision combined with a scar
markers such as neurotropism, SLNB. Patients w i t h primary irregularization technique (geomet-
extracapsular spread, multiple melanoma tumors measuring <1 ric broken line closure, W-plasty)
lymph node involvement (>4 m m i n thickness should also be would be indicated likely followed
nodes), or recurrence. Primary considered if they have poor prog- by dermabrasion 6 to 8 weeks
tumor ulceration alone is not a nostic indicators such as tumor later. A 2-month-old with a scar 2
common reason for adjuvant extension to the deep margin, weeks after closure should be fol-
radiation. ulceration, extensive regression to lowed to see how the wound
1.0 m m , young age, high mitotic appears after 6 to 12 months of
3 d rate, or Glark level IV invasion. maturation. An edematous bilobed
The two most important factors i n Patients with regional metastasis flap 6 weeks out is normal and
T classification of localized disease warrant a therapeutic lymph node does not need surgical scar revi-
are primary tumor thickness and dissection and, therefore, are not sion but would likely benefit from
ulceration. The number of metasta- deemed candidates for the SLNB intradermal steroid injection. A red
tic lymph nodes, as opposed to the staging procedure. Other exclusion scar i n the preauricular area 2
gross size of metastatic l y m p h criteria include distant metastasis, months out is likely undergoing
nodes, is used to define the N cate- previous surgical disruption of the normal healing, and the erythema
gory. Delineation of occult (micro- lymphatics and previous resection usually fades with increased scar
scopic) nodal metastasis vs of the primary tumor with wide maturation and does not need sur-
clinically/radiographically apparent margins. Punch biopsy alone will gical scar revision.
Answers Key 267

3. c CHAPTER 25 lize, and, if needed, a local or


All of the statements are true Facial Trauma: Soft-Tissue regional flap may be planned i n a
regarding tissue expansion except Lacerations and Burns delayed fashion. This will likely
that during expansion the cur- result i n a better outcome.
rently accepted understandings on 1. d
the fate of tissue layers during tis- The phenomenon of debris tattoo- 4. c
sue expansion are: Epidermis is ing results when particulate matter If a burn spontaneously heals
thickened, melanin production gets embedded i n the dermis and within 10 to 14 days, then scarring
increases, mitotic activity is the epidermis heals over it. The will be minimized and skin quality
increased, dermis is thinned best treatment for debris tattooing will be good. After 14 days, the
(30%-50%), collagen synthesis is is prevention. Therefore, at the chances of unsatisfactory hyper-
enhanced, hair follicle number time of primary repair, foreign trophic scarring increase dramati-
remains unchanged, hair density material should be aggressively cally. Therefore, at approximately
decreases, muscle thins and can removed from the wound by scrub- 10 days, the burn should be closely
atrophy, and blood vessels pro- bing and irrigation. After debris evaluated for its viability, and if at
liferate. tattooing has resulted, full-thick- this time healing seems to be
ness excision is the most effective delayed, then eschar excision and
4. c treatment. With simple facial lacer- skin grafting should be performed.
Dermabrasion is best performed ations, no soft tissue debridement Oral commissure burns often result
at 6 to 8 weeks. Wire fraises are is usually necessary. Fine (nothing from a child biting an electrical
best reserved for experienced der- larger than 6-0), nonabsorbable, cord. Injury occurs from the cur-
mabraders, and diamond fraises monofilament suture should be rent passing through the tissues
are best suited for neophyte der- used, and the sutures should be rather than from heat on the skin
mabraders. Prophylaxis against removed i n 3 to 5 days to obtain surface; therefore, the initial injury
herpetic outbreak should be pro- the optimal cosmetic result. may seem limited. Tissue necrosis,
vided to any patient w i t h a his- however, may progress over the
tory of previous herpetic 2 b next 36 to 48 hours. Because of the
outbreak. When performing der- Although injury to Stensen's duct difficulty distinguishing viable from
mabrasion, one should move the is uncommon, an unrecognized nonviable tissue, early debride-
bit perpendicular to the axis of and unrepaired duct can result i n a ment is generally not recom-
rotation of the bit. Dermabrasion sialocele or salivary fistula that can mended. With periorbital burns, a
should proceed into the papillary easily be avoided with repair. The major concern is corneal protec-
dermis. most common sign of injury is tion. Tarsorrhaphy was once rou-
saliva draining from the wound, tinely recommended, but now is
5 b but sometimes the injury may be reserved difficult cases. With
Irregularization would not be the subtle. I n these cases, the best way meticulous corneal lubrication and
surgical scar revision technique of to confirm injury is by cannulating appropriate ophthalmologic consul-
choice for an unsightly scar that the duct intraorally with a lacrimal tation, often tarsorrhaphy can be
falls between two facial aesthetic probe. Repair is best performed avoided. Colonization and superin-
units. I n this situation, re-excision with 8-0 or 9-0 nylon suture under fection of the eschar is a major
with primary closure or intrale- magnification and possibly using a source of morbidity with burns and
sional steroids would be a reason- silastic stent left i n place for 7 to in the past has been a major
able alternative. The limbs of a 10 days after repair. source of sepsis i n burn patients.
W-plasty should be 5 to 7 m m i n Topical antimicrobials have served
length. Shorter limbs undergo con- 3. a to decrease the risk of eschar
traction and are too small to create Placement of a skin graft i n the superinfection.
sufficient irregularization. acute setting when facial soft tissue
Geometric broken line closure is has been lost stops secondary heal- 5. e
an excellent choice for a long l i n - ing, which results i n a poor tissue Second-degree burns extend into
ear scar that does not fall within match and a noticeable contour the dermis for a variable distance.
relaxed skin tension lines. defect. Therefore, skin grafts Clinically, they are characterized
Dermabrasion should be routinely should be avoided i n these situa- by severe pain, erythema, and blis-
offered to patients 6 to 8 weeks tions, unless there is massive tissue tering. Third-degree burns extend
after scar irregularization to obtain loss or third-degree burns. Instead, the full thickness of the dermis
maximum camouflage. the wound should be left to stabi- into subcutaneous and sometimes
268 ANSWERS

even down to bone. Clinically, suppression of circulating dihy- CHAPTER 29


there is no bleeding, blistering, or drotestosterone (DHT) while hav- Management of Aging Skin
pain. First-degree burns are super- ing no affinity or effect on
ficial injuries only involving the testosterone and other hormones. 1. Type I I
epidermis. Clinically, they cause In 5-year studies, 65% of men tak- 2. e
only pain and erythema. ing finasteride maintained or 3. c
improved their hair counts, and 4. Systemic toxicity if first sug-
CHAPTER 26 90% of men were noted i n clinical gested by CNS stimulation
Maxillofacial Trauma photography as having no further (tremors, hyperreflexia, and
visible hair loss compared with hypertension) followed by CNS
1. a baseline. These studies also found depression with respiratory fail-
2. c that the incidence of sexual side ure, hypotension, and cardiac
3 b effects decreased with duration of arrhythmia, which can be fatal.
4 d therapy from 0.8% to 1.8% to 0.3% Preventing such toxicity is pos-
5. c after the fifth year. As with minoxi- sible with IV fluid volume load-
dil, continued therapy with finas- ing before, during, and after a
CHAPTER 27 teride is required to preserve peel; forced diuresis with
Reconstruction of Facial Defects renewed hair growth. furosemide 10 minutes before
the peel, waiting 20 to 30 m i n -
1. c 4 b utes between each facial region
2 d Follicular-unit hair transplantation to be peeled (25%-30% surface
3 d has become the established method area), and careful patient selec-
4. c of choice i n hair restoration surgery. tion, avoiding patients with
5. e Individual follicular units are dis- hepatorenal disease.
sected based on the natural group- 5. Tretinoin induces type I procol-
CHAPTER 28 ing of one to four terminal hairs and lagen formation, thickens the
Hair Restoration: Medical include all of their support struc- epidermis, reduces melanin
and Surgical Techniques tures. I n terms of ideal candidates, content, increases glycosamino-
minimal contrast should exist glycans, and stimulates angio-
1 d between hair color and skin color to genesis, giving a "rosy"
Different androgens have various improve the illusion of a natural appearance to the skin.
influences on hair growth based on anterior hairline. This illusion also Histologically, tretinoin-treated
the region. Testosterone affects p r i - relies on recreating a "feathering" skin is more stable because of
marily axillary and pubic hair transition zone with fine, single hair increased anchoring fibrils and
growth. Dihydrotestosterone (DHT) grafts placed i n recipient incisions decreased activity of collage-
affects beard growth and scalp hair, made to emulate the surrounding nase enzyme. Tretinoin can
primarily i n frontal and vertex natural hair in terms of proper hair also reduce UV-induced colla-
regions of the scalp, resulting i n direction and angulation. gen destruction and therefore
androgenetic alopecia. helps prevent photodamage
5. c (Fisher and others 1999). Side
2 d Although modified scalp reductions effects include m i l d to moder-
During telogen effluvium, the hair still have a role i n select patients ate dryness, skin irritation,
shaft is shed as the follicle abruptly with alopecia, both the surgeon peeling, and photosensitivity;
transitions from the growth or ana- and the patient need to be aware of therefore, sun protection is
gen phase to the resting of telogen the potential complications, most important.
phase. Telogen effluvium often noticeably misdirected hair,
occurs i n response to certain stres- decreased hair density, and poten- CHAPTER 30
sors, including hormonal and sys- tially detectable scars. The vertical Rhytidectomy
temic conditions, as well as "slot" defect is created i n the
exposure to a broad range of med- occipital scalpel by closing an ellip- 1 d
ications. tical resected areas into an elon- The buccal branch of the facial
gated scar. This unfavorable scar nerve is at risk during subpe-
3. e can be prevented or corrected by riosteal dissection of the midface
Finasteride is a type 2 5-oc-reduc- use of a series of three rotational at the time of release of the
tase inhibitor that results i n the flaps as described by Frechet. periosteum from the inferior
Answers Key 269

aspect of the zygoma and zygo- aspect of the forehead and hairline the level of the supraorbital r i m .
matic arch. or is placed just within the hairline Near the supraorbital rim, a thick-
to further camouflage the scar. A ening of periosteum termed the
2 b beveled incision to allow hair folli- conjoint tendon is incised sharply
The transfacial approach is used to cle growth through the scar adds or bluntly. Adequate release of the
perform an extended supra-SMAS additional camouflage, and an conjoint tendon at the lateral
rhytidectomy i n which the midface irregularized scar pattern is cre- supraorbital r i m is an essential fac-
is lifted by dissecting to the upper ated. The temporal incision is con- tor of the periosteal release. The
lip while remaining superficial to nected to the pretrichial incision dissection occurs over the deep
the SMAS but deep to the cheek and is posterior to the temporal temporal fascia and temporalis
fat. I t is not used as an approach hairline similar to the coronal lift. muscle without release.
for subperiosteal midface lifts. The advantage of the pretrichial
incision is that the forehead is not 5 d
3 d elevated, and the frontal hairline is The "sentinel" vein is a reliable
The platysma is a rhomboidal sub- preserved. The pretrichial forehead marker for the frontal branch of the
cutaneous sheet of muscle. The lift treats all aspects of the aging facial nerve, which lies superficial
muscle crosses the entire length of forehead and brow. to the dissection on the undersur-
the mandible and accounts for the face of the temporoparietal fascia.
mobility of the skin along the jaw- 2 b If the vein is cauterized, the bipolar
line. I t continues above the lower In general, the selected brow eleva- forceps are placed at the base of
cheek as a superficial aponeurotic tion procedure should be per- the sentinel vein to help prevent a
fascia that invests the muscles of formed before upper thermal neuropraxic injury to the
facial expression located i n the blepharoplasty so that the facial frontal branch of the facial nerve.
midface. plastic surgeon can judge the pre- Lateral and slightly inferior to the
cise amount of upper eyelid skin to sentinel vein, the zygomaticotem-
4. c be removed. This helps prevent poral sensory nerve is encountered
A SMAS rhytidectomy is performed excessive elevation of the brow-lid and is usually considered the lat-
by approximately 20% of facelift complex with the potential for eral border of the dissection.
surgeons. This may take the form causing lagophthalmos. I n some
of dissecting a SMAS flap limited to cases, the need for upper blepharo- CHAPTER 32
the area over the parotid gland or plasty maybe eliminated after brow Management of the Aging
extending the SMAS flap anterior lifting procedures. Periorbital Area
to the parotid gland. The SMAS
flap is then suspended postero- 3 b 1. c
superiorly. The brow depressor musculature As the nerve crosses the zygomatic
include the corrugator, procerus, arch, it lies between the periosteum
5. e depressor supercilii, and supraor- of the zygoma and the SMAS. This is
By resecting redundant platysmal bital orbicularis oculi muscles. The an important relationship to keep i n
muscle and advancing the platysma frontalis muscle is the only brow mind, because dissection i n the
medially toward the midline, the elevator. region of the arch should be carried
surgeon is addressing the deformity out subcutaneously or subpe-
at its origin and is advancing tissue 4 d riosteally. The nerve courses from
in the same direction as the gravi- The term "release" means to ele- the parotid gland toward its final
tational forces on the neck. vate, incise, and spread. The tech- destination, where i t pierces the
nique that achieves excellent brow undersurface of the frontalis muscle
CHAPTER 31 elevation is release of the perios- 1.5 cm above the lateral canthus.
Management of the Aging Brow teum from one inferolateral orbit to
and Forehead the other and release of the brow 2 a
depressor musculature (corrugator, The ideal female brow has medial
1. a procerus, depressor supercilii, and and lateral ends that lie on the
In patients with an elongated fore- supraorbital orbicularis oculi). The same horizontal plane. I n addition
head and high hairline, the pret- temporal conjoint fascia (fusion of to the other descriptions, the lat-
richial forehead lift may be used. the galea and the temporoparietal eral extent of the brow should
The pretrichial incision is located fascia) is released with a periosteal reach a point on a line drawn from
at the junction of the cephalic elevator i n an inferior direction to the nasal alar facial junction
270 ANSWERS

through the lateral eanthus of the patients with pseudoherniation of remaining fat cells to weight loss or
eye. I n men, there should be less of fat and little need for skin excision. gain.
an arch to the brow position and It is also good for patients prone to
more of a horizontal contour along hypertrophic scar formation and 5. c
the supraorbital ridge. patients unwilling to accept an When selecting a patient for lipo-
external incision. suction, young elastic skin, a
3 b strong skeletal structure with a
The major vascular supply lies CHAPTER 33 favorable position of the hyoid lar-
within the subcutaneous fat and ynx complex, and fat that is not
the superficial fascia of the Suction-Assisted Lipocontouring responsive to weight loss are the
frontalis muscle. Dissecting i n this ideal characteristics. Older patients
plane thus results in a higher inci- 1. b with loose inelastic skin will not
dence of vascular compromise with The gender of the patient does achieve significant improvement
skin slough and hair loss. This have an impact on the effective- with liposuction alone and will
plane is most useful i n brow lifting ness of liposuction. The thicker require some form of skin reduc-
techniques involving incisions skin of the male beard may help to tion, either through direct cervical
anterior to the hairline, where hair camouflage irregularities that can excision or facelift. Any patient
loss is not an issue. Subgaleal and occasionally occur but does not that considers cosmetic surgery to
subperiosteal dissection result in determine the ability to contour. create a major improvement i n his
flaps that are thicker and well vas- 2 b or her socioeconomic status should
cularized but also more limited as The remaining adipocytes i n areas be counseled and avoided; only
a result of inelasticity. that have undergone lipocontouring patients with realistic expectations
will respond to weight gain and should be treated.
4. c weight loss in relation to overall
When performed in conjunction body fat deposition. The evidence CHAPTER 34
with a browlift, the blepharoplasty indicates that weight gain occurs Mentoplasty and Facial Implants
incisions must be marked only through the enlargement of individ-
after the browlift has been per- ual fat cells and not by the addition 1. c
formed and all incisions closed. of new cells. If excessive weight gain 2 d
Upward repositioning of the brow occurs, then the original contour 3. a
reduces the amount of upper eyelid could be obtained through hyper- 4. c
skin excess, resulting i n a decrease trophy of the remaining fat cells. 5. c
i n the amount of skin to be excised
with blepharoplasty. Failure to 3. c CHAPTER 35
adhere to this may result i n the Maintaining bridges of uninter-
development of postoperative rupted tissue between the deep and Rehabilitation of Facial Paralysis
lagophthalmos. superficial tissue is important to
ensure viability of the elevated skin 1 b
5. e flap. The fat cells are selectively The zygomatic and buccal
The subciliary lower eyelid ble- aspirated as a result of their lack of branches of the facial nerve have
pharoplasty approach is most use- structural integrity, whereas the equal importance i n reinnervation
ful for patients with large amounts vessels, nerves, and muscles are procedures. These branches inner-
of excess skin. I n this technique, protected. vate the midface structures respon-
an incision is made 2 m m below sible for the smile and some
the lash line extending from 1 m m 4 b orbicularis oculi function.
lateral to the inferior punctum to With the undulations of the ultra- Restoration of innervation to these
10 m m lateral to the lateral can- sonic liposuction cannula, heat can muscles provides gross facial sym-
thus. I t extends through the skin be generated, which could lead to a metry. The cervical branch is least
and orbicularis oculi muscles. After thermal injury. No advantage is important for facial expression,
the skin-muscle flap is elevated obtained by the liberal use of lubri- although selective disruption of
and redraped and bulging orbital cating jelly. Other risks associated this nerve can result i n subtle lip
fat addressed, a variable amount of with liposuction are not signifi- asymmetry noted on mouth open-
skin and muscle are excised as cantly influenced by the use of ing. The frontal branch is relatively
needed. The transconjunctival ultrasonic liposuction. There is less important i n comparison as
approach is more appropriate for also no impact on the response of well. The mandibular nerve branch
innervates the depressor muscles
Answers Key 271

of the mouth and, although impor- PTFE is very biocompatible, infec- reestablishment of the antihelical
tant, does not take the same prece- tion rates of up to 9% have been fold by suture technique.
dence as reinnervation of the reported with its use.
midface muscles. 4. a
5. e 5. e
2 b EMG is indicated i n any facial
Electrical stimulability i n the distal paralysis lasting longer than 1 CHAPTER 37
nerve branches remains intact for year. I t can indicate whether rein- Physiology of Olfaction
up to 72 hours. After acute facial nervation is occurring and can also
nerve transection, exploration provide information about the via- 1 b
should be performed within this bility of facial musculature. If The cell body of the olfactory
time frame to repair injured nerves there is evidence of reinnervation, receptor neuron resides in the
if possible. If reinnervation is not procedures could be delayed to olfactory epithelium within the
performed i n this time frame, then observe for return of function. If nasal cavity. It is a bipolar cell with
marking the position of the distal there is total electrical silence, a dendrite extending to the
nerve branches after identification indicating severe facial muscle mucosal surface, where the olfac-
is suggested to aid subsequent rein- atrophy, reinnervation procedures tory knob gives rise to several cilia
nervation efforts. would not be indicated. Muscle containing the olfactory receptors.
biopsy is useful i n selected cases The axon extends into the lamina
3. c to confirm lack of viable facial propria and travels through the
This patient has a proximal nerve musculature. cribriform plate to synapse within
injury, and delayed nerve grafting glomeruli of the olfactory bulb.
to the brainstem will not provide CHAPTER 36
satisfactory results. The individual Otoplasty 2 b
has a viable hypoglossal nerve that This patient most likely has anos-
can be used. Muscle transfers could 1. e mia related to an upper respiratory
be performed, but the function Although autosomal dominance tract infection, which decreases
with nerve transposition is supe- may be the mode of inheritance, i t the flavor of food. The inability to
rior. Static techniques and upper is descriptive of the embryology detect smoke at close distances
eyelid adjunctive procedures could underlying protruding ears. The reflects the severity of the smell
be used i n selected cases to pro- otic placode is the first precursor loss. Ammonia is a strong stimula-
vide immediate restoration of sym- to the ear; however, the antihelix tor of the trigeminal system, which
metry and eyelid closure, but by derives from the fourth hillock of usually remains intact. I t is rare for
themselves would provide subopti- His and the conchal cartilage from both cranial nerve I and cranial
mal results. the ectoderm of the first branchial nerve V to be damaged at the same
groove. The hillocks derive from time.
4. c the mesoderm of the first and sec-
Like autologous material, PTFE ond arch. 3 d
and acellular dermis (Alloderm) There are approximately 1000
provide immediate restoration of 2. c genes encoding olfactory receptor
symmetry when used for static Although each of these is consid- proteins. A mutation i n one of the
midface suspension. Autologous ered i n the preoperative assess- "functional" genes may result i n an
materials generally require gross ment, i t is really the cartilage size odorant specific anosmia. A muta-
overcorrection to compensate for that determines whether to pro- tion i n genes encoding G-protein,
laxity developing i n the postopera- ceed with otoplasty. cyclic AMP, or the calcium/sodium
tive period, but this is not neces- channel would result i n the inabil-
sary for PTFE. Advantages of 3 d ity to detect all odorants. Inositol
biocompatible alloplastic materials Octyl-2-cyanoacrylate is reserved phosphate is not thought to play a
such as PTFE include no need to for neonatal otoplasty to avert role i n the olfactory receptor signal
harvest tissue, which prevents future surgical otoplasty. A postau- transduction cascade.
donor site morbidity and can ricular skin incision may set the
shorten the operative time. The ear back but does not address the 4 b
infection rate with alloplastic graft antihelix. I n the case of stiff carti- Over all testable ages, females do
materials is not less than w i t h lage, scoring may impede the carti- better than males i n olfactory iden-
autologous materials. Although lage spring and facilitate tification. A rapid drop i n odorant
272 ANSWERS

identification testing occurs i n the that allows the investigator to 5 b


seventh decade. Odorant identifi- determine the cross-sectional area The question refers to the situation
cation testing is thought to be of the nose at various distances of a combined structural problem
unreliable i n young children. into the nose. This allows the cal- as outlined i n the algorithm. I n this
culation of the MGA (minimal instance, a patient gets some relief
5. d cross-sectional area of the nose) by from decongestion, but i t is not
This patient most likely had anos- noting the distance into the nose complete. Furthermore, there is
mia develop i n relation to an upper where the greatest restriction to some observed anatomic pathology
respiratory tract infection. Her his- airflow occurs. Because AR gives an on the same side as the subjective
tory has been unchanged for the anatomic picture of the nasal cross- sensation of obstruction, which is
past 6 years, and she is too young sectional area, i t makes sense that confirmed by objective testing.
for age-related loss. Her nasal the curve generated can also be According to the algorithm, these
examination is without any evi- used to calculate the total volume patients benefit from a trial of
dence of obstruction, neoplasia, or of one or both sides of the nose. medical management followed by
inflammation. Because her symp- surgical intervention if the
toms are not atypical and her neu- 3. c response is unsatisfactory.
rologic history and examination Rhinomanometry measures the
are otherwise negative, an intracra- pressure differential across the CHAPTER 39
nial lesion is highly unlikely. A n nose from front to back. This can Manifestations of Systemic
MRI or GT scan would, therefore, be done on one side at a time as i n Diseases of the Nose
be unnecessary. Electroolfactograms anterior rhinometry or on both
are limited to research settings and sides at the same time as i n poste- 1. a
would only confirm her anosmia rior rhinometry. Active rhinometry 2 b
but may be helpful i n patients sus- refers to data collected using the 3. a
pected of confabulation. High-dose patients own respiratory effort, 4. a
steroids have not been shown to whereas passive refers to data col- 5. e
offer dramatic improvements i n lected while the patient holds his
anosmia of this type. The most or her breath and a known rate of CHAPTER 40
important step i n all patients with air is introduced into the nose. The Epistaxis
complete smell loss is to educate rhinomanometric data obtained are
the patient on health risks associ- typically displayed as a pressure- 1 b
ated with anosmia. flow curve. This curve is sigmoidal 2. c
in shape. The data from this curve 3 d
CHAPTER 38 can be then be used to calculate 4. a
Evaluation of Nasal Breathing resistance i n the nose. Both poste- 5. e
Function w i t h Objective Airway rior and anterior methods can be
Testing used to calculate total resistance. CHAPTER 41
By tradition, the data are displayed Nasal Fractures
1. d with pressure on the x-axis and
According to Kasperbauer and flow on the y-axis. Therefore, i n a 1 d
Kern, the nasal valve area is the more obstructed nose, greater pres- Radiographs are not indicated for
functional unit that "includes the sures are required to generate the evaluation of nasal fractures. GT
distal end of the upper lateral carti- same flow and the axis rotates scans may be used when an injury
lage, the head of the inferior toward the pressure or x-axis. associated with a high velocity
turbinate, the caudal septum, and impact has occurred or when phys-
the remainder of the tissues sur- 4. e ical findings are indicative of addi-
rounding the piriform aperture." Objective airway testing has many tional maxillofacial fractures.
The nasal valve alone was the slit- factors that lead to variability i n Immediate closed nasal reduction
like opening between the caudal results. The difficulty encountered is appropriate when the patient is
end of the upper lateral cartilage in controlling for all of the vari- seen soon after the trauma and
and the nasal septum. ables severely limits the clinical when nasal edema does not
usefulness of these tools. Other fac- obscure nasal bone position. For
2. e tors influencing objective testing the remaining cases, outpatient
Acoustic rhinometry results are dis- include secretions, exercise, med- reassessment is performed 48 to 72
played as an area-distance curve ications, temperature, and race. hours once swelling has subsided
Answers Key 273

and the results of a delayed closed does not provide adequate exposure 2. c
reduction can be more easily deter- in this case and would not allow for The Gell and Coombs type I
mined. proper reduction and fixation. (immediate or anaphylactic) reac-
tion is the mechanism of allergic
2. c 5 d rhinitis (hay fever) commonly
Nasal bone comminution, sur- Children are more likely than encountered by the otolaryngolo-
rounding facial fractures, and adults to acquire nasal septal gist. I n addition, i t is the mecha-
greenstick fractures all can be hematomas because of softer nasal nism of hypersensitivity reactions
potential causes for persistent tissues that are more susceptible to drugs, contrast materials, and
nasal deviation after CNR. They to shear forces. The hematoma insect stings. Thus, i t is important
should be considered after a nasal collection under the mucoperi- to understand i t , be able to recog-
septal fracture has been ruled out. chondrium separates the tenuous nize it, and treat i t appropriately.
Fibrous tissue formation associated blood supply to the quadrangular
with healing is generally not signifi- cartilage, resulting i n necrosis 3. c
cant i n adults less than a week w i t h i n 3 days. This destruction A high total IgE does not necessar-
after injury. then often leads to significant ily indicate the presence of allergy.
internal and external deformities. Skin tests and RAST (and other
3. e Nasal septal hematomas tend to be in vitro) tests demonstrate the
Because of faster rates of healing, compressible masses and usually presence of allergen-specific IgE.
closed nasal reduction should be are not discolored. Bacterial seed- However, the sine qua non of
performed early i n children when ing of a hematoma can result i n allergy is the production of specific
edema no longer obscures nasal abscess formation that then has and typical symptoms on exposure
bone position. Unless there is a the ability to spread to contiguous to one or more allergens.
medical contraindication, general areas including the intracranial
anesthesia should be used for most vault. 4. d
pediatric cases to ensure patient It is generally accepted that preven-
comfort and ease of reduction for CHAPTER 42 tion of the allergic reaction is much
the surgeon. Nasal septal Allergic Rhinitis preferred to treating its conse-
hematomas are considered emer- quences. Although immunotherapy
gent conditions that should be 1. d may also provide protection, avoid-
drained immediately on diagnosis. 2 d ance remains the best and safest
Pediatric nasal fractures should be 3 b treatment when i t is feasible.
treated conservatively to avoid fur- 4. e Although allergy patients will gener-
ther disruption of important 5. c ally require "rescue medications,"
growth centers. Open techniques no specific type of pharmacother-
should only be considered for CHAPTER 43 apy is universally effective or appli-
extensive fractures (i.e., nasal- Nonallergic Rhinitis cable, and all methods have
orbital-ethmoid fractures) or when drawbacks.
closed reduction cannot reasonably 1. d
reduce the deformity. Allergy may affect all aspects of the 5 a
ear, nose, and throat, including oti- Because immunotherapy carries a
4. a tis media, Meniere's disease, r h i - risk of severe reactions, even
Bicoronal scalp flaps are a good nosinusitis, laryngitis, chronic sore though the likelihood of anaphy-
choice for exposure of nasal-orbital- throats, as well as asthma and laxis is small when quantitative
ethmoid fractures, especially i n chronic cough. A l l otolaryngolo- testing is used, i t should not substi-
cases when a split calvarial bone gists should be able to suspect tute for simpler and safer measures
graft may be necessary to recon- allergy based on history, prescribe such as environmental control and
struct severely comminuted nasal appropriate pharmacotherapy, and pharmacotherapy. I t offers benefits
bones. Lateral rhinotomy incisions give advice regarding empiric to patients who have perennial
are only useful i n unilateral nasal avoidance measures. Depending on allergy and those with seasonal
injuries. Both lateral rhinotomy the training they have received, symptoms (typically i t is best used
and open-sky incisions leave con- otolaryngologists should be able to in patients with symptoms cover-
spicuous facial scarring that may either refer patients for appropriate ing several seasons or severe single
not be desirable. An intranasal inci- immunotherapy or administer i t season symptoms). The use of
sion or open rhinoplasty approach themselves. anti-IgE shows promise, because i t
274 ANSWERS

provides nonspecific results the septal floor from the maxil- mechanism. Tip retroprojection
(regardless of allergens involved) lary crest and creating a superi- ordinarily results from this inci-
and must only be given over a orly based swinging door. The sion, and thus i t is often the initial
short period of time. septum can then be replaced step i n retroprojecting an overpro-
into the midline and secured jecting tip.
CHAPTER 44 with suture to the periosteum of
The Nasal Septum the nasal spine. 5 b
Although microosteotomes are pre-
1. During the 10th week of embry- CHAPTER 45 ferred for percutaneous
onic development, the nasobuc- Rhinoplasty osteotomies, the latter are not the
cal membrane ruptures and only circumstance i n which
creates a communication 1 d microosteotomes are useful. These
between the nasal passages and Dissection of the soft tissues cov- small osteotomes create less dam-
the nasopharynx. Failure of this ering the nasal dorsum is best car- age to the nasal side walls, produce
membrane to rupture may result ried out i n the favorable tissue less bleeding and swelling, and thus
i n the membranous variant of dissection plane between the aid i n rapid healing.
posterior choanal atresia. overly SMAS fascia covering
and the underlying cartilage and CHAPTER 46
2. Nasal valve compromise can be bone of the supporting structures Special Rhinoplasty Techniques
appreciated by visual inspec- of the nose. If dissection proceeds No answers
tion, use of the Cottle maneu- w i t h i n the SMAS layer, various
ver, or by lateralization of the arteries, veins, nerves, and l y m - CHAPTER 47
ULC. Lateralization of the ULC phatics are damaged, leading to Revision Rhinoplasty
by insertion of a cotton-tipped increased intraoperative
applicator or a thin metal bleeding, swelling, and prolonged 1. b
curette provides perhaps the healing. Overresection of the lower lateral
best assessment tool. crura causes lack of support of the
2. a supratip area. A common deformity
3. The hemi-transfixion incision If overwide domal angles combined seen with this is supra-alar pinch-
allows superb access to the cau- with a wide interdomal distance ing and alar retraction. On lateral
dal septum and minimally dis- (bifidity) is found i n the nasal tip, view one should see 2 to 4 m m of
rupts supporting tip structures. suture reorientation of the domal columellar show.
Conversely, the full-transfixion angles and wide tip defining points
incision and the open approach is recommended as the most effec- 2 d
to the septum significantly dis- tive and safe tip technique. Vertical Preoperative photography is essen-
rupt tip support and are used division of the domes runs the very tial i n rhinoplasty. I t is useful dur-
primarily when septal perfora- real risk of asymmetric healing and ing the initial consultation with the
tion repair is being performed, loss of tip support. patient, as well as postoperative
or i n conjunction with an exter- follow-up. The base view provides
nal rhinoplasty procedure. The 3 b information about the size and
Killian incision created within Although noses are encountered i n shape of the columella, alar base,
the respiratory epithelium does which the nasal spine is overlarge nostrils, and the lobule. I n a true
not allow access to the caudal or even deviated, and thus requires base view, the tip should obscure
septal edge. correction, surgery of the nasal tip the radix.
size and shape per se is not princi-
4. Perforations are avoided by pally affected by the nasal spine. 3. c
careful membrane elevation and The spine may, however, play a The nasolabial angle i n men is
closure of all membrane rents role i n nasal tip projection or devi- should be between 90 and 95
that may occur. Interposition of ation and require alteration. degrees, and i n women it is
crushed cartilage bolsters the between 95 and 105 degrees.
repair and ensures against per- 4. e Depending on the amount of tissue
manent perforation. A complete transfixion separates excess or deficiency at the premax-
the medial crural footplates from illa, this angle may not reflect the
5. The significantly bowed septum the caudal septum, which i n most amount of rotation at the tip and
is approached by first freeing patients is a major tip support infratip lobule.
Answers Key 275

4 d C H A P T E R 52 3. c
Costal cartilage is most commonly Medical Management of Nasosinus Dividing the infundibulum into
taken from the seventh, eighth, or Infectious and Inflammatory thirds, Van Alyea found the ostium
ninth ribs. The medial portion of Disease to be i n the superior third i n 10%
the rib is taken, leaving the inner of cases, middle third i n 25% of
perichondrium intact to prevent 1. c cases, and inferior third i n 65% of
entry into the pleural space. 2. a cases. (Van Alyea OE . The ostium
3 d maxillare. Anatomic study of its
5. a 4 d surgical accessibility. Arch
A n open roof deformity occurs 5. e Otolaryngol 24:553-569, 1936.)
when a bony hump is removed and
the osteotomies do not adequately CHAPTER 53 4. c
medialize the nasal bones. If stan- Primary Sinus Surgery Minor complications such as
dard lateral osteotomies do not suf- hyposmia, headache, periorbital
ficiently mobilize the bones, then 1. e ecchymosis, periorbital emphy-
one can try percutaneous In a study by Meyers and sema, and facial pain can all occur.
osteotomies. Valvassori, 400 preoperative CT The most common minor compli-
scans were reviewed with attention cation is the formation of synechia,
CHAPTER 48 to anatomic variations. They found which usually does not require
Reconstructive Rhinoplasty six specific variations that may revision surgery. (Stammberger H,
predispose a surgeon to inadver- Posawetz W. Functional endoscopic
1. a tent penetration of the orbit or the sinus surgery. Concept, indications
2. a anterior cranial cavity. These varia- and results of the Messerklinger
3 d tions include (1) lamina papyracea technique. Eur Arch Oto-Rhino-
4 d lying medial to the maxillary Laryngol 247[2]:63-76, 1990.)
5 a ostium; (2) maxillary sinus
hypoplasia; (3) fovea ethmoidalis 5. a
CHAPTER 49 abnormalities, such as low or slop- Superiorly, the uncinate process
Radiology of the Nasal Cavity ing fovea; (4) lamina papyracea has three possible attachments: the
and Paranasal Sinuses dehiscence; (5) sphenoid sinus wall lamina papyracea, skull base, or
variations, such as septa attached middle turbinate. These variants
1. b to the carotid, or dehiscence of the are important to identify preopera-
2. e carotid or optic nerve; (6) sphe- tively because of variations i n
3. c noethmoid cells. (Meyers RM, frontal sinus drainage.
4. a Valvassori G. Interpretation of (Stammberger HR, Kennedy DW.
5. c anatomic variations of computed Paranasal sinuses: anatomic termi-
tomography scans of the sinuses: a nology and nomenclature. The
CHAPTER 50 surgeon's perspective. Anatomic Terminology Group. Ann
Infectious Causes Laryngoscope 108[3]:422-425, Otol Rhinol Laryngol Suppl 167:7,
of Rhinosinusitis 1998.) 1995.)

l a 2 b CHAPTER 54
2. b Mucoceles, CSF rhinorrhea, com- Revision Endoscopic Sinus
3. a plications of rhinosinusitis, and Surgery
4. c tumors are all considered absolute
5. a indications for endoscopic sinus 1. b
surgery. Headaches are a relative Culture results are not required to
CHAPTER 51 indication. Surgery should only make the diagnosis of chronic r h i -
Neoplasms be considered as a last resort after nosinusitis but may provide a use-
a thorough evaluation. ful adjunct for therapy. History
1. b (Stankiewicz JA . Directed func- alone is now not considered suffi-
2. c tional endoscopic sinus surgery cient to make the diagnosis of
3. a and headaches [letter]. Arch chronic rhinosinusitis. Objective
4. c Otolaryngol Head Neck Surg findings on CT scans and nasal
5. no answer 126[10]: 1277-1278, 2000.) endoscopy are required adjuncts
276 ANSWERS

to appropriate history to make CHAPTER 55 small amounts of sample for ade-


the diagnosis of chronic rhinosi- Cerebrospinal Fluid (CSF) quate results. The test is noninva-
nusitis. Rhinorrhea sive; the sample is collected as the
nasal fluid passively drains from
2. c 1. c the nose. Obviously, i t cannot
Although involvement/scarring Use of the term "spontaneous CSF provide specific information about
of the nasolacrimal duct can rhinorrhea" should be discouraged, the location of the CSF leak; i t
commonly occur when the maxil- because i t implies that the underly- only indicates the presence of a
lary ostium is widened too far ing etiology cannot be determined. leak.
anterior w i t h a backbiter, i t rarely In reality, the underlying etiologic
causes recurrent chronic rhinosi- factors i n CSF rhinorrhea can be 4. e
nusitis. determined. Only those cases that CSF rhinorrhea associated with
are idiopathic are truly "sponta- closed head trauma typically
3 d neous." CSF rhinorrhea is best cat- resolves with conservative manage-
The lateral wall and lamina egorized as traumatic and ment, but nontraumatic CSF rhin-
papyracea are important first nontraumatic. The traumatic group orrhea is much less likely to resolve
landmarks to identify i n revision includes head injury and surgery. with conservative management and
endoscopic sinus surgery, because The nontraumatic group may be hence is more likely to require
identifying these landmarks will further broken down i n to those operative repair. Numerous reports
help ensure protection against cases with normal intracranial confirm that endoscopic repair is
orbital injury. The posterior wall pressure and those cases with the preferred surgical technique if
of the maxillary sinus is often at abnormal intracranial pressures. operative repair is indicated.
the same depth as the sphenoid Altered CSF physiology and tumors Although a wide variety of grafts
face, which helps gauge the depth are both causes increased intracra- have been used with great success
of ethmoid dissection. The supe- nial pressure. in CSF leak repair, only pedicled
rior turbinate can be a critical mucosal grafts have been
landmark to help identify the nat- 2 b associated with a relatively high
ural ostium of the sphenoid Several studies have noted an asso- rate of failure.
sinus. The anterior wall of the ciation between nontraumatic CSF
maxillary sinus is not a commonly rhinorrhea and unrecognized ele- 5 b
used landmark i n revision sinus vated intracranial pressure and P-2 transferrin testing accurately
surgery. suggest a relationship between CSF confirms the presence of a CSF
rhinorrhea, the empty sella syn- leak. High-resolution CT can iden-
4. c drome (ESS) and benign intracra- tify the presence of skull base
In the patient with recalcitrant nial hypertension (BIH). Despite dehiscences that may represent
chronic rhinosinusitis who may this relationship, not all patients the site of leakage. Together a
require further surgery, all antibi- with nontraumatic CSF rhinorrhea positive p-2 transferrin study plus
otics administered should be i n have ESS or BIH; other causes of a CT showing a suspected skull
culture-directed format. nontraumatic CSF rhinorrhea base defect warrant operative
include intracranial and skull base exploration. As a result, tradi-
5. e neoplasms. The MRI finding of an tional CSF tracer studies may not
Magnetic resonance image of the empty sella suggests possible ele- be routinely necessary. Today,
sinuses with and without gadolin- vated intracranial pressure, not prophylactic antibiotics are not
ium is important to evaluate any lowered intracranial pressure. An routinely recommended, because
areas of dehiscent bone, opacifica- association between nontraumatic they do not seem to change out-
tion at the skull base to rule out CSF rhinorrhea and middle-aged comes, and they may lead to the
encephalocele, and in completely obese women has been described. development of resistant bacteria.
opacified sphenoids to help evalu- Radionuclide studies and CT cis-
ate for fungus and aneurysm or 3. a ternography at best can identify
pseudoaneurysm. MRI is notorious p-2 transferrin assay provides a 80% of CSF leaks. MR cisternogra-
for overestimating the extent of reliable method for confirming the phy does not require intrathecal
mucosal disease and has no role presence of a CSF leak. (3-2 trans- contrast; this imaging study relies
for evaluation of mucosal disease ferrin is specific marker of CSF, on the intrinsic signal characteris-
only. and this assay requires relatively tics of CSF.
Answers Key 277

CHAPTER 56 exercise compared with a no-exer- acid, which have antimicrobial


cise control day. I t is thought that properties. I n addition, the saliva
Physiology of the Salivary Glands exercise-related increases i n sym- from the submandibular and sub-
pathetic nervous system activity lingual glands contains high molec-
1 d cause constriction of blood vessels ular weight glycoproteins that
The average daily volume of saliva to the salivary glands, leading to a competitively inhibit bacterial
produced is between 1000 m L and reduction i n saliva secretion. attachment to the epithelial cells of
1500 mL. Sixty to seventy percent Dehydration during exercise is the salivary ducts.
of the total volume of saliva formed believed to be a contributing factor
in a day is secreted by the sub- for the reduced flow. 2. c
mandibular gland, 20% to 30% is Initial treatment of acute suppura-
secreted by the parotid glands, and 5. a tive sialadenitis begins with aggres-
5% to 10% is secreted by the sub- Immunoglobulin A (IgA) is the pre- sive medical management. This
lingual glands and minor salivary dominant immunoglobulin i n saliva includes prompt fluid and elec-
glands. Hyposalivation is defined as and plays an important role i n the trolyte replacement, oral hygiene,
an unstimulated salivary flow local immune defense system. The reversal of salivary stasis, and
<0.25 m L / m i n . parotid saliva contains 30 to 160 antimicrobial therapy. Stimulation
2. e jig/mL of IgA. Although IgG and of salivary flow is accomplished by
Myoepithelial cells are elongated or IgM are also present i n the parotid the use of sialogogues such as
star-shaped nonsecreting cells with saliva, their concentration is signif- lemon drops and orange juice. I n
long branching processes that sur- icantly lower. IgA exerts its antiin- addition, capable patients should
round the acinus and proximal flammatory protective functions be instructed on regular external
ducts. The observation that myoep- and down-regulates inflammation and bimanual massage, starting
ithelial cells possess adenosine by inhibiting IgG- and IgM-modu- from the distal bed of the gland
triphosphate activity, have intercel- lated functions. The role of IgA as a and working i n the direction of
lular gap junctions, and contain protective agent against dental duct drainage. Analgesics and local
myofilaments has led to the plaque formation i n humans is heat application ease the discom-
hypothesis that these cells have controversial. fort. Antimicrobial therapy is an
contractile properties and play a essential part of the management
role i n expelling preformed secre- CHAPTER 57 of acute salivary gland infections.
tions. Diagnostic Imaging and Fine- Antimicrobial therapy is initiated
Needle Aspiration of the Salivary empirically toward gram-positive
3 d Glands and anaerobic bacteria. However,
Operation of the Na -K -ATPase is
+ + the recovery of p-lactamase-pro-
essential for secretion of saliva. 1. e ducing bacteria i n 75% of patients
Hydrolysis of one ATP results i n 2. c requires the use of augmented
the active transport of 3 Na out of +
3. a penicillin and antistaphylococcal
the cell and 2 K into the cell. The
+
4 b penicillin or a first-generation
Na -K -ATPase is localized on the
+ +
5 d cephalosporin. Culture results
basolateral domain of the acinar should be used to further direct
cells. The Na -K -ATPase thus
+ +
CHAPTER 58 antimicrobial treatment.
maintains a high intracellular con- Inflammatory Disorders Methicillin-resistant S. aureus infec-
centration of K and a low intracel-
+
of the Salivary Glands tion may require the use of van-
lular concentration of Na , thus + comycin or linezolid. The use of
contributing to the resting mem- 1 b clindamycin or the addition of
brane potential. The parotid gland is most suscepti- metronidazole to the first-line
ble to such infections. The parotid agents to broaden anaerobic cover-
4 b gland produces saliva that is age has been advocated by some
A role for the sympathetic nervous mainly serous as opposed to saliva authors. Response to antimicrobial
system i n decreased salivary flow from the submandibular and sub- therapy is seen within 48 to 72
after exercise has been proposed. lingual glands that is primarily hours of initiating treatment and
In addition, an anticipatory mucoid. Serous saliva, unlike muci- should continue for 1 week after res-
decrease i n saliva flow rate has nous saliva, is deficient i n lyso- olution of symptoms. Rarely, conser-
been shown immediately before somes, IgA antibodies, and sialic vative measures fail to eradicate the
278 ANSWERS

infection, and surgical drainage of SS-A and SS-B ribonuclear proteins, the minor salivary glands, specifi-
a loculated abscess is necessary. is necessary for the diagnosis of cally the palate. The presence of
The surgical approach involves ele- Sjogren's syndrome. The presence squamous cell carcinoma i n a sali-
vation of an anterior-based facial of another autoimmune disorder, vary gland should immediately
flap with abscess drainage by way such as rheumatoid arthritis or sys- raise the question of a second p r i -
of radial incisions i n the parotid temic lupus erythematosus, would mary tumor, because this is clearly
fascia parallel to the facial nerve mandate a diagnosis of secondary more common than primary
branches. A drain should be Sjogren's syndrome. Patients who disease.
placed, and the wound edges have objective signs of sicca com-
should be loosely approximated plex but no evidence of an autoim- 3 d
with the central aspect left to heal mune process should be evaluated Elective neck dissection is not rou-
by secondary intention. for other causes. tinely advocated for salivary malig-
nancy. However, the indications for
3. e CHAPTER 59 elective neck dissection are stage
Several factors may account for the Trauma of the Salivary Glands III/IV tumors, high-grade mucoepi-
propensity of salivary stones to dermoid carcinoma, squamous cell
form i n the submandibular gland. 1. a carcinoma, and adenocarcinoma.
Wharton's duct is longer, has a 2. c The presence of cervical metastasis
larger caliber, and is angulated 3 d is an indication for neck dissection
against gravity as i t courses around 4 b but would be classified as thera-
the mylohyoid muscle, all of which 5. e peutic rather than elective.
results i n slower salivary flow rates.
Also, the saliva produced by the CHAPTER 60 4. c
gland itself is more viscous and has Benign Neoplasms of the Salivary The most common site of distant
a higher calcium and phosphorous Glands failure i n patients with parotid
concentration. malignancy is the lungs. Liver,
1. c bone, and brain metastases can
4. c 2. a occur but are clearly less common.
In most cases, no active therapy is 3 d Although cervical and lung metas-
required. The patient should be 4. e tasis occur with similar frequency,
reassured that the lymphadenopa- 5 d cervical metastases are classified as
thy is self-limited and usually will regional failure, not distant.
resolve spontaneously i n 2 to 4 CHAPTER 61
months. However, i n patients who Malignant Neoplasms 5. d
are systemically ill, highly sympto- of the Salivary Glands Postoperative radiation is indicated
matic antibiotic therapy is recom- in high-grade malignancies, nodal
mended. The (3-lactam antibiotics 1. a involvement, stage III/IV tumors,
are ineffective i n the treatment of Acinic cell carcinoma occurs most and i n those with positive margins.
GSD. The antibiotics reported to be frequently in the parotid gland. In Neutron therapy is advocated for
most effective are rifampin, eryth- fact, i t is rare that another gland adenoid cystic carcinoma, although
romycin, gentamycin, would contain acinic cell carcinoma. it is clearly not the standard tech-
azithromycin, and ciprofloxacin. nique, because very few centers
2 b provide this modality. Although
5 d The most common parotid malig- postoperative radiation does not
In general, the diagnosis consists of nancy is mucoepidermoid carci- improve survival, i t is believed to
establishing the presence of kerato- noma, with the second most improve regional control, espe-
conjunctivitis sicca and xerostomia common histologic pattern being cially i n advanced and high-grade
by clinical examination and objec- adenoid cystic carcinoma. Adenoid malignancy.
tive testing. This testing should cystic carcinoma is the most com-
include objective measurements of mon primary malignancy of the CHAPTER 62
decreased salivary and tear flow submandibular glands. Acinic cell
along with a minor salivary gland carcinoma occurs most frequently Physiology of the Oral Cavity
biopsy. I n addition, laboratory evi- in the parotid gland, whereas poly-
dence suggesting a systemic autoim- morphous low-grade adenocarci- 1. c
mune disease, specifically against noma is most commonly found i n Oral sensation is mediated by
nerves with multiple sensitivities
Answers Key 279

and functions. Thus, the lingual stimulus binding to G-protein-cou- under brainstem or involuntary
nerve responds to mechanical, pled receptors (bitter, sweet, and control.
thermal, and chemical stimuli. amino-acid stimuli) make up the
Likewise, interdental discrimina- first step of transduction. 2. a
tion and three-dimensional recog- Interestingly, entry of acid stimuli Videofluoroscopy shows all stages
nition (stereognosis) is mediated into cells is accompanied by small, of swallow i n detail. A patient with
by multiple nerves, not just those reliable changes i n intracellular chemoradiation may have oral
innervating the periodontal liga- pH, which are correlated with the problems related to xerostomia
ment. Although oral pain is fre- intensity of sourness. that could affect the pharyngeal
quently associated with neural function. Endoscopy does not view
processing i n the subnucleus cau- 5 b the oral stage of swallow.
dalis, other brainstem structures Each chorda tympani nerve inner-
are likely involved as well. vates the ipsilateral taste buds in 3. c
the fungiform papillae on the ante- Aspirating as soon as food, particu-
2. c rior two thirds of the tongue. If a larly liquid, enters the mouth is
There are numerous oral reflexes precise testing procedure is used, so usually caused by an abnormality
mediating both digestive and pro- that taste stimuli are applied only in tongue control to hold the bolus
tective functions. The central sub- to this part of the tongue, destruc- cohesively or a delay i n triggering
strates for many of these reflexes tion of the chorda tympani nerve the pharyngeal swallow. If there is
are only partially understood. would be obvious, because the indi- a pharyngeal delay, liquid can
Several reflexes that result i n vidual would not be able to detect quickly enter the airway before the
mandible elevation (jaw-closure) any type of taste sensation. pharyngeal swallow triggers.
are monosynaptic reflexes from However, if the person was allowed
muscle spindle afferents directly to take the stimulus into the 4. e
exciting jaw closer motoneurons i n mouth, so that i t contacted recep- With both an oral and a tongue
the motor trigeminal nucleus. tors on the back of the tongue or base disorder, you will want to
Cephalic phase reflexes include palate, taste loss would be subtle, examine the oral and pharyngeal
insulin release i n response to gus- because the innervated taste buds stages of swallow simultaneously.
tatory stimulation that influences on the back of the tongue and This requires videofluoroscopy.
parasympathetic preganglionic palate seem to be able to compen- With a tongue base disorder,
neurons i n the dorsal vagal sate for chorda tympani loss. A sim- endoscopy will allow you to visual-
complex. ilar type of testing procedure could ize the degree to which the tongue
detect glossopharyngeal damage if base and pharyngeal wall make
3 b stimuli were restricted to the foliate contact.
The dentine is permeated with or circumvallate papillae. However,
fluid-filled tubules that allow this nerve is not as vulnerable to 5. e
osmotic, thermal, and mechanical iatrogenic damage Although ani- Because supraglottic laryngectomy
stimuli to activate A-delta fibers mals with lesions of their chorda involves removal of a part of the
located i n the proximal end of the tympani nerve do demonstrate pro- tongue base, the top two sphincters
tubule. The fluid-filled dentinal found, specific losses i n the ability of the larynx, and disconnection of
tubule allows "hydrodynamic" to discriminate sodium, such a strap muscles from the hyoid to
forces set up by a distal stimulus to deficit is not as apparent i n the larynx, these swallowing disor-
activate a nociceptor. Once this humans, implying that the different ders are predictable based on the
process is underway, other regions of the mouth may not be as structures resected.
processes, such as the release of specialized as in animals.
neuropeptides into the pulp, or CHAPTER 64
central sensitization can exasper- CHAPTER 63 Oral Mucosal Lesions
ate the hypersensitivity. Mechanisms of Normal
and Abnormal Swallowing 1 d
4. e See Table 1. Aphthous ulcers are
Because there are many different 1. c distributed over the nonkeratinized
types of molecules that the taste The oral and oral preparatory mucosa but do not have a virally
system must transduce, at a m i n i - stages of swallow are under cortical associated vesicular phase or cyto-
mum, both direct stimulus entry voluntary control, whereas the pathic effect as does the infection
into cells (e.g., Na and acids) and
+
pharyngeal stage of swallow is produced by herpes simplex.
280 ANSWERS

TABLE 1

RECURRENT (2-DEGREE) H S V VS R A S (MINOR)

HSV RAS
Etiology Herpes simplex I/II Varied/immune dysfunction
Location Keratinized tissue mucosa Moveable/Nonkeratinized
Vesicle phase Yes No
Duration 7-14 days Varies (usually 7-10 days)
Management Topical (docosanol, penciclovir) Topically based steroids
Oral antivirals
Prodrome Often Uncommonly
Triggers Stress, trauma Stress, ultraviolet light, foods
Biopsy findings Viral cytopathic effect Nonspecific

2 b 3. c 3. c
Oral melanoma precursor phase or 4 b Ameloblastoma has a significant
developmental biology may best be 5 d recurrence rate and can become
compared with nodular or acral quite large and locally destructive.
lentigenes melanoma i n the CHAPTER 66 It is not a malignant process.
absence of the usual corresponding Odontogenesis and Odontogenic Recurrence with only simple enu-
phase(s) associated with cutaneous Cysts and Tumors cleation or enucleation followed
melanoma. with curettage is unacceptably
1 d high. One centimeter margins and
3 d Most odontogenic cysts and tumors extension to the adjacent unin-
Epithelial dysplasia is a microscop- are slow growing. They rarely per- volved soft tissue plane is ade-
ically defined term. Choices a, b, forate cortical bone or natural tis- quate.
and c are clinically distinguishable sue boundaries despite getting
from each other and, by virtue of quite large. This slow growth tends 4. a
their appearance, can be diagnosed to produce a sclerotic border that Most lesions of this type are
with relative confidence. is radiopaque. I t also allows expan- inflammatory responses to pulpal
sion of bone rather than perfora- involvement secondary to dental
4. c tion, and this expansion, if disease/caries. The large amalgam
Proliferative verrucous leukoplakia, between teeth, will also push the indicates previous caries. I t would
unlike the more common form of teeth apart and produce some be appropriate to first evaluate the
leukoplakia, carries a significant blunting of the roots because of tooth for restorability, and, if i t is
risk of carcinoma development and resorption. salvageable, root canal therapy
higher rate of recurrence. Location would remove inflammation and
does not enter into this separation, 2. c generally lead to resolution of the
given the widespread nature of pro- Multiple odontogenic keratocysts lesion without surgical interven-
liferative verrucous leukoplakia. are found i n Gorlin's syndrome, tion. This approach does not pro-
also known as basal cell nevus duce tissue for diagnosis and so
5 b syndrome. Although genetic coun- follow-up is needed to ensure reso-
Both mucosal pemphigoid and ero- seling may be appropriate, the lution.
sive lichen planus may involve the most significant problem that
attached gingiva and might, these patients face is that they 5. c
because of basement membrane have basal cell carcinoma develop The term "benign odontogenic
zone alterations, present as a i n non-sun-exposed areas. cyst" is often used by general
desquamative process. Frequent and thorough total skin pathologists to describe any cyst
evaluation must be done fre- within the jaws. I n this case, deter-
CHAPTER 65 quently by a dermatologist. mination of only that the lesion is
Oral Manifestations of Systemic These lesions do not undergo benign is not adequate. There are a
Disease malignant change, and i n and of number of different cysts, some of
themselves are no more likely to which require more aggressive
1. c recur than OKC i n nonsyndromic treatment than others and with
2. a patients. higher recurrence rates. A vague
Answers Key 281

diagnosis should always be investi- 5 a 3. b


gated and tissue evaluated by Diffuse sclerosing osteomyelitis The age of the patient is an impor-
someone with specific knowledge can widely vacillate i n its clinical tant consideration i n the surgical
of odontogenic lesions. presence and symptoms over the management of mandibular ankylo-
course of a lifetime, necessitating sis, because the associated lack of
CHAPTER 67 both palliative and definitive sup- mandibular growth caused by dam-
Odontogenic Infections port. Florid osseous dysplasia age of the condyle, as well as the
resembles an odontogenic tissue ankylosis, need to be addressed. To
1 b pathosis and should be treated maintain the new j o i n t space, i t is
It has been estimated that only 5% very conservatively. Osteora- important to use an interpositional
of odontogenic infections are dionecrosis is a complication of material. I n the child, a costochon-
caused exclusively by aerobic bac- radiation therapy i n which the soft dral graft provides both growth
teria, 60% are caused exclusively tissue, as well as the bone, must be potential and an excellent interpo-
by anaerobic bacteria, and 35% are definitively treated on an aggres- sitional tissue. I n treating ankylo-
caused by mixed infections. sive basis, such as hyperbaric oxy- sis, an understanding of the cause
gen therapy, debridement, and of the problem is also essential,
2. e reconstruction. Chronic because an autogenous graft or flap
Constipation may be associated osteomyelitis of childhood shows should not be used i n patients with
w i t h a dehydrated patient taking marked resolution w i t h skeletal active rheumatoid arthritis, i n
narcotic analgesics for an odonto- aging. whom such interpositional materi-
genic infection but is not as com- als can be destroyed by the inflam-
mon as facial swelling. Mental CHAPTER 68 matory process. Such patients
confusion is a late finding i n Temporomandibular Joint require an alloplastic j o i n t replace-
patients w i t h odontogenic infec- Disorders ment. Active, prolonged physical
tions. Diarrhea and productive therapy is necessary i n all cases of
cough are best described as possi- 1. c ankylosis to maintain movement
ble comorbid conditions i n Condylar hypoplasia is a condi- and to strengthen the masticatory
patients w i t h odontogenic infec- tion caused by trauma to the muscles. Although the amount of
tions. mandibular condyle occurring jaw limitation can be a problem i n
during the growth period. Because performing general anesthesia, and
3 d the condyle is an important the surgery can be more difficult i n
By definition, Ludwig's angina growth site for the mandible, those patients with severe limita-
involves the bilateral submandibu- injury i n this area results i n tion, this factor is less important
lar and sublingual spaces and the mandibular deformity. Therefore, than the others noted.
submental space. The lateral pha- early diagnosis and treatment are
ryngeal space may become second- important. Tile features described, 4. c
arily involved i n the patient with except for the absence of antego- Patients with locking caused by an
Ludwig's angina, but its primary nial notching, are characteristic anteriorly displaced, nonreducing
involvement is not required to of the changes produced by disk will not respond to medical
make the diagnosis of Ludwig's condylar hypoplasia. Increased management and require immedi-
angina. antegonial notching is pathogno- ate surgical intervention. The least-
monic of retarded mandibular invasive, highly effective procedure
4. c growth and is an important diag- is lysis of adhesions and lavage of
External bandaging is only used to nostic feature. the j o i n t by arthrocentesis. I n
support soft tissue wounds. those patients who do not respond
External skeletal fixation, 2. a, b positively to such treatment, surgi-
although acceptable, is technically Unless there is gross displacement cal disk repositioning, or disk
difficult and creates patient com- of the condyloid processes, removal if i t is not salvageable, are
pliance problems. Internal skeletal patients with bilateral fractures can indicated.
fixation is technically efficient and be treated by closed reduction and
physiologically acceptable if maxillomandibular fixation as long 5 b, c
applied correctly. Wire stabiliza- as the occlusion can be reestab- Although masticatory muscle over-
tion is not much better than soft lished. This is not a problem i n a contraction or overextension can
tissue bandaging and offers no real dentate patient and is also possible cause myofascial pain and dysfunc-
immobility except w i t h regional i n edentulous patients who have tion i n some patients, the most
fixation. dentures. common cause is increased muscle
282 ANSWERS

tension caused by psychological lower and will recreate the patients yet even more a primary goal
stress. Whereas the associated commissure at the same setting. before preservation of any tissues.
presence of chronic clenching or
grinding of the teeth is not essen- 4. a 2. c
tial, patients who engage i n such Byers and others demonstrated Whether anterior or posterior,
parafunctional activities are more that with lateralized oral tongue mandibular discontinuity usually
likely to have clinical symptoms. carcinomas skip metastases i n the creates problems associated with
Malocclusion of the teeth is not a N to level I I I and/or IV occur i n
0 swallowing from either interference
contributing factor. approximately 16% of patients. The with lip sealing, pull of unopposed
use of a supraomohyoid neck dis- pterygomasseteric slings, suprahy-
CHAPTER 69 section could potentially miss this oid muscles, or tethered oral
Benign Tumors and Tumor-Like regional disease, as such the tongue used for wound closure.
Lesions of the Oral Cavity authors advocated a level I to IV Mastication and speech are
neck dissection i n this setting. secondary effects of mandibular
1. d discontinuity depending on their
2. c 5. e location.
3 b In a patient with an oral lesion that
4. e will require soft tissue free flap 3 b
5 d reconstruction and the potential The primary role of the tongue is
need for postoperative radiation for swallowing. Respiration, speech
CHAPTER 70 therapy, removal of the lesion w i t h - production, and assistance during
Malignant Neoplasms of the Oral out the creation of a mandibulo- mastication are important func-
Cavity tomy is the best available option. A tions as well. However, swallowing
pull-through technique allows the is best achieved through both the
1. c flap access to the neck vasculature oral actions of the tongue and pha-
In patients with oral tongue carci- and spares the clinician from hav- ryngeal as well.
nomas and a depth of invasion of ing to radiate a recent mandibulo-
greater than 2 m m , the rate of tomy site. 4. c
regional metastasis exceeded 40%. Velopharyngeal insufficiency is a
Elective treatment of the neck has CHAPTER 71 common result of resection of the
been advocated when the risk of Reconstruction of the Mandible soft palate. Frequently, the soft
occult metastases exceeds 20%. and Maxilla palate is "insufficient i n form" to
close off the nasopharynx for swal-
2 d 1. e lowing and speech to occur.
The specific situation presented 2. c Incompetency and paralysis are
does not represent a situation that 3. c possible sequelae with surgery of
requires bony reconstruction. An 4 b this type but are uncommon.
elderly edentulous patient with a 5 b
lateral mandibular lesion can toler- 5 d
ate composite resection of the CHAPTER 72 A palatal lift prosthesis serves to
mandible with soft tissue recon- Maxillofacial Prosthetics for Head close off the nasopharynx with the
struction and maintain adequate and Neck Defects incompetent soft palate. A second-
speech and swallowing function. ary effect may be achieved by stim-
1. c ulating the soft palate into increased
3. e The hard palate is the primary sta- functional level. Pharyngeal muscle
With the average lip length of bilizing structure necessary for activity should be present, and the
approximately 6 to 7 cm, this prosthetic support. Without the extension should be aimed at the
lesion involves about half of the hard palate, the prosthesis will be first cervical vertebrae. Although a
lower lip. Primary closure is not an further seated into the defect, palatal lift does not directly
option, given the size of the lesion. resulting i n impingement of the decrease oral transit times, addi-
The Bernard-von Burow flap is structures within the nasal cavity. tion of material to the palate por-
intended for lesions greater than Removal of the inferior conchae, tion may serve the purpose of a
two thirds the length of a lip. A n grafting of the cheek flap and sinus combination palatal augmentation
Estlander flap (lip-switch) can use with split-thickness skin are also as well for patients who also have
the upper lip to reconstruct the important. Disease eradication is paralytic tongue effects.
Answers Key 283

CHAPTER 73 take into account lateral parapha- CHAPTER 74


Benign and Malignant Tumors ryngeal extension. The revised Pharyngitis i n Adults
of the Nasopharynx 1997 UICC/AJCC classification has
been prognostically validated both 1. e
1 d in Asia and the West. A viral etiology, most commonly
Surgery is the treatment of choice from rhinovirus, is the most com-
for JNA. Combination therapy or 4 b mon cause of acute sore throat i n
radiation alone is reserved for Radiotherapy alone is used i n the adults. The patient described i n
more advanced tumors. The main treatment of stage I and I I disease. this question has symptoms char-
considerations for the approach are Meta-analyses of six randomized acteristic of the common cold.
tumor location and extent, as well controlled trials found a progres- Nasal stuffiness and nonproductive
as the available surgical expertise. sion-free and overall survival cough are not characteristic for a
Endoscopic surgery, although ideal improvement of 34% and 2 1 % with streptococcal pharyngitis.
for avoiding soft tissue and bony the use of chemotherapy. Neck
disruption, is at present indicated irradiation is routinely given even 2. c
for low-volume disease with m i n i - in the absence of cervical l y m - The Centor scoring system is help-
mal extension. Radiation therapy phadenopathy because of the high ful for determining the likelihood
alone has been able to achieve incidence of subclinical disease. for a streptococcal etiology i n
local control rates of 80% for Although three-dimensional CRT adults that are seen with an acute
locally advanced disease. Recur- has improved tumor coverage, sore throat. This scoring system
rence is often difficult to predict there is no conclusive evidence does not include odynophagia but
even after complete extirpation that locoregional and long-term includes the remainder of the
and ranges between 30% and complications are improved. choices i n the question.
46%. Accelerated fractionation may
improve local control, but a ran- 3 d
2 d domized controlled trial using Penicillin is considered the first
Keratinizing SCC accounts for 20% accelerated hyperfractionation had choice antibiotic for patients not
to 30% of cases seen i n North to be prematurely terminated allergic to i t because of its narrow
America, but less than 3% of all because of excessive complications spectrum of action that includes
NPC i n Asia. WHO histopathologic without improvement i n local group A-p hemolytic Streptococcus
types I I and I I I have similar ultra- control. pyogenes (GABHS), low cost, and
structural features that make i t dif- safety. No evidence exists for toler-
ficult sometimes to distinguish 5. c ance or resistance of GABHS to
between the two. Type I is the least Surgery is never used i n the initial this antibiotic. Patients with m u l t i -
radiosensitive and exhibits the management of even bulky neck ply recurrent infections may have
poorest local control. EBV titers disease, because high response copathogens i n the oropharyngeal
have an 82% to 100% correlation rates can be achieved with tissues that produce p-lactamases,
with types I I and I I I compared with chemotherapy and radiotherapy. and an alternative antimicrobial
between 20% and 40% for type I . The main role of surgery is for would be indicated for these cases.
Fine-needle aspiration biopsy is the treatment of recurrent or residual
most expeditious; open biopsy disease i n the neck. The surgical 4. e
should be avoided if at all possible. approach must take into account Acute retroviral syndrome caused
the tumor extent, the required by human immunodeficiency virus
3. e exposure, and often control of the type 1 (HIV-1) is characterized by
Ho's classification comprises three internal carotid from the neck for a high viral load caused by the
T and N stages, and five overall tumors extending into the parapha- initial burst of viremia. ELISA and
stages, which differs from the TNM ryngeal space. Surgical morbidity, Western blot are not positive until
classification. Ho's classification is especially if the patient has approximately 4 weeks after
the most widely used i n Asia and received more that one course of acquiring the infection. The CD4
has been prognostically validated. radiation, is significant and count is normal during acute infec-
No survival difference has been includes skull base osteomyelitis, tion.
shown between T and T disease
t 2 necrosis, or bleeding. Surgery for
based on the 1992 UICC/AJCC recurrent disease can achieve local 5 b
classification. The 1992 control rates of between 43% and A painless, unilateral oral ulcer i n a
UICC/AJCC classification does not 67%. young patient with risk factors for
284 ANSWERS

sexually transmitted diseases tion as the cause of the obstruc- and types of upper airway recon-
should be highly suspicious for p r i - tive sleep apnea. This is the struction.
mary syphilis. Proper recognition rationale for a 75% surgical suc-
and treatment are essential for pre- cess rate w i t h adenoidectomy and CHAPTER 76
venting the infection from going tonsillectomy alone i n the pedi- Oropharyngeal Malignancy
into a latent phase and then re-pre- atric patient. Adults have prima-
senting as secondary syphilis. rily upper airway hypotonia as the 1. c
Secondary syphilis manifestations main factor and frank tissue The epithelium of anterior tongue
in the oropharynx reveal enlarged, obstruction as the second factor. up to the terminal sulcus is derived
reddened tonsils, and a rash com- This is one of the reasons why a from the first pharyngeal arch. The
monly involves the palms and uvulopalatopharyngoplasty has other tissues are correctly associ-
soles. Exudative pharyngitis is only a 39% to 40% surgical success ated with their embryologic struc-
more characteristic of gonococcal rate as defined as an apnea-hypop- tures of origin.
pharyngitis and infectious mononu- nea index <20 or an apnea index
cleosis. The role of chlamydia i n <10 w i t h at least a 50% improve- 2 d
causing pharyngitis in the absence ment. Nonkeratinizing SCC is less com-
of bronchitis or pneumonia is now mon than keratinizing SCC.
being questioned. 4. a-d Verrucous carcinoma essentially
Twenty percent of patients with does not metastasize. More than
CHAPTER 75 obstructive sleep apnea have 90% of all malignant oropha-
Sleep Apnea and Sleep-Disordered depression, and many others may ryngeal neoplasms are SCC.
Breathing have personality changes including Basaloid SCC occurs most
irritability. Cardiovascular associa- commonly i n the tongue base,
1. a tions are well known to be associ- followed by larynx, hypopharynx,
Dyssomnia is a sleep disorder that ated with OSAS, with 35% of and tonsil.
produces either difficulty initiating patients with hypertension, 85%
or maintaining sleep (insomnia) or with three medications to control 3 b
excessive daytime sleepiness. A n hypertension, 50% of congestive CT, MRI, and clinical evaluation
intrinsic disorder originates or heart failure patients, 35% of are equivalently accurate at detect-
develops within the body or arises angina and stroke patients having ing neck metastases, at a rate of
from causes within the body. an underlying diagnosis of obstruc- 70% to 80%. Contralateral metasta-
tive sleep apnea. Because of the sis occurs i n 20% of tongue base
2. a, c increased intrathoracic pressure tumors. A staging neck dissection
Upper airway resistance syndrome during the upper airway obstruc- has low morbidity, so i t is indicated
by definition has an apnea-hypop- tions during sleep, up to 45% of for most patients with oropharyn-
nea index <5 or i t would be classi- patients experience indigestion. geal SCC. After a margin-negative
fied as mild obstructive sleep apnea OSAS causes insulin resistance, resection of a T primary tumor
2

syndrome. I t is associated with fre- and 70% of patients with OSAS are with minimal or no neck disease,
quent respiratory-related arousals obese; these factors cause this postoperative radiotherapy can be
during sleep, two-thirds of patients association. avoided.
snore, and the main complaint is
daytime sleepiness or fatigue. It is 5. a, b, c 4. a
diagnosed during a sleep study by Cautious use of intraoperative and For resections of up to 50% of the
an elevated esophageal pressure, postoperative use of narcotics is tongue base, there is no adverse
elevated diaphragmatic eleetromyo- imperative. A PCA device is not effect on function; the defect can
gram, or other respiratory monitor recommended, because patients be closed primarily, through sec-
illustrating a respiratory effort asso- can suppress their respiratory ondary intention, or with a small
ciated with an arousal. This is one drive to a dangerous level. These thin flap. Tumors of the upper pha-
of the reasons that ambulatory or patients should receive the degree ryngeal wall are considered chal-
home sleep studies have limita- of monitoring that the surgeon lenging, because access and
tions, because they rarely suggest believes ensures a safe recovery, reconstruction are difficult. The
this diagnosis. taking into account many variables transoral route is used for resec-
including the body habitus of the tion of most tonsillar cancers. I n
3. c patient, the patient's medical con- soft palate cancer, radiotherapy is
I n general, the pediatric popula- dition, the severity of the obstruc- favored when surgical resection
tion has frank anatomic obstruc- tive sleep apnea, and the number would result i n considerable func-
Answers Key 285

tional impairment, which is usually accepted, and the other option is changes over time. A n examination
the case when the lesion is >2 cm to perform a folding technique to performed 1 month after therapy
in diameter. Resection of the tonsil reestablish the velopharynx. would serve primarily as a baseline
and surrounding soft tissue does for further tests. Endoscopy is l i m -
not usually result i n impairment of 3 b ited to mucosal recurrences. PET is
function. The goals of tongue base recon- sensitive for residual tumor but not
struction i n order of importance as accurate as combined PET/GT,
5. e are maintenance of the airway, particularly i n the head and neck.
It is difficult to provide sensation swallowing, and articulation. These
with the rectus abdominis flap. objectives are achieved with a form 2 d
Advantages of the rectus abdominis of reconstruction that provides the If a perforation or leak is sus-
flap include its ease of harvest, ver- necessary bulk to position the pected, barium should not be used
satility, length of its vascular pedi- neotongue above the laryngeal initially. Barium may inspissate i n
cle, and reliability. The forearm inlet. The perception of taste is not the soft tissues and cause a granu-
free flap uses the lateral ante- a goal i n reconstructing defects of lomatous reaction. If Gastrografin
brachial cutaneous nerve to provide the tongue base. is aspirated, i t may induce respira-
sensation. The lateral arm free flap tory distress; this is of particular
has a small-caliber feeding vessel. 4. e concern after a supraglottic laryn-
The muscle and soft tissue compo- The platysma flap has been under- gectomy. Non-ionic GT contrast
nents of a fibular free flap can be used for oropharyngeal reconstruc- agents are the most appropriate
epithelialized with a split-thickness tion. I t provides a reliable amount first choice; barium may then be
skin graft with acceptable results. of pliable tissue for reconstruction. used if no leak is detected.
Overall flap survival i n the tongue
CHAPTER 77 base has been reported to be 3. a
greater than 90%. Radiotherapy Endoscopy may cause strictures i n
Reconstruction of the Oropharynx and ligation of the facial artery are patients with epidermolysis bul-
not contraindications for the use of losa, so fluoroscopy is preferred.
1. c this flap. Manometry is most useful i n nut-
It has been shown that quality of life cracker esophagus. The other dis-
and functional status can be 5 d eases are best assessed
restored at 6 months and usually is The lateral arm flap is thought to endoscopically.
improved 1 year after microvascular be ideal for reconstructing com-
reconstruction of advanced oropha- bined defects of the pharyngeal 4. c
ryngeal tumors. Despite the fact that wall and tongue base. The distal Intramural pseudodiverticulosis is
the overall prognosis is poor, recon- aspect of the flap can be harvested dilation of mucous glands and is
struction is justified to achieve the over the upper forearm, providing not confused with ulceration on an
highest level of function possible. thin pliable tissue for the pharyn- esophagram. The other disease
There is no one free flap that is geal wall, while the bulky upper may all present with esophageal
superior for all oropharyngeal arm component may be used i n the ulcers.
defects, and despite our advances in tongue base. The flap is supplied
reconstruction, the overall survival by the posterior radial collateral 5 b
rate has not changed. Several fac- artery, which tends to be smaller Staging of esophageal carcinoma
tors come into play when deciding than the radial artery. However, i t relies on cross-sectional tech-
on which form of reconstruction is is a terminal artery, thus not put- niques for extent of tumor and
best for each individual patient. ting the arm at risk of ischemia. lymph node involvement.
2 d Neurotization may be performed Prevertebral spread of hypopha-
The Gehanno technique has been with the posterior cutaneous nerve ryngeal tumors is best assessed f l u -
used for defects that encompass of the arm. oroscopically by examining the
50% or more of the soft palate. motion of the larynx against the
Kimata and others reported that CHAPTER 78 vertebral column. Sinus tracts and
the incidence of flap dehiscence is Diagnostic Imaging of the Pharynx fistulas, i n general, are not well
higher i n the above case when this and Esophagus seen on GT. The GT appearance of
technique was not used. This leads a jejunal graft may be confused
to contracture and subsequent 1. e with recurrent tumor, whereas
velopharyngeal insufficiency. This Gross-sectional imaging, such as esophagram shows a characteristic
technique is not universally GT and MR, rely on morphologic mucosal pattern.
286 ANSWERS

CHAPTER 79 important parameter to measure typically a disease of African-


Endoscopy of the Pharynx during ambulatory 24-hour American men and is associated
and Esophagus esophageal pH monitoring. A reflux with alcohol and tobacco abuse.
episode is defined when the Patients with a history of caustic
1. a esophageal pH drops below 4.0. esophageal injury are also at
Rigid esophagoscopy, although i t can This value is chosen based on the increased risk. Adenocarcinoma is
be performed on the awake patient, proteolytic activity of pepsin, predominately a disease of white
is not well tolerated and carries an which is most active at and below men and has a well-documented
increased risk of complications. this pH. I n addition, pH less than association with GERD and
4.0 best distinguishes between Barrett's esophagus.
2. c symptomatic patients and asymp- Adenocarcinoma occurs predomi-
Functional endoscopic evaluation tomatic controls. nately i n the distal esophagus and
of swallowing (FEES) is typically at the GE junction.
used to assess delay i n swallowing, 2. a
laryngeal penetration, aspiration, The distinction between oropha- 5. c
and pharyngeal residue. As part of ryngeal and esophageal dysphagia Strong alkali and acids are the
the FEES vocal cord, mobility is is crucial, because they have differ- most likely to produce esophageal
examined as well. ent causes. Oropharyngeal dyspha- injury when ingested. Strong alka-
gia is most commonly caused by lis are contained i n drain cleaners
3. e disruptions i n the finely coordi- and other household cleaning prod-
Complications of rigid and flexible nated act of swallowing secondary ucts. Lye is a generic term for a
esophagoscopy include dental to neuromuscular dysfunction. I n strong alkali, usually sodium or
trauma, bleeding, perforation, this setting, the symptoms may be potassium hydroxide, used i n these
mediastinitis, cardiac arrhythmia, more severe when swallowing l i q - cleansing agents. Injury to the
and carotid artery dissection. uids. Any disease that affects the esophagus is more severe with
nerves or muscles can produce alkali substances than acid sub-
4. e oropharyngeal dysphagia, with the stances, because they produce a
Indications for pharyngoscopy are more common associations includ- liquefactive necrosis. This results
varied and are listed i n Table 1. ing ALS, myasthenia gravis, and in rapid and deep tissue injury
Parkinson's disease. compared with acidic agents,
5. c which produce a coagulative necro-
Although dental trauma is a com- 3. c sis that limits penetration and
plication of any peroral endoscopy, There is a clear association injury.
preoperative dental x-rays are not between asthma and GERD, and
routinely obtained. Instead, the 70% to 80% of patients with asthma CHAPTER 81
dentition is carefully examined have GERD. There are two main Zenker's Diverticulum
before and after the procedure. proposed mechanisms of acid-
Preoperative evaluation includes a induced asthma. The first is that 1 b
thorough physical examination, proximal esophageal reflux leads to Zenker's diverticulum is classically
review of medical and surgical his- microaspiration/bronchospasm. defined as a pulsion diverticulum
tory, review of current medicine The second is that a vagally medi- found bulging through the
regimen and drug allergies, and ated esophageal-bronchial reflex hypopharyngeal mucosa and sub-
possibly radiographic evaluation. results i n bronchospasm. GERD is mucosa between the cricopharyn-
Cervical spine instability should be the third most common cause of geal muscle and inferior constrictor
suspected i n any patient w i t h con- chronic cough after postnasal drip muscle i n an area of weakness
genital anomalies and evaluated and asthma. I t accounts for 21% of called Killian's dehiscence or trian-
preoperatively, especially if rigid cases of chronic cough. gle. However, there are other areas
esophagoscopy is planned. of weaknesses where pulsion diver-
4 b ticula may form, including Killian-
CHAPTER 80 The two main types of esophageal Jamieson's area between the
The Esophagus: Anatomy, carcinoma are squamous cell carci- oblique and transverse fibers of the
Physiology, and Diseases noma (SCCA) and adenocarci- cricopharyngeus and Laimer's t r i -
noma. More than half of the cases angle formed between the
1. a i n the United States are now ade- cricopharyngeal muscle and the
The percent time that the pH is nocarcinoma. The epidemiology of most superior esophageal wall cir-
less than 4 is the single most the two is quite different. SCCA is cular muscles. More rarely, they
Answers Key 287

also may be found i n the postero- for the ESD procedures. Among the which may be easily perforated
lateral or lateral areas of the phar- many theories attempting to with insertion of the laryngoscope
ynx and hypopharynx explain the etiology of Zenker's blades. One should insert the
(pharyngoeele). diverticulum formation is the dis- laryngoscope just far enough to
coordination of the cricopharyn- expose the superior border of the
2. b geal muscle during glutition, a common wall completely.
The diagnostic test of choice is bar- theory first proposed by Bell i n
ium swallow radiography. The test 1816. Even with the development CHAPTER 82
will allow the size and position of and numerous advantages of endo- Neoplasms of the Hypopharynx
the sac to be defined. Although the scopic techniques, there is still a and Cervical Esophagus
chest x-ray may reveal a hazy role for external approaches to
opacity over a lung apex suggesting address Zenker's diverticulum. 1. c
a diverticulum, i t does not have These include difficult and/or 2 d
any surgical value. I t may have impossible exposure of the diver- 3 b
value i n assessing preoperative pul- ticulum caused by patient anatomy 4. e
monary status of the patient, how- such as kyphosis, large cervical 5. c
ever. Occasionally, a diverticulum osteophytes, or small oropharyn-
may be first discovered inciden- geal opening. Also, retraction of the CHAPTER 83
tally during esophagogastroduo- common wall may not be possible Radiotherapy and Chemotherapy
denoscopy or rigid cervical in patients with recurrent small ZD of Squamous Cell Carcinomas of
esophagoscopy but is unnecessary from prior external approaches the Hypopharynx and Esophagus
in diagnosis. However, if other secondary to scarring, making
causes of dysphagia are suspected, exposure and divisibility of the 1. c
these tests may be worthwhile. GT cricopharyngeal muscle difficult if Advanced hypopharyngeal cancers,
scans are unnecessary, unless one not impossible, even with stitches when resectable, are better con-
suspects a neck mass contributing to help retract the common wall. trolled by radical surgery (i.e., total
to the patient's symptoms. Last, ESD should not be performed laryngectomy and partial pharyn-
if diverticular carcinoma is highly gectomy and radical neck dissec-
3. b, c, d suspected or confirmed on intraop- tion) and postoperative irradiation.
All endoscopic procedures take erative biopsy of a diverticular This treatment may control four of
approximately 30 minutes to per- lesion. External diverticulectomy five patients above the clavicles,
form. ESD does not induce thermal should be performed i n this case. but most of the patients subse-
injury to the mucosa and sur- quently have distant metastases
rounding tissues as laser and 5. a, b, d develop.
cautery methods. Such thermal Retraction sutures, placing the
injury could potentially injure the longer stapling blade containing 2 b
recurrent laryngeal nerve. ESD the cartridge into the esophagus, All studies have concluded that the
simultaneously incises and seals and using multiple stapler car- only one adjunct that has been
the mucosa with staples. ESD does tridges for large diverticula are all able to improve locoregional con-
have a lower complication rate good methods to help completely trol after radical surgery of the
compared with other endoscopic, divide the common wall between hypopharynx. Preoperative irradia-
as well as external, techniques. A l l the esophagus and diverticulum tion has demonstrated a deleteri-
endoscopic techniques do not pro- during ESD. Sawing off the distal ous impact, whereas chemotherapy,
duce an external scar. part of the stapler anvil as sug- whatever the setting, has no
gested by Collard is unnecessary improved the locoregional
4. a, e when using retraction sutures and control.
An internal cricopharyngeal potentially may even adversely
myotomy is performed with endo- affect the integrity of the stapler 3. e
scopic techniques when dividing introducing unnecessary risk to the Induction chemotherapy-based
the common wall that contains the procedure. One should never place larynx-preserving strategies have
cricopharyngeal muscle. ESD pro- the blades of the Weerda laryngo- demonstrated an ability to allow
cedures take approximately 30 scope directly into the diverticu- preservation of the larynx in good
minutes to perform, whereas exter- lum and esophagus as distally as responders to chemotherapy. On
nal techniques take several hours possible. The reason is because the the contrary, there was no impact
to complete. We have not found diverticular walls are composed of (favorable or unfavorable) on locore-
perioperative antibiotics necessary only mucosa and submucosa, gional control, distant metastases,
288 ANSWERS

second primary tumors, and overall the muscular process of the ary- 2. a, b, e
survival. tenoids, rotating that cartilage so 3. a, b, d , e
that the vocal process of the ary- 4. A l l would cause decreased
4. c, e tenoids moves rostrally and later- mucosal wave.
None of the studies that have ally. 5. a, c, d
examined the role of either postop-
erative radiation or postoperative 2 d CHAPTER 87
chemotherapy have observed any Laryngospasm is most like to occur Voice Analysis
benefit with the adjuvant treat- in response to laryngeal stimula-
ment. Increasing radiation dose did tion i n a well-oxygenated patient i n 1 b
not translate into enhanced sur- a light plane of anesthesia. Patient scales are extremely variable
vival but provides increased mor- and can measure many different
bidity and mortality rates. 3. e things. Some scales are well con-
Protracted radiation therapy During normal breathing, the PCA structed and demonstrate both relia-
increases the 2-year disease-free begins contracting just before bility and validity. Others do not.
survival i n a definitive chemoradia- onset of inspiration and is silent Using a scale such as the VHI brings
tion regimen compared with split- during exhalation. With increasing consistency and structure to ques-
course radiation therapy. A respiratory demand, the PCA con- tions about how the disorder affects
Cochrane systematic review favors tinues contracting after the onset physical, functional, and emotional
the preoperative chemotherapy. of exhalation to facilitate the well-being. They are a valuable addi-
egress of air. tion to a thorough voice evaluation.
5. a, c, e
Continuing chemoradiation is an 4. a 2. e
alternative to surgery i n locally Thyroarytenoid muscle contraction Hypernasality is often an indica-
advanced operable cancer respond- shortens and thickens the vocal tion of a structural or neurologic
ing to chemoradiation. Preopera- fold, lowering vocal pitch. process affecting voice and speech.
tive chemoradiation does not A clenched jaw, neck extension,
improve survival but improves dis- 5. c and decreased thyrohyoid space
ease-free survival. Up to now, we The phonated voice produced by reflect increased musculoskeletal
still do not know whether the larynx is articulated into words tension, which often adversely
chemotherapy provides any benefit by actions of the upper aerodiges- affects voice.
in terms of survival vs best sup- tive tract.
portive care i n metastatic disease. 3 d
Definitive chemoradiation with CHAPTER 85B The semitone scale is often used to
5-fluorouracil and cisplatin signifi- Evaluation and Management state frequency range, because i t
cantly increases survival compared of Hyperfunctional Disorders equalizes the differences between
with radiation therapy alone. two frequencies. To illustrate, a
1 d 100-Hz difference between two
CHAPTER 84 2. c tones is perceived as a greater dif-
Reconstruction of Hypopharynx 3 d ference at low frequencies than at
and Esophagus 4 b high frequencies. There are 12
5 d semitones between 98 Hz and 196
1 d Hz, whereas there are only 2 semi-
2 d CHAPTER 86 tones between 880 Hz and 988 Hz.
3. c Visual Documentation Frequency is only one dimension
4 b of the Larynx of pitch, so answer "a" is incorrect.
5. c Loudness is the perceptual corre-
1. c late of intensity, and frequency and
CHAPTER 85A A moire pattern is a colored fringe intensity are interrelated.
Laryngeal and Pharyngeal effect produced when there is over-
Function lap of linear features i n an image. 4. a
It can be reduced by slightly defo- Narrow-band spectrograms show
1 d cusing the image or with a filter the fundamental frequency and
The posterior cricoarytenoid mus- placed between the eyepiece and harmonic structure. Jitter and
cle pulls medially and inferiorly on the camera. shimmer are influenced by many
Answers Key 289

factors and not necessarily reliable tion. GT tends to be excellent for ography has no role i n cartilage
or valid, especially for acoustic sig- neck and chest evaluation, and MRI invasion.
nals that lack a single fundamental is superb for skull base evaluation.
frequency. GPP seems to correlate Perineural infiltration of the nerves 5. e
with breathiness but is not based by distant disease is very rare. Imaging of the posttherapy neck
in frequency analysis. Mean Signs of paralysis include parame- remains challenging, although MRI
nasalance below 50% for sustained dian position of the cords, dis- has emerged as the most reliable
In! generally corresponds to placed arytenoid cartilage, readily available cross-sectional
hyponasality, not above 50%. ipsilateral dilation of the pyriform modality. I n general, scar tends to
sinus, tilting of the thyroid carti- remain stable or even contract
5. c lage, and prominent laryngeal with time, whereas recurrent
EGG traces show degree of vocal ventricle. tumor presents as expanding nodu-
fold contact but i n a relative man- lar scars within the posttherapy
ner. EGG does not show actual 3 d field. Despite these guidelines,
degree of closure. The other MRI offers significant advantages hemorrhage and edema may per-
answers are true. for evaluation of complex disease sist for 4 to 6 weeks and confound
of the neck. Its superior soft tissue interpretation. Therefore, a base-
CHAPTER 88 differentiation provides excellent line is best postponed for 6 to 8
Diagnostic Imaging of the Larynx information about primary lesion weeks after therapy.
location and extent of spread.
1. e Submucosal disease is especially CHAPTER 89
Retropharyngeal abscess consists well studied. One of the major Neurologic Evaluation
of a masslike collection of purulent advantages of MRI is the capability of the Larynx and the Pharynx
fluid i n the retropharyngeal space. of multiplanar display. This fea-
Imaging findings reflect this patho- ture permits evaluation of 1. e
physiology. Plain films, which anatomy and lesion i n three 2 d
poorly define soft tissues, never- dimensions. The remaining 3 b
theless reveal the masslike proper- choices represent some of the 4. c
ties by demonstrating other limitations of MRI. 5. e
displacement of the airway, thick-
ening of the soft tissues, and occa- 4 b CHAPTER 90
sionally soft tissue emphysema. GT Although GT can detect the differ- Laryngeal and Tracheal
and MR imaging better display the ence i n densities of soft tissues Manifestations of Systemic Disease
actual fluid w i t h i n the retropha- such as tumor and fat, i t is much
ryngeal space. Fluid tends to be more limited i n differentiating sim- 1. a
hypodense compared w i t h soft ilar tissues such as tumor and mus- Clinical studies have demonstrated
tissue on GT and hyperintense on cle. Therefore, infiltration of the the efficacy of all of the above
T2-weighted MR images. thyroarytenoid muscle with tumor treatments except humidified air.
Administration of contrast on may appear similar to a flaccid par-
either GT or MRI often reveals a alyzed thyroarytenoid muscle. MRI 2. c
ring pattern bordering the fluid. better demonstrates tumor infiltra- The factors that have been shown
Because of its availability, speed tion into fatty spaces, such as the to correlate with likelihood of i n t u -
of imaging, and excellent paraglottic and preepiglottic bation are those that present with
anatomic display, GT has become spaces, because of its superior soft stridor, tachycardia, rapid progres-
the preferred modality for con- tissue differentiation. Phase of res- sion of symptoms, or blood cul-
firming retropharyngeal abscess. piration can dramatically alter cord tures positive for H. influenza.
configuration and lead to misdiag-
2. c nosis of tumor extent. The anterior 3. e
Evaluation of vocal cord paralysis commissure should be no greater Incidence is rising i n infants,
should include the entire course of than 1 m m thick. Values >1 m m teenagers, and adults. Multiple
the vagus nerve from the skull base imply tumor infiltration. Both GT causes for the increase have been
to the pulmonary hila. Most causes and MRI are preferred for detecting hypothesized, but none proven.
of paralysis are peripheral, and, cartilage invasion. MRI may have Acquiring pertussis by contact from
therefore, brain imaging alone is advantages over GT according to another infected individual protects
inadequate for thorough evalua- some investigators. Plain film radi- one from future infection for at
290 ANSWERS

least 3 years, but then this protec- ship with tracheotomy. 3. d


tion starts to wane. Infants (or Tracheotomy does not prevent 4. b
adults) often do not exhibit the clas- aspiration but does decrease dead 5. d
sic "whooping" cough, although this space and improve pulmonary
sign is commonly seen i n children. toilet for patients with chronic CHAPTER 94
aspiration. The Professional
4 b Voice
Chest radiographs are positive only 3 d
60% to 80% of the time, and other The sensitivity and specificity of 1.
manifestations of pulmonary dis- FEES and videofluoroscopic swal- 2.
ease (such as bloody sputum) are low studies are similar, thus they 3.
even less common. HIV has not should be performed according 4.
been shown to be a risk factor for hospital and provider preference. 5.
laryngeal tuberculosis, but there is For unstable patients who are
a strong association between smok- unable to be transferred to the
ing and alcohol use and the pres- radiology suite, a FEES is a better CHAPTER 95
ence of laryngeal TB. PPD test is option i n that i t may be performed Benign Vocal Fold Mucosal
usually positive i n patients with at the bedside. Disorders
laryngeal tuberculosis.
4. a 1 b
5. d Fifty percent of normal, healthy 2. a
Pseudoepitheliomatous hyperplasia patients have some degree of aspi- 3 d
can be seen i n all of the above ration while sleeping. This is typi- 4. e
except actinomycosis. cally clinically insignificant. 5 d

CHAPTER 91 5 b CHAPTER 96
Chronic Aspiration Patients w i t h chronic aspiration Medialization Thyroplasty
should not be started on empiric
1 b antibiotics. Antibiotic therapy 1 d
Laryngotracheal separation may be should be initiated if and when 2. c
performed at the bedside, making their clinical picture suggests 3. c
this procedure possible for unsta- pneumonia. Discontinuation of 4 d
ble patients who are unsafe for oral intake combined w i t h 5 d
transportation. Successful reversals establishment of an alternative
of this procedure have been route of alimentation and swal- CHAPTER 97
demonstrated, and several series lowing therapy are standard Arytenoid Adduction
have reported efficacy i n children. nonsurgical treatments for
Laryngotracheal separation is fre- aspiration. 1. c
quently performed i n patients with 2 d
prior tracheotomy, because the CHAPTER 92 3. e
diversion procedure may be Laryngeal and Esophageal 4. a
technically difficult i n such Trauma 5. c
patients. The chronic pooling of
secretions i n the laryngeal pouch 1. d CHAPTER 98
has not been found to be clinically 2. f Laryngeal Reinnervation
significant. 3 d
4. e 1 d
2 d 5 b Most of the time after an RLN
A causal role for tracheotomy i n anastomosis, vocal fold motion
the development of aspiration has CHAPTER 93 does not return. Instead, a laryn-
yet to be demonstrated. The only Surgical Management of Upper geal synkinesis occurs, with adduc-
prospective trial with preoperative Airway Stenosis tor and abductor nerve fibers
evaluation of aspiration, although nonselectively innervating the
limited by small sample size, failed 1 d laryngeal muscles. The result of
to demonstrate a causal relation- 2. c this neuromuscular mismatching is
Answers Key 291

that counteracting forces are CHAPTER 99 higher i n patients w i t h positive


applied to the arytenoid by all the Malignant Tumors of the Larynx adenopathy. Therefore, the
muscles innervated by the RLN and Hypopharynx 5-year overall survival remains
and little or no functional move- poor even w i t h more aggressive
ment occurs. 1. d treatment or multimodality treat-
Ensley and others, demonstrated ment.
2. c that a positive response to
The neuromuscular pedicle (NMP) chemotherapy, especially a patho- 4. a
technique and the ansa cervicalis logic GR, had a favorable prognosis. Multiple series have reported that a
to recurrent laryngeal nerve (ansa- This concept was challenged by modified neck dissection is all that
RLN) anastomosis both use a Poulsen, but even that study is necessary for an N or N posi-
0 x

branch of the ansa cervicalis as a showed that in a select group of tive neck. However, recent studies
donor nerve. The NMP typically patients, the need for laryngectomy confirm that a bilateral modified
uses the branch to the superior would be reduced. The NIH neck dissection is required. I n
belly of the omohyoid muscle, Intergroup study showed that there some series, the incidence of
whereas the ansa-RLN anastomosis was a positive effect from recurrence is highest i n the con-
typically uses the branch to the chemotherapy/radiation therapy tralateral neck, even if i t has been
sternothyroid or the sternohyoid when given concomitantly in treated with postoperative radia-
muscle. extending the length of time until tion therapy. The submandibular
laryngectomy. gland is included when level I is
3 b suspected to be positive, and i n
Of the muscles listed, the thyroary- 2. c many cases because chronic
tenoid muscle is the fastest. I n the Wolf and others provide long-term sialadenitis may be confused with
body as a whole, the extraocular follow-up i n the original VA laryn- recurrent disease.
muscles are faster. The soleus mus- geal preservation study. Persistent
cle is one of the slowest. The speed neck disease was best treated by 5. e
of contraction is related to the neck dissection as the result of per- Decreased vocal cord mobility
fiber type of the muscle, which is sistent or recurrent disease i n the implies invasion into the thyroary-
related to its myosin heavy chain neck, even when response at the tenoid muscle or involvement of
composition. primary tumor was poor. The p r i - the cricothyroid joint. This can
mary laryngeal tumor and neck lead to paraglottic space involve-
4. a disease should be evaluated sepa- ment, which limits the capability
The ansa cervicalis-to-recurrent rately. There is still controversy on of a limited resection and likeli-
laryngeal nerve anastomosis is the management of N neck disease
2 hood of cure by radiation therapy.
indicated for unilateral vocal fold as to the necessity of a routine
paralysis. Bilateral vocal fold paral- neck dissection after chemother- CHAPTER 100
ysis and glottic stenosis are con- apy/radiation therapy if there is a Management of Early Glottic
traindications, because vocal fold clinical GR. All authors agree, Cancer
movement is not expected as a though, that persistent disease
result of the procedure. needs to be addressed surgically, 1. e
especially if not resolved by 2. c
5 b 3 months. 3. a
The recurrent laryngeal nerve is 4. c
divided, and the distal portion of 3. e 5 a
the RLN is attached to the proxi- Most patients w i t h hypopharyn-
mal ansa cervicalis nerve during geal cancer are initially seen w i t h CHAPTER 101
an ansa-RLN anastomosis. This T N disease, usually w i t h the p r i -
3 + Transoral Laser Micro Resection
effectively eliminates the possibil- mary malignancy i n the pyriform of Advanced Laryngeal Tumors
ity of spontaneous recovery of the sinus. These patients have a high
RLN nerve. I t is important to incidence of comorbid disease 1 b
wait until the likelihood of sponta- (Carpenter's study) and a high 2 d
neous recovery of RLN is minimal incidence of second primary 3. a
before performing an ansa-RLN tumors (Raghavan study). The 4 d
anastomosis. incidence of distant disease is 5. c
292 ANSWERS

CHAPTER 102 CHAPTER 103 ately adjacent to the airway. The


Total Laryngectomy low paraesophageal lymph nodes
Conservation Laryngeal Surgery and Laryngopharyngectomy are, however, situated away from
the carina and passage of the left
1 d 1. a mainstem bronchi; therefore, under
The cricoarytenoid unit is the 2 a normal circumstances, i t is better
essential functional unit of the lar- 3. a examined and sampled by way of
ynx i n the organ preservation para- 4. a upper GI endoscopic ultrasonogra-
digm for laryngeal cancer. I t 5. a phy. I n addition to the sampling of
includes the recurrent and superior regional lymph nodes, trans-
laryngeal nerves, the associated CHAPTER 104 bronchial needle aspiration (TBNA)
musculature, along with the ary- Radiation Therapy for the Larynx techniques can also be used i n con-
tenoid and cricoid cartilage. and Hypopharynx junction with transbronchial biop-
2. e sies and washes to sample
The T-staging system for glottic 1. a peripheral lung nodules. Nodules
and supraglottic cancer does not 2. e <2 cm are locatable with the help
indicate the conservation laryn- 3. e of fluoroscopy, CT scan, and, i n the
geal procedure to be performed. 4. b future, by electromagnetically
The T-staging system does not 5. c guided systems. However, the yield
include the precise, detailed is definitely lower for smaller and
anatomic information necessary CHAPTER 105 more peripherally located lesions.
to be able to perform these surger- Vocal Rehabilitation Following Historically, rigid TBNA needles
ies. For example, a T glottic
3 Laryngectomy were used to sample the left-atrial
lesion will be amenable to pressure, although this is no longer
supracricoid laryngectomy, 1. e performed as a primary indication.
whereas a T lesion w i t h interary-
2 2. c
tenoid involvement is not a candi- 3 d 2. a
date for this procedure. 4. e Although i t is advisable not to take
5 b undue risks when performing a
3. b bronchoscopy, there is, i n fact, no
The supracricoid laryngectomy can CHAPTER 106 required standard set of preproce-
be performed to include the entire Management of the Impaired dure laboratory studies that will
preepiglottic space along w i t h the Airway i n the A d u l t predict or preclude hemorrhagic
epiglottis for transglottic tumors complications. Cancer patients
affecting this area. The reconstruc- 1. c undergoing cytotoxic treatment
tion would require a cricohy- 2. e may become pancytopenic with
oidopexy. 3 d opportunistic infections requiring
4. e diagnosis; i n these cases, transfu-
4. c 5. c sion is attempted to maintain nor-
The supraglottic laryngectomy mal platelet counts, but when
requires a cut that is performed at CHAPTER 107 indicated, lavage and careful biop-
the level of the ventricle to pre- Endoscopy of the sies may be performed. Sedation
serve the true vocal folds. If tumor Tracheobronchial Tree with propofol (Diprivan) is consid-
involves the ventricle, this cut can- ered deep sedation, requiring very
not be made safely. 1 d close monitoring by qualified per-
With slim and ultrathin broncho- sonnel, because respiratory arrest
5. c scopes measuring less than 3 m m can otherwise easily occur. General
The supracricoid laryngectomy in diameter, a FOB can be steered anesthesia (GA) with the use of
requires at least one functional into the 12th to 15th generation paralytic agents is generally
cricoarytenoid unit. This includes bronchi. Endobronchial ultrasound reserved for rigid bronchoscopy,
the cricoid cartilage. A tumor that can provide a real-time image of and rarely is rigid bronchoscopy
extends into the subglottis below transbronchial structures such as performed with them; however, GA
the level of the cricoid cartilage lymph nodes and blood vessels and with paralysis may also be required
will not allow for preservation of to help direct sampling of lymph for certain interventional proce-
the cricoid. nodes and other structures immedi- dures performed with a flexible
Answers Key 293

fiberoptic bronchoscope, such as coagulation with a much shallower variants may be used for covering
laser or argon plasma coagulation depth of penetration is much less over tracheobronchialesophageal
debridement when the risk of mis- likely to do so. Certain tumor abla- fistulas and other causes of airway
fire is increased i n a spontaneously tive therapies such as cryotherapy, perforation. The perfect airway
breathing and coughing patient. PDT, and brachytherapy have a stent has not been made, one that
Lidocaine i n dosages i n excess of delayed response such that critical is easy to deploy, removable
500 mg may be systemically airway narrowing should be man- when desired, but w i l l not u n i n -
absorbed i n sufficient quantities so aged by other techniques. tentionally migrate, and does not
as to cause seizures and other Conversely, PDT and brachyther- cause granulation or promote
complications. The choice of bron- apy can both have a prolonged infection. Future stents may be
choscope type (rigid vs flexible), effect, and this may account for coated w i t h special coverings
size and passage of entry (oral vs the 3% to 25% incidence of mostly that will make them useful drug
nasal) depends on many parame- delayed fatal hemoptysis. Lesions delivery devices for the local dep-
ters, including the stability of the in the right upper lobe take off, osition of antineoplastic, antifi-
patient's neck and facial bones, and the distal left mainstem, per- brotic, antiinfective, or gene
size of the patient and hence his haps by its relationship to the therapy.
nares, and planned procedures. For respectively pulmonary arteries, is
example, with flexible FOB, place- most prone to this potential devas- 5 d
ment of endobronchial brachyther- tating complication. Endoscopic lung volume reduction
apy afterloading catheters will be (ELVR) will attempt to replicate
easier with a nasal route, whereas 4. b surgical lung volume reduction
anticipated retrieval of a foreign Although the earliest stents for tra- surgery (LVRS) by causing
body would be more easily cheobronchial uses were made of selected regional atelectasis of
removed orally. hard polymers (the Montgomery hyperinflated lung segments. The
T-tube), stents are currently made FDA is currently favoring trials
3. e from a range of materials, ranging w i t h only removable valve devices
Low-grade fever can commonly from silicone to various types of (i.e., a reversible process). Use of
occur after a diagnostic bron- metal. There are also silicone existing approved drugs i n combi-
choscopy and is most often self- stents with embedded stainless nation with approved devices w i l l
limited. Informing the patient steel support struts. Silicone stents require further testing for safety
ahead of time will help to alleviate are removable; because they are by and efficacy. Autofluorescence
much anxiety. Although pneumoth- nature completely covered, they bronchoscopy (AF) makes use of
oraces occur most often with trans- are, however, also more prone to the properties of tissue autofluo-
bronchial biopsies, needle migration and by their thickness rescence and does not require an
aspirations or brushings of the also lead to impaction of secre- exogenous photosensitizer.
peripheral lung with unintentional tions. Although most silastic stents Advances i n airway imaging
trauma to the visceral pleura, i t are incompressible and require a include the creation of 3D images
can occasionally happen i n rigid bronchoscope or a suspension and virtual fly-throughs, w i t h the
patients with severe GOPD and laryngoscope for delivery and capability of presenting even a ret-
bullous lung diseases, who may deployment, there is now also rograde view up the airway.
perform a Valsalva maneuver and available a compressible and self- However, false-positive results
cough vigorously during the proce- expanding polyester-silicone stent. from airway secretions and the
dure. Airway perforation is a risk Most metallic stents are self- present resolution is insufficient
with interventional procedures, expandable (SEMS), but not the for i t to replace diagnostic bron-
including by the tip of the rigid earliest Palmaz and Gianturco choscopy, and imaging cannot
bronchoscope, inadvertent passage stainless steel stents that require substitute for tissue sampling.
of other firm instruments, or bal- balloon dilation for deployment. Endobronchial ultrasound (EBUS)
loon bronchoplasty through a false These earlier stents are no longer helps to direct TBNA sampling of
lumen because of the necrotic used i n the airways because of regional l y m p h nodes; however, i t
tumor debris that has replaced the their tendency to perforate airways can only do so for l y m p h node sta-
normal bronchial wall. Although and lack of covering that render tions adjacent to the airway,
laser, especially when set at a high them ineffective for stenting tumor hence the low paraesophageal and
wattage setting and fired i n a con- infiltrated airways. Newer SEMS lateral aortopulmonary lymph
tinuous mode, can definitely cause come i n both covered and uncov- nodes are still not accessible by
airway perforation, argon plasma ered versions, and the covered this technique.
294 ANSWERS

CHAPTER 108 examination, because i t can be 3 b


Diagnosis and Management masked laterally by the mandible. The histologic pattern of alternat-
of Tracheal Neoplasms ing regions containing compact,
5. a spindle cell Antoni type A areas
1. b Because of mucosa redundancy and more loosely arranged Antoni
2. e and proximal location of the type B areas is characteristic of
3. a esophageal inlet, penetrating schwannomas. Paragangliomas typ-
4. c injuries can be missed when intro- ically contain two types of cells:
5. e ducing the scope. type I or chief cells, and type I I , or
sustentacular cells. Neurofibromas
CHAPTER 109 CHAPTER 112 typically demonstrate histologically
Upper Airway Manifestations of Differential Diagnosis of Neck interlacing bundles of spindle cells.
Gastroesophageal Reflux Disease Masses Fibrosarcomas present histologi-
cally as fibroblastic proliferation of
1. b 1. e variable amounts of collagen and
2. e 2. c reticulin forming a "herringbone"
3. e 3. a pattern. Synovial sarcomas typi-
4 d 4 b cally demonstrate a predominant
5. c 5. c spindle cell component, with
cuboidal and columnar cells sur-
CHAPTER 110 CHAPTER 113 rounding glandular areas, and may
Deep Neck Infection have calcifications i n up to 30% of
Primary Neoplasms of the Neck cases.
1 d
2. a 1. c 4. c
3. b The etiology of paragangliomas is Fine-needle aspiration (FNA)
4. c multifactorial and includes familial biopsy is a technique both sensi-
5. e syndromes, such as MEN types IIA tive and specific i n the diagnosis of
and B; genomic imprinting; living squamous cell carcinoma metasta-
CHAPTER 111 at elevated altitudes; and condi- tic to cervical lymph nodes. The
Blunt and Penetrating Trauma tions causing chronic arterial need for a complete history and
to the Neck hypoxemia, such as cyantoic heart physical examination cannot be
disease. Previous radiation expo- understated, and FNA should be
1. a sure has not been described as an the next step i n the algorithm
A neurologic deficit signifies higher inciting agent i n the development when evaluating a neck mass i n an
probability that a vascular injury is of paragangliomas. adult.
present because of anatomic prox- 2 a
imity. As carotid paragangliomas enlarge, 5 d
progressive symptoms attributable The most common soft tissue
2. a to cranial nerve deficits of IX, X, sarcoma of the head and neck
Zone I I I is more difficult to explore X I , or X I I may appear and may i n children is rhabdomyosarcoma.
surgically than zones I I and I result i n dysphagia, odynophagia, Head and neck rhabdomyosarco-
because of the presence of the or hoarseness. Functional para- mas have the highest incidence
skull base and mandible. gangliomas only make up 1% to 3% i n the first decade, w i t h
of paragangliomas but may be her- another peak occurring i n
3 b alded by symptoms such as the second and t h i r d
A diagonal incision along the ante- headaches, palpitations, flushing, decade.
rior sternocleidomastoid muscle or perspiration because of neu-
gives the best exposure for carotid ropeptide secretion. Although CHAPTER 114
artery inspection. carotid paragangliomas may pres- Lymphomas Presenting
ent as a pulsatile neck mass, pul- i n the Head and Neck
4 d satile tinnitus is a symptom
Severe injuries i n zone I I I may not typically characteristic of jugulo- 1 b
be evident initially on clinical tympanic paragangliomas. 2 d
Answers Key 295

3. c not receiving antibiotics, and a continued suction drains, pres-


4 b study of 201 clean head and neck sure dressings, replacement of flu-
5 a cases i n which the rate of wound ids and electrolytes, and an
infection after neck dissection was enteral diet restricted to medium-
CHAPTER 115 13% vs 1% for all other procedures. chain triglycerides or TPN. If the
Radiation Therapy and Two quoted studies have shown output is high (>600-1000 mL/24
Management of the Cervical that shorter time courses of antibi- h r ) , reoperation is indicated.
Lymph Nodes otics were found to be as effective Doxycycline has been used as a
at controlling wound infections i n sclerosing agent, although
1. a both clean head and neck cases caution is advised, because i t is
2. c and clean-contaminated head and neurotoxic. Subcutaneous
3. a neck cases compared with longer somatostatin has also been
4. c courses of antibiotics. A cost analy- reported i n the treatment of both
5 b sis study showed that treatment of chylothorax and chylous fistula of
three patients with wound infec- the neck.
CHAPTER 116 tion outweighed prophylaxis of 100
Neck Dissection patients. Finally, i t is well known 5 b
that wound infection rate increases Eleventh nerve syndrome was first
1. e dramatically when a procedure described by Ewing and Martin.
2. c involving spillage of oral secretions This syndrome includes a dull
3. a is performed. ache, stiffness or soreness, droop-
4. c ing of the shoulder, aberrant scapu-
5 b 3. e lar rotation, limited forward
Radiation therapy has both early shoulder flexion, and limited active
CHAPTER 117 effects (DNA damage and cell shoulder abduction. Because this
Surgical Complications death) and late effects (microvas- syndrome can be absent after radi-
of the Neck cular damage leading to capillary cal neck dissection i n which the
dilation and obliteration and spinal accessory nerve is sacrificed,
1. e increased fibrous tissue) that but present after 11th nerve spar-
In the study by Yii and others, the impact wound healing, whereas ing neck dissections, i t seems to
only complication that was chemotherapy affects wound heal- have a more complex explanation.
increased i n a comparison between ing by its effect on WBCs. I n 69 One theory put forth is that adhe-
triradiate, modified MacFee and patients who underwent neck dis- sive capsulitis of the shoulder may
apron incisions was the incidence section after chemoradiotherapy, be a contributing factor after
of wound dehiscence i n previously complications included flap necro- nerve-sparing neck dissections.
radiated necks when the triradiate sis, need for tracheotomy, nerve
incision was used. The authors sug- injury, and hypocalcemia. Timing CHAPTER 118
gest using the apron incision i n of surgery after chemoradiotherapy Disorders of the Thyroid Gland
these patients, because they seems to be most advantageous i n
believe the exposure is better than the range of 5 to 17 weeks after 1 b
the modified MacFee. treatment. Finally, treatment time 2 a
seems to be an important factor i n 3 b
2. c the response of early-responding 4 d
Although no prospective, random- tissues, and fraction dose is impor- 5 b
ized trials have shown a benefit of tant i n the effect on late-respond-
perioperative wound infections, a ing tissues. CHAPTER 119
number of retrospective analyses Management of T h y r o i d
indicate a trend favoring the use of 4. g Neoplasms
a short course (24 hours) of antibi- As i n many complications, pre-
otics when performing a neck dis- vention of injury to the thoracic 1 b
section. These include a study of duct is the best form of treatment. The superior parathyroid glands
192 patients undergoing neck dis- Once a chyle fistula is recognized, are derived from the fourth
section i n whom three times more it can be treated conservatively branchial pouch, and the inferior
wound infections occurred i n those w i t h elevation of the head of bed, parathyroid glands originate from
296 ANSWERS

the third branchial pouch. The p r i - and may be deep or superficial to hypercalcemia will demonstrate
mary blood supply of the superior the inferior thyroid artery low 24-hour urinary calcium levels.
and inferior parathyroid glands is branches. The RLN enters the Vitamin D levels are usually nor-
the inferior thyroid artery. The laryngeal framework between the mal, and serum phosphate levels
inferior thyroid artery is a branch arch of the cricoid cartilage and are low i n patients with primary
of the thyrocervical trunk. the inferior cornu of the thyroid hyperparathyroidism.
Occasionally, the superior parathy- cartilage after penetrating deep to
roid glands will also receive blood the lowermost fibers of the inferior 4 b
supply from the superior thyroid constrictor muscle. The external Although both MRI and GT may be
artery. branch of the superior laryngeal used as correlative adjuncts i n local-
nerve innervates the cricothyroid izing hyperfunctional parathyroid
2. c muscle. glands in the reoperative setting,
Although advanced age increases they are not sufficient as an initial
the likelihood for malignancy when CHAPTER 120 localizing study. Ultrasonography
evaluating a patient with a head Surgical Management may not be effective in localizing
and neck lesion, age is a significant of Parathyroid Disorders enlarged glands i n the retroe-
prognostic factor for patients with sophageal, retrotracheal, retroster-
thyroid carcinoma. Every prognos- 1. e nal, and deep cervicothoracic inlet
tic classification, including the At present, osteitis fibrosa cystica regions. Technetium 99m sestamibi
AJCC TNM staging system, occurs i n 1% of patients and only is preferred over subtraction imag-
includes age at initial presentation 10% to 20% are initially seen with ing because of overall greater accu-
as an important variable i n deter- renal stones. Postmenopausal racy and ease of performance.
mining risk categorization. women with the disorder are at
greater risk for osteoporosis devel- 5. e
3. c oping but do not represent the Ectopic locations for inferior
Papillary carcinoma is the most majority of presenting manifesta- parathyroid glands include an
common form of thyroid cancer, tions. Some signs of muscle fatigue intrathymic location, the anterior
accounting for 60% to 70% of all and malaise may be found i n as superior mediastinum, and within
cases. Follicular carcinomas many as 40% of symptomatic the carotid sheath. Although
account for approximately 10% to patients. ectopic superior glands may
15% of all thyroid malignancies. occupy an intrathyroidal location,
Medullary carcinomas account for 2 d they more commonly will migrate
approximately 5% of all thyroid Adherence of glands to surrounding to a retroesophageal position.
carcinomas. cervical soft tissue is common with
parathyroid carcinoma but may be CHAPTER 121
4 b found i n adenoma with hemor- Paranasal Sinuses: Management
Follicular carcinoma extends from rhage, resulting i n periglandular of Thyroid Eye Disease (Graves'
the primary disease site mainly by fibrosis with adherence and thyroid Ophthalmology)
local extension. Unlike papillary parenchyma involvement. Broad
and medullary carcinomas, follicu- separated fibrotic bands may be 1 b
lar carcinomas are less likely to noted i n both carcinoma and atypi- 2. c
metastasize to the cervical lymph cal adenoma. Similarly, mitotic fig- 3 b
nodes. The presence of cervical ures may also be seen i n 4. a
lymph node disease should raise parathyroid adenoma and hyper- 5. c
suspicion for significant local dis- plasia, the absence of which does
ease and visceral invasion. not eliminate the presence of carci- CHAPTER 122
noma. Metastases are the only cer- Anatomy of the Skull Base,
5. c tain sign of malignancy. Temporal Bone, External Ear,
The RLN is found within a triangle and Middle Ear
defined by the trachea medially, 3. c
the carotid sheath laterally, and Hypercalcemia is the principal 1 d
the undersurface of the retracted defining manifestation of primary 2 b
inferior thyroid pole superiorly. hyperparathyroidism. I n contrast 3. a
The inferior thyroid artery has a to patients with primary HPT, 4. b
variable relationship to the RLN those with familial hypocalciuric 5 b
Answers Key 297

CHAPTER 123 t i o n " allows a relatively accurate 2. c


Neural Plasticity in Otology representation of the outside world A spontaneous leak of cere-
at the cortical level. I t is mainly or brospinal fluid may occur from
1. d only at this level where complex arachnoid granulations weakening
The main projection system within processing, memory storage and the dura of the middle or posterior
the auditory system is one i n retrieval, cross-modality compar- cranial fossa. If persistent clear
which the topographic arrange- isons, and so on can be carried out. drainage occurs through a ventila-
ment of cochlear afferent neurons tion tube, the fluid should be col-
is apparently maintained through- 5. c lected and tested for (3-2
out the system to cortex. I t is sim- Age-related plasticity definitely transferrin.
ply because (spectral) sound exists i n the auditory system, as i t
frequency is "place coded" along clearly does i n visual and other 3. a
the cochlear length that a neuron sensorimotor systems. I n experi- Eagle's syndrome involves ear pain
connected up to a certain cochlear mental studies of plasticity, i t is secondary to stretching or irrita-
position responds best to a certain important to distinguish those that tion of the glossopharyngeal nerve
frequency of sound. As noted i n reveal plastic change i n the adult from an elongated styloid process.
the text, the interchangeability of subject and those changes i n a The other choices (b, c, d, e) are
these terms requires caution when developing organism. I n the audi- all associated with the symptom of
the place coding of sound fre- tory system, plasticity of tonotopic aural fullness.
quency is disrupted (e.g., i n map reorganization looks similar at
cochlear pathology). the cortical level; however, i n sub- 4. e
cortical regions, the age-related Ramsay-Hunt syndrome represents
2 d plasticity effects become very a viral polyneuropathy primarily
Many mechanisms, both at the apparent. affecting the cochleovestibular and
presynaptic site and associated facial nerves. I t occurs through
w i t h the postsynaptic neuron, can CHAPTER 124 reactivation of latent varicella virus
produce an alteration i n the effi- Tinnitus and Hyperacusis within the cranial nerve ganglia. I t
cacy of information transfer. Some rarely involves additional cranial
of these mechanisms are outlined 1. c nerves (V, IX, X, X I , XII).
in Figure 123-7. 2. a
3 b 5. e
3. c 4 d Grave's disease is not associated
A number of studies exploring plas- 5 d with sudden SNHL. A n uncommon
ticity i n the auditory system have 6. c presentation of a CPA tumor is a
involved making a total or partial sudden hearing loss that may or
cochlear deafferentation. Many CHAPTER 125 may not recover with a course of
studies have taken advantage of the Management of Temporal Bone steroids. Membranous labyrinthine
ototoxic effects of aminoglycoside Trauma injury from barotrauma or head
antibiotics as an experimental tool trauma without temporal bone
to cause cochlear hair cell lesions. 1. c fracture may also cause sudden
A number of these aminoglyco- 2. c sensorineural hearing loss.
sides, including amikacin, are very 3 b
predictable i n causing damage i n 4 d CHAPTER 127
basal cochlear areas. 5 d Otologic Manifestations
of Systemic Disease
4. a CHAPTER 126
The visual, somatosensory, and Otologic Symptoms 1 d
auditory systems have a common and Syndromes Langerhans cell histiocytosis,
organizational feature. They each tuberculosis, and Wegener's granu-
have a regular, topographically T b lomatosis are all granulomatous
organized system of connections Bullous myringitis is a painful diseases that can affect the middle
that maintains patterns of neural infection of the tympanic mem- ear and mastoid, and, i n so doing,
activity generated at the sensory brane resulting i n a mixed hearing they may closely mimic the symp-
epithelium, up to central cortical loss that typically resolves with toms and signs of chronic otitis
areas. This "main line organiza- treatment. media.
298 ANSWERS

2 f restore the vital nutrients that tone audiogram. Furthermore, by


There is no consistent pathologic power their intracellular machin- indicating the pattern of OHC sur-
basis for the air-bone gap i n Paget's ery. Noise-induced ischemia within vival, they can be used to predict
disease. Specifically, the apparent the cochlea has been indexed by which frequencies need to be
conductive hearing loss is not use of laser-Doppler flowmetry amplified and which do not, thus
caused by a middle ear lesion such measures either through the optimizing the fitting of a digital
as ossicular fixation or obliteration round-window membrane or hearing aid to a particular patient
of the oval or round windows. through the lateral bony wall of the with NIHL. Their objectivity, sim-
Therefore, attempts at surgical cor- cochlea. Finally, ionic poisoning ple set-up procedures, and rapid
rection of the conductive hearing caused by the mixing of the dis- test times also make OAEs ideal for
loss are unlikely to be of benefit. parate cochlear fluids consisting of monitoring the progression of hear-
endolymph and perilymph has ing impairment i n situations i n
3. a been shown to occur through the which exposure to noise continues.
Progressive narrowing of the exter- use of i n vivo tracers to demon-
nal auditory canal with conductive strate microbreaks i n both the 3. c
hearing loss is the most common reticular lamina and apical mem- It is well established that both
manifestation of fibrous dysplasia branes of sensory and supporting noise damage and hearing loss can
affecting the temporal bone, occur- cells i n noise-damaged cochleas. be augmented by being exposed
ring i n about 80% of cases. Chronic bleeding has never been simultaneously (and, i n some case,
observed i n the acoustically over- before and after) to certain agents.
4. c stimulated cochlea probably These include the ototoxic drug
Positive Hennebert's sign, believed because of the unique pattern of cisplatin and the aminoglycosidic
to be due to fibrous adhesions blood supply to the inner ear, antibiotics. I n addition, a great deal
between the stapes footplate and which, i n the organ of Corti is of accumulating research has also
the membranous labyrinth, is seen reduced to a system of microcapil- shown such synergism between
as a result of inner ear involvement laries and venules. noise and particular industrial
by otosyphilis. chemicals such as the asphyxiant,
2 b carbon monoxide. Furthermore,
5 b Otoacoustic emissions (OAEs) can other stresses such as whole-body
This combination of histopatho- be used to accomplish all these or segmental vibration have also
logic findings is characteristic of efforts i n the patient with NIHL, been shown to enhance the
Wegener's granulomatosis. except to specify damage to the adverse effects of loud sounds.
inner hair cell system. There is However, although microwaves
CHAPTER 128 accumulating evidence, like that per se have been shown to have
Noise-Induced Hearing Loss presented i n Figure 128-5, which adverse effects on hearing, i t is
demonstrates that OAEs detect their thermal rather than their
1. e cochlear dysfunction where NIHL mechanical properties that seem to
The proposed anatomic substrates can potentially occur before i t has produce any related hearing loss.
of NIHL include all of these possi- been measured by other audiomet-
bilities except chronic bleeding. ric tests, including the clinical 4. a
Sound-induced mechanical damage audiogram. I n addition, there are a Many studies of chronic noise
includes such observations, at number of lines of evidence exposure of the sort experienced
either the high-power light micro- (reviewed i n 10) that OAEs are by people who live near airports
scope or scanning electron predominately generated by the that operate 24 hours a day have
microscopy level, as buckled sup- OHC system. Because OHCs repre- demonstrated noise-induced bio-
porting cells (e.g., pillar bodies that sent the organ of Corti component logic and/or psychological stress i n
support the tunnel of Gorti) or that is initially damaged i n NIHL, such individuals. These effects
bent or broken stereocilia. OAEs make ideal measures with include pathologic conditions as
Metabolic exhaustion caused by which to identify the onset stages emotional unrest, hypertension,
sound overexposure, which has of this pathology. Thus, i n NIHL, and peptic ulcers. Although there
been evaluated using biochemical, OHC abnormalities should be the is some evidence using sophisti-
histochemical, or immunohisto- primary pathology. Therefore, cated rotary-chair testing of
chemical methods, can occur dur- OAEs are capable of approximating vestibular pathways that support
ing constant overstimulation, when the pattern of noise-induced hear- the sensitive compensatory
cells have no opportunity to ing loss as measured by the pure- processes of balance function, out-
Answers Key 299

right vertigo is not a typical com- 3. e include vertigo, nausea, oscillopsia,


plaint of the patient with NIHL. 4 b and disequilibrium. Patients can
readily distinguish these symptoms
5 b CHAPTER 130 from their typical Meniere's dis-
The regulatory control of sound in ease-related symptoms. These
the workplace is promulgated by Vestibular and A u d i t o r y Toxicity symptoms become progressively
federal, state, and local authorities. worse until they peak 1 week after
These basic regulations are essen- 1. c onset. During this peak, patients
tially patterned after a ruling made Arsenic. Each of the other drugs usually require 2 to 3 days of bed
more than 20 years ago by OSHA preferentially attack the hair cells rest. Gradual resolution is achieved
that was subsequently enacted into of the basal turn of the cochlea, in 2 to 4 weeks i n most patients.
law by Congress. Part of the ruling causing high-frequency sen- Because acute vestibular deaf-
states that if employees are exposed sorineural hearing loss. ferentation syndrome is an
to >85 dB SPL i n the workplace, 2 d expected outcome of therapy, some
their employer must implement a Cisplatin. This drug is most likely authors recommend that a vestibu-
hearing-conservation program that to cause permanent sensorineural lar rehabilitation team be available
includes baseline audiometric hearing loss among the choices to work with severely affected
assessment, annual audiometric provided. Erythromycin, patients.
monitoring, and, if a hearing loss is furosemide, and torsemide can
documented, the employee must be cause primarily reversible ototoxic- 2. a
notified and educated about the use ity. Vancomycin usually not oto- There is, indeed, no evidence i n rig-
of personal hearing protectors. As toxic when given alone but can orous clinical studies (i.e., random-
part of a hearing-conservation pro- cause ototoxicity when given i n ized and controlled) documenting a
gram, all workers who toil i n areas combination w i t h aminoglycosides. clinical benefit of steroids i n treat-
of high noise levels (i.e., >85 dBA) ing Meniere's disease. There has
must wear ear protectors and par- 3. c been a stinging on injection docu-
ticipate i n a noise-education pro- Blood levels of aminoglycosides are mented by some patients with
gram on the hazardous effects of not predictive of vestibular dam- methylprednisolone, and some
noise and the correct fitting of per- age. The other options are all true. practitioners recommend coadmin-
sonal hearing protectors. I n identi- istration with 0.1 m L of 1% lido-
fying unsafe sound levels in 4. a caine with 0.9 m L standard IV
industry and i n calculating the Mutations involving connexin 26 methylprednisolone solution (40
durations of safe exposure times, can cause hearing loss but have not m g / m L ) . There have been no
141

the concepts of the time-weighted been associated with increased reports of IT steroids upsetting the
average and the principles of equal sensitivity to ototoxicity from inner ear flora. Because steroids
energy and equivalent continuous aminoglycosides. are fairly benign, treating patients
sound level are paramount. with bilateral Meniere's disease
However, the standard measure of 5 d with steroids is not the relative
sound level is in dBA units rather Carboplatin seems to selectively contraindication that i t is with IT
than in sound pressure level (dB damage inner hair cells. The other gentamicin. Although i t is true that
SPL) units that represent a linear drugs are more likely to damage the mechanism of action of
scale of measurement. The dBA outer hair cells. steroids i n the inner ear are not
scale is used to gauge the magni- yet fully described, this is not a
tude of occupational noise, because CHAPTER 131 problem that would preclude
it best estimates the configuration Pharmacologic Treatment steroid use i n clinical settings.
of the human threshold for hearing of the Cochlea and Labyrinth
and thus reduces the influence of 3. e
sounds at very low and very high 1. c There is no way to completely pre-
frequencies. Acute vestibular deafferentation vent anacusis when gentamicin
syndrome, also known as acute therapy is used. Remember that the
CHAPTER 129 chemical labyrinthine upset, is the lowest dose recorded to
Autoimmune Inner Ear Disease consequence of unilaterally insult- cause total hearing loss was only
ing the vestibular apparatus. This 0.24 m g . Even when using titra-
203

1. c phenomenon usually occurs 3 to 5 tion protocols, there is no evidence


2. a days after the injection. Symptoms showing that immediately stopping
300 ANSWERS

therapy preserves or improves cholesteatoma: (1) invagination of have been made to explain this
hearing outcomes. Although steroid the tympanic membrane (retrac- observation: the hereditary theory,
injections as salvage therapy may tion pocket cholesteatoma); which states that children with
prove to be a useful therapy i n the (2) basal cell hyperplasia; (3) hypoaeration of the mastoid are
future, there are no published stud- epithelial ingrowth through a perfo- prone to OME, and the environ-
ies yet documenting the efficacy of ration (the migration theory); and mental theory, which states that
this intervention. The trend toward (4) squamous metaplasia of middle chronic OME results i n hypop-
the use of microdoses of gentamicin ear epithelium. Transdifferentiation neumatization of the mastoid.
or delivering gentamicin through means converting one sort of cell
sustained-release devices seems to into another and has not been CHAPTER 134
be improving hearing outcomes. shown for acquired cholesteatoma. Complications of Temporal Bone
Infections
4. e 2. c
All of these compounds are The infectious and noninfectious 1 b
approved for use i n humans and complications of otitis media may 2 b
could potentially be used on an off- result i n significant morbidity and 3 d
label basis. Of these compounds, the complications, including acute and 4 d
one that has been shown to improve chronic mastoiditis, petrositis, and 5 d
outcomes after noise-induced intracranial infection. The nonin-
trauma is riluzole. However, fectious sequelae, including CHAPTER 135
intratympanic use of this compound chronic perforation of the t y m - Infections of the Labyrinth
has never been attempted i n panic membrane, ossicular erosion,
humans and systemic application labyrinthine erosion, and t y m - 1 d
can cause significant side effects. panosclerosis, are major causes of Since the introduction of the
hearing loss. rubella vaccine, most congenital
5. e hearing loss i n developed countries
Although neurotrophins represent 3 d occurs from cytomegalovirus. I n
an exciting class of potential thera- Symptoms of petrositis usually are most cases, the maternal
peutic compounds, their wide subtle. Typically, a patient who has cytomegalovirus infection occurs i n
range of actions are only beginning had previous mastoid surgery will nonimmune women early i n the
to be understood. I n fact, under complain of persistent infection pregnancy. The maternal infection
certain pathologic conditions, neu- and deep facial pain. The diagnosis is usually asymptomatic. However,
rotrophins can exacerbate, rather of petrous apicitis is suspected on occasional cases of congenital
than alleviate, injury. For a good clinical grounds, the most appro- cytomegalovirus infection occur i n
review of this topic, see the review priate diagnostic procedure is CT. immune mothers who apparently
article by Behrens and others High-resolution CT scanning usu- developed a recurrent asympto-
(Neurotrophin-mediated potentia- ally shows details of the petrous matic viremia from a latent infec-
tion of neuronal injury, Microsc apex and provides important detail tion. Treponema pallidum
Res Tech 45[4-5]:276, 1999). about potential surgical routes. (congenital syphilis) and rubella
virus can cause occasional cases of
CHAPTER 132 4. e congenital hearing loss i n the
Infections of the External Ear Tympanosclerosis is a conse- United States. Rubeola and mumps
quence of resolved otitis media or viruses cause acquired hearing
1. c trauma and was often seen after loss.
2 d recurrent bouts of acute otitis
3 d media. There is no relation to oto- 2. c
4. e sclerosis, but i t may be present i n More than 99% of congenital
5. c cholesteatoma but is not associ- cytomegalovirus infections are
ated with i t . asymptomatic, w i t h virus detected
CHAPTER 133 i n the infant's urine at b i r t h . The
Chronic Otitis Media, Mastoiditis, 5. a virus disappears from the urine
and Petrositis It has been observed that patients over several months. However,
with a history of chronic OME have occasional asymptomatically
1 b more sclerotic mastoids with infected infants subsequently
There are four basic theories of the decreased pneumatization than have bilateral or unilateral hear-
pathogenesis of acquired aural healthy subjects. Two suggestions ing loss develop during the first
Answers Key 301

decade of life. The pathogenesis of CHAPTER 138 4 b


this delayed hearing loss is Anatomy of Vestibular End Organs 5. e
unclear. and Neural Pathways
CHAPTER 140
3. b 1. e Evaluation of the Patient
Most viruses that cause congenital All of the above. Answers a-d are w i t h Dizziness
hearing loss produce cochlear dam- all correct.
age involving endolymphatic struc- 1. e
tures. The specific endolymphatic 2 b Some forms of nystagmus are
structures damaged depend on the Afferent fibers arrive at the sensory "unstable" and may oscillate, such
virus. For example, epithelium before hair cells differ- as periodic alternating nystagmus,
cytomegalovirus tends to infect entiate. The remaining answers are but this is not the situation here.
cells i n the stria vascularis and all correct. The neural integrator is ineffective
Reissner's membrane. after a vestibular lesion but does
3 d not cause a reversal of nystagmus.
4. a Endolymph is resorbed i n the Tilting of the head can "dump" the
Congenital rubella remains a seri- endolymphatic sac. I t is produced velocity storage mechanism and
ous cause of congenital hearing by so-called vestibular "dark" cells, cause nystagmus to cease early,
loss i n developing countries that do which are mitochondria-rich, and but the reason nystagmus automat-
not administer the rubella vaccine. it resembles extracellular fluid by ically changes its direction after
Most hearing loss i n congenitally having low Na and high K con-
+ +
head-shaking is because of adapta-
infected infants is bilateral and tent. Perilymph is low i n amino tion of the vestibular system.
involves all frequencies. I n more acids, especially glycine, compared Subject to high levels of stimula-
than half the infants, the hearing with blood, and i t leaves the inner tion, the system gradually adapts.
loss is profound and permanent. ear by drainage through venules This effects of this adaptation last
and through the middle ear longer than those from the initial
5 d mucosa. stimulus, causing an undershoot.
Hearing loss from mumps is
acquired during childhood mumps 4. c 2. c
and is usually unilateral. Hearing The optokinetic reflex depends on The effect of velocity storage is
loss from mumps continues to rep- the integrity of the visual system, shortened after deafferentation. The
resent a common cause of acquired because i t relies on visual input remainder of the answers are true.
deafness i n developing countries from moving targets as a stimulus
that do not administer the mumps to move the eyes. The other 3. a, e
vaccine. The hearing loss usually reflexes do not rely on visual input; There is no proven role for postur-
develops toward the end of an in fact, they can function i n com- ography in evaluating treatment
uncomplicated parotitis and is plete darkness. with gentamicin. I t is a very ineffi-
often profound. cient modality i n screening patients
5. a in general. I t is not an effective
CHAPTER 136 The vestibular nuclei do not proj- diagnostic tool for perilymphatic
Tympanoplasty and Ossiculoplasty ect to the labyrinth itself. There fistula, which is better diagnosed by
are efferent vestibular neurons, pressure changes i n the inner ear
1. c which contain acetylcholine and and middle ear exploration.
2 b other transmitters and neuromod-
3. a ulators, that project to the 4 b
4. c labyrinth. The vestibular nuclei The most common cause of benign
5. c project to all the remaining struc- paroxysmal vertigo is debris i n the
tures. posterior canal. Canalithiasis and
CHAPTER 137 cupulolithiasis of the horizontal
Mastoidectomy CHAPTER 139 canal are known and can cause the
Principles of Applied Vestibular symptoms of choice e, but this is
1 b Physiology much less common. Downbeat ver-
2 b tical nystagmus with a torsional
3 d 1. a component is typical of superior
4. e 2 b semicircular canal dehiscence syn-
5 d 3. c drome.
302 ANSWERS

5 d CHAPTER 142 lar infarction. Central ocular motor


Studies i n environments where Meniere's Disease and Other signs such as direction-changing or
convection cannot occur because Peripheral Vestibular Disorders downbeat nystagmus and profound
of lack of gravity (such as i n space) gait imbalance increase this likeli-
have shown that the caloric 1 b hood. When the brainstem is
response still exists, although 2. c affected, other cranial nerve symp-
somewhat attenuated. A direct 3 d toms and signs are generally pres-
effect of heating on the vestibular 4. a ent. Patients should undergo
apparatus, causing stimulation, is 5 d emergent head imaging with non-
therefore understood to exist. contrast CT or MRI, because an
CHAPTER 143 expanding cerebellar hematoma or
CHAPTER 141 Central Vestibular Disorders swelling from a cerebellar infarc-
Imbalance and Falls i n the Elderly tion can cause brainstem compres-
1 d sion and death without
1. c A patient with acute vertigo from a neurosurgical intervention.
More than one third of all elderly peripheral cause will usually have a
falls i n the home occur i n the bed- positive head thrust sign (i.e., a 3 b
room, usually getting into or out of rapid 15-degree passive rotation of Vestibular migraine may be the
bed. the head i n one direction while the most common cause of recurrent
patient attempts to fixate on the spontaneous (nonpositional) ver-
2. a examiners nose elicits a corrective tigo among young and middle-aged
A result of aging is loss of lower saccade that brings the gaze back people and is more common is
body strategies, particular ankle to the fixation target). This occurs women. Often there is a remote or
strategies, to compensate for sway. because the slow phase of the high- current history of headaches that
Instead, stepping strategies are frequency VOR is inadequate to may occur during or independently
used and because of the amount of maintain fixation during head rota- from attacks of vertigo. Other
time to relocate the stepping foot, tion and implies a unilateral loss of migrainous and even aural symp-
falls frequently ensue after mild horizontal semicircular canal func- toms may accompany attacks.
balance perturbations. tion. A mixed horizontal-torsional A history of motion sensitivity is
nystagmus is characteristic of an common among migraineurs, and a
3 b acute unilateral peripheral family history of migraines (possi-
Elderly patients most commonly labyrinthine lesion affecting all bly never diagnosed) is typical.
are seen with complaints of dise- three semicircular canals, i n which Distinguishing from Meniere's dis-
quilibrium or loss of balance. True pure vertical or torsional nystag- ease can sometimes be difficult,
vertigo is primarily a result of a mus would be more likely central. but examination, audiogram, and
unilateral vestibular loss, which is Peripheral nystagmus characteristi- vestibular testing are generally nor-
not usually an aging problem. cally increases with removal of mal, and diagnosis is based largely
visual fixation through Frenzel on the appropriate history.
4. e lines or occlusive ophthalmoscopy.
Gustatory sensations play no role Peripheral nystagmus is also unilat- 4. a
in balance, whereas the other four eral and increases with gaze in the Superficial siderosis is a rare con-
listed do. direction of the quick phases and dition characterized by progressive
decreases with gaze i n the direc- sensorineural deafness and ataxia
5 d tion of the slow phases and is due to hemosiderin deposi-
Although an ABR test gives impor- (Alexander's law). tion along the leptomeninges, cra-
tant information on auditory func- nial nerves, subpial tissues, and
tion i n the brainstem, i t is not 2. d spinal cord. I t is caused by recur-
useful for assessing balance and/or Cerebellar infarction or hemor- rent subarachnoid bleeding, usually
gait. rhage should be suspected i n any from an occult vascular malforma-
patient with acute vertigo when tion, aneurysm, tumor, or previous
6. c vascular risk factors (including age) intracranial surgery. The eighth
The pursuit (tracking) test is the are present. In one study, as many nerve, cerebellum, and olfactory
ENG test most susceptible to as one-fourth of older patients seen bulb are particularly susceptible.
abnormalities caused by age- in the emergency department with T2-weighted MRI scans reveal a
related processes. acute isolated vertigo had cerebel- margin of hypointensity surrounding
Answers Key 303

the brainstem and cerebellum been fully transected. As a result, 4. a


reflecting the paramagnetic hemo- afferent information from the dis- The symptom complex described
siderin deposition. eased labyrinth is still being trans- in the first answer is the classic
mitted to the vestibular nucleus on presentation of the uncompensated
5. e the involved side. Destruction of but stable peripheral vestibular
Because of the often disabling the vestibular end organ by any lesion. Surgical treatment i n this
symptoms, patients with vestibular technique would be expected to setting is almost never beneficial,
disorders frequently become anx- control ongoing vertigo, recogniz- and the patient should be referred
ious or depressed. Dizziness has a ing that either of the labyrinthec- for a customized program of
more deleterious effect when i t is tomy procedures would result i n vestibular rehabilitation. Answers b
associated with a psychiatric disor- complete hearing loss on the and c suggest two common variants
der such as depression, and treat- involved side. If the hearing of endolymphatic hydrops,
ment of the vestibular disorder remained excellent, intratympanic Meniere's disease and delayed ipsi-
may lead to a suboptimal response gentamicin injections would be the lateral hydrops, which are appropri-
unless the psychiatric disorder is most conservative approach. ately treated with surgery, assuming
also addressed. Thus, vestibular Alternatively, performing a more medical therapy for hydrops was
and psychiatric disorders should be selective vestibular neurectomy not successful. Answer d suggests a
aggressively managed together. distally i n the internal canal by the perilymphatic fistula with perhaps
Depending on the situation, this middle fossa approach could be subluxation or fracture of the stapes
can be accomplished by explana- considered. Vestibular rehabilita- footplate. Although further vestibu-
tion and reassurance to the tion, although desirable for treating lar rehabilitation (customized
patient, medication management disequilibrium and/or motion-pro- habituation exercises) may be
by the otolaryngologist, or referral voked vertigo after any ablative helpful i n refractory BPPV, this is a
to a psychiatrist. vestibular operation, would not be fluctuating peripheral disorder and
expected to provide any benefit would be responsive to surgical
CHAPTER 144 in the setting described i n this therapy.
Surgery for Vestibular Disorders question.
5. c
1 b 3. e In any endolymphatic sac opera-
Singular neurectomy selectively The presence of a sensorineural tion, the surgeon should remem-
sections the afferent innervation of hearing loss is a highly reliable ber that although the operation is
the posterior semicircular canal indicator of peripheral pathology, generally quite safe, the efficacy is
and, thus, is effective and appropri- provided that the opposite ear has questionable, and any beneficial
ate for any disorder afflicting that completely stable hearing and no effect may be nonspecific. Thus,
canal. Nevertheless, i t is true that auditory symptoms. The patient the patient should always be
this procedure has been largely with profound deafness and counseled that the outcome is
supplanted by the technically sim- episodic vertigo is the ideal candi- uncertain and that more definitive
pler canal occlusion procedure. date for labyrinthectomy. A unilat- treatment may be required. Safety
Both operations are associated with eral weakness of vestibular is paramount, and i t is appropri-
the risk of hearing loss, although function may be confirmatory but ate to decompress the sigmoid
this complication is quite rare with is not absolute in localizing the sinus and/or skeletonize the facial
canal occlusion procedures. unhealthy ear i n the absence of nerve to safely gain access to the
Particle repositioning and other hearing loss. Likewise, the pres- region of the sac. The literature
vestibular rehabilitation techniques ence of normal caloric responses suggests that no matter how the
are very effective, and surgery of should not dissuade the surgeon if sac is manipulated, the outcomes
any kind is rarely required i n this hearing loss is present. Tinnitus are similar if not identical.
condition. and fullness are nonspecific symp- Therefore, any procedure that
toms and, again, may be confirma- violates the deep wall of the sac
2 d tory but are not sufficient to (posterior fossa dura) w i l l place
If the dizziness is truly character- securely base a surgical decision the patient needlessly at risk for a
ized as episodic spells of vertigo on. Rotary chair asymmetry sug- CSF leak or meningitis. Likewise,
after any posterior fossa vestibular gests that an uncompensated if the sac is extremely difficult to
neurectomy procedure, one must peripheral lesion is present but is access, the surgeon should
assume that the vestibular division of little help in lateralizing the remember that any salutary effect
of the eighth cranial nerve has not lesion. of the sac operation may well be a
304 ANSWERS

nonspecific result of the general generic form, if designed appropri- macologic paralysis sufficient to
anesthetic or drilling i n the tem- ately, will stimulate improvement allow the anesthesiologist to con-
poral bone. Thus, one could never but not to the degree as the cus- trol the patient's ventilation will
be criticized for backing out if tomized format. still permit EMG facial nerve
there is believed to be substantial monitoring. A l l the other state-
danger to vulnerable critical struc- 5. e ments are true.
tures, particularly the dura, the Although appropriate given symp-
posterior semicircular canal, the tom complaints, patients with head 5. c
sigmoid sinus, or the facial nerve. injury as the cause of their vestibu- Electrogustometry has been shown
lar injury (peripheral and central) to be abnormal i n virtually all
CHAPTER 145 do not as a group achieve the same cases of Bell's palsy (even with
Vestibular and Balance degree of success with a VBRT pro- incomplete paralysis), which
Rehabilitation Therapy: Program gram as other etiologies. makes i t nearly useless i n the very
Essentials early stages of this disorder. If the
CHAPTER 146 stapedius reflex is present i n a case
1. a of complete paralysis of the facial
In VBRT, the main overall goal is to Tests of Facial Nerve Function muscles, one should doubt the
promote the naturally occurring diagnosis of Bell's palsy and should
central compensation process. The 1. c consider imaging studies to rule
other responses are subgoals or Glass I injury is also called "con- out a parotid or temporal bone
techniques by which that can be duction block" or "neuropraxia." lesion. Tests of salivary and
accomplished. Glasses I I and I I I are called lacrimal function have been sug-
axonotmesis and neurotmesis, gested as prognostic tests but have
2. c respectively. "Axonotomy" and failed to demonstrate added value
The static phase of the central "neurotomy" are unrelated to the after clinical data and electrical
compensation process, tonic rebal- Sunderland classification. tests are available.
ancing, occurs at the level of the 2 b
vestibular nuclei and serves to sig- No paralyzed nerve can be success- CHAPTER 147
nificantly reduce symptoms of ver- fully stimulated proximal to the Clinical Disorders of the Facial
tigo after a stable peripheral lesion. Distal stimulation, i n a class Nerve
system insult. This occurs stimu- II-V lesion, will produce muscle
lated by the significant asymmetry contraction only until 3 to 4 days 1. c
in neural activity recognized by the after the onset of the injury. Herpes simplex virus (HSV). HSV
central nervous system and does DNA has been detected i n per-
not require any other external 3. a ineural fluid of patients with Bell's
stimulus. All tests baced on distal electrical palsy, whereas VZV DNA has not
stimulation (including NET, MST, been recovered from any.
3 d and ENOG) can yield useful prog- Conversely, VSV DNA was recov-
Spontaneously occurring symp- nostic data i n cases of Bell's palsy ered from all the patients with
toms of dizziness are a strong indi- but only when paralysis is total, Ramsay-Hunt syndrome, whereas
cation of an unstable peripheral or less than a month (6 weeks at the none had HSV-1 DNA.
central lesion. A n unstable lesion is most) has elapsed, and until
a major indicator as to why central excitability is lost or recovery 2. a
system compensation has not gone begins. None of these tests has Desynchronization can cause an
to completion. Typically, patients been shown to be superior to the artifactual depression of the CAP i n
with significant spontaneous events others. When excitability has been the presence of voluntary motor
are not able to use VBRT as the totally lost, incomplete recovery is responses on EMG. The desynchro-
primary form of management. certain, and fibrillation potentials nization causes a "spreading out"
seen on needle EMG are also har- of the CAP response, so that i t is
4 b bingers of incomplete recovery. not clearly seen on EnoG. This is
Double-blinded control research extremely important if a patient is
has shown superior results i n indi- 4 d being considered for surgical
vidually customized VBRT vs a Somewhat surprisingly, i t has decompression that both tests are
generic form. That said, the been well documented that phar- abnormal.
Answers Key 305

3. e CHAPTER 151 latency significantly. Because elec-


Birth trauma. Intrauterine injury is Electrophysiologic Assessment trical stimulation bypasses the
suspected, because the incidence is of Hearing traveling wave, the peaks of the
equal between forceps, vaginal, and EABR do not show a significant
cesarean deliveries. 1. e change i n latency with decreasing
Both sensorineural hearing loss stimulus current levels.
4 d >25 to 30 dB HL and middle ear
Preeclampsia. Maternal facial paral- pathology will result i n absent tran- CHAPTER 152
ysis is not associated with any fetal sient otoacoustic emissions. Diagnostic and Rehabilitative
abnormalities. I t is most common Auditory neuropathy is a condition Audiology
i n the third trimester of pregnancy characterized by an abnormal ABR
and is increased sixfold i n in the face of normal otoacoustic 1. a
preeclampsia. emissions. Similarly, subjects with 2 b
normal hearing can be expected to 3. c
5 f have measurable transient otoa- 4. e
Orofacial edema. Edema of the coustic emissions. 5. a
lips, buccal area, and sometimes
the periorbital tissues of the face is 2. c CHAPTER 153
the defining symptom i n Historically, an SP:AP ratio greater Auditory Neuropathy
Melkersson-Rosenthal syndrome. than approximately 0.4 has been
Facial paralysis and fissured considered to be diagnostically sig- 1. c
tongue occur i n half of patients nificant for Meniere's disease. Auditory neuropathy is character-
and the complete triad i n only one Unfortunately, the sensitivity of ized by the following: (1) The
fourth. this test is relatively low. Although patient must complain of a hearing
the figures vary to some extent, loss i n at least some settings. (2)
CHAPTER 148 research has shown that only 60% Patients have evidence of normal
Intratemporal Facial Nerve to 70% of individuals with con- outer hair cell function as demon-
Surgery firmed Meniere's disease actually strated by normal otoacoustic
have an enlarged SP:AP ratio. emissions and/or normal cochlear
1. d microphonics. (3) The patient
2. c 3 d demonstrates auditory nerve dys-
3 d Although the individual peaks of function as displayed by abnormal
4. a the ABR represent activity from a auditory brainstem response. (4)
5. c range of different generator sites, Patients with auditory neuropathy
the neural generator site that con- demonstrate poor speech recogni-
CHAPTER 149 tributes most significantly to wave tion scores that seem to be out of
Cochlear Anatomy and Central V is the lateral lemniscal tracks. proportion to the degree of hearing
Auditory Pathways loss depicted by the pure tone
4. c thresholds. (5) Patients with audi-
1 b Click-evoked ABR thresholds cor- tory neuropathy typically have
2 d relate most strongly with the aver- absent middle ear stapedial
3. a age of the 2000 Hz and 4000 Hz air reflexes. Normal radiologic imaging
4. e conduction thresholds. Click- studies of the brain and brainstem
5 b evoked ABR thresholds are not are also important i n making the
accurate indicators of low fre- diagnosis of auditory neuropathy.
CHAPTER 150 quency loss. Auditory neuropathy is currently a
Molecular Basis of A u d i t o r y diagnosis of exclusion. There is no
Pathology 5. e enhancement of the auditory nerve
The peaks of the EABR are typi- on MRI i n auditory neuropathy.
1. c cally larger i n amplitude and
2. e shorter i n level than the peaks of 2 d
3. c the ABR. I n addition, as stimula- Neonatal hypoxia and hyperbiliru-
4 b tion level is decrease, wave V of binemia are risk factors associated
5 b the ABR is known to increase i n with the development of auditory
306 ANSWERS

neuropathy. This is a true state- auditory neuropathy. I t is not agement of auditory neuropathy.
ment, (a) Current newborn hearing known precisely how these insults Some authors will argue that hear-
screening protocols frequently use contribute to the pathogenesis of ing aids are contraindicated
otoacoustic emissions as the first auditory neuropathy, (e) Auditory because of the presence of intact
step i n assessing newborns for neuropathy is inherited both in an outer hair cells and the risk of
hearing loss. Under this method, autosomal-recessive pattern and an noise-induced damage to these cells
only those children who fail otoa- autosomal-dominant pattern. I n from amplification systems. Because
coustic emissions initially are pur- the autosomal-dominant inheri- of this, when conducting a hearing
sued further with ABR testing. tance pattern, the patients are aid trial, i t is recommended that
Because children with auditory more likely to have a slowly pro- hearing aids initially be fitted con-
neuropathy will display normal gressive hearing loss and an associ- servatively, with a low maximum
otoacoustic emissions, this screen- ated peripheral neuropathy. The power output i n an effort to pre-
ing method will miss the diagnosis autosomal-recessive form generally serve functioning outer hair cells.
of auditory neuropathy. There are is seen i n infancy, with profound Furthermore, otoacoustic emissions
some institutions across the coun- hearing loss and no associated should be frequently monitored
try that are using auditory brain- peripheral neuropathy. The during the amplification trial to
stem response as a part of the Otoferlin gene, which is localized assess any damage to the outer hair
initial newborn screening protocol on chromosome 2, has been identi- cells, (d) Even though the auditory
for this reason, (b) Most children fied to be responsible for the non- nerve may be damaged i n patients
with auditory neuropathy do not syndromic recessive form of with auditory neuropathy, there is
have an associated peripheral neu- auditory neuropathy. At present, strong evidence to support that
ropathy. The peripheral neuropa- there is no genetic test available to cochlear implantation does provide
thy is typically demonstrated i n identify the presence of auditory reliable consistent nerve conduc-
adult patients with auditory neu- neuropathy. tion, despite the presence of a dis-
ropathy, (c) To date, there are eased or demyelinated nerve. This
many studies and case reports 3 b results in a restoration of neural
demonstrating successful auditory The statement that hearing aids synchrony, as well as the promotion
rehabilitation through cochlear generally offer long-term successful of neural survival, (e) Of the cases
implantation i n children w i t h audi- auditory rehabilitation for patients reported i n the literature, cochlear
tory neuropathy. Although long- w i t h auditory neuropathy is false. implantation for patients with audi-
term studies are not available to In general, i t has been a consistent tory neuropathy is associated with
demonstrate durable results, initial finding that amplification has not the same low complication rates as
data seem promising that cochlear provided successful auditory reha- cochlear implantation performed for
implantation will be a beneficial bilitation for most cases of auditory other causes of hearing loss.
therapeutic intervention for the neuropathy. Typically, patients
auditory rehabilitation of children with auditory neuropathy will 4. e
with auditory neuropathy. The report frustration with amplifica- Because of various anatomic stud-
decision to perform cochlear tion, complaining that the sound is ies, especially Spoedlin (1996). His
implantation i n children with audi- louder, and they can hear you but very detailed drawings of surface
tory neuropathy still represents cannot understand you. Hearing preparations of human inner ears
clinical dilemma. These children aids do sometimes improve the showed that 95% of the afferent
frequently demonstrate a large pure tone threshold level; however, fibers of the auditory nerve ener-
amount of residual hearing on pure the speech recognition scores are vate the inner hair cells. Each hair
tone thresholds. However, when often not improved, (a) If the deci- cell receives multiple fibers,
diligently observed for progress i n sion is made to conduct a hearing whereas a single fiber may ener-
speech recognition and language aid trial i n cases of auditory neu- vate several different hair cells.
acquisition, these children often ropathy, the audiologist should t r y This finding has resulted i n many
demonstrate a failure to make to maximize benefit from the additional anatomic and physio-
progress even with an adequate amplification by use of directional logic studies of the inner ear.
trial of amplification. It is at this microphones or person FM systems
time that a cochlear implant in an attempt to decrease back- 5. c
should be seriously considered for ground noise and improve the signal Patients with auditory neuropathy
these children, (d) Neonatal to noise ratio, (c) There is some typically exhibit phase-reversing
hypoxia and hyperbilirubinemia controversy regarding the appropri- cochlear microphonics and abnor-
are risk factors associated w i t h ate role of amplification i n the man- mal ABRs. Several examples are
Answers Key 307

presented i n the chapter. See refer- of the stapes carries more signifi- 4. c
ence 4 i n the book chapter (Berlin cant risk, requiring input from the Improvement i n hearing after
and others: Reversing click polarity child into the determination for steroid therapy is frequently seen
may uncover auditory neuropathy surgical intervention. with acoustic neuroma.
in infants, Ear Hear 19:37-47,
1998) and Figures 153-3 and 153¬ 5. a 5. c
4, which show phase-reversing See discussion i n chapter. At least i n the early stages of dis-
cochlear microphonics and no ease, hearing loss i n benign
neural potentials. C H A P T E R 155 intracranial hypertension,
Sensorineural Hearing Loss: endolymphatic hydrops, basilar
CHAPTER 154 Evaluation and Management migraine, and syphilis tends to be
Evaluation and Surgical i n Adults primarily low frequency and fluctu-
Management of Conductive ating. Presbycusis is predominately
Hearing Loss 1 d a high-frequency hearing loss.
Well-defined risk factors for amino-
1 d glycoside-induced hearing loss CHAPTER 156
Obliteration of the round and oval have been established and include Otosclerosis
windows does not allow a sound (1) presence of renal disease; (2)
wave to move into and through the longer duration of therapy; (3) 1. c
cochlear fluids. This results i n a 60- increased serum levels (either peak Otosclerosis is a unique process of
dB loss. The hearing loss from mid- or trough levels); (4) advanced age; changes i n the bone of the otic
dle ear effusion and perforations and (5) concomitant administra- capsule. The lesions of the bone
depends on the thickness of the tion of other ototoxic drugs, partic- start as spongification, which pro-
fluid and the size of the TM defect. ularly the loop diuretics. gresses to sclerosis. The lesions
Ossicular disruption behind an typically involve the otic capsule
intact TM results in a 55-dB loss. 2 d adjacent to the oval window and
There is considerable variability i n may spread through the cochlea.
2. e hearing loss among subjects with
The specific type and materials identical exposure. Age, gender, 2. e
making up the prosthesis has little race, and coexisting vascular dis- Otosclerosis usually causes a
to do with hearing results i n m i d - ease have been carefully studied, purely conductive hearing loss but
dle ear reconstruction. Staging and when adequately controlled for may cause a mixed conductive
cholesteatoma surgery can assist in other factors, they have not been sensori neural hearing loss. Rarely,
hearing recovery but to a varying shown to correlate with suscepti- the loss is purely sensori neural
degree. Cartilage interposition pre- bility to NIHL. with no involvement of the stapes
vents extrusion. Perpendicular footplate. A significant number of
placement i n respect to the TM 3. c patients undergoing cochlear
with mild tension on the head of Although the use of antiviral drugs implants have a profound hearing
the prosthesis ensures stability of would seem logical, no study to loss due to advanced otosclerosis.
the synthetic ossicle. date has demonstrated their effec-
tiveness i n sudden sensorineural 3 b
3. c hearing loss. Given their low side In large series of patients with oto-
Incudostapedial joint erosion is the effect profile and theoretical basis, sclerosis, about 70% are female.
most common cause of ossicular many use antivirals i n SSNHL The onset is usually i n the early
erosion associated with chronic despite the absence of proven effi- 20s but may be i n the late 30s. I t is
otitis media. Malleus head fixation cacy. There has never been a trial not usual to find a history that the
and incus head erosion are rare. showing benefit from anticoagula- female patient first noticed her
Calcification and superstructure tion, and this is not considered hearing loss at the time of her first
erosion are less common findings reasonable therapy by most practi- pregnancy. The hearing loss is pro-
as well. tioners. Several large studies have gressive and not associated with
shown benefit from steroid treat- vertigo.
4. c ment i n selected subgroups of
Reconstruction of the lateral ossic- patients with SSNHL. Only isolated 4. c
ular chain is not problematic i n reports have demonstrated benefit The Weber and Rinne are an
patients of any age. Manipulation from carbogen or Hypaque. important component i n the clinical
308 ANSWERS

evaluation of patients with otoscle- conversions maximizes transduc- otosclerosis i n combination with
rosis. The 512 Hz tuning fork is tion efficiency and minimizes 2 through 4 above. (6) Patients
used to establish the conductive distortion. With a conventional with profound single-side sen-
component of the hearing loss. The hearing aid, acoustic waves i n air sorineural loss may also benefit
Weber will lateralize to the ear impinge on a microphone and from ipsilateral BAHA use for
with the greater conductive hear- are converted to an electric cur- contralateral routing of sound.
ing loss. The Rinne will reveal bone rent; the current signal is ampli-
conduction greater than air con- fied and drives an 5. (1) Pure tone average bone
duction when the air bone gap is electromagnetic speaker, creat- thresholds worse than 45 dB HL
greater than 15 dB. When the air ing acoustic waves in air again or word discrimination score
bone gap is greater than 25 dB, the (but much more intense); these <60% i n the target ear; (2) emo-
1024 Hz fork will reverse. Tuning waves then cause ossicular tional instability, development
forks are essential for confirming motion. Directly driving the delay, or drug abuse; (3) age
the audiometric findings. The posi- ossicular chain with an <5 years (NOTE: relative con-
tive Schwartze sign (a red blush implantable aid obviates the con- traindication, depending on
over the promontory) is seen i n version back into air acoustic skull thickness). Implantation of
only about 10% of patients with waves. Less amplification is osseointegrated fixtures i n irra-
active otosclerosis. The tympanic required in the aid circuitry, and diated or otherwise diseased
membrane may be opaque, but i t is the required incus (or stapes) bone and i n bone <3-mm thick
usually clear. The blue sclera is motion is less than required of has a higher incidence of failure
associated with osteogenesis imper- the speaker coil in a conven- and device extrusion.
fecta and the white forelock is seen tional aid, so both distortion and
in Waardenburg's syndrome. power use can be minimized. CHAPTER 158
Patient Evaluation and Device
5. e 3. Just as feedback from the Selection for Cochlear
The one stage fenestration was first speaker to the microphone of a Implantation
described and popularized by conventional hearing aid can
Julius Lempert i n the late 1930s. cause a squeal, the same can 1 b
In the late 1800s, attempts were happen when the TIGA drives Genetic syndromal deafness repre-
made to correct the hearing loss the ossicular chain, making the sents a small proportion of all sig-
with stapedectomy, but these tympanic membrane behave like nificant hearing loss. Studies
attempts were associated with a a speaker generating acoustic indicate that up to 50% of all NSHL
high incidence of meningitis and waves i n air that feed back to cases are due to a mutation i n a
death. I n 1953 Sam Rosen the nearby TIGA microphone i n single gene encoding connexin 26
described the stapes mobilization the ear canal. Gutting the (Gx26). The gene coding for Gx26
followed by John Shea's introduc- malleus neck breaks this feed- (gap junction protein p or GJB2)
2

tion of the stapedectomy. back path but also compromises is located at locus DFNB1 on
native conductive hearing. human chromosome 13ql2. The
CHAPTER 157 diagnosis of auditory neuropathy/
Surgically Implantable Hearing 4. (1) Any patient who uses a con- auditory dyssynchrony (AN/D) has
Aids ventional bone conduction (BG) been specified as a hearing disor-
hearing-aid; (2) air conduction der i n which normal cochlear outer
1. Physical factors: insufficient (AG) hearing aid user with hair cell function is found in con-
gain, acoustic feedback, alter- chronic otorrhea; (3) AG hear- junction with absent or abnormal
ation of spectral shape and ing aid user experiencing too auditory neural responses, which is
phase, nonlinear distortion, much discomfort because of indicative of poor neural syn-
occlusion effects, externally visi- chronic otitis media/externa; chrony. Prenatal infection with
ble, poor transduction efficiency (4) AG hearing aid user experi- TORCH organisms (toxoplasmosis,
(thus short battery life), lack of encing uncontrollable feedback syphilis, rubella, cytomegalovirus
directionality. caused by a radical mastoidec- [CMV] and herpes) is commonly
tomy or large meatoplasty; associated with deafness. This
2. Each conversion of energy from (5) otosclerosis, tympanosclero- spectrum of infections can result i n
one physical domain to another sis, canal atresia with a con- reduced ganglion cell counts, cog-
incurs some loss and distortion, traindication to repair, such as nitive dysfunction, and abnormal
so minimizing the number of in an only hearing ear. Also, position of the facial nerve.
Answers Key 309

Bilateral temporal bone fractures therefore contraindicated. spectral cues of the input signal
resulting i n deafness can be reha- Cochlear implantation was initially envelope as rapidly as possible.
bilitated with cochlear implants. viewed as contraindicated i n young The important spectral informa-
children with chronic suppurative tion is sent by designated elec-
2 a otitis media (CSOM) because of the trodes that are tonotopically
Current adult selection criteria i n potential risk of infection. organized (i.e., high- and low-fre-
the most recent clinical trials However, selective retrospective quency information to basal and
include: (1) severe or profound studies have shown that the preva- apical electrodes, respectively).
hearing loss w i t h a pure-tone lence and severity of OM does not The greater the n , the more spec-
average (PTA) of 70 dB H L , increase after implantation, leading tral information may be provided,
(2) use of appropriately f i t hearing surgeons to advocate cochlear given that the electrodes are able
aids or a trial w i t h amplification, implantation if the ear is dry at the to be perceived as independent
(3) aided scores on open-set sen- time of implantation. The diagnosis stimulation channels. N-of-m is
tence tests of <50%, (4) no evi- of auditory neuropathy does not available with the Med-El device.
dence of central auditory lesions preclude a child from cochlear Pulses with the High Resolution
or lack of an auditory nerve, and implant candidacy. (HiRes) strategy are available w i t h
(5) no evidence of contraindica- the HiRes 90K device manufac-
tions for surgery i n general or 4. d tured by the Advanced Bionics
cochlear implant surgery i n par- For children implanted between 4 Corporation. Continuous inter-
ticular. I n addition, cochlear and 5 years, expectations include leaved sampling (CIS) is a speech
implant centers generally recom- improvement i n speech perception processing strategy that has been
mend at least 1 to 3 months of with excellent closed-set perform- implemented i n cochlear implant
hearing aid use, realistic expecta- ance and varied open-set abilities, devices i n recent years. Clarion,
tions by the patient and family improvements i n speech produc- Nucleus, and MED-EL implement a
members, and willingness to com- tion, use of hearing to support version of the CIS speech-process-
ply w i t h follow-up procedures as improvements i n language, and ing strategy i n their respective
defined by the center. reduced dependence on visual cues devices. Spectral peak extraction,
for communication. or SPEAK, is implemented i n the
3. c Nucleus device. The Advanced
Although the average postoperative 5. e Combination Encoder (ACE) strat-
scores for individuals with prelin- The most common preimplant fac- egy was designed for the Nucleus
gual hearing loss are generally tors that affect performance for device to incorporate the spectral
lower than those with postlingual children include age at implanta- representation benefits of SPEAK
hearing loss, there have been sig- tion, hearing experience (age at w i t h a high rate CIS.
nificant preoperative to postopera- onset of profound hearing loss,
tive improvements i n speech amount of residual hearing, pro- CHAPTER 159
perception reported for this group. gressive nature of the hearing loss, Medical and Surgical
Therefore, adults with prelingual aided levels, consistency of hearing Considerations in Cochlear
onset of severe-to-profound hearing aid use), training with amplifica- Implants
loss may be appropriate candidates tion (in the case of some residual
for cochlear implantation. hearing), presence of other disabili- 1. e
Audiologic results for cochlear ties, and parent and family sup- 2. a
implant users ages 65 to 80 years port. Postimplant factors that 3 d
indicate significant improvements contribute to performance levels 4. c
for both preoperative and postoper- include length of cochlear implant 5. d
ative comparisons and for varied use, rehabilitative training, and
speech stimulus presentation lev- family support. CHAPTER 160
els. Therefore, increased age is not Cochlear Implants: Results,
a contraindication for cochlear 6. c Outcomes, and Rehabilitation
implant candidacy. When congeni- W i t h the n-of-ra pulsatile strategy,
tal or acquired narrow internal n is the number of electrodes stim- 1. c
auditory canals are identified on ulated out of a total of m elec- 2 d
preoperative CT scanning, primary trodes available. The goal of the 3 b
afferent innervation may be lack- n-of-m strategy is to transmit the 4. c
ing, and cochlear implantation is most prominent and important 5 d
310 ANSWERS

CHAPTER 161 embolization may be lost. Gelfoam arteries, the stylomastoid branch of
Diagnostic and Interventional is usually hand-cut on the table, the occipital artery, the posterior
Neuroradiology and so is less easy to use than PVA. auricular artery, and the tentorial
Finally, a tissue adhesive is like branch of the internal carotid
1. a water i n that i t will flow into the artery. Preoperative embolization is
MRI is the modality of choice for smallest tributaries, such as the an excellent technique to decrease
evaluating sensorineural hearing tiny feeders to the cranial nerves. surgical blood loss. Surgery is made
loss, because i t is the most sensi- There is usually no reason to sub- more difficult by the close quarters
tive detector of tumors and other ject a patient to the risk of cranial of the bony skull base.
diseases affecting the internal audi- nerve palsy for a preoperative Embolization is more difficult if
tory canal and the cerebellopon- embolization, especially when PVA multiple feeders are present.
tine angle, and i t is able to detect is universally available. Finally, MRI of the entire head and
abnormal signal intensities from neck is an excellent way to detect
the parenchyma, such as the 3. b, d multiple chemodectomas present
involvement of the brainstem by The "passage" of the BOT relies on in approximately 10% of patients.
multiple sclerosis. I t is able to the status of collateral circulation
demonstrate enhancement of the to the hemisphere fed by the vessel 5. a, d
meninges without obscuration by being temporarily occluded. If The incidence by location is true,
the contiguous bony structures, there are no neurologic deficits according to numerous sources
which hampers GT i n such detec- during the test, the only thing the (see text and references). Although
tion. I t can demonstrate enhance- performer knows is that the blood many meningiomas are hypervas-
ment of the cranial nerves passing flow must be above the threshold cular, those in certain locations are
through the bony foramina, indica- to produce a deficit, 20 mL/100 notorious for not being vascular at
tive of perineural spread of tumor, g/min. If the flow was 22 m L , the angiography; the suprasellar
especially if fat-suppression tech- patient might be at risk for a post- meningioma is an example.
niques are used. The contiguity of operative stroke developing after Embolization caries the risk of
bone again is the detriment to the permanent vascular occlusion if stroke and cranial nerve palsy.
use of GT. However, GT is the there were superimposed hypoten-
modality of choice when evaluating sion of decreased cardiac output. A CHAPTER 162
the middle ear and the ossicles for GBF study is the only way to make Temporal Bone Neoplasms and
a condition producing conductive such a determination. Although Lateral Cranial Base Surgery
hearing loss. The spatial resolution quantitative GBF studies can define
of GT is superior to MRI, so that ischemic or infracted tissue simply 1. c
small bony structures are far better by the blood flow numbers, a much 2. c
evaluated with GT than MRI. I n simpler way to define infarction is 3 d
addition, air i n the middle and MRI with diffusion-weigh ted imag- 4 b
outer ears is a natural contrast ing. GBF studies will not define the 5. e
agent for GT. potential for clot propagation,
which may be a major reason for CHAPTER 163
2. c postoperative strokes even with a Extra-Axial Neoplasms Involving
PVA is the perfect choice for "negative" preoperative BOT. the Anterior and Middle Cranial
embolizing a vascular tumor such Fossa
as a chemodectoma. I t is very easy 4. a, b, c\ d, e
to use, coming from the manufac- Although the glomus jugulare 1 d
turer as dried particles of well- tumor is histologically "benign," i t 2 b
defined sizes from which to select. produces irregular bone destruc- 3 b
Small particles (approximately 150 tion that simulates a more aggres- 4. c
microns) will block the small arter- sive, even malignant, tumor. I t is 5 d
ies i n the tumor bed, not just the always hypervascular; if, on angiog-
larger feeding arteries that coils raphy, the tumor i n question is not CHAPTER 164
can only do. Vascular recanaliza- vascular, i t is not a chemodectoma. Surgery of the Anterior
tion will take weeks to months or Small tumors may be fed primarily and Middle Cranial Base
might not occur; Gelfoam breaks from the ascending pharyngeal
down in 72 hours, and so if surgery artery, but a large tumor will also 1. a
is delayed, the effect of the receive supply from the meningeal 2 b
Answers Key 311

3. e 2. e venous plexus inferiorly. I t is the


4. c The anterior ethmoidal artery, a largest communicating channel
5. a terminal branch of the ophthalmic between the paired cavernous
artery, exits the ethmoid foramen sinuses. Although midline transfa-
CHAPTER 165 at or just superior to the frontal cial approaches to midline skull
Extra-Axial Neoplasm ethmoid suture and enters the base structures are advantageous
of the Posterior Fossa anterior cranial fossa as the lateral for their direct access, they are
edge of the cribriform plate. I t sup- restricted by critical neurovascular
1 b plies the mucosa of the anterior structures such as the internal
2. b and middle ethmoid air cells and carotid artery, optic nerve, cav-
3. e the dura covering the cribriform ernous sinus, and the basilar
4. a plate and the planum sphenoidale. venous plexus. Large lesions often
The pterygopalatine fossa is situ- compress the basilar venous
CHAPTER 166 ated between the posterior wall of plexus, but profuse bleeding can
Auditory Brainstem Implants the maxillary sinus anteriorly and occur when the external layer of
the pterygoid process of the sphe- clival dura is incised and the
1 b noid posteriorly. I t contains the plexus is not compressed. Bleeding
2. e pterygopalatine ganglion, which can usually be controlled with
3 d receives the Vidian nerve, the max- Surgicel, but this area must be
4. e illary nerve as i t leaves the fora- approached with caution.
5 d men rotundum, the internal
maxillary artery and its two termi- 5 d
CHAPTER 167 nal branches, the posterior lateral Most orbital complications stem
Transnasal Endoscopic-Assisted nasal artery and the septal artery. from direct injury to the optic
Surgery of the Skull Base nerve or the extraocular muscles
3. e or from bleeding w i t h i n the bony
1 b The transnasal endoscopically orbit. Direct or indirect damage to
The transseptal approach was assisted approach for repair of CSF the optic nerve usually occurs at
conceived to provide midline fistula has many advantages over the superolateral sphenoid sinus
access to the sphenoid sinus craniotomy. I t allows for precise wall or i n the posterior ethmoid
region through the nasal septum. localization of defects and direct cells. The location of the optic
This midline access avoids dam- repair with minimal morbidity. The nerves along the superolateral
age to the structures i n the nasal type of repair depends on the size wall of the sphenoid sinus must
cavity and the carotid artery and and location of the defect. Small be appreciated when performing
optic nerve along the lateral wall defects may be covered with procedures through the sphenoid
of the sphenoid sinus. I t is partic- mucoperiostcal grafts from the sinus to avoid injury.
ularly useful to access the clivus, middle or inferior turbinates. When
sella, and parasellar regions, located i n the fovea ethmoidalis, an CHAPTER 168
because these are all midline ethmoidectomy is required for Intraoperative Monitoring of
structures. The transmaxillary identification. Total middle Cranial Nerves i n Neurotologic
approach is ideal for approaching turbinate resection is indicated Surgery
lesions involving the medial por- specifically for repair of defects in
tion of the maxillary sinus, the the cribriform plate. 1. e
pterygopalatine fossae, or zygo- 2 d
matic fossae. The transnasal 4 b 3 b
direct approach is ideal for lesions The basilar venous plexus is 4 b
involving the roof of the nasal located between the two layers of 5 a
cavity without involvement of the the dura of the upper clivus and is
ethmoid sinus, lesions of the related to the dorsum sella and the CHAPTER 169
nasopharynx, and some lesions posterior wall of the sphenoid Radiation Therapy of the Cranial
involving the sphenoid sinus. The sinus. I t forms interconnecting (Skull) Base
transethmoidal approach is i n d i - venous channels between the infe-
cated for lesions extending into rior petrosal sinuses laterally, the 1 d
or involving the ethmoid and cavernous sinuses superiorly, and 2. e
sphenoid sinuses. the marginal sinus and epidural 3 b
312 ANSWERS

4. a exchange as described previously, 2. a


5. c hypoxemia can develop rapidly, This is straightforward descriptive
and the first sign is usually brady- anatomy.
CHAPTER 170 cardia. During surgery, any unex-
General Considerations plained episode of bradycardia 3. c
should be initially treated with This is also straightforward
1. a oxygen and increased ventilation. anatomy, although you should be
When newly born, infants have a During hypoxemia, neonatal pul- aware that on occasion the trans-
low ratio of type I to type I I muscle monary vasoconstriction and verse cervical arises as an inde-
fibers i n their diaphragm. This hypertension occur more dramati- pendent branch from the third part
gives a higher proportion of fatiga- cally than i n adults. This can shift of the subclavian artery.
ble type I I fibers. If the work of them back into fetal circulation,
breathing increases, they may soon compounding the problem. 4 b
fatigue, faster than older children. Answer a is incorrect, because the
This is compounded by a high 5 b stapes is derived from the second
compliance of the chest wall Because the total blood volume of arch; c is incorrect, because the
decreasing the efficiency at which an infant is small, significant blood long process of the malleus and
ventilation occurs. loss can accompany relatively incus are from the second arch; d
minor surgical blood loss. I t has is also incorrect, because although
2. a been observed during exchange the malleus is from the first arch,
The laryngeal chemoreflex (LCR) transfusions that withdrawal of the long process of the incus is
causing laryngospasm seems to be blood parallels a decline i n systolic from the second.
most sensitive to water and is blood pressure and cardiac output.
ablated by saline application. Acid, This is reversible to normal param- 5 b
base, and pressure can also induce eters with replacement of the same Straightforward descriptive
the reflex and may be important i n blood volume removed. Changes i n anatomy.
the cycle of laryngospasm, airway arterial blood pressure with normal
obstruction, hypoxia, bradycardia, heart rates are thus proportional to 6. d
and death seen in SIDS. the degree of hypovolemia. A new- Straightforward embryology
born's ability to adapt the intravas- description.
3. c cular volume to the available blood
The usual Starling curves of con- volume is limited because of less 7. a
tractility we are familiar w i t h i n efficient control of capacitance ves-
the adult cardiovascular physiology sels and immature or ineffective Straightforward clinical description.
do not hold true for the neonatal baroreceptors. The infant's systolic
heart. Cardiac output is rate arterial blood pressure is closely 8 b
dependent i n the neonatal heart. related to the circulating blood vol- Straightforward clinical descrip-
Bradycardia invariably equates ume. Blood pressure is then an tion.
with reduced cardiac output. excellent guide to the adequacy of 9. d
Because of the differences i n com- blood or fluid replacement during Injury i n the supraclavicular fossa
pliance and contractility i n the anesthesia, a fact that has been only involves the distal part of X I ,
neonatal heart, increased contrac- confirmed by extensive clinical thus i t has no effect on the ster-
tility is not possible to maintain experience. nomastoid muscle and only affects
cardiac output during bradycardia. the trapezius on the paralyzed
The low compliance of the relaxed CHAPTER 171 side.
ventricle limits the size of the Developmental Anatomy
stroke volume and, therefore, 10. Only thyroglossal cysts and
increases i n preload are not as 1. a dermoid present as midline
important i n neonatal physiology The carotid sheath begins at the masses of the neck. Branchial
as is heart rate. base of the skull surrounding the cysts are i n the anterior
carotid and jugular canal and thus triangle and follow the ante-
4 b receives the vessels and 9, 10, and rior border of the sternomas-
Because of a relatively high meta- 11 cranial nerves. The sheath also toid muscle. Pharyngoceles,
bolic rate seen i n neonates and the includes the hypoglossal canal and laryngoceles, and carotid
relative low reserve for gas the emerging 12th nerve. body tumors present as
Answers Key 313

masses i n the carotid 5 d immune system, they are more


triangle. Embolization must be of the nidus, likely to have upper respiratory
or epicenter, of the AVM. There is tract viral infections and associated
CHAPTER 172 no place for ligation or proximal acute sinusitis. There is a strong
Anesthesia embolization of feeding vessels. association between sinusitis and
This will lead to rapid recruitment respiratory viral infections. Viral
1. e of flow from nearby arteries and infections are thought to cause sig-
2. a denies access for embolization. nificant ciliary dysfunction by
3. e decreasing the ciliary beat fre-
4. e C H A P T E R 175 quency or destroying the ciliary
5. c Craniofacial Surgery for blanket. This results i n edema,
Congenital and Acquired which obstructs the ostium and
CHAPTER 173 Deformities increases the chance of establish-
Characteristics of Normal and ing a bacterial infection of the
Abnormal Postnatal Craniofacial 1 d sinuses. This edema will interrupt
Growth and Development 2. c the drainage of the anterior eth-
3 b moid sinuses and maxillary sinuses
1. e 4 d and predispose the patient to acute
2. a 5 b and chronic sinusitis.
3. c
4. e CHAPTER 176 2 d
5. e Cleft Lip and Palate It is now clear that plain films do
not adequately image the pedi-
CHAPTER 174 1. c atric sinuses. I n the setting of
Vascular Tumors and 2 d acute sinusitis, we would expect
Malformations of the Head and 3. a plain films and CT scans to be
Neck 4. c positive. Gwaltney's and Glasier's
5 b work showed a high incidence of
1. e opacification of the anterior eth-
Hemangioma grows rapidly (the CHAPTER 177 moid and maxillary sinuses w i t h
proliferative phase) during the first Velopharyngeal Dysfunction acute rhinovirus infections. For
6 to 8 months of life followed with assessing the status of sinuses,
slow regression (the involution 1 b the coronal CT remains the image
phase). 2. c method of choice. I n general,
3. e sinusitis is a clinical diagnosis,
2. e 4. e and radiographic imaging is not
The empiric dose for IFN is 2 to 3 5 d necessary i n children to confirm
million units/m , injected subcuta-
2 the diagnosis. CT scans should be
neously everyday. CHAPTER 178 obtained when both the parents
Congenital Malformations of the and the surgeon believe surgical
Nose intervention is warranted.
The usual dosage of systemic corti- The CT scan is used primarily to
costeroid is 2 to 3 mg/kg/day of 1 b look for anatomic abnormalities
prednisone. If no response is seen 2 d that would increase the risk of
in 7 to 10 days, steroids should be 3. c surgical complications and to
tapered and stopped. 4. c help document the presence of
5 a disease. The CT scan should
4 d been obtained after a trial of
It is clinically useful to separate CHAPTER 179 m a x i m u m medical management
the vascular malformations into Pediatric Chronic Sinusitis that would include broad-spec-
"slow-flow" (capillary, venous, l y m - t r u m antibiotics and topical nasal
phatic, or combined form) or "fast- 1. d steroid sprays for at least 4 weeks.
flow" (arteriovenous fistula [AVF] Age is clearly one of the most sig- The CT scan should be obtained
and arteriovenous malformation nificant factors i n pediatric sinusi- at the end of this course of m a n -
[AVM]) lesions. tis. Because of their immature agement.
314 ANSWERS

3 b with 12 standard facial measure- before a slow but progressive invo-


Chronic sinusitis is associated with ments on both groups. A facial lution. Ninety percent of heman-
more resistant bacteria and there- plastic expert performed blinded giomas involute by the age of
fore will need to be treated with qualitative facial analysis on stan- 9 years.
broader spectrum antibiotics. For dardized photographs. Both quanti-
the most resistant strains of pneu- tative and qualitative analyses 4 b
mococcus, File has found the showed no trends or statistical sig- Mucoepidermoid carcinoma is
newer form of amoxicillin/clavu- nificance i n changes of facial the most common malignant
lanate (AMX/CA) 2000/125 mg and growth between children who epithelial salivary gland neoplasm,
the fluoroquinolones were highly underwent FES surgery and those accounting for approximately
active against these cultured iso- who had chronic sinusitis but did 50% of salivary gland malignancies.
lates from patients with commu- not undergo FES surgery. Their Most of these are low-grade
nity-acquired respiratory tract data also showed no deviations, or lesions that have a good prognosis.
infection. These drugs, however, trends toward deviation, from the The treatment is surgical excision
should be saved for the most resist- standard norms i n children. They (superficial parotidectomy),
ant infections. concluded there was no evidence including a generous cuff of
that FES surgery affected facial normal salivary gland tissue around
4. c growth i n children. it, with preservation of the facial
Absolute indications include (1) nerve. The facial nerve is
complete nasal airway obstruction CHAPTER 180 preserved i n all cases, unless the
in cystic fibrosis caused by massive Salivary Gland Disease tumor is grossly invading the
polyposis or closure of the nose by nerve.
medialization of the lateral nasal 1. c
wall; (2) antrochoanal polyp; (3) Extensive use of the mumps vacci- 5. c
intracranial complications; (4) nation has lead to a significant Sublingual gland excision is not a
mucoceles and mucopyoceles; (5) decline in the number of reported realistic treatment option for c h i l -
orbital abscess; (6) traumatic cases of mumps i n the United dren w i t h excessive salivation.
injury to the optic canal; (7) dacry- States. Although not common, the A good medical option is a trial of
ocystorhinitis caused by sinusitis other inflammatory processes listed glycopyrrolate. Surgical options
and resistant to medical treatment; occur far more often than mumps. include bilateral parotid duct liga-
(8) fungal sinusitis; (9) some tion w i t h submandibular gland
meningoencephaloceles; and (10) 2. e excision, bilateral parotid duct
some neoplasms. Relative or possi- The most common organisms lead- and submandibular duct ligation,
ble indications, which include most ing to acute bacterial sialadenitis or submandibular duct rerouting.
patients, are (1) chronic rhinosi- are Staphylococcus aureus and
nusitis that persists despite optimal Streptococcus viridans. The use of CHAPTER 181
medical management and after the appropriate antimicrobial agents Pharyngitis and Adenotonsillar
exclusion of any systemic disease. against these organisms is essential. Disease
The use of hydration, massage, sial-
5 a ogogues, and warm compresses is 1 d
Bothwell and others sought to also beneficial to treat this process. 2 b
determine whether functional 3 d
endoscopic sinus (FES) surgery 3. e 4 b
performed i n children with chronic Hemangioma is the most common 5. c
rhinosinusitis alters facial growth. neoplasm found i n the parotid
Sixty-seven children with a mean gland i n the pediatric population. CHAPTER 182
age of 3.1 years at presentation These are usually discovered at Obstructive Sleep Apnea
were evaluated for facial growth 10 b i r t h or shortly after b i r t h . i n Children
years later at a mean age of 13.2 Physical findings include facial
years. I n this group, there were 46 asymmetry and a fluctuant mass. 1 b
children who underwent FES sur- The hemangioma generally grows The "gold standard" for diagnosis
gery and 21 children who did not very rapidly over the t h i r d to of OSAS in children is polysomnog-
undergo FES surgery and acted as twelfth month of life u n t i l reach- raphy i n a pediatric sleep labora-
a control. Quantitative anthropo- ing a plateau. The lesions then tory. Tonsil and adenoid size does
morphic analysis was performed remain stable for a period of time not directly predict the presence of
Answers Key 315

OSAS. Daytime somnolence is 5 b mended. Neck dissection is per-


unusual i n children with OSAS. Adenotonsillectomy is the most formed only i n the setting of obvi-
History and clinical examination common treatment of childhood ous nodal extension or FNA-proven
do not distinguish between primary OSAS and is usually curative, espe- metastases.
snoring and OSAS i n children. cially i n otherwise healthy children.
Home testing for OSAS i n children Tracheotomy is sometimes indi- 4. e
is not widely used and needs to be cated for the management of severe Monostotic, local LCH can be
validated against studies i n the OSAS i n children with complicated treated by all of the methods listed.
sleep laboratory. anatomic or neuromotor issues. Interestingly, spontaneous resolu-
Children who fail to respond to or tion can also occur. Chemotherapy
2. e are not candidates for surgical inter- is reserved for multisystem or
Several retrospective studies have vention can also often be managed widespread disease.
detailed clinical risk factors for res- successfully with nasal continuous
piratory compromise after adeno- (CPAP) or bilevel positive airway 5. a
tonsillectomy for OSAS. These risk pressure. Nocturnal oxygen supple- Removal of the tonsils and adenoids
factors include young age, severe mentation has been studied as a is often curative i n the posttrans-
OSAS on sleep study, craniofacial temporary treatment for hypoxemia plant pediatric patient with PTLD
anomalies, neuromotor disease, associated with OSAS until defini- affecting the tonsils and adenoids.
and chromosomal abnormalities. tive therapy can be provided. The rate of PTLD is highest in liver,
Supplemental oxygen therapy may heart, and heart-lung transplant
3 b suppress hypoxic ventilatory drive recipients because of the increased
Sleep-related upper-airway obstruc- and lead to significant hypercarbia immunosuppression needed i n
tion in children may manifest as in some OSAS patients, so i t must these patients. Therefore, observa-
obstructive apnea or obstructive be administered with caution and tion is never warranted.
hypoventilation. Obstructive initiated i n a monitored setting.
hypoventilation results from con- CHAPTER 184
tinuous partial airway obstruction, CHAPTER 183 Differential Diagnosis of Neck
which leads to paradoxical respira- Pediatric Head and Neck Masses
tory efforts, hypercarbia, and often Malignancies
hypoxemia. Diagnosis of obstruc- 1. c
tive hypoventilation i n children 1. e First branchial derivatives are
requires end-tidal G 0 monitoring
2 All of the patient situations listed found along the mandible and third
during polysomnography. Despite put children at increased risk for derivatives near the upper pole of
the absence of complete airway childhood malignancies developing the thyroid gland.
obstruction during sleep, children on the basis of altered immune sys-
with obstructive hypoventilation tem capabilities. 2 d
are at risk for all of the reported Ultrasonography of the neck is the
complications of OSAS. Clinical 2 d most practical and economical of
measures of tonsil size alone do Fresh tissue, with its mRNA, is the studies listed above to identify
not predict the need for surgery. necessary for the pathologist to normal thyroid tissue.
perform molecular genetic
4. c analysis, cytogenetics, and cell cul- 3 b
Numerous studies have demon- ture. This is critical i n the workup All of the modalities are
strated that OSAS cannot be distin- of pediatric small round blue cell useful. Treatment of cutaneous
guished from PS i n children on the tumors. hemangiomas may result i n later
basis of clinical history and physi- malignant transformation.
cal examination alone. Overnight 3. c
polysomnography in a sleep labora- Cervical metastases are rare in 4. c
tory is the current "gold standard" pediatric salivary gland malignan-
for differentiating childhood OSAS cies. The presence of cervical 5 a
from PS. An accurate diagnosis of adenopathy i n a pediatric patient is Pseudomonas is a virulent organ-
OSAS will ensure that appropriate more likely to reflect reactive ism i n the neonatal period but is
treatment is provided when needed hyperplasia of cervical lymph not a pathogen i n infants and chil-
and will avoid unnecessary surgery nodes. Therefore, routine elective dren with normal immune systems.
in patients with PS. neck dissection is not recom- The other three organisms are all
316 ANSWERS

common pathogens i n infants and 2. e 4. b, c


children. All of the above. The staging sys- 5. a, b, c, d
tem for laryngeal clefts and laryn-
6. c gotracheoesophageal clefts has CHAPTER 188
Serologic testing for Bartonella evolved over the past few decades. Gastroesophageal Reflux
DNA is the best method for confir- The most commonly used staging and Laryngeal Disease
mation. Bartonella henselae is dif- systems are the Benjamin/Inglis
ficult to culture, and viral and Myer/Cotton system. When 1. b, c, d
inclusions within biopsied nodes describing intraoperative findings, GER is frequent i n children; at
are nonspecific. the surgeon should specify which least 20% of children experience
staging system is used. clinical GER disease. Postprandial
7 b GER can be considered physio-
The treatment of Mycobacterium 3. c logic. I n this particular case, pH
tuberculosis is with two-drug ther- Gastroesophageal reflux disease. monitoring is normal. I n contrast
apy. Use of one drug fosters resist- Laryngomalacia is commonly asso- with GER i n adults, heartburn is
ance, and surgery is not a usual ciated with GERD. There is some not a frequent sign of GERD i n
option. debate whether the laryngomalacia children. However, major complica-
causes the GERD or the GERD tions, such as obstructive apnea,
8. a contributes to the laryngomalacia choking, and failure to thrive are
Coronary artery aneurysm is a late (the proverbial chicken and egg not exceptional.
sequela of Kawasaki's disease. problem.) The presence of GERD
should be considered and possibly 2. a, b, d
9. d treated i n all patients with laryngo- GER is frequently associated with
malacia. laryngeal diseases. Esophageal motor
10. c disorders are frequently associated
While patients have elevated titers 4. e with laryngomalacia, probably
to Epstein Barr virus, a mono spot Laryngeal and laryngotracheoe- because of the neurologic dysfunc-
test should be negative. sophageal clefts. Children with tion that determines laryngomalacia.
Nasopharyngeal carcinoma typi- laryngeal clefts typically have sig- Posterior laryngeal clefts include
cally metastasizes to the neck and nificant aspiration problems, malformations of the tracheal mus-
may produce unilateral otitis because the liquids/foods they con- cular posterior wall. GER seems to
media with effusion. sume pass into the trachea and be important i n the development of
bronchi through the cleft. acquired subglottic stenosis but not
CHAPTER 185 in pure congenital stenoses.
Congenital Disorders 5 a
of the Larynx Thyroglossal duct cyst i n the val- 3. a, b, c
leculas Autopsy reports of several The first steps of GER treatment
1 d infants who died i n their beds have are always diet measures and
Cardiac evaluation. Laryngeal webs revealed a thyroglossal duct cyst at lifestyle assessment. Antireflux or
are commonly associated with the the foramen cecum. antacid drugs are the second line
chromosome deletion 2 2 q l l . of treatment.
Microscopic and submicroscopic CHAPTER 186
deletions of this chromosome Managing the Stridulous Child CHAPTER 189
cause a wide range of phenotypes, Aspiration and Swallowing
including DiGeorge syndrome, 1. a, b, c Disorders
velo-cardio-facial syndrome, 2. a, b
conotruncal anomaly face syn- 3. a, b, e, d CHAPTER 190
drome, and sporadic or familial 4. a, c Voice Disorders
heart defects. Features related to 5 b, d
these syndromes include cardiac 1 d
defects, abnormal facies, thymus CHAPTER 187 Unilateral vocal cord paralysis
hypoplasia, cleft palate, and Glottic and Subglottic Stenosis rarely requires airway intervention.
hypocalcemia (CATCH-22). A car- Vocal cord paralysis spontaneous
diac defect should be ruled out 1. a, b, c recovers over 6 to 12 months by
before surgery to minimize the 2. a, b, c, d contralateral compensation.
risks of the procedure. 3. a, c Recovery is hastened by speech
Answers Key 317

therapy. Only few patients w i t h 5. c horseshoe shape and a flaccid


persistent dysphonia or aspiration Epidermoid cysts are unilateral but posterior membranous wall.
will need surgery such as vocal occur w i t h associated edema of the These symptoms often resolve
cord injection, medialization, or contralateral vocal cord and thus w i t h time.
reinnervation. are often mistaken for vocal nod-
ules. Laryngeal stroboscopy shows 5 a
2 b decreased vibrating amplitude and Innominate artery compression is
Posterior glottic stenosis i n chil- incomplete glottic closure. the most common form of vascular
dren is most commonly secondary Epidermoid cysts do not resolve compression. I t occurs when the
to airway trauma from intubation. spontaneously and require micro- artery arises more to the left than
To distinguish cricoarytenoid fixa- surgical excision. usual, passing from left to right and
tion from vocal cord paralysis, pal- compressing the anterior wall of
pation of the cricoarytenoid j o i n t CHAPTER 191 the trachea.
at rigid endoscopy is necessary. Congenital Disorders
Arytenoidectomy should be of the Trachea CHAPTER 192
avoided to prevent aspiration, dete- Tracheal Stenosis
rioration of voice, and difficulty 1 d
with future airway repair. The sur- Most infants with tracheal stenosis 1. a
gical procedure of choice is ante- have stridor and respiratory symp- 2 b
rior and posterior cricoidotomy toms i n the prenatal period. 3. e
w i t h posterior graft. The postopera- Classically, biphasic stridor with a 4. c
tive voice is functional; however, marked expiratory component is 5. e
persistent hoarseness and breathi- described. Associated symptoms
ness are common. may include cough, wheezing, CHAPTER 193
apnea, croup, and feeding Caustic Ingestion
3. a difficulty.
Recurrent respiratory papillomato- 1. c
sis (RRP) is usually diagnosed 2. e Liquid NaOH quickly comes i n
between ages 2 and 3 years and is Primary tracheomalacia can be contact with mucosa distally. Due
most often associated w i t h human seen i n premature infants, i n to its alkaline nature, it leads to a
papillomavirus types 6 and 1 1 . infants with connective tissue dis- mucosa injury i n seconds.
RRP often involves the laryngeal orders, and i n otherwise healthy
surface of the epiglottis, the upper full-term infants. I t is a weakness 2 d
and lower margins of the ventri- of the tracheal wall, resulting i n Stridor indicates laryngeal edema
cles, and the undersurface of the marked exaggeration of movement and is evidence that the alkaline
vocal cords, thus resulting i n with respiration. The clinical pres- substance passed to or (generally)
symptoms of hoarseness and air- entation typically includes expira- beyond the level of the cricoid, and
way obstruction. The need for tory stridor, often reminiscent of thus the esophageal inlet.
repeated procedures usually asthmatic wheezing, and varying
causes scarring of degrees of respiratory distress. 3. c
the vocal cords and long-term It is best to immediately remove
deterioration of the voice. 3. c any coin-shaped object that has the
The most common presentation of potential to be a battery, because
4 b tracheoesophageal fistula is proxi- injury from a battery is progressive
Vocal cord granulomas are often mal esophageal atresia with distal and quite destructive. Direct obser-
secondary to intubation trauma tracheoesophageal fistula. vation with airway protection dur-
and are causally related to GERD. ing removal is paramount for safety
Most granulomas resolve w i t h 4. e and prevention of further injury to
medical management of GERD. Postoperatively, patients often a weakened esophageal wall.
Surgical removal of vocal cord have symptoms of tracheomala-
granuloma is reserved for lesions cia, esophageal dysmotility, and 4. c
causing airway obstruction. gastroesophageal reflux. Tracheo- At a m i n i m u m , this represents a
Regardless of the surgical tech- malacia is related to tracheal car- circumferential burn. The exact
nique chosen, recurrences are tilages having an indented depth may be difficult to immedi-
common. semicircular rather than a normal ately determine.
318 ANSWERS

5 d impactions occur i n the cervical CHAPTER 195


Esophageal reflux must be maximally esophagus below the cricopharyn- Infections of the Airway
controlled in patients with esopha- geus muscle. Airway obstruction
geal burns from ingested agents to can result from direct compression 1 b
minimize stricture formation. of the trachea anteriorly by the 2. c
object or by inflammation. A long- 3. e
6. d standing esophageal foreign body 4 d
Such a severe burn can extend into may migrate to an extraluminal 5 b
the mediastinum and stomach, with position.
complete necrosis of the esophagus. CHAPTER 196
3. e Recurrent Respiratory
7. c Esophageal perforations may be Papillomatosis
This is based on work by Vancura caused by the object itself, the
and others (Toxicity of alkaline solu- length of time the object is lodged, 1 b
tions, Ann Emerg Med 9:118, 1980). or by the attempt at removal. Pill HPV types 16 and 18 have been
ingestion has been reported to associated with malignancies of the
8 b result i n esophageal perforation. aerodigestive tract and the cervix.
This prevents a protective eschar Fever, tachycardia, and increased HPV types 6 and 11 are most
that might limit extent of injury pain may indicate an early closely associated with RRP and
from forming. This leads to a esophageal perforation. Open are generally not associated with
deeper, more severe injury per drainage may be necessary to treat malignancies. However, RRP can
amount consumed when compared an esophageal perforation but is undergo malignant degeneration.
to similar acid burns. not always indicated. HPV types 31 and 33 are some-
where between HPV 6/11 and
CHAPTER 194 4. a 16/18 i n malignant potential.
Foreign Bodies of the Airway Disk battery ingestions that impact
and Esophagus in the esophagus may cause 2. e
esophageal injury i n as quickly as 1 HPV types 6 and 11 are associated
1. b hour and need to be removed as with greater than 90% of genital
Esophageal foreign bodies are quickly as possible. Disk batteries condylomata, and there is a strong
more common. The three phases are usually ingested and rarely are association between maternal
of foreign body aspiration are the seen as a bronchial foreign body. condylomata and the transmission
i n i t i a l , asymptomatic, and compli- Radiographs are helpful to localize to her offspring. Overt condylomata
cation phases. Radiographs may disk battery impaction within the are seen in more than 50% of moth-
aid i n the diagnosis of bronchial esophagus. Disk batteries <15 m m ers who give birth to children with
foreign body but cannot in size may traverse through the RRP. There has been no demonstra-
absolutely rule out the presence gastrointestinal tract. tion of the eradication of HPV by
of a foreign object. Hyperinflation any treatment modality. HPV has
of the affected lung is an early 5. e been demonstrated even in histolog-
finding. Optical forceps aid i n Disk battery impaction within the ically normal mucosa, and RRP may
visualization during bronchial for- esophagus is an emergency, and recur at any age after remission.
eign body removal but can actu- the battery should be removed as Cesarean-section delivery does not
ally decrease the ventilatory quickly as possible. Public educa- completely eliminate the risk of
capacity of the bronchoscope tion has decreased mortality from maternal-fetal transmission, suggest-
because of the increased diameter acute airway obstruction from for- ing that there may be placental
of the forceps. eign bodies. Food and nonfood transmission in some rare cases.
objects may cause acute airway Malignant degeneration has been
2. c obstruction. The Heimlich maneu- shown to occur rarely in patients
Coins are the most common ver should only be performed i n with RRP. In a reported series of 244
esophageal foreign body. Multiple complete airway obstruction from patients with RRP, 4 underwent doc-
objects found i n the esophagus are a foreign object; i n children umented malignant transformation
associated with an esophageal younger than 1 year, i t should be (1.6%). In general, the younger the
anomaly i n up to 80% of patients. performed with back blows and age at diagnosis, the more aggressive
Most esophageal foreign body chest thrusts. the RRP clinical course should be.
Answers Key 319

3. a 4 b plication of atresiaplasty, occurring


The finding of pulmonary dissemi- 5. c in up to 22% of cases.
nation of RRP is a grave develop- Sensorineural hearing loss occurs
ment. The clinical course of CHAPTER 199A i n 2% or fewer cases, and with the
pulmonary spread of RRP is insidi- Reconstruction Surgery of the routine use of preoperative high-
ous and may progress over years Ear: Microtia Reconstruction resolution temporal bone CT scan-
but eventually manifests i n respira- ning and facial nerve monitoring,
tory failure from destruction of lung 1. c injury to the facial nerve is rare.
parenchyma. Furthermore, pul- 2 a Auricular reconstruction precedes
monary dissemination is anecdo- 3 d atresiaplasty because of the
tally associated with a higher risk of 4. e demand for an excellent blood sup-
malignant transformation of RRP. 5 b ply for the rib cartilage graft.
The other sites listed pose no spe- Devascularization of cartilage
cial consideration for prognosis. CHAPTER 199B grafts used for auricular recon-
Reconstruction Surgery struction rarely occurs after
4 d of the Ear: Auditory Canal atresiaplasty.
Although multiple methods of treat- and Tympanuum
ment have been tested, and many 4. e
adjuvant therapies are i n ongoing 1 b The stapes footplate derives from
clinical trials, no single modality or The malleus/incus complex is both the second branchial arch and
combination of modalities has been deformed and fixed i n cases of con- from the otic capsule. This dual
consistently shown to eradicate RRP genital aural atresia, but this does embryologic origin may account for
HPV or to guarantee long-term not preclude successful atresi- the fact that i n most cases of con-
remission. aplasty. Most often, the gential aural atresia, the footplate
malleus/incus complex can be is normal and mobile. Meckel's car-
5. e mobilized, and the tympanic mem- tilage gives rise to the neck and
The cardinal triad of new-onset brane graft is placed directly on the head of the malleus and the body
laryngeal RRP includes relentlessly ossicular mass. I n exceptional of the incus, whereas Riechert's
progressive hoarseness accompa- cases, a severe deformity or fixation cartilage forms the long processes
nied by the development of inspira- prevents use of the malleus/incus of the malleus and incus and the
tory stridor, progressing to complex, and a partial ossicular stapes superstructure.
respiratory distress. Cough is a fre- reconstruction prosthesis can be
quently associated symptom. used with a degree of success. Major 5. a
Snoring, i n contrast, is more a malformations that signify poor can- Poor pneumatization is the p r i -
manifestation of upper airway didacy for atresiaplasty include mary cause for inoperability i n
(nasopharyngeal/ poor pneumatization, abnormal or congenital atresia; however, most
oropharyngeal) obstruction. absent oval window/footplate, patients have a well-pneumatized
abnormal course of the facial nerve, tympanic cavity/mastoid air cell
CHAPTER 197 abnormalities of the inner ear. system. Inner ear, facial nerve, and
Early Detection and Diagnosis oval window/footplate anomalies
of Infant Hearing Impairment 2. c are seen less frequently. A high-
The superior landmark for drilling resolution temporal bone CT scan
1. d the ear canal is the tegmen, the identifies these significant abnor-
2 d anterior landmark is the glenoid malities that would preclude sur-
3 b fossa, and the medial landmark is gery and is necessary for all
4. a the malleus/incus complex i n the elective cases of atresiaplasty.
5 b epitympanum. The lateral semicir-
cular canal is not identified in a CHAPTER 200
C H A P T E R 198 standard surgical approach until Acute Otitis Media and Otitis
Congenital Malformations the epitympanum and ossicular Media w i t h Effusion
of the Inner Ear mass have been localized.
1. b
1. a 3 d Studies suggest that more than 80%
2. e Tympanic membrane graft lateral- of children will have at least one
3. e ization is the most common com- bout of OM i n their childhood, and
320 ANSWERS

approximately 25% will have six or 5. A l l of the answers are factors 2. c


more bouts. Although OM can that may warrant intervention The hearing impairment i n bran-
occur at any age, i t is far more w i t h tympanostomy tubes. chiootorenal syndrome (BOR) is
prevalent i n children younger than Patients w i t h cleft palate fre- conductive i n 30%, sensorineural i n
2 who attend daycare. I t is more quently have long-standing 20%, and mixed i n 50% of individu-
common i n Native Americans and eustachian tube dysfunction, als. Outer ear abnormalities include
rare i n newborns. and long-lasting tympanostomy preauricular pits (82%), preauricu-
tubes should be considered for lar tags, microtia, and stenotic
2. e the initial insertion. The hear- external auditory canals. Enlarged
The effect of the hearing loss pro- ing loss associated w i t h vestibular aqueducts, cochlear dys-
duced by OME on speech, lan- eustachian tube dysfunction plasia, and hypoplasia of the lateral
guage, and cognitive development may greatly complicate rehabili- semicircular canal may be seen on
has been hotly debated. Studies tative efforts for patients with GT of the temporal bone. Branchial
show conflicting results and long- an underlying sensorineural cleft fistula and renal anomalies are
term adverse effects i n the typical hearing loss. Reliable drainage also part of this autosomal-domi-
patient are likely to be subtle at of the middle ear by tympanos- nant syndrome.
most. Furthermore, whether these tomy tube placement may sim-
differences, if present, can be plify management of 3. b
attributed to the hearing loss, or to suppurative complications of Waardenburg syndrome type I is
other factors, such as frequent AOM. Tympanostomy tube characterized by sensorineural
coexistent illness, has not been set- placement may be an alterna- hearing loss, pigmentary distur-
tled. I n this setting, any effect by tive to or may augment t y m - bances (white forelock, heterochro-
tympanostomy tubes would clearly panoplasty i n patients w i t h mia irides), and dystopia
be difficult to tease out. severe TM retractions w i t h canthorum. Synophrys; broad
impending cholesteatoma of the nasal root and patent metopic
3. a, b pars tensa. Approximately half suture are other findings that may
In general, most patients will have of conscious patients undergo- be present. I n type I I , dystopia
a spontaneous resolution of the ing hyperbaric oxygen therapy canthorum is absent, although
bulk of their symptoms w i t h i n 2 to will have middle ear complica- there is a greater likelihood of sen-
3 days of the onset whether they tions, including severe pain, sorineural hearing loss. Type I I I is
receive antibiotics or not. The hemotympanum, and OME; also known as Klein-Waardenburg
minority of patients who do not these patients are candidates syndrome and is similar to type I ,
improve is greatest in younger chil- for ear tube placement. along with the presence of upper
dren (2 years old or younger) with limb abnormalities. Type IV
culture-proven disease found by CHAPTER 201 Waardenburg syndrome includes
tympanocentesis. I n this subset, Genetic Sensorineural Hearing Hirschsprung's disease and is also
more than 40% may have pain and Loss known as Waardenburg-Shah syn-
fever lasting over 7 days, and these drome. The hearing impairment is
children are most likely to be 1. c often profound, bilateral, and sta-
helped by antibiotic therapy. I n Mitochondrial mutations account ble over time.
studies using less-rigorous inclu- for 1% to 2% of genetic deafness.
sion criteria, there seems to be l i t - Maternal transmission is the hall- 4. c
tle improvement i n the naturally mark of mitochondrial diseases, Long QT syndrome is found i n
good outcome by using broad-spec- because sperm cells do not Jervell and Lange-Nielsen syn-
t r u m antibiotics or longer courses donate mitochondria to the fertil- drome. The long QT interval may
of treatment, and initial therapy ized egg. Homoplasmy refers to be visualized on EGG. Treatment,
with analgesics alone may be con- the presence of all abnormal if instituted early, can signifi-
sidered i n older children. mtDNA; heteroplasmy refers to cantly reduce the risk of sudden
the presence of a m i x of normal death caused by cardiac a r r h y t h -
4. c and abnormal mtDNA. Random mias. Genetic counseling allows
Tympanostomy tubes confer no transmission to progeny cells the opportunity to inform the
protection after they are extruded. accounts for the variable expres- family and patient about recur-
The remainder of the statements sion that is often seen i n m i t o - rence chances, data interpreta-
are true. chondrial diseases. tion, and treatment options. If a
Answers Key 321

diagnosis of DFNB1 (GJ52-related globes along the orbital roof to lower orbit around age 7. Up to
deafness) is made by genetic test- expose the nasal dorsum. 86% of orbital roof fractures are
ing, no further investigations are associated w i t h intracranial injury.
necessary, because there are no 2. e The orbit and globe rarely sustain
other comorbidities associated Computed tomography scans have long-term damage, and thus sur-
w i t h this form of deafness. Usher revolutionized the care of NOE gery is rarely necessary. Orbital
syndrome includes sensorineural fractures. With both the coronal encephaloceles have been
hearing loss, variable vestibular and axial CT cuts, i t is possible to reported as a late, but uncommon,
dysfunction, and retinitis pigmen- develop a three-dimensional under- sequela.
tosa. Consultation w i t h ophthal- standing of the fracture. This
mology is appropriate for the allows the surgeon to decide 5. e
early detection and follow-up of whether surgical intervention is In prepubescent children, the fre-
eye disease i n patients suspected warranted and, if indicated, a sur- quent absence of teeth and the
of having Usher syndrome. Alport gical plan of repair. Townes view poor retentive shape of the decidu-
syndrome may present w i t h a does not give enough information ous teeth make the use of arch bars
positive urinalysis that demon- on the fractured area. The Panorex and interdental wiring for maxillo-
strates the presence of red blood is helpful to search for a concomi- mandibular fixation (MMF) unfeasi-
cells. tant mandible fracture but does not ble to apply. Fortunately, 2 to
directly give suitable information 3 weeks of mandibular immobiliza-
5 a on the NOE complex. tion i n children younger than 12 is
Genetic hearing impairment adequate. To obtain MMF, one must
accounts for 50% of childhood deaf- 3. c consider the age and development
ness and is nonsyndromic i n 70% The available resorbable plating of the teeth. In children younger
of individuals. Connexins oligomer- systems (1.5- and 2.0-mm screw than 2 and between 5 and 9, immo-
ize to form a connexon that docks diameters) provide flexural and bilization requires unconventional
to a neighboring connexon, tensile strength comparable to the fixation techniques, because the
thereby forming a gap junction. microplate titanium systems (1.0 dentition will not support arch
Aminoglycoside susceptibility is to 1.3 m m diameter screws). bars. One approach is the use of an
caused by an mtDNA. Definitive long-term studies i n overlay acrylic mandibular splint
Nonsyndromic can be either con- facial trauma using resorbable held i n place by circummandibular
genital or late onset. plates have yet to answer whether and transnasal wires. Another
any patient experienced problems approach pioneered and described
CHAPTER 202 with growth restriction. At this by Eppley takes advantage of
Pediatric Facial Fractures time, absorbable plates are not rec- resorbable screws. Between age 2
ommended for all types of pediatric and 5, the deciduous incisors have
1. c facial fractures. The use of firm roots, and if the deciduous
Nasoethmoid fractures are rela- absorbable plates i n the mandible molars have formed, then they can
tively uncommon i n the pediatric and load-bearing" bone is still
a be used for cap splints or arch bars.
population. The most important investigational i n children, and In general, after age 10, the devel-
component of the repair is to over- long-term results are limited. At opment of permanent teeth pro-
correct the fracture core. The use this time, the indications for the vides for safe anchors. However,
of absorbable plates is often diffi- use of absorbable systems i n pedi- children develop at different rates,
cult i n this region, and conse- atric trauma are on non-load-bear- and the strength of the teeth should
quently plates and wires are often ing regions i n the upper and be carefully examined before the
more practical. For cosmesis, i t is middle third of the craniofacial placement of any type of MMF.
important to set the intercanthal skeleton. Condylar fractures presenting with
distance narrower than antici- an open bite, mandibular retrusion,
pated. Exposure is best obtained by 4. a or movement limitation are best
preexisting lacerations or coronal The pattern of orbital fractures treated with 2 to 3 weeks of
incisions, with mobilization of the changes from roof fractures to the immobilization.

You might also like